Thanks to visit codestin.com
Credit goes to www.scribd.com

0% found this document useful (0 votes)
42 views1,604 pages

Syllabus

Uploaded by

Steven Godelman
Copyright
© © All Rights Reserved
We take content rights seriously. If you suspect this is your content, claim it here.
Available Formats
Download as PDF, TXT or read online on Scribd
0% found this document useful (0 votes)
42 views1,604 pages

Syllabus

Uploaded by

Steven Godelman
Copyright
© © All Rights Reserved
We take content rights seriously. If you suspect this is your content, claim it here.
Available Formats
Download as PDF, TXT or read online on Scribd
You are on page 1/ 1604

COMPREHENSIVE

REVIEW OF
GENERAL SURGERY
directed by
Pardon R. Kenney, MD, MMSc, FACS
BRIGHAM AND WOMEN’S FAULKNER HOSPITAL

OAKSTONE CME | BOARD PREP 438


Comprehensive Review of General Surgery
Provided By:
Oakstone Publishing
_____________________________________________________________________________________________________________________________________

DATE OF ORIGINAL RELEASE: February 1, 2022


DATE CREDITS EXPIRE: February 1, 2025
TARGET AUDIENCE:
This educational activity was designed for general surgeons and general surgery residents.
ESTIMATED TIME TO COMPLETE:
It is estimated that it should take the average learner 42 hours to complete the activity.
ACCREDITATION:
Oakstone Publishing is accredited by the Accreditation Council for Continuing Medical
Education (ACCME) to provide continuing medical education for physicians.
DESIGNATION:
Oakstone Publishing designates this enduring material for a maximum of 42 AMA PRA Category
1 Credits™. Physicians should claim only the credit commensurate with the extent of their
participation in the activity.

Successful completion of this CME activity, which includes participation in the evaluation
component, enables the learner to earn credit toward the CME and Self-Assessment requirements
of the American Board of Surgery’s Continuous Certification program. It is the CME activity
provider's responsibility to submit learner completion information to ACCME for the purpose of
granting ABS credit.

SPECIAL PREREQUISITES FOR PARTICIPANTS: There are no prerequisites for


participants.
METHOD OF PARTICIPATION: Review Video/Audio program, score 70% or greater on the
required posttest to assess the knowledge gained from reviewing the program and complete the
comprehensive activity evaluation.
LEARNING OBJECTIVES:
At the completion of this course, you should be able to:

 Explain the principles of prophylaxis of hospital-acquired surgical infections and


appropriate antimicrobial agents.
 Differentiate between the management of benign breast disease and malignant breast
disease.
 Outline the key features of the assessment of trauma and acute care surgery.
 List the current management guidelines for emergency surgery of the large intestine.
 Describe the principles of melanoma excision.
 Summarize evaluation of the swollen leg and DVT diagnosis.
 Discuss open versus laparoscopic surgical approaches to inguinal, femoral, umbilical, and
ventral hernias.
 List the treatment options for primary and secondary liver neoplasms.
____________________________________________________
 Compare management options for benign neoplasms of the esophagus and malignant
neoplasms of the esophagus.
 Differentiate between operative and non-operative treatment of chronic pancreatitis.
 Describe the indications for and extent of resection for inflammatory bowel disease.
 Compare endoscopic versus resectional management of colon neoplasms.

FACULTY AFFILIATIONS DISCLOSURE:


Oakstone Publishing has assessed conflict of interest with its faculty, authors, editors, and any
individuals who were in a position to control the content of this CME activity. Any identified
relevant conflicts of interest have been mitigated. Oakstone Publishing’s planners, content
reviewers, and editorial staff disclose no relationships with ineligible entities.
Disclosure information for all individuals in control of the content of the activity is located
on the disclosure slides in the PDF and printed syllabus.

WARNING:
The copyright proprietor has licensed the picture contained on this recording for private
home use only and prohibits any other use, copying, reproduction, or performance in
public, in whole or in part (Title 17 USC Section 501 506).

CMEinfo is not responsible in any way for the accuracy, medical or legal content of this
recording. You should be aware that substantive developments in the medical field covered
by this recording may have occurred since the date of original release.

© 2022 Ebix, Inc. DBA Oakstone Publishing.


CMEinfo is a registered trademark of Oakstone Publishing.
Comprehensive Review of General Surgery

Faculty List
_____________________________________________________________________________________________________________________________________

Course Director:

Pardon R. Kenney, MD, MMsc, FACS


Staff Surgeon,
Brigham and Women’s Faulkner Hospital;
Senior Lecturer in Surgery,
Harvard Medical School,
Boston, MA

Speakers:

Reza Afrasiabi, MD David Brooks, MD


Fellow, Acute Care Surgery, Brigham and Associate Professor of Surgery, Harvard
Women’s Hospital, Boston, MA Medical School; Senior Surgeon, Brigham
and Women’s Hospital, Boston, MA
Geoffrey Anderson, MD
Associate Surgeon, Brigham and Women’s Matthew J. Carty, MD
Hospital; Harvard Medical School, Boston, Staff Surgeon, Division of Plastic Surgery,
MA Brigham and Women’s Hospital; Associate
Professor of Surgery, Harvard Medical
Stanley W. Ashley, MD School, Boston, MA
Frank Sawyer Professor of Surgery, Brigham
and Women’s Hospital; Harvard Medical Nancy L. Cho, MD
School, Boston, MA Assistant Professor of Surgery, Harvard
Medical School; Section Chief, Endocrine
Reza Askari, MD, FACS Surgery, Associate Surgery Clerkship
Assistant Professor of Surgery, Harvard Director, Brigham and Women’s Hospital,
Medical School; Trauma/Burns and Surgical Boston, MA
Critical Care, Department of Surgery,
Director, Surgical Intensive Care Unit, Thomas Clancy, MD
Director, SICU and ACS Fellowship, Division of Surgical Oncology, Brigham and
Director, Surgical Clerkship, Brigham and Women’s Hospital; Gastrointestinal Cancer
Women’s Hospital, Boston, MA Center, Dana-Farber Cancer Institute;
Assistant Professor of Surgery, Harvard
Ronald Bleday, MD Medical School, Boston, MA
Chief, Section of Colon and Rectal Surgery,
Vice Chairman of Surgery for Quality and Roger Clark, DO
Safety, Brigham and Women’s Hospital; Lecturer in Medicine, Harvard Medical
Associate Professor of Surgery, Harvard School; Assistant Professor of Medicine,
Medical School, Boston, MA Tufts University School of Medicine,
Boston, MA
_____________________________________________________________________________________

Nathan T. Connell, MD, MPH James A. Greenberg, MD


Assistant Professor of Medicine, Harvard Chief of Gynecology, Brigham and Women’s
Medical School; Clinical Chief of Faulkner Hospital; Associate Professor of
Hematology, Brigham and Women’s Obstetrics, Gynecology, and Reproductive
Faulkner Hospital; Associate Physician, Biology, Harvard Medical School,
Hematology Division, Brigham and Women’s Boston, MA
Hospital, Boston, MA
Mohamad A. Hussain, MD
Zara Cooper, MD Vascular and Endovascular Surgeon,
Michele and Howard J. Kessler Distinguished Brigham and Women’s Hospital, Harvard
Chair of Surgery and Public Health, Kessler Medical School; Junior Core Faculty, Centre
Director, Center for Surgery and Public for Surgery and Public Health, Boston, MA
Health, Chair, Executive Advisory
Committee for Diversity, Equity and Jennifer L. Irani, MD
Inclusion, Director, Center for Geriatric Assistant Professor of Surgery, Harvard
Surgery, Associate Surgeon, Division of Medical School; Associate Surgeon, General
Trauma, Burns and Surgical Critical Care, and Gastrointestinal Surgery, Brigham and
Brigham and Women’s Hospital; Associate Women’s Hospital and Dana-Farber Cancer
Professor of Surgery, Harvard Medical Institute, Boston, MA
School, Boston, MA
Michael J. Malone, MD
Laura S. Dominici, MD Section Chief, Division of Urology, Brigham
Section Chief of Breast Surgery, Brigham and and Women’s Faulkner Hospital, Boston, MA
Women’s Faulkner Hospital; Associate
Surgeon, Dana-Farber Cancer M. Blair Marshall, MD
Institute/Brigham and Women’s Hospital; Michael A. Bell Family Distinguished Chair
Assistant Professor of Surgery, Harvard in Healthcare Innovation, Associate Chief for
Medical School, Boston, MA Quality, Promotions, Mentorship and
Inclusion, Brigham and Women’s Hospital;
Mark Fairweather, MD Associate Professor of Surgery, Harvard
Associate Surgeon, Division of Surgical Medical School, Boston, MA
Oncology, Brigham and Women’s
Hospital/Dana-Farber Cancer Institute; Nelya Melnitchouk, MD, MSc, FACS
Assistant Professor of Surgery, Harvard Assistant Professor of Surgery, Harvard
Medical School, Boston, MA Medical School; Associate Surgeon, Brigham
and Women’s Hospital, Boston, MA
Jason S. Gold, MD
Associate Professor of Surgery, Harvard Matthew Menard, MD
Medical School; Department of Surgery, Associate Professor of Surgery, Harvard
Brigham and Women’s Hospital; Chief of Medical School; Co-Director of Endovascular
Surgical Oncology, Surgery Service, VA Surgery, Brigham and Women’s Hospital,
Boston Healthcare System, Boston, MA Boston, MA

Joel Goldberg, MD, MPH Faina Nakhlis, MD


Associate Surgeon, Brigham and Women’s Assistant Professor of Surgery, Harvard
Hospital; Assistant Professor of Surgery, Medical School; Dana-Farber/Brigham and
Harvard Medical School, Boston, MA Women’s Cancer Center, Boston, MA
_____________________________________________________________________________________

Prathima Nandivada, MD Rosh Kumar Viasha Sethi, MD


Assistant Professor of Surgery, Harvard Assistant Professor of Otolaryngology-Head
Medical School; Pediatric Surgeon, Boston and Neck Surgery, Harvard Medical School;
Children’s Hospital, Boston, MA Associate Surgeon, Brigham and Women’s
Hospital; Head and Neck Oncology, Dana-
Matthew A. Nehs, MD, FACS Farber Cancer Institute, Boston, MA
Assistant Professor of Surgery, Harvard
Medical School, Boston, MA David Shaff, MD
Chief of Anesthesiology, Brigham and
Louis L. Nguyen, MD, MBA, MPH Women’s Faulkner Hospital, Boston, MA
Vascular and Endovascular Surgery, Vice Scott A. Shikora, MD
Chair for Digital Health Systems, Director, Center for Metabolic and Bariatric
Department of Surgery, Brigham and Surgery, Brigham and Women’s Hospital;
Women’s Hospital; Associate Professor of Professor of Surgery, Harvard Medical
Surgery, Harvard Medical School, School, Boston, MA
Boston, MA
Douglas S. Smink, MD, MPH
Stephanie Nitzschke, MD Chief of Surgery, Brigham and Women’s
Assistant Professor of Surgery, Harvard Faulkner Hospital; Vice Chair for Education,
Medical School; Brigham and Women’s Department of Surgery, Brigham and
Hospital, Boston, MA Women’s Hospital; Associate Professor of
Surgery, Harvard Medical School,
Luise Pernar, MD, MHPE Boston, MA
Surgeon, Division of Minimally Invasive and
Bariatric Surgery, Boston Medical Center; Kristin Sonderman, MD, MPH
Assistant Professor of Surgery, Boston Associate Surgeon, Division of Trauma, Burn
University School of Medicine, Boston, MA and Critical Care, Brigham and Women’s
Hospital; Harvard Medical School,
Chandrajit P. Raut, MD, MSc Boston, MA
Chief, Division of Surgical Oncology,
Brigham and Women’s Hospital; BWH David Spector, MD
Distinguished Chair in Surgical Oncology; Director of Bariatric and Anti-Reflux
Surgery Director, Center for Sarcoma and Surgery, Brigham and Women’s Faulkner
Bone Oncology, Dana-Farber Cancer Hospital, Boston, MA
Institute; Professor of Surgery, Harvard
Medical School, Boston, MA Ali Tavakkoli, MD
Chief, Division of General and GI Surgery,
Nakul P. Raykar, MD Brigham and Women’s Hospital, Boston, MA
Trauma and Emergency Surgery, Faculty,
Center for Surgery and Public Health, Thomas Tsai, MD
Brigham and Women’s Hospital; Fellowship Associate Surgeon, Brigham and Women’s
Director, Program in Global Surgery and Hospital; Assistant Professor of Surgery,
Social Change, Harvard Medical School, Harvard Medical School, Boston, MA
Boston, MA
_____________________________________________________________________________________

Stefan G. Tullius, MD, PhD, FACS Jon O. Wee, MD


Joseph E. Murray, MD, Distinguished Chair Section Chief, Esophageal Surgery, Director
in Transplant Surgery, Chief Division of of Robotics in Thoracic Surgery, Co-Director
Transplant Surgery, Director, Transplant of Minimally Invasive Thoracic Surgery,
Surgery Research Laboratory, Brigham and Associate Program Director, Division of
Women’s Hospital; Professor of Surgery, Thoracic Surgery, Brigham and Women’s
Harvard Medical School, Boston, MA Hospital; Assistant Professor of Surgery,
Harvard Medical School, Boston, MA
Ashley Haralson Vernon, MD
Associate Surgeon, Brigham and Women’s Anna Weiss, MD
Hospital; Assistant Professor of Surgery, Assistant Professor of Surgery, Harvard
Harvard Medical School, Boston, MA Medical School, Dana-Farber/Brigham and
Women’s Cancer Center, Boston, MA
James Vogel, MD Director of Breast Surgery Research, Dana-
Attending Anesthesiologist, Medical Director Farber/Brigham and Women’s Cancer Center
of Center for Preoperative Evaluation, South Shore affiliate, South Weymouth, MA
Brigham and Women’s Faulkner Hospital;
Instructor in Anaesthesia, Harvard Medical James Yoo, MD
School, Boston, MA Associate Professor of Surgery, Harvard
Medical School, Boston, MA
Jiping Wang, MD, PhD, FACS
Associate Surgeon, Surgical Oncology, Charles H. Yoon, MD
Department of Surgery, Brigham and Director of Surgery for Cutaneous Oncology
Women’s Hospital; Dana-Farber Cancer and Melanoma, Director of Surgical
Institute; Associate Professor of Surgery, Oncology, Merkel Cell Carcinoma Center of
Harvard Medical School, Boston, MA Excellence, Dana-Farber/Brigham and
Women’s Cancer Center, Boston, MA
Oakstone Publishing
Comprehensive Review of General Surgery

Book
Topic/Speaker Page #
APPLIED SCIENCE
Surgical Infection
1
Roger Clark, DO
Fluids, Electrolytes, and Acid-base Disorders
27
Reza Askari, MD, FACS
Transfusion Medicine and Anticoagulant Management
58
Nathan T. Connell, MD, MPH
Concepts in Anesthesia
91
David Shaff, MD
Perioperative Care
118
James G. Vogel, MD
Head and Neck Evaluation
146
Rosh Kumar Viasha Sethi, MD
Thoracic Surgery for General Surgeons
204
Jon O. Wee, MD
Surgical Critical Care
280
Mohammad Afrasiabi, MD
Ethics
330
Zara Cooper, MD, MSc
Transplantation
356
Stefan Tullius, MD, PhD
Pediatric Surgery
399
Prathima Nandivada, MD
VASCULAR
Arterial Disease
444
Matthew Menard, MD
Venous Disease
476
Louis L. Nguyen, MD, MBA, MPH
Vascular Access
501
Mohamad Hussain, MD
SURGICAL SPECIALTIES
Plastic Surgery
529
Matthew J. Carty, MD
Gynecologic Challenges for General Surgeons
552
James A. Greenberg, MD
Genitourinary Surgery
570
Michael J. Malone, MD
Oakstone Publishing
Comprehensive Review of General Surgery

Book
Topic/Speaker Page #
SKIN AND SOFT TISSUE
Tumor Biology
612
Charles H. Yoon, PhD, MD, FACS
Soft Tissue Sarcoma
630
Chandrajit P. Raut, MD, MSc
Melanoma and Dysplastic Nevi
673
Charles H. Yoon, PhD, MD, FACS
ENDOCRINE
Adrenal Surgery and Endocrine Dysfunction
694
Matthew A. Nehs, MD, FACS
Thyroid and Parathyroid Surgery
736
Nancy L. Cho, MD
BREAST
Benign Breast Disease
764
Anna Weiss, MD
Malignant Breast Disease
808
Laura S. Dominici, MD
Paget’s Disease and Male Breast Cancer: Surgical Techniques
844
Faina Nakhlis, MD
ABDOMINAL - GENERAL
General Abdominal Surgery
863
Luise Pernar, MD, MHPE
ABDOMINAL - HERNIA
Abdominal Hernias - Incidence, Cause, and Treatment
896
David C. Brooks, MD
Incisional Hernia
932
Douglas S. Smink, MD, MPH
ABDOMINAL - BILIARY
Benign Gallbladder Disease
955
Ashley Haralson Vernon, MD
Common Duct Diseases
975
Thomas C. Tsai, MD
ABDOMINAL - LIVER
Primary and Secondary Liver Neoplasms
989
Thomas E. Clancy, MD
Oakstone Publishing
Comprehensive Review of General Surgery

Book
Topic/Speaker Page #
ABDOMINAL - PANCREAS
Pancreatic Neoplasms
1043
Mark Fairweather, MD
Inflammatory Pancreatic Disease
1079
Stanley W. Ashley, MD
ABDOMINAL - SPLEEN
Surgical Diseases of the Spleen
1124
Ali Tavakkoli, MD
ALIMENTARY TRACT - ESOPHAGUS
Benign and Malignant Neoplasms of the Esophagus
1160
M. Blair Marshall, MD
Diseases of Gastroesophageal Junction
1180
David Spector, MD
ALIMENTARY TRACT - STOMACH
Gastric Cancer Treatment
1231
Jiping Wang, PhD, MD
Bariatric Surgery and Peptic Ulcer Disease
1248
Scott A. Shikora, MD
ALIMENTARY TRACT – SMALL BOWEL
Small Bowel - Management Challenges
1304
Jason S. Gold, MD
Inflammatory Bowel Disease
1333
Joel E. Goldberg, MD, MPH
LARGE INTESTINE
Emergencies of the Large Intestine
1361
James Yoo, MD
Colon Neoplasms
1380
Jennifer L. Irani, MD
ANORECTAL
Management of Rectal and Anal Cancer and Dysplasia
1413
Nelya Melnitchouk, MD
Anal Fissure and Fistula
1434
Ronald Bleday, MD
TRAUMA / ACUTE CARE SURGERY
Initial Assessment in Trauma and Acute Care Surgery
1454
Kristin Sonderman, MD, MPH
Oakstone Publishing
Comprehensive Review of General Surgery

Book
Topic/Speaker Page #
TRAUMA / ACUTE CARE SURGERY
Thoracic Trauma
1503
Geoffrey Anderson, MD
Abdominal Trauma
1526
Nakul P. Raykar, MD
Management of > 20% TBSA Burns - The First 24 Hours
1552
Stephanie Nitzschke, MD
Top 6 Surgical Gastrointestinal Emergencies
1574
Reza Askari, MD, FACS
Copyright © Oakstone Publishing, 2021. All Rights Reserved.

COMPREHENSIVE REVIEW OF GENERAL SURGERY

Surgical Infections
Roger P. Clark, D.O.
Infectious Diseases Staff
Associate Hospital Epidemiologist
Brigham and Women’s Faulkner Hospital, Boston
Brigham and Women’s Hospital, Boston

Disclosures
• None

1
Copyright © Oakstone Publishing, 2021. All Rights Reserved.

Diagnostics

Diagnostics
• Begins with obtaining the best sample available
• Tissue and fluid more sensitive and specific than swabs
• Often only one chance to obtain culture data

• Proper labelling is important in interpretation


• “wound”
Vs
• “ left metatarsal head – bone”

• Prompt delivery to micro lab helps to ensure good results

2
Copyright © Oakstone Publishing, 2021. All Rights Reserved.

Specimen Collection, Device, Temperature


and Transportation

Clinical Infectious Diseases 2018;67(6):e1–e94

Specimen Collection, Device, Temperature


and Transportation

Clinical Infectious Diseases 2018;67(6):e1–e94

3
Copyright © Oakstone Publishing, 2021. All Rights Reserved.

Specimen Collection, Device, Temperature


and Transportation

Clinical Infectious Diseases 2018;67(6):e1–e94

Antimicrobial Agents

4
Copyright © Oakstone Publishing, 2021. All Rights Reserved.

Antimicrobial Agents
Empiric choice
• First establish – is it an infection?
• Presence of bacteria does not necessarily establish an infection
• Presence of bacteria in non-sterile space may indicate commensal organisms
• Example: Asymptomatic bactiuria
• Presence of bacteria in a sterile space may be contaminant
• Example: ¼ blood culture bottles with Coagulase negative Staphylococcus

Antimicrobial Agents
Empiric choice
• Treat empirically vs awaiting culture data?
• Can antibiotics wait until cultures can be obtained?
• In certain cases (ie: longstanding osteomyelitis without systemic
signs/symptoms or severe associated cellulitis)
• Accurate culture data may allow more specific treatment and narrowing of
coverage

5
Copyright © Oakstone Publishing, 2021. All Rights Reserved.

Antimicrobial Agents
Empiric choice

• Choose antibiotics based on:


• site of infection
• allergies
• previous infections
• route of administration
• local antibiogram
• toxicities
• cost
• host factors

Empiric Antibiotics
Site of infection
• Does the antibiotic achieve adequate levels in the affected tissue?
• Examples:
• Prostate
• CNS

• Is the antibiotic effective in the affected tissues?


• Example:
• Daptomycin in lungs
• Good levels in lungs, but…
• Inactivated by pulmonary surfactant
• Bad choice for airspace infections
• OK to use for septic pulmonary emboli

6
Copyright © Oakstone Publishing, 2021. All Rights Reserved.

Empiric Antibiotics
Allergies
• Is it a true allergy?
• Anaphylaxis, rash, hives, etc
• Intolerance to drug
• Nausea, diarrhea
• Are there protocols in place to challenge certain allergies:
• Ie: childhood penicillin allergies
• Skin testing
• Test dose
• OK to give some cephalosporins?
• Desensitization
• Use of a systematic guideline can often allow safe use of drugs in same
class

1) Infect Control Hosp Epidemiol 2019 May; 40(5): 528–535

Using previous cultures to guide empiric


antibiotics:
• Previous culture results should be taken into account when selecting
empiric antibiotics
• For gram negative infections, good correlation exists between prior
blood stream infections and current resistance patterns (92%
specificity in on large observational multicenter cohort study)1
• One year lookback is reasonable, prior cultures become less useful
the more distant past they were obtained

1) Clinical Microbiology and Infection;2018


24(5):493-499

7
Copyright © Oakstone Publishing, 2021. All Rights Reserved.

Antimicrobial Agents
Antibiogram
• Performed yearly
• Minimum 30 isolates
• Avoid repeat samples from same patient (1st culture per pt per year)
• Avoid skewing1
• Can be broken down to specific areas in Hospital (eg: ICU)
• Community Hospital vs Academic Medical Center

Clinical Infectious Diseases 2007; 44:867–73

8
Copyright © Oakstone Publishing, 2021. All Rights Reserved.

Empiric Antibitiotics: other considerations


Toxicity Cost
• Consideration of any antibiotic • Secondary – but still important
involves Risk/Benefit analysis consideration.
• What benefit can a specific • If 2 equivalent drugs, the less
antibiotic offer expensive should be chosen
• What are the risks? • Someone has to be a steward of
• How can risks be ameliorated? healthcare costs. If not you, then
who?
• Example: QTc prolongation
from FQs – obtain ECG

9
Copyright © Oakstone Publishing, 2021. All Rights Reserved.

Empiric Antibitiotics: other considerations


• Host factors:
• Pregnancy
• Lactation
• Renal or Hepatic dysfunction
• Age

Selecting Antibiotics based on Cultures


• 2 major methods to determine susceptibility:
• Kirby-Bauer Disk Diffusion
• MIC method

10
Copyright © Oakstone Publishing, 2021. All Rights Reserved.

Selecting Antibiotics based on Cultures


Kirby Bauer Disk Diffusion Method

https://microbenotes.com/kirby-bauer-disc-diffusion/

Selecting Antibiotics based on Cultures


MIC Method

11
Copyright © Oakstone Publishing, 2021. All Rights Reserved.

How to interpret MIC results

CLSI M100 Performance Standards for Antimicrobial


Susceptibility Testing. 31st Ed. 2021

Principles of Prophylaxis

It’s a horse, of course!

12
Copyright © Oakstone Publishing, 2021. All Rights Reserved.

Prevention of Surgical Site Infections (SSI)


• NHSN defines SSI as an infection occurring
within 30 days of index surgery (90 days if
implant present).
• Classified as:
• Superficial Incisional SSI
• Deep Incisional SSI
• Organ Space SSI

Antimicrobial Prophylaxis
• Used in conjunction with Infection Control practices, optimization of
patient factors (nutrition, smoking cessation, etc) and other means to
prevent surgical site infections.
• Ideally, antimicrobial prophylaxis will1:
• (1) prevent SSI
• (2) prevent SSI-related morbidity and mortality
• (3) reduce the duration and cost of health care
• (4) produce no adverse effects
• (5) have no adverse consequences for the microbial flora of the patient or the
hospital.

1) Am J Health-Syst Pharm. 2013; 70:195–283

13
Copyright © Oakstone Publishing, 2021. All Rights Reserved.

Timing of antimicrobial prophylaxis


• The optimal time for administration of preoperative doses is within 60
minutes before surgical incision.
• Some agents, such as fluoroquinolones and vancomycin, require
administration over one to two hours; therefore, the administration
of these agents should begin within 120 minutes before surgical
incision.
• repeat intraoperative dosing is warranted for procedures that exceed
two half-lives of the drug and for procedures in which there is
excessive blood loss (>1500 mL)

Am J Health-Syst Pharm—Vol 70 Feb 1, 2013

Duration of antimicrobial prophylaxis


• If antimicrobial prophylaxis is continued postoperatively, the duration
should be less than 24 hours, regardless of the presence of
intravascular catheters or indwelling drains

14
Copyright © Oakstone Publishing, 2021. All Rights Reserved.

Society Guidelines Available


• Including Joint Guidelines from:
• American Society of Health-System Pharmacists (ASHP)
• Infectious Diseases Society of America (IDSA)
• Surgical Infection Society (SIS)
• Society for Healthcare Epidemiology of America (SHEA)

• Local (Hospital Specific) guidelines generally build from Society


guidelines, employing local antibiograms in select cases.

Am J Health-Syst Pharm—Vol 70 Feb 1, 2013

Am J Health-Syst Pharm—Vol 70 Feb 1, 2013

15
Copyright © Oakstone Publishing, 2021. All Rights Reserved.

Am J Health-Syst Pharm—Vol 70 Feb 1, 2013

16
Copyright © Oakstone Publishing, 2021. All Rights Reserved.

Hospital Acquired Infections (HAI)


• Per the CDC, “On any given day, about one in 31 hospital patients has
at least one healthcare-associated infection”.
• HAIs Largely tracked through CDC’s National Heathcare Safety
Network (NHSN).
• Central Line Associated Bloodstream Infection (CLABSI)
• Catheter Associated UTI (CAUTI)
• SSI
• Ventilator Associated Pneumonia (VAP)

Central Line Associated Bloodstream Infection


(CLABSI)
• Attributable mortality estimated at 18%
• Resulting in 28,000 annual deaths in the US1
• Incidence can be reduced by following evidence based guidelines

1) https://www.ahrq.gov/hai/clabsi-tools/guide.html

17
Copyright © Oakstone Publishing, 2021. All Rights Reserved.

CLABSI Prevention
• Follow recommended central line insertion practices to prevent infection when
the central line is placed, including:
• Perform hand hygiene
• Apply appropriate skin antiseptic (ie: 2% chlorhexidine) and allow to completely dry before
inserting the central line
• Use all five maximal sterile barrier precautions:
• Sterile gloves and gowns
• Cap
• Mask
• Large sterile drape
• Once the central line is in place:
• Follow recommended central line maintenance practices
• Perform hand hygiene before and after touching the line
• Remove a central line as soon as it is no longer needed. The sooner a catheter is
removed, the less likely the chance of infection.
https://www.cdc.gov/hai/bsi/clabsi-resources.html
Infect Dis Clin North Am. 2017 Sep; 31(3): 551–559.

CAUTI Facts
• UTIs are the most common type of healthcare-associated infection
reported to the NHSN.
• Among UTIs acquired in the hospital, ~ 75% are associated with a
urinary catheter.1
• Between 15-25% of hospitalized patients receive urinary catheters
during their hospital stay.
• The daily risk of acquisition of bacteriuria varies from 3% to 7% when
an indwelling urethral catheter remains in situ.2 Therefore, catheters
should only be used for appropriate indications and should be
removed as soon as they are no longer needed.
1) infection control and hospital epidemiology may 2014, vol. 35, no. 5
2) https://www.cdc.gov/hai/ca_uti/uti.html

18
Copyright © Oakstone Publishing, 2021. All Rights Reserved.

Ventilator Associated Pneumonia (VAP)


• NHSN VAC, IVAC, Possible or Probable VAP: Stable Vent pt. > 2days, FiO2 & PEEP :
• VAC Minimum daily 1) ↑ FiO2 >.20 x 2 days, or
2) ↑ PEEP > 3 cm H2O x 2 days
• IVAC At least 2 of the following:
• 1) Fever (> 380C or > 100.40F) with no other recognized cause for fever or Leukopenia
• 2) New antimicrobials x 4 days
• Possible VAP (1 needed)
•1) New onset of purulent sputum or change in character of sputum
•2) Positive culture
• Probable VAP (1 needed)
• 1)Purulent secretion & positive culture
• 2)+ pleural fluid Cx, lung path, Legionella lab or Viral respiratory test

VAP Prevention
• Avoid intubation if possible (NIPPV)
• Minimize sedation
• Maintain and improve physical conditioning
• Minimize pooling of secretions above the endotracheal tube cuff
• Elevate the head of the bed
• Maintain ventilator circuits

19
Copyright © Oakstone Publishing, 2021. All Rights Reserved.

Antimicrobial Stewardship
• Increase • Decrease
• Infection cure rates • Treatment failures
• C.difficile Infections
• Adverse effects
• Hospital costs and lengths of
stay
• Antibiotic resistance

Antimicrobial Stewardship - Antibiotic


resistance

https://www.youtube.com/watch?v=plVk4NVIUh8

20
Copyright © Oakstone Publishing, 2021. All Rights Reserved.

https://www.youtube.com/watch?v=plVk4NVIUh8

https://www.youtube.com/watch?v=plVk4NVIUh8

0 1 10 100 1000 100 10 1 0

21
Copyright © Oakstone Publishing, 2021. All Rights Reserved.

https://www.youtube.com/watch?v=plVk4NVIUh8

11
Days

0 1 10 100 1000 100 10 1 0

Principles of immune response to injury

22
Copyright © Oakstone Publishing, 2021. All Rights Reserved.

Bacterial Defenses
• Physical
• Chemical
• Resident microbiota
• Innate immunity
• Adaptive immunity

Innate Immunity
• Skin
• Mucosa Physical Barriers
• Physical expulsion (cilia, motility, flow)
• Commensal microbiota
• Pattern Recognition Receptors
• Complement
• Innate cells (Dendritic cells, macrophages, NK cells, granulocytes, etc)

23
Copyright © Oakstone Publishing, 2021. All Rights Reserved.

Importance of commensal microbiota

• Direct competition with pathogens


• > 1 billion years of practice
• Direct killing
• Competing for resources

• Dynamic role in calibrating the immune system

• All bacteria are not bad!!!

Adaptive immunity
• B Lymphocytes produce antibodies with assistance from certain T
cells
• Antibodies are key mechanism in clearing
• Bacteria
• Fungi
• Parasites
• Viruses
• Exotoxins

24
Copyright © Oakstone Publishing, 2021. All Rights Reserved.

Immune response to Surgery and Trauma


• Surgical trauma can lead to exuberant immune response
• Tissue trauma can lead to pro-inflammatory cytokines
• IL-1β, IL-6, TNF
• Also compensatory anti-inflammatory response
• IL-4, IL-10, sTNFR1, IL-1 receptor antagonist (IL-1Ra), transforming growth
factor β (TGF-β)

Cent Eur J Immunol. 2014; 39(4): 532–537

Immune response to Surgery and Trauma


• Some inflammation is good
• Too much inflammation is not so good
• ERAS guidelines can reduce some of the negative effects of post-op
inflammation

25
Copyright © Oakstone Publishing, 2021. All Rights Reserved.

Summary of Discussion
• Diagnostics
• Antimicrobial agents
• Principles of prophylaxis
• Opportunistic and hospital-acquired infection
• Stewardship
• Principles of immune response to injury

I Know you are all busy…..

26
Copyright © Oakstone Publishing, 2021. All Rights Reserved.

Reza Askari, MD FACS


Fluids, Assistant Professor of
Surgery
Electrolytes, and Trauma/Burns and
Acute Care Surgery

Acid-Base Department of Surgery


Brigham and Women’s

Disorders Hospital

• No Disclosures to report

27
Copyright © Oakstone Publishing, 2021. All Rights Reserved.

Overview

• Fluid homeostasis

• Electrolyte abnormalities

• Acid/Base Disorders

Fluid compartments
• ~60% of total body mass is
water
• Lower in females
• Decreases with age
• 2/3 of total body water is
intracellular
• Of the remaining 1/3
• 75% is interstitial
• 25% is plasma

28
Copyright © Oakstone Publishing, 2021. All Rights Reserved.

Fluid homeostasis
• Fluid balance between intravascular &
interstitial compartments
• Described by the Starling equation
• Capillary hydrostatic pressure (Pc)
drives fluid out of the bloodstream
• Capillary oncotic pressure (πc) draws
fluid into the bloodstream
• Primarily driven by albumin

Fluid homeostasis
• Renin-Angiotensin-Aldosterone System
• Modulated by JGA in kidneys
• Renin → angiotensin I → angiotensin II
• Glomerular efferent capillary constriction
• ↑ capillary hydrosta?c pressure →
↑ filtra?on
• ADH release from posterior pituitary
• Free water resorption from renal
collecting ducts
• Aldosterone release from adrenal cortex
• ↑ tubular Na+ resorption
• Also ↑ K+ excretion
• Free water follows Na+

29
Copyright © Oakstone Publishing, 2021. All Rights Reserved.

Fluid homeostasis

• Baroreceptor system
• Modulates sympathetic tone
• Sympathetic tone -> renin release from JGA
• Controls release of ADH
• Free water resorption from renal collecting ducts
• Also released in response to ↑ plasma osmolarity
• Natriuretic peptides
• Volume overload → stretch of cardiac ?ssue
• ANP & BNP
• Na+ & free water excretion from kidneys

Hypovolemia
• Hemorrhage, GI losses, burns, etc
• Usually isotonic fluid loss
• Ac?va?on of RAAS → Na+ resorption
• Low FENa (< 1%) & FEUrea (< 35%)
• High BUN:Cr
• Orthostasis
• Treat with balanced crystalloids
• E.g. LR
• Cl- load from NS → metabolic acidosis

30
Copyright © Oakstone Publishing, 2021. All Rights Reserved.

Fluid overload
• Over-resuscitation, heart/renal/liver failure, SIADH
• ↑ capillary hydrosta?c pressure → piKng edema
• CXR findings:
• Cephalization & hilar fullness
• Kerley lines & thickening of interlobar
fissures
• Alveolar infiltrates & pleural effusion
• RV distention & IVC non-callapsibility on US
• Treatment
• Loop diuretics
• Thiazides to augment diuresis
• Spironolactone for cirrhosis
• Dialysis or CVVH for refractory cases

Electrolyte balance
• Na+/K+ pump
• Maintains gradient between intra- &
extracellular compartments
• Extracellular: Na+ & Cl-
• Intracellular: K+ & PO43-

31
Copyright © Oakstone Publishing, 2021. All Rights Reserved.

Sodium disorders
• Usually reflect disorder of total body water
• Change in plasma tonicity = particular risk
for CNS
• Rapid correc?on of hyponatremia →
osmotic demyelination
• Rapid correc?on of hypernatremia →
cerebral edema

Hyponatremia
• Evaluation
• Correct for hyperglycemia
• Add 2 mEq/L for each 100 mg/dL increase in glucose
Signs and symptoms of hyponatremia
• Consider pseudohyponatremia
• Spurious result from unmeasured solutes Mild Fatigue
• Mannitol, bilirubin, glycerine/sorbitol from Nausea
TURP or hysteroscopy, etc
• Check serum osm if suspected Headache
• Assess volume status Moderate Lethargy
• Hypervolemic: Heart failure or cirrhosis Weakness
• Euvolemic: SIADH, water intoxication
• Urine osm < 100: water intoxication Hyporeflexia
• Urine osm > 100: SIADH Severe Obtundation
• Hypovolemic: Renal or extrarenal losses Seizure
• Urine Na+ < 25: Extrarenal (GI losses, burns)
Coma
• Urine Na+ > 40: Renal (diuretic, adrenal
insufficiency, cerebral salt wasting)

32
Copyright © Oakstone Publishing, 2021. All Rights Reserved.

Hyponatremia
• Treatment
• Severe hyponatremia (seizure,
obtundation, respiratory depression)
• Hypertonic saline bolus
• ICU admission & hourly Na+
monitoring
• Target 4-6 mEq/L in 1st 6 hrs
• Mild hyponatremia
• Assess underlying cause
• Hypovolemic: Fluid resuscitation
with isotonic crystalloids
• Euvolemic or hypervolemic: Free
water restriction & salt tabs
• Target correction 4-6 mEq/L per day

Hypernatremia
• Evaluation
• Assess for free water loss
• Insensible losses Signs and symptoms of hypernatremia
• Upper GI losses Mild Irritability
• Loop diuretics
Restlessness
• Osmotic diuretics or cathartics
• Assess for iatrogenic causes Moderate Lethargy
• Hypertonic saline Hyperreflexia
• Inadequate free water (e.g. tube feed Spasticity
dependent)
Severe Obtundation
• Urine osm if etiology unclear
• > 600 suggests extrarenal free water loss Seizure
• 300-600 suggests osmotic diuresis Coma
• < 300 suggests diabetes insipidus

33
Copyright © Oakstone Publishing, 2021. All Rights Reserved.

Hypernatremia
• Treatment
• Severe hypernatremia (seizure,
obtundation, coma)
• D5W bolus
• ICU admission & hourly Na+
monitoring
• Target 1-2 mEq/L per hour
• Mild hypernatremia
• Correct total body water deficit
• Use enteral route if possible
• Target correction <10 mEq/L per day

Potassium disorders
• K+ is primarily intracellular
• Plasma concentration determined by:
• Total body K+ content
• Acid-base balance
• Intracellular K+ exchanged with
extracellular H+

34
Copyright © Oakstone Publishing, 2021. All Rights Reserved.

Hypokalemia
• Evaluation
• ECG to assess severity
• Flattened or inverted T wave
• Prominent U wave
• Assess for common causes
• Vomiting
• Diarrhea
• Diuretics
• Consider urine K+:Cr ratio & acid-base
status for rare causes
• Renal tubular acidosis
• Mineralocorticoid excess

Hypokalemia
• Treatment Potassium Symptoms

• Correct severe or acute Asymptomatic 3-3.5 None


hypokalemia parenterally Mild 2.5-3 Smooth muscle
• If refractory, check & dysfunction (e.g.
ileus)
replete Mg2+
• Oral supplementation Severe <2.5 Muscle cramps
preferred for mild Weakness
Dysrhythmias
hypokalemia

35
Copyright © Oakstone Publishing, 2021. All Rights Reserved.

Hyperkalemia
• Evaluation
• ECG to assess severity
• Peaked T waves
• Widened QRS complex (late sign)
• Consider spurious result (e.g.
hemolysis)
• Assess for common causes
• Crush injuries & burns
• Renal insufficiency
• Adrenal insufficiency
• Acidosis

Hyperkalemia
• Treatment Treatment of severe hyperkalemia – “C BIG K Di”
• Severe hyperkalemia
• Stabilize cardiac membrane
• Calcium gluconate Calcium gluconate
• Shift K+ intracellularly to temporize
• Insulin/glucose & albuterol Beta agonist (high-dose albuterol)
• Reduce total body K+
• K+ binder, loop diuretic, or Insulin
dialysis
• Mild hyperkalemia Glucose

• Shifting K+ intracellularly is
counterproductive K+ binder

• Reduce total body K+


Diuresis or Dialysis
• K+ binder, loop diuretic, or
dialysis
• Bicarbonate is not effective

36
Copyright © Oakstone Publishing, 2021. All Rights Reserved.

Calcium disorders
• Half of serum Ca2+ is protein bound
• Ionized (unbound) fraction is physiologically
active
• Homeostasis
• Vitamin D
• Stimulates absorption from gut
• Decreases excretion from kidney
• Increases resorption from bone
• PTH
• Decreases excretion from kidney
• Increases resorption from bone

Hypocalcemia
• Evaluation
• Correct for hypoalbuminemia Signs and symptoms of hypocalcemia
• Ca2+ = 0.8(4 - albumin) + Ca2+measured
Mild Muscle cramps
• Consider common causes
• Hypoparathyroidism (check PTH) Abdominal pain
• Chronic kidney disease
Hyperreflexia
• Calcium depletion
• Blood transfusion Chvostek and Trousseau signs
• Pancreatitis
Severe Tetany
• Treatment
• Symptomatic: IV calcium Seizure

• Chronic hypoparathyroidism: Vitamin D & Ca2+


QT prolongation
supplementation

37
Copyright © Oakstone Publishing, 2021. All Rights Reserved.

Hypercalcemia
• Evaluation
• High PTH: Hyperparathyroidism Signs and symptoms of hypercalcemia
• Low PTH: Paraneoplastic syndrome (PTHrP), bony
cancer metastasis, thiazide diuretics, Mild Bone pain
hyperthyroidism Kidney stones
• Treat if symptomatic or Ca2+ > 14 mg/dL Abdominal pain/constipation
• Volume expansion with NS
Depression
• Loop diuretics when euvolemic only if renal failure
or CHF is present Severe Confusion
• Calcitonin & bisphosphonate Stupor
• Denosumab if bisphosphonates
Coma
contraindicated
• Hemodialysis for refractory cases Dysrhythmia

Refeeding syndrome
• PO43- is primarily intracellular (component
of ATP)
• Hypophosphatemia typical with
malnourishment & liver resection
• Compounded by glycolysis
• Causes neuromuscular dysfunction,
including cardiopulmonary collapse in
severe cases
• Treat with phosphate supplementation
(IV or PO depending on severity)

38
Copyright © Oakstone Publishing, 2021. All Rights Reserved.

Acid base regulation


• Carbonic acid/bicarbonate buffer system
• Carbonic anhydrase converts H2CO3 ↔
CO2
• Ventilation regulates CO2
• Kidneys regulate HCO3-
• Renal acid excretion
• H2PO4- & NH4+ directly excreted

Primary acid-base disorders


• Metabolic derangements
• HCO3- & pCO2 move in same direction
as pH Disorder pH HCO3- pCO2
• Rapid respiratory compensation
Metabolic acidosis ↓ ↓ ↓
• Respiratory derangements
Metabolic alkalosis ↑ ↑ ↑
• HCO3- & pCO2 move in opposite
direction as pH Respiratory acidosis ↓ ↑ ↑
• Gradual metabolic compensation
Respiratory alkalosis ↑ ↓ ↓

39
Copyright © Oakstone Publishing, 2021. All Rights Reserved.

Metabolic acidosis
Etiology Evaluation
Winter’s Formula
Expected PCO2 = 1.5 x HCO3- + 8 ± 2
• Increased acid • Arterial pH & pCO2
generation (e.g. lactic • Assess respiratory • If PCO2 > predicted: 2° respiratory acidosis
acidosis) compensation
• HCO3- loss (e.g. diarrhea (Winter’s formula) • If PCO2 < predicted: 2° respiratory alkalosis
or ileal conduit) • Anion gap
• Reduced renal acid • Correct for albumin
excretion (e.g. ESRD) • Narrow etiology based Anion Gap
on anion gap
• High gap: Excess AG = Na+ - (Cl- + HCO3-)
acid • If > 12 there is an elevated anion gap
• Normal gap: Loss of
base • Add 2.5 for every 1 g/dL reduction in albumin

Metabolic Acidosis
• The kidney’s role in regulating acid-base balance includes
• Reclamation of filtered HCO3 and
• Regeneration of HCO3 consumed by net acid production.
• Quantitatively, the most important urinary buffer is the NH3/NH4 + (Ammonia/Ammonium)
system. H+ + NH3 → NH4+
• The rate of NH3 production and excretion can be varied according to physiologic needs.
• Under normal circumstances, urine excretion of NH4 + accounts for more than half of the net
acid excreted per day.
• Acidosis and hypokalemia stimulate NH4+ excretion.
• Alkalosis and hyperkalemia diminish NH4+ excretion.

40
Copyright © Oakstone Publishing, 2021. All Rights Reserved.

Renal Causes
Non gap / Renal tubular acidosis

Hyperchloremic Carbonic anhydrase inhibitors

Metabolic GI Causes
Severe diarrhea
Acidosis Uretero-enterostomy or Obstructed ileal conduit
Drainage of pancreatic or biliary secretions
Small bowel fistula

Other Causes
Recovery from ketoacidosis
Addition of HCl, NH4Cl

• Useful for Non gap (or hyperchloremic)


acidosis that is not obvious
Urinary Anion • Only Na+, K+ and Cl- are commonly measured
in urine so the other charged species are the
Gap unmeasured anions (UA) and cations (UC).
• Urinary Anion Gap = ( UA - UC ) = [Na+]+ [K+] -
[Cl-]
• Urinary Anion Gap (UAG) provides a rough
index of urinary ammonium (NH4+) excretion.
• With normal kidney function and acidosis,
urine NH4Cl should increase driving the UAG
negative.
• A negative UAG = GI loss of bicarbonate
(diarrhea) “neGUTive”
• A positive UAG = impaired renal distal
acidification (RTA).

41
Copyright © Oakstone Publishing, 2021. All Rights Reserved.

• Anions associated with:


• endogenous organic acidosis (lactate, beta-
Anion Gap hydroxybutyrate, and acetoacetate)
• Exogenous poisons (glycolate, salicylate)
• Renal failure (sulfate, phosphate, and other
anions)
• Negative charges on plasma albumin are a large
component of the unmeasured anions that make
up the calculated AG.
• Hypoalbuminemia will lead to an underestimate
of the size of the AG and potentially to the failure
to recognize a clinically important high AG
acidosis.
• A “corrected anion gap” avoids this problem: For
every 1 g/dl decrease in serum albumin below
4.4 g/dl, 2.5 mEq/L is added to the calculated AG.

• An elevated Anion Gap always strongly


Anion Gap suggests a Metabolic Acidosis

• AG 20-30: high chance of metabolic


acidosis
• AG > 30: metabolic acidosis is certainly
present

42
Copyright © Oakstone Publishing, 2021. All Rights Reserved.

Delta/Delta
Increase in Anion Gap / Decrease in bicarbonate

If one molecule of metabolic acid (HA) is added to the ECF and dissociates, the one H+
released will be buffered by one molecule of HCO3- to produce CO2 and H2O.

The net effect will be an increase in unmeasured anions by the one acid anion A- (ie anion
gap increases by one) and a decrease in the bicarbonate by one.

Significant intracellular buffering with extracellular retention of the unmeasured acid


anion (A- ) will cause the value of the delta ratio to be greater than one in a high AG
metabolic acidosis.

Delta/Delta example
“Normal” AG 10 Normal HCO3 24

Case 1
Anion gap 20 HCO3 17
Delta AG/ Delta HCO3 = 20-10/24-17 = 1.42 -> uncomplicated high-AG acidosis

Case 2
Anion gap 20 HCO3 12
Delta AG/ Delta HCO3 = 20-10/24-12 = 0.08 -> less HCO3 than expected
High AG acidosis with additional nongap acidosis

Case 3
Anion gap 20 HCO3 20
Delta AG/ Delta HCO3 = 20-10/24-20 = 2.5 -> more HCO3 than expected
High AG acidosis with additional metabolic alkalosis or additional compensated respiratory acidosis

43
Copyright © Oakstone Publishing, 2021. All Rights Reserved.

Delta/Delta

Increase in Anion Gap / Decrease in bicarbonate

< 0.4
Hyperchloremic nongap acidosis
Less HCO3 present than
expected by increase in AG
0.4 - 0.8
Combined High AG and nongap acidosis
acidosis associated with renal failure Delta/Delta < 1

1 to 2
Usual for uncomplicated high-AG acidosis
Lactic acidosis, DKA

>2 More HCO3 present than


Pre-existing elevated HCO3 level so consider:
expected by increase in AG
concurrent metabolic alkalosis, or
pre-existing compensated respiratory acidosis

Delta/Delta

Delta Ratio Assessment Guidelines in patients with a metabolic acidosis

< 0.4 Hyperchloremic non gap acidosis


0.4 - 0.8 Combined high AG and non gap acidosis
1 to 2 Typical pattern in high AG metabolic acidosis
>2 Either a co-existing Metabolic Alkalosis or a co-existing
Chronic Respiratory Acidosis

44
Copyright © Oakstone Publishing, 2021. All Rights Reserved.

Osmolar gap

The osmolar gap is the difference between the measured osmolality and the
calculated osmolarity.

Measured Osmolality is measured in the lab via an osmometer


Calculated Osmolarity = ( 2 x [Na+] ) + glucose/18 + BUN/2.8 + ethanol/4.6

Osmolar gap

Screening test for detecting abnormal low molecular weight solutes

methanol (Osm gap + high AG)


ethylene glycol (Osm gap + high AG)
ethanol (Osm gap + normal AG)
acetone (Osm gap + normal AG)
isopropyl ethanol (Osm gap + normal AG)
propylene glycol (Osm gap + high AG, vehicle in Ativan IV)

osmolar gap > 10 mOsm/l is often stated to be abnormal


osmolar gap≥ 25 is suggestive of methanol or ethylene glycol ingestion

45
Copyright © Oakstone Publishing, 2021. All Rights Reserved.

Metabolic acidosis
Etiology Evaluation
Winter’s Formula
Expected PCO2 = 1.5 x HCO3- + 8 ± 2
• Increased acid • Arterial pH & pCO2
generation (e.g. lactic • Assess respiratory • If PCO2 > predicted: 2° respiratory acidosis
acidosis) compensation
• HCO3- loss (e.g. diarrhea (Winter’s formula) • If PCO2 < predicted: 2° respiratory alkalosis
or ileal conduit) • Anion gap
• Reduced renal acid • Correct for albumin
excretion (e.g. ESRD) • Narrow etiology based Anion Gap
on anion gap
• High gap: Excess AG = Na+ - (Cl- + HCO3-)
acid • If > 12 there is an elevated anion gap
• Normal gap: Loss of
base • Add 2.5 for every 1 g/dL reduction in albumin

Metabolic alkalosis
Respiratory compensation in metabolic
Etiology Evaluation alkalosis
Expected PCO2 = 0.7(HCO3- - 24) + 40 ± 2
• H+ loss (e.g. vomiting) • Assess for upper GI
• Excess HCO3- losses & diuretic use • If PCO2 > predicted: 2° respiratory acidosis
administration or • Urine Na+, Cl-, & pH if • If PCO2 < predicted: 2° respiratory alkalosis
production (e.g. cause unclear
massive transfusion – • Assess respiratory
citrate converted to compensation
HCO3-) (formula)
• “Contraction” alkalosis • Assess anion gap
(actually due to Cl- • If elevated there is a
loss, not volume loss) 2° metabolic acidosis

46
Copyright © Oakstone Publishing, 2021. All Rights Reserved.

Initiation of Metabolic Alkalosis

Gain of alkali
Exogenous source (IV NaHCO3 infusion, citrate in transfused blood pheresis or
renal replacement)
Endogenous source (metabolism of ketoanions to produce bicarbonate)

Loss of H+
Kidneys (diuretics)
Gut (vomiting, Gastric suction)

Maintenance of Metabolic Alkalosis


Chloride depletion
Loss of gastric acid and diuretic use account for 90% of clinical cases of
metabolic alkalosis
Loop / Thiazide Diuretics (Urinary chloride losses are greater than Urine
bicarbonate losses). Seen only in volume depleted (high aldosterone)
patients with low dietary chloride intake ('salt restricted' diet)

Potassium depletion
Bicarbonate reabsorption in both the proximal and distal tubules is
increased in the presence of potassium depletion.
Primary hyperaldosteronism
Cushing’s syndrome – high dose corticosteroids

47
Copyright © Oakstone Publishing, 2021. All Rights Reserved.

Urinary Chloride in Metabolic Alkalosis


If the cause is not apparent then Urine Cl can be helpful

Urine Cl- < 10 mmol/l


Volume depletion (increased proximal tubular reabsorption of HCO3)
Responsive to IV NaCl (replaces chloride and volume)
Cause: Vomiting (90% of cases), Prior thiazide diuretic use

Urine Cl- > 20 mmol/l


Volume expansion and hypokalemia
Not responsive to IV NaCl
Cause: Excess aldosterone, severe K+ deficiency, Current diuretic therapy

Metabolic Alkalosis
Compensation
Hypoventilation causes a compensatory rise in arterial pCO2
Magnitude of the response has generally been found to be quite variable.
Expected pCO2 = 0.7 [HCO3] + 20 mmHg ± 5

Correction
Treat underlying disorder
Repletion of chloride, potassium and volume will improve renal bicarbonate excretion

48
Copyright © Oakstone Publishing, 2021. All Rights Reserved.

Metabolic alkalosis
Respiratory compensation in metabolic
Etiology Evaluation alkalosis
Expected PCO2 = 0.7(HCO3- - 24) + 40 ± 2
• H+ loss (e.g. vomiting) • Assess for upper GI
• Excess HCO3- losses & diuretic use • If PCO2 > predicted: 2° respiratory acidosis
administration or • Urine Na+, Cl-, & pH if • If PCO2 < predicted: 2° respiratory alkalosis
production (e.g. cause unclear
massive transfusion – • Assess respiratory
citrate converted to compensation
HCO3-) (formula)
• “Contraction” alkalosis • Assess anion gap
(actually due to Cl- • If elevated there is a
loss, not volume loss) 2° metabolic acidosis

Respiratory
acidosis and
alkalosis
• Hypoventilation (build up of CO2) → acidosis
• Hyperventilation (excess loss of CO2) →
alkalosis
• Must discern between acute & chronic (chart)
• If HCO3- < predicted: 2° metabolic
acidosis
• If HCO3- > predicted: 2° metabolic
alkalosis
• Treat underlying cause
• Acidosis: Consider oversedation airway
obstruction, massive PE, & ARDS
• Alkalosis: Consider pain, anxiety,
sepsis, & sub-massive PE
• For chronic respiratory acidosis (e.g. OHS):
Monitor for posthypercapneic metabolic
alkalosis after correction

49
Copyright © Oakstone Publishing, 2021. All Rights Reserved.

Respiratory Acidosis

An adult at rest produces 200mls of CO2 per minute

Increased production of CO2 leads to a respiratory acidosis if ventilation remains


constant.

Increases in pCO2 very promptly results in a large increase in ventilation


maintaining the at pCO2 40 mmHg.

Respiratory Acidosis
Inadequate Alveolar Ventilation (most common)
Central Respiratory Depression & Other CNS Problems
Inadequate mechanical ventilation
Nerve or Muscle Disorders
Lung or Chest Wall Defects
Airway Disorders (obstruction)

Over-production of Carbon Dioxide


Hypercatabolism (Malignant Hyperthermia)

Increased Intake of Carbon Dioxide


Rebreathing of CO2-containing expired gas
Insufflation of CO2 into body cavity (laparoscopic surgery)

50
Copyright © Oakstone Publishing, 2021. All Rights Reserved.

Respiratory Acidosis
The compensatory response to an acute respiratory acidosis is limited to buffering rapidly inside red
blood cells via carbonic anhydrase and hemoglobin
CO2 + H2O <-> H2CO3 <-> H+ + HCO3-

With continuation of the acidosis, the kidneys respond by retaining bicarbonate.

The renal response begins by 6 - 12 hours and is maximal at 3 - 4 days.


Increased HCO3 production
Increased NH3 production to 'buffer' the H+ to increase urinary excretion of NH4Cl

The pCO2 quickly returns to normal with restoration of adequate alveolar ventilation

Respiratory Alkalosis
A respiratory alkalosis is a primary acid-base disorder in which arterial pCO2 falls to a level
lower than expected

A respiratory alkalosis is due to increased alveolar ventilation


The low arterial pCO2 will be sensed by the central and peripheral chemoreceptors and the
hyperventilation will be inhibited unless the patient’s ventilation is controlled

Consequences:
decreased cerebral blood flow
decrease in myocardial contractility
shift of the oxygen dissociation curve to the left

51
Copyright © Oakstone Publishing, 2021. All Rights Reserved.

Respiratory Alkalosis
Central Causes
Head Injury, Stroke
Anxiety, Pain, fear, Drugs (salicylate intoxication)
progesterone during pregnancy, cytokines during sepsis

Hypoxemia
peripheral chemoreceptors stimulate respiration

Pulmonary Causes (act via intrapulmonary receptors)


Pneumonia, Asthma, Pulmonary Embolism, Pulmonary edema

Iatrogenic
Excessive controlled ventilation

Respiratory Alkalosis
Acute
The buffering is by intracellular proteins
slight decrease in HCO3-
Renal compensation has insufficient time to respond
The lower limit of 'compensation' is 18mmol/l

Chronic
Renal loss of bicarbonate causes a further fall in plasma bicarbonate
This maximal response takes 2 to 3 days to reach.
The limit of compensation is a [HCO3-] of 12 to 15 mmol/l.

52
Copyright © Oakstone Publishing, 2021. All Rights Reserved.

Respiratory Alkalosis

A patient cannot have both a respiratory alkalosis and a respiratory acidosis

Hyperventilation is used to acutely decrease intracranial pressure as the onset is


rapid.

The effect on cerebral blood flow is time-limited as equilibration of bicarbonate


across the blood-brain barrier occurs over 4 to 6 hours and cerebral blood flow
and intracranial pressure return towards normal

Respiratory
acidosis and
alkalosis
• Hypoventilation (build up of CO2) → acidosis
• Hyperventilation (excess loss of CO2) →
alkalosis
• Must discern between acute & chronic (chart)
• If HCO3- < predicted: 2° metabolic
acidosis
• If HCO3- > predicted: 2° metabolic
alkalosis
• Treat underlying cause
• Acidosis: Consider oversedation airway
obstruction, massive PE, & ARDS
• Alkalosis: Consider pain, anxiety,
sepsis, & sub-massive PE
• For chronic respiratory acidosis (e.g. OHS):
Monitor for posthypercapneic metabolic
alkalosis after correction

53
Copyright © Oakstone Publishing, 2021. All Rights Reserved.

Acute Respiratory Acidosis


Expected HCO3= 24 +1[(Actual pCO2-40)/10]

Chronic Respiratory Acidosis


Expected HCO3 = 24 +4[(Actual pCO2-40)/10]

Acute Respiratory Alkalosis


Expected HCO3 = 24 -2[(40-Actual pCO2)/10]
Rules for Acid-
Base Disorders Chronic Respiratory Alkalosis
Expected HCO3 = 24 -5[(40-Actual pCO2)/10]

Metabolic Alkalosis
Expected pCO2 = 0.7 [HCO3] + 20

Metabolic Acidosis
Expected pCO2 = 1.5 [HCO3] + 8

Acid-Base Interpretation

2. pH: <7.40 Acidemia


1. Clinical stem or >7.40 Alkalemia 3. Pattern of HCO3 and
4. Is compensation
(vomiting/ nasogastric • pH=7.40 can occur with pCO2 (primary
appropriate?
suction / diarrhea) mixed acid base disorders disorder)

6. If AG then 7. Hints in other


5. Is Anion Gap (AG)
Delta/Delta and investigations (urine
present?
Osmolar Gap ketones)

54
Copyright © Oakstone Publishing, 2021. All Rights Reserved.

High anion gap


Metabolic acidosis

Hypokalemia and/or hypochloremia


Metabolic alkalosis

Hints Hyperchloremia
Normal anion gap acidosis

Elevated creatinine and urea


Uremia acidosis or prerenal renal failure

Hyperglycemia
DKA or hyperosmolar non-ketotic syndrome

1. Clinical stem
2. pH
3. Pattern of HCO3 and pCO2 (primary disorder)
4. Is compensation appropriate?
5. Is Anion Gap (AG) present?
6. If AG then Delta/Delta and Osmolar Gap
Summary 7. Hints in other investigations

Respiratory Acidosis and Alkalosis cannot occur together


More than one metabolic acid-base disorder can be
present at one time
Urine AG for Non gap acidosis [NeGUTive]
Urinary Chloride in Metabolic Alkalosis

55
Copyright © Oakstone Publishing, 2021. All Rights Reserved.

Question 1
A 57-year-old female with chronic kidney disease is admitted after an
elective right hemicolectomy. Her urine output is marginal on
postoperative day 1. On postoperative day 2 her potassium is 6.9. An
ECG is obtained showing peaked T waves and a widened QRS complex.
Which of the following is the most appropriate initial treatment?
A. Emergent hemodialysis
B. Insulin and glucose
C. Potassium binder
D. Calcium gluconate

Question 1 - Answer
A 57-year-old female with chronic kidney disease is admitted after an elective right hemicolectomy. Her urine output is marginal on
postoperative day 1. On postoperative day 2 her potassium is 6.9. An ECG is obtained showing peaked T waves and a widened QRS
complex. Which of the following is the most appropriate initial treatment?
A. Emergent hemodialysis
B. Insulin and glucose
C. Potassium binder
D. Calcium gluconate

This patient has potentially life-threatening hyperkalemia, as evidenced by potassium of 6.9 and associated ECG changes. Patients with
severe hyperkalemia (K+ > 6.5) or symptomatic hyperkalemia (ECG changes or muscle weakness) should receive rapidly acting therapies
to temporize as total body potassium is lowered.
Cardiac membrane stabilization is the highest treatment priority. Calcium gluconate antagonizes the effects of hyperkalemia on
cardiac membranes, preventing the development of lethal arrhythmias.
Shifting potassium intracellularly is the next priority. Insulin and glucose in addition to high-dose albuterol can drive potassium into
cells, providing immediate reduction in serum potassium levels. These temporizing measures last 4-6 hours, allowing time for other
measures to promote potassium excretion. Note that sodium bicarbonate has not been shown to be effective and is no longer
recommended.
Ultimately, the goal is to remove potassium from the body. This can be accomplished with potassium binders, loop diuretics, and/or
hemodialysis.

56
Copyright © Oakstone Publishing, 2021. All Rights Reserved.

Question 2
A 32-year-old male with arrives in the trauma bay after a high-speed motor vehicle collision. He
undergoes damage control laparotomy and is transferred to the ICU with tenuous hemodynamics and an
open abdomen. He is manually ventilated en route from the OR to the ICU. On arrival, the following
laboratory data are obtained:
ABG: 7.24 / 29 / 112 / 19
BMP: 131 │ 97 │ 34 /
97
4.8 │ 19 │ 1.7 \
Albumin: 3.1
Which of the following most accurately describes his acid-base disorder?
A. Primary respiratory acidosis with secondary anion gap metabolic acidosis
B. Primary non-anion gap metabolic acidosis with secondary respiratory acidosis
C. Primary anion gap metabolic acidosis with secondary respiratory alkalosis
D. Primary respiratory acidosis with secondary non-anion gap metabolic acidosis

Question 2 - Answer
A 32-year-old male with arrives in the trauma bay after a high-speed motor vehicle collision. He undergoes damage control laparotomy and is
transferred to the ICU with tenuous hemodynamics and an open abdomen. He is manually ventilated en route from the OR to the ICU. On arrival,
the following laboratory data are obtained:
ABG: 7.24 / 29 / 112 / 19
BMP:
131 │ 97 │ 34 /
97
Albumin: 3.1 4.8 │ 19 │ 1.7 \
Which of the following most accurately describes his acid-base disorder?
A. Primary respiratory acidosis with secondary anion gap metabolic acidosis
B. Primary non-anion gap metabolic acidosis with secondary respiratory acidosis
C. Primary anion gap metabolic acidosis with secondary respiratory alkalosis
D. Primary respiratory acidosis with secondary non-anion gap metabolic acidosis

A stepwise approach should be taken for any acid-base derangement.


First, the pH of 7.24 shows that there is a primary acidosis. The decreased bicarbonate shows that this is a metabolic acidosis.
For any metabolic acidosis, the anion gap should then be assessed. This is determined by subtracting the bicarbonate (19) and chloride (97) from
the sodium (131), which gives us a gap of 15, which is elevated (normal ≤ 12). There is thus a primary anion gap metabolic acidosis, presumably
reflecting lactic acidosis given apparent shock.
Next the adequacy of respiratory compensation should be assessed using Winter’s formula. The expected partial pressure of CO2 is 1.5 x
bicarbonate +8 ± 2. This gives us an expected value of 34.5 to 38.5. The observed partial pressure of CO2 is only 29, indicating a secondary
respiratory alkalosis. He may have been over-ventilated en route from the OR to the ICU.

57
Copyright © Oakstone Publishing, 2021. All Rights Reserved.

Transfusion Medicine and


Anticoagulant Management

Nathan T. Connell, MD, MPH, FACP


Assistant Professor of Medicine, Harvard Medical School
Associate Physician, Hematology Division, Brigham and Women’s Hospital
Clinical Chief of Hematology, Brigham and Women’s Faulkner Hospital

Disclosures
No relevant disclosures
Off-label drug use:
Tranexamic acid, aminocaproic acid, desmopressin
No investigational drug use

58
Copyright © Oakstone Publishing, 2021. All Rights Reserved.

Objectives

• Recognize the clinical signs and


laboratory findings of transfusion
reactions
• Identify management options for
acute transfusion reactions
• Identify management options for
patients who require transfusion
after a prior transfusion reaction

Punchline First!

• If there’s a transfusion reaction, stop the transfusion and notify


the blood bank/transfusion medicine service!

59
Copyright © Oakstone Publishing, 2021. All Rights Reserved.

Blood products

• Whole blood (500mL) collected


– In-line leukoreduction
– 40mL diverted to reduce risk of skin
contamination (donor testing)
• Centrifuged into component parts
– Red blood cells
– Plasma
– Buffy coat (x 4 donors) + one plasma unit
= pooled donor platelet unit

Storage

60
Copyright © Oakstone Publishing, 2021. All Rights Reserved.

Informed Consent

• When: As early as possible


• What:
– Description
– Risks/benefits
– Alternatives
– Patient questions

Risks

• Red cell sensitization: 1 in 13


• Febrile non-hemolytic transfusion reactions: 1 in 20
• Transfusion Associated Circulatory Overload (TACO): 1 in 100
• Minor allergic reactions (urticaria): 1 in 100
• Delayed hemolytic transfusion reaction: 1 in 7000
• Transfusion Related Acute Lung Injury (TRALI): 1 in 10,000
• Bacterial sepsis
– Platelets: Symptomatic 1 in 10,0000; Death 1 in 200,000
– Red Cells: Symptomatic 1 in 250,000; Death 1 in 500,000
• West Nile: < 1 in 1,000,000
• Hepatitis B: 1 in 7,500,000
• Hepatitis C: 1 in 13,000,000
• HIV: 1 in 21,000,000

61
Copyright © Oakstone Publishing, 2021. All Rights Reserved.

Blood Bank Tests

• Type/Screen
• Crossmatch
• Transfusion Order
• Transfusion Reaction Workup
• Eluate
• DAT
• Other specialized tests (RBC phenotype, platelet antibody testing,
etc.)

From Type/Screen to Crossmatch

62
Copyright © Oakstone Publishing, 2021. All Rights Reserved.

Transfusion Reactions

• All transfusion reactions must be reported to the transfusion


medicine service (Joint Commission, CAP, AABB requirements)
• Symptoms:
– Fever, Dyspnea, Urticaria, Hypotension, Hemolysis, Cytopenias after
transfusion, Viral/Parasite/Prion infections

Fever

63
Copyright © Oakstone Publishing, 2021. All Rights Reserved.

Fever

Bacterial Sepsis or Contamination

• Skin commensals from donor, unrecognized bacteremia in donor,


contamination of product from environment/handling
• Gram positive and gram negative

64
Copyright © Oakstone Publishing, 2021. All Rights Reserved.

Bacterial Sepsis or Contamination

• Clinical Presentation:
– Rigors, fever, tachycardia, hypotension, nausea/vomiting, dyspnea, DIC
– Symptoms may be delayed after platelets (small bacterial load)
• Management:
– Stop the transfusion and notify transfusion medicine!
– Return residual blood products
– Blood cultures
– Broad-spectrum antibiotics

Acute Hemolytic Transfusion Reactions

• Etiology: ABO incompatibility, other (29 systems: 346 known


antigens) incompatibility, rare cases of high-titer Anti-A or –B in
type O platelets
• ABO incompatibility: Clerical error or patient identification issue,
improperly labeled blood specimen
• Clinical presentation:
– Common: Fevers, chills, HEMOGLOBINURIA
– Less common:
• Pain, hypotension, nausea/vomiting,
• renal failure, DIC

65
Copyright © Oakstone Publishing, 2021. All Rights Reserved.

Connell and Sweeney Hematol Oncol Clin N Am (2012)

Acute Hemolytic Transfusion Reactions

• Management:
– Stop the transfusion and notify the transfusion medicine service
– Check patient identifiers
– Send specimens to recheck ABO group along with returning blood
product
– First post-transfusion urine specimen for urinalysis
– Supportive care:
• Maintain good urine output
• Check coags/fibrinogen for DIC and support if needed

66
Copyright © Oakstone Publishing, 2021. All Rights Reserved.

Febrile Non-hemolytic Transfusion Reactions

• Etiology: Soluble factors (cytokines) in plasma of transfused


component or recipient antibodies reacting to antigens on the
cells (usually WBCs)
• Clinical Presentation: Fever during or up to 4 hours post (may be
associated with chills, rigors, nausea/vomiting, hypotension)
• Management:
– Stop the transfusion and notify the blood bank
– Acetaminophen
– Cultures depending on clinical scenario

Febrile Non-hemolytic transfusion reactions

• Prevention:
– Pre-medication with acetaminophen/diphenhydramine has not been
shown to be effective in preventing FNHTR
– With recurrent FNHTR, many will use: acetaminophen, corticosteroids,
fresh components, washed red cells, plasma-depleted components, but
EFFICACY UNKNOWN
– Antihistamines are not effective

67
Copyright © Oakstone Publishing, 2021. All Rights Reserved.

Dyspnea

Transfusion-Related Acute Lung Injury (TRALI)

• Acute onset hypoxemia, bilateral


infiltrates on CXR, no circulatory
overload
• Definite: within 6 hours of
completion of transfusion and no
other risk factors for acute lung
injury
• Possible: within 6 hours of
completion of transfusion and one
or more risk factors for ALI

68
Copyright © Oakstone Publishing, 2021. All Rights Reserved.

TRALI
• Etiology:
– Antibody-mediated (passive transfer of HLA or granulocyte antibodies
from donor to product)
– Neutrophil priming (Biologic response modifiers)

TRALI

• Management:
– Supportive care
– Avoid diuretics and steroids (no data to support use)
– Report to transfusion medicine service
• Especially important to avoid TRALI in other recipients
• Prevention
– Plasma for transfusion from predominantly male donors
– Buffy coat platelet pools in male plasma
– Plateletpheresis from male donors or never pregnant female donors
– Donor deferral if prior TRALI from transfused unit

69
Copyright © Oakstone Publishing, 2021. All Rights Reserved.

Transfusion associated circulatory overload (TACO)

• Etiology: Impaired cardiac function and/or excessive rapid rate of


transfusion
• Clinical Presentation: Dyspnea, orthopnea, cyanosis, tachycardia,
increased JVP, hypertension
• Management:
– Stop the transfusion
– Administer oxygen/diuretics
– Chest x-ray
– Transfusion at reduced rate
– Notify transfusion medicine service

Transfusion associated circulatory overload (TACO)

• Prevention:
– Pre-transfusion assessment of
cardiac function/reserve
– Transfusion of one unit at a time
– Transfusion over longer time
period (4 hours)
– Pre-emptive diuretics

70
Copyright © Oakstone Publishing, 2021. All Rights Reserved.

Uricaria and Other Allergic Reactions/Anaphylaxis

Uricaria and Other Allergic Reactions/Anaphylaxis

71
Copyright © Oakstone Publishing, 2021. All Rights Reserved.

Uricaria and Other Allergic Reactions/Anaphylaxis

• Etiology: Most unexplained


– Anti-IgA in IgA deficiency patients
– Antibodies to polymorphic serum proteins (IgG, albumin, haptoglobin…)
– Transfusing an allergen to sensitized patient (penicillin, ASA, food…)
consumed by the donor
– Passive transfer of IgE
• IgA deficiency (1 in 500 blood donors with 1:500 anti-IgA)
• Haptoglobin deficiency (Asian: 1 in 1000)
• Anaphylaxis accounts for 5% of transfusion associated fatalities

Uricaria and Other Allergic Reactions/Anaphylaxis

• Management:
– Stop the transfusion and notify transfusion medicine service
– Urticarial reactions: Benadryl 25-50mg
– Anaphylaxis: Epinephrine, corticosteroids, Benadryl, vasopressors,
supportive fluids
• Prevention of recurrent anaphylaxis
– Premedication with corticosteroids and Benadryl
– IgA deficiency products

72
Copyright © Oakstone Publishing, 2021. All Rights Reserved.

Hemolysis After Transfusion

• Hemolysis Not Related to RBC antibodies


– Hypotonic IV solutions
– Mechanical: Cell saver, blood warmer, small bore needle
– Overheating: (Don’t put the RBCs on top of the radiator!)
– Mostly benign
• Delayed Hemolytic Transfusion Reactions
– Alloantibodies in recipient below detection threshold on initial screen
– 3 days to 2 weeks after transfusion
– Commonly E, Jka, c, Fya, K
– Also with transmitted malaria and babesiosis

Hemolysis after transfusion

• Mostly benign, but in severe hemolysis can develop anemia and


renal failure
• Notify the transfusion medicine service
• Transfuse compatible blood (may require phenotyping)
• Avoid RBC transfusions and make sure the patient carries a wallet
card regarding their alloantibody

73
Copyright © Oakstone Publishing, 2021. All Rights Reserved.

Massive Transfusion

• More than 10 units of RBCs or >1 blood volume in 24 hours


• Independent risk factor for multi-organ failure
• Complications
– Dilutional Coagulopathy
– Hypothermia
– Hypocalcemia/Hypomagnesemia/Citrate Toxicity
– Metabolic acidosis
– Hyperkalemia

Summary

• Recognize the signs/symptoms of transfusion reactions


• Stop the transfusion, provide supportive care, and notify the
transfusion medicine service
• Only transfuse with clear indications

74
Copyright © Oakstone Publishing, 2021. All Rights Reserved.

Anticoagulation

Anticoagulants
Thrombosis Risk

75
Copyright © Oakstone Publishing, 2021. All Rights Reserved.

Anticoagulants
Coagulation Testing

Anticoagulants
Coagulation Testing
• Screening coagulation studies are poor predictors of perioperative
bleeding risk
• Normal in many patients at risk for bleeding
• Abnormal in many patients who will not bleed (e.g. lupus anticoagulant)
• Medical conditions, history of bleeding with prior procedures, or
family history of bleeding should guide decisions for additional
workup
• Liver disease, medications
• Personal or family history of von Willebrand disease or hemophilia
• Postpartum hemorrhage, bleeding with wisdom tooth extraction

76
Copyright © Oakstone Publishing, 2021. All Rights Reserved.

Anticoagulants
Anticoagulants
• Prevent thrombosis
• Directly
• Indirectly
• Increase bleeding risk
• Trauma
• Underlying anatomic defect
• Spontaneous
• Prophylactic versus therapeutic

Common Anticoagulants
• Unfractionated heparin • Vitamin K Antagonists
• Low-molecular weight heparin • Warfarin (COUMADIN®,
JANTOVEN®)
• Dalteparin (FRAGMIN®)
• Enoxaparin (LOVENOX®) • Direct factor Xa Inhibitors
• Rivaroxaban (XARELTO®)
• Synthetic pentasaccharide
• Apixaban (ELIQUIS®)
• Fondaparinux (ARIXTRA®)
• Edoxaban (SAVAYSA®)
• Direct thrombin (IIa) Inhibitors
• Argatroban
• Bivalirudin (ANGIOMAX®)
• Dabigatran (PRADAXA®)

77
Copyright © Oakstone Publishing, 2021. All Rights Reserved.

Warfarin (COUMADIN®, JANTOVEN®)


• Not an anticoagulant on its own
• Start same day
• Initiation: Usually 5mg daily
• Adjustment: 3-4 days to see effect

Heparin

78
Copyright © Oakstone Publishing, 2021. All Rights Reserved.

LMWH Dosing
Agent Route Prophylaxis Therapeutic
Dalteparin SC 5000 units once daily Dose in units varies (Once daily)

Enoxaparin SC 40mg daily 1 mg/kg twice daily


30mg twice daily 1.5 mg/kg daily (Avoid)

Fondaparinux SC 2.5mg daily (>50kg) <50kg: 5mg daily


50 – 100kg: 7.5mg daily
>100kg: 10mg daily

DOAC Dosing: Factor II Inhibitor


Agent Route Prophylaxis Therapeutic
Dabigatran PO SPAF VTE:
Treat with parental agent days 5-10, then
CrCl > 30: 150mg BID CrCl > 30: 150mg BID
CrCl 15-30: 75mg BID CrCl < 30 or dialysis: AVOID
CrCl<15/dialysis: AVOID

79
Copyright © Oakstone Publishing, 2021. All Rights Reserved.

DOAC Dosing: Factor Xa Inhibitors


Agent Route Prophylaxis Therapeutic
Rivaroxaban PO Knee: 10mg daily X 10d 15mg BID x 21 days, then 20mg daily
Hip: 10mg daily X 35d Extended (> 6 months): 10mg daily
SPAF: 20mg daily

Apixaban PO Knee: 2.5mg BID X 12d 10mg BID x 7 days, then 5mg BID
Hip: 2.5mg BID X 35d Extended (>6 months): 2.5mg BID
SPAF: 5mg BID

Edoxaban PO SPAF: 60mg daily Treat with parental agent days 5-10, then
>60kg: 60mg daily
≤60kg: 30mg daily

Increasing use in malignancy associated VTE (HOKUSAI VTE Cancer; SELECT-D; ADAM VTE; CARAVAGGIO)

DOACs: Cautions
• ISTH Guidance: Avoid if weight >120kg or BMI >40
‒ Emerging data that DOACs in these groups are likely reasonable
• Mechanical valves: CONTRAINDICATED
• Antiphospholipid syndrome: “CONTRAINDICATED”
‒ Triple positive: AVOID (TRAPS Trial: Rivaroxaban vs. Warfarin)
‒ Single/double positive: Unclear risk/benefit balance
• Bariatric surgery

80
Copyright © Oakstone Publishing, 2021. All Rights Reserved.

DOACs and Bariatric Surgery

Am J Med. 2017 May;130(5):517-524

Anticoagulants
Perioperative Anticoagulation
• Warfarin may or may not require bridging, but not necessary in most
cases
• Exceptions: antiphospholipid syndrome, high risk cardiac valves
• DOACs have a short half life so bridging is unnecessary
• PAUSE-AF trial
• Postoperatively, once adequate hemostasis has been achieved
prophylactic or therapeutic anticoagulation may be restarted,
typically within 12-24 hours

81
Copyright © Oakstone Publishing, 2021. All Rights Reserved.

Anticoagulants
Perioperative Anticoagulation

JAMA Intern Med. 2019;179(11):1469-1478.

Bleeding: Approach to Lab Evaluation


• Prothrombin time (PT  INR)
• Activated partial thromboplastin time (aPTT)
• Fibrinogen
• Serial blood counts (CBC)
• Specialized Assays (if they are available and only in certain
clinical contexts)

82
Copyright © Oakstone Publishing, 2021. All Rights Reserved.

Bleeding: Antifibrinolytic Agents


• Tranexamic acid (TXA) and aminocaproic acid (ACA): off-label
• Help to maintain hemostatic clot once formed
• Particularly useful for mucosal bleeding
• TXA: variety of dosing schemes
‒ 10-20 mg/kg IV followed by 10mg/kg IV every 6-8 hours until bleeding
stops
‒ 10-15 mg/kg IV followed by 1 mg/kg/hour for 6-10 hours
‒ 1 gram PO followed by 1 gram every 8-12 hours
‒ Dependent on renal function
• ACA: 2 grams IV every 6 hours (escalating dose to 1 g/h IV)

Bleeding: Uremia
• Leads to platelet dysfunction
• Desmopressin (DDAVP) increases plasma levels of von
Willebrand factor, factor VIII, and t-PA contributing to shortened
aPTT
• Dose depends on product and location:
‒ Canada: 0.3 mcg/kg IV over 20-30 minutes (maximum dose 20mcg)
‒ U.S (off-label): 0.4 mcg/kg over 10 minutes (Watson and Keogh 1984)
• Risks: hyponatremia, flushing, fluid retention

83
Copyright © Oakstone Publishing, 2021. All Rights Reserved.

Reversal Strategies
Coagulation Factor Replacement

• Prothombin Concentrate Complex (II, VII*, IX, X, Proteins C/S)


• FEIBA (IIa, VIIa, IXa, Xa), recombinant VIIa, FFP

Specific Antidotes

• Humanized monoclonal antibody fragment against dabigatran


• Inactive factor Xa derivatives

Adjunctive

• Dialysis
• Desmopressin
• Antifibrinolytic Agents

J Am Coll Cardiol. 2020 Aug, 76 (5) 594–622

84
Copyright © Oakstone Publishing, 2021. All Rights Reserved.

• Critical Site Bleeds


• Intracranial hemorrhage
• Other CNS bleeding
• Pericardial tamponade
• Airway (including posterior epistaxis)
• Hemothorax, intra-abdominal bleeding, retroperitoneal hematoma
• Extremity bleeds
J Am Coll Cardiol. 2020 Aug, 76 (5) 594–622

J Am Coll Cardiol. 2020 Aug, 76 (5) 594–622

85
Copyright © Oakstone Publishing, 2021. All Rights Reserved.

When specialized assays are available

DRUG CLINICAL OBJECTIVE


Exclude Clinically Relevant Drug Level Measure Therapeutic Drug Levels
Test Interpretation Suggested Test
Dabigatran Dilute Thrombin Time Normal probably excludes Dilute Thrombin Time
Ecarin Clotting Time clinically relevant levels Ecarin Clotting Time
Ecarin Chromogenic Ecarin Chromogenic Assay
Assay
Apixaban, Anti-Xa Absent chromogenic anti-Xa Anti-Xa (with calibrated reagents)
edoxaban, assay activity probably excludes
and clinically relevant levels
rivaroxaban

When specialized assays are NOT available

J Am Coll Cardiol. 2020 Aug, 76 (5) 594–622

86
Copyright © Oakstone Publishing, 2021. All Rights Reserved.

J Am Coll Cardiol. 2020 Aug, 76 (5) 594–622

J Am Coll Cardiol. 2020 Aug, 76 (5) 594–622

87
Copyright © Oakstone Publishing, 2021. All Rights Reserved.

Summary: Anticoagulation
• Many options for anticoagulation!
• DOACs are first line therapy in most indications
‒ No bridging in perioperative setting
• Need to be careful with renal function and obesity/absorption
• Reversal should be reserved for life threatening bleeding
• Use an algorithmic approach to the assessment of a bleeding patient
‒ Standard labs (PT/INR, PTT, fibrinogen, CBC) +/- specialized assays
‒ While anticoagulant-specific reversal agents are becoming more available,
don’t underestimate the value of adjunctive measures
• Transfusion
• Antifibrinolytics
• Local pressure/surgical intervention

Question 1
A 58-year-old man presents to the emergency room with diarrhea with bright
red blood. He is known to have diverticulosis based on screening
colonoscopies and was recently started on apixaban 5 mg twice daily for new
onset atrial fibrillation. The last dose of apixaban was 6 hours ago. He is
hemodynamically stable, with hemoglobin that is just 1 gm/dL lower than
baseline. Treatment should involve:

A. Transfusions with 4 bags of FFP


B. Idarucizumab 5 grams IV bolus
C. IV vitamin K
D. Aggressive supportive care with IV fluids and red cell transfusions if
necessary
E. Andexanet alfa according to package insert

88
Copyright © Oakstone Publishing, 2021. All Rights Reserved.

Question 1
A 58-year-old man presents to the emergency room with diarrhea with bright
red blood. He is known to have diverticulosis based on screening
colonoscopies and was recently started on apixaban 5 mg twice daily for new
onset atrial fibrillation. The last dose of apixaban was 6 hours ago. He is
hemodynamically stable, with hemoglobin that is just 1 gm/dL lower than
baseline. Treatment should involve:

A. Transfusions with 4 bags of FFP


B. Idarucizumab 5 grams IV bolus
C. IV vitamin K
D. Aggressive supportive care with IV fluids and red cell transfusions if
necessary
E. Andexanet alfa according to package insert

Question 2
A 67-year-old man is seen by his PCP for advice about duration of
anticoagulation. Two months ago, he had had urgent cholecystectomy with
the development of a left calf vein DVT. The surgical team sent a
hypercoagulable work up which revealed the patient is heterozygous for a
Factor V Leiden mutation. What is the appropriate approach to
management?

A. Treatment for 1 additional month


B. Treatment for 3 additional months
C. Lifelong anticoagulation
D. Stop the anticoagulant at this time

89
Copyright © Oakstone Publishing, 2021. All Rights Reserved.

Question 2
A 67-year-old man is seen by his PCP for advice about duration of
anticoagulation. Two months ago, he had had urgent cholecystectomy with
the development of a left calf vein DVT. The surgical team sent a
hypercoagulable work up which revealed the patient is heterozygous for a
Factor V Leiden mutation. What is the appropriate approach to
management?

A. Treatment for 1 additional month


B. Treatment for 3 additional months
C. Lifelong anticoagulation
D. Stop the anticoagulant at this time

References
Agnelli G, et al. Apixaban for extended treatment of venous
thromboembolism. N Engl J Med. 2013 Feb 21;368(8):699-708.
Agnelli G, et al. Apixaban for the Treatment of Venous Thromboembolism
Associated with Cancer. N Engl J Med. 2020 Apr 23;382(17):1599-1607.
doi: 10.1056/NEJMoa1915103.
Connors JM. Thrombophilia Testing and Venous Thrombosis. N Engl J Med.
2017 Sep 21;377(12):1177-1187. doi: 10.1056/NEJMra1700365.
Martin KA, et al. Oral Anticoagulant Use After Bariatric Surgery: A Literature
Review and Clinical Guidance. Am J Med. 2017 May;130(5):517-524.
doi: 10.1016/j.amjmed.2016.12.033.
Tomaselli GF, et al. 2020 ACC Expert Consensus Decision Pathway on
Management of Bleeding in Patients on Oral Anticoagulants: A Report of
the American College of Cardiology Solution Set Oversight Committee. J
Am Coll Cardiol. 2020 Aug, 76 (5) 594–622.

90
Copyright © Oakstone Publishing, 2021. All Rights Reserved.

CONCEPTS IN ANESTHESIA

David A. Shaff, MD
Chief of Anesthesiology
Brigham and Women’s Faulkner Hospital

DISCLOSURE

I have no disclosures to report

91
Copyright © Oakstone Publishing, 2021. All Rights Reserved.

INTRODUCTION

What is the surgeon’s definition of an


endotracheal tube?

A 6-inch piece of plastic with someone who


is half asleep on either end.

INTRODUCTION

• Sometimes our approaches are complementary, occasionally adversarial

• Important for all disciplines to maintain an open mind and understand the challenges
each other faces

• Despite the occasional friction between surgery and anesthesia, it’s critical to
remember that we both have the patient’s best interest in mind

• The patient will have the best outcome when Anesthesia understands surgical
concerns, and Surgery understands anesthetic considerations

92
Copyright © Oakstone Publishing, 2021. All Rights Reserved.

BRIEF HISTORY OF ANESTHESIA

• Anesthesia has been around for a long time


• Greek physician Dioscorides wrote about
anesthetic effects of certain medicinal plants
between 40-90 AD
• Madragora/Henbane are highly biologically active
plants than contain alkaloids (atropine,
scopolamine, cocaine)

BRIEF HISTORY OF ANESTHESIA

• Theodoric of Lucca, Italian physician and


bishop, used sponges soaked with opium and
mandragora for surgical pain relief in 1298
• Fast forward 500 years, Nitrous oxide (N20)
and di-ethyl ether used recreationally in
“frolics” during Colonial America

93
Copyright © Oakstone Publishing, 2021. All Rights Reserved.

BRIEF HISTORY OF ANESTHESIA

• Finally, on October 16, 1846 Dr. Morton


anesthetized Edward Abbott with diethyl ether at
MGH
• To the amazement of the crowd in the Ether
Dome, the jaw tumor was removed without any
response from the patient causing Dr. Warren to
exclaim “Gentlemen, this is no humbug”.
• Debate whether the first demonstration of ether
as an anesthetic took place 4 years earlier by
Crawford Long in 1842…but didn’t publish the
procedure until1849
• We’ve come a long way since the Ether Dome…

BRIEF HISTORY OF ANESTHESIA

• As late as the 1970s, one in 10,000 people would die just from the anesthesia alone
• Contemporary rates are closer to 1:300,000
• Improved techniques
• Improved monitoring
• Improved medications

94
Copyright © Oakstone Publishing, 2021. All Rights Reserved.

TYPE OF ANESTHESIA

ANESTHESIA AS A CONTINUUM

Minimal Moderate Deep General


Sedation Sedation/ Sedation/ Anesthesia
Anxiolysis Analgesia Analgesia
(“Conscious Sedation”)
Responsiveness Normal Purposeful** Purposeful** Unarousable
response response to response even with
to verbal verbal or following painful
stimulation tactile stimulation repeated or stimulus
painful
stimulation
Airway Unaffected No intervention Intervention Intervention
required may be often required
required
Spontaneous Unaffected Adequate May be Frequently
Ventilation inadequate inadequate

Cardiovascular Unaffected Usually Usually May be


Function maintained maintained impaired

95
Copyright © Oakstone Publishing, 2021. All Rights Reserved.

GENERAL ANESTHESIA

• A drug-induced loss of consciousness during which patients are not arousable, even
by painful stimulation.
• General anesthesia is not defined by the presence of an airway

Continuum of Depth of Sedation: Definition of General Anesthesia and Levels of Sedation/Analgesia, 2019

GENERAL ANESTHESIA

• General anesthesia is a three-legged stool:


1. Amnesia
2. Immobility
3. Antinociception
• The classic approach to a general anesthetics is the so called “balanced technique”
• Selecting specific medications to produce general anesthesia

96
Copyright © Oakstone Publishing, 2021. All Rights Reserved.

GENERAL ANESTHESIA

• Can be accomplished with no airway, an LMA, or an ETT


• Decision for LMA vs. ETT depends on:
• Surgical position and conditions
• Patient comorbidities
• Duration of case
• Disposition of the patient post-operatively
• NPO status

LARYNGEAL MASK AIRWAY (LMA)

• Popular airway for certain cases


• Essentially an oral airway that connects to the
anesthesia machine
• Can be used for positive pressure ventilation
with limited positive pressure requirements
• Doesn’t prevent aspiration

97
Copyright © Oakstone Publishing, 2021. All Rights Reserved.

MONITORED ANESTHESIA CARE (MAC)

• Monitored Anesthesia Care (“MAC”) does not define the depth of sedation
• MAC is “a specific anesthesia service performed by a qualified anesthesia provider,
for a diagnostic or therapeutic procedure”
• Incorporates pre-procedure and post-procedural responsibilities

Continuum of Depth of Sedation: Definition of General Anesthesia and Levels of Sedation/Analgesia, 2019

MONITORED ANESTHESIA CARE (MAC)

• Monitored anesthesia care includes the spectrum of sedation, awareness, analgesia


and anxiolysis
• If the patient loses consciousness and the ability to respond to noxious stimulus, it is
a general anesthetic by definition
• Irrespective of whether airway instrumentation is required

98
Copyright © Oakstone Publishing, 2021. All Rights Reserved.

MONITORED ANESTHESIA CARE (MAC)

• A good MAC starts with good analgesia


• Local infiltration
• Formal nerve block

• Without analgesia, a MAC becomes a GANA (General Anesthesia No Airway)

REGIONAL

• Formal peripheral nerve block


• Local anesthetic is injected around nerves or in a
fascial plane containing somatic nerves
• Effective for both surgical anesthesia and post-
operative analgesia
• Risk
• Intraneural or intravascular injection
• Mechanical injury
• Local anesthetic toxcitiy (LAST)

99
Copyright © Oakstone Publishing, 2021. All Rights Reserved.

NEURAXIAL

• Epidural
• The epidural space is a “potential space” filled
with unnamed blood vessels and adipose tissue
• Catheter or single shot in the epidural space
• Creates a “band” of anesthesia
• Based on injection location, medication, and
volume
• Can be placed cervical to caudal
• Autonomic nerves most sensitive

NEURAXIAL

• Spinal
• Intrathecal space
• Single shot or catheter
• “Transection” at the highest level
• Highest level predicated on medication, baricity,
dose, patient position, and concentration
• Can only be safely placed in the lumbar region

100
Copyright © Oakstone Publishing, 2021. All Rights Reserved.

TYPE OF ANESTHESIA

• Sometime multiple types of anesthesia are necessary to accomplish the task


• Choice of primary anesthetic should be a discussion between the anesthesiologist
and the patient
• The surgeon influences the decision by explaining the surgical needs and expectations to
the anesthesia team
• Regardless of the intended type of anesthesia, a general anesthesia is always a
consideration

PHARMACOLOGY

101
Copyright © Oakstone Publishing, 2021. All Rights Reserved.

MECHANISMS OF ANESTHESIA

• No one unifying receptor or process could explain how anesthesia is produced by


such a diverse group of compounds
• May involve suppression of excitatory synapses OR stimulation of depressive
synapses
• May be related to disruption of phospholipid bilayer
• Bottom line: WE STILL DON’T KNOW HOW THEY WORK

INDUCTION AGENTS

Propofol
• Most common, hemodynamic depression, painful
Ketamine
• Dissociative, sympathomimetic, certain contraindications
Etomidate
• Hemodynamically stable, can cause adrenal suppression, painful, myoclonus, PONV
Volatile Breathe-Down
• Primarily used for non-reassuring airways, rarely used

102
Copyright © Oakstone Publishing, 2021. All Rights Reserved.

MAINTENANCE AGENTS

• Intravenous or inhaled
• All must be continuously delivered to maintain anesthetic effects

VOLATILE ANESTHETICS

• Sevoflurane, Isoflurane, Desflurane


• Despite their chemical similarities, they have different potencies/physical properties

103
Copyright © Oakstone Publishing, 2021. All Rights Reserved.

INTRAVENOUS ANESTHETICS

• Primarily propofol
• Total Intravenous Anesthetics (TIVA) preferentially used for:
• Neuromonitoring
• Patients with severe history of PONV

NITROUS OXIDE (N 2 0)

• Inhaled, but not a volatile anesthetic


• Not potent
• Hemodynamically stable
• Can be associated with PONV
• Contraindicated with air bubble expansion

104
Copyright © Oakstone Publishing, 2021. All Rights Reserved.

NEUROMUSCULAR BLOCKERS
(RELAXATION)

• Used to facilitate:
• Endotracheal intubation
• Decrease muscle tone in the patient for surgical considerations
• 2 basic types:
• Depolarizing (ultra short)
• Non-Depolarizing
• Often need to be “reversed”

OPIOIDS

• Alone have no amnestic properties


• Used to decrease autonomic response to noxious stimulus
• Wide range of ultra-short to long-acting preparations
• Most problematic class of medications:
• Often the cause of PONV
• Addictive
• Hallucinations
• Constipation/gut motility
• Respiratory depression

105
Copyright © Oakstone Publishing, 2021. All Rights Reserved.

OVERVIEW OF PERIOPERATIVE PAIN


MANAGEMENT

MULTIMODAL APPROACHES

• Current trend to approach perioperative pain management with a multimodal


approach
• Facilitate earlier discharge
• Limit opioid induced side effects
• Stewardship for addiction (up to 17% of patients are still using narcotics one year after
surgery)

Anesthesiology 2020; 132:1304–6

106
Copyright © Oakstone Publishing, 2021. All Rights Reserved.

NON-OPIOID PREMEDICATIONS

• Acetaminophen (oral or intravenous)


• NSAIDS (non-selective or COX-II)
• Gabapentin
• Became heavily incorporated in ERAS pathways before adequate data supported its role
• Gabapentinoids were associated with statistically lower but clinically unimportant decrease in
postoperative opioid use
• Less PONV, but more adverse effects, including dizziness and visual disturbances
• Increased odds of postoperative pulmonary complications (respiratory failure, pneumonia,
reintubation, pulmonary edema, or noninvasive ventilation) and ICU admission

Anesthesiology 2020; 133:251–4

REGIONAL TECHNIQUES FOR


POSTOPERATIVE PAIN MANAGEMENT

• “Nothing beats numb”


• Depending on surgical location, there are many options for postoperative pain
control
• These “blocks” last somewhere between 8-24 hours unless a catheter is placed
• Some can be done while the patient is asleep while others are done while the
patient can offer feedback

107
Copyright © Oakstone Publishing, 2021. All Rights Reserved.

TRUNCAL BLOCKS

Paravertebral • Generally last 12-24 hours


Erector Spinae Plane • Injection of local anesthetic into specific fascial
Serratus Anterior planes using ultrasound guidance

Intercostal • In addition to bleeding/infection, risk of PTX

PECS I/II

ABDOMINAL BLOCKS

Lumbar Plexus • Some can be done on the field by surgery


Quadratus Laborum • Can involve femoral nerve (ipsilateral quadriceps
Fascia Iliaca weakness)

TAP • Local needs to be injected into a specific fascial


plane
Rectus Sheath

108
Copyright © Oakstone Publishing, 2021. All Rights Reserved.

LOCAL ANESTHETICS

• Frequently used by both surgery and anesthesia


• Amide vs. Esther type
• Important to stay within the limits to avoid local anesthetic systemic toxicity (LAST)
• Important to check with Anesthesia to make sure local hasn’t already been used

LOCAL ANESTHETICS

Drug Onset Maximum Dose Duration (with


(with Epinephrine) epinephrine)

Lidocaine 10-20 min 4.5 mg/kg (7 mg/kg) 120 min (240 min)
Mepivicaine 10-20 min 5 mg/kg (7 mg/kg) 180 min (300 min)
Bupivicaine 15-30 min 2.5 mg/kg 360 min (8 hrs)
Ropivicaine 20-30 min 3 mg/kg 300 min (6 hrs)
Procaine rapid 8 mg/kg (10 mg/kg) 45 min (90 min)
Chloroprocaine rapid 10 mg/kg(15 mg/kg) 30 min (90 min)

109
Copyright © Oakstone Publishing, 2021. All Rights Reserved.

EXPAREL

• Nine trials were included in a meta-analysis


examining the difference in 24- to 72-h rest
pain severity scores for liposomal and
nonliposomal bupivacaine
• The area under the curve pain scores for the
24- to 72-h period were statistically but
probably not clinically significant
• Secondary outcome analysis likewise failed to
uncover benefits for liposomal bupivacaine
regarding analgesic consumption, length of
stay, and functional recovery

Anesthesiology 2021; 134:147–64

EXPAREL

• When infiltrated surgically and compared with


unencapsulated bupivacaine or ropivacaine, only
11% of trials (4 of 36) reported a clinically
relevant and statistically significant improvement
in the primary outcome favoring liposomal
bupivacaine.
• 92% of trials (11 of 12) suggested a peripheral
nerve block with unencapsulated bupivacaine
provides superior analgesia to infiltrated
liposomal bupivacaine.

Anesthesiology 2021; 134:283–344

110
Copyright © Oakstone Publishing, 2021. All Rights Reserved.

CONVERSATIONS WITH
SURGEONS

ANTICOAGULATION

• Myriad classes of anticoagulant medications for at least as many indications


• When deciding to hold perioperative anticoagulation, must consider:
• Risks/benefits to the patient
• How long the medication needs to be held
• When it can be restarted

111
Copyright © Oakstone Publishing, 2021. All Rights Reserved.

POSITIONING

• Positioning in the operating room is a shared responsibility


• The patient is unable to indicate discomfort during the case
• The OR team needs to position by proxy
• Principles of positioning:
• Nothing constricting
• Avoid brachial plexus stretch
• Extra care of the eyes and ulnar nerve
• Avoid leaning on the arms

SPECIALIST CONSULTATIONS

• Preoperative specialist input can be important


• Anticoagulation management
• Cardiac device management
• Always ask a specific question – don’t ask for “clearance”

112
Copyright © Oakstone Publishing, 2021. All Rights Reserved.

CASE CANCELLATIONS

• Case cancellation does occur


• Well functioning OR should have a cancellation < 5%
• High cancellation rate might indicate poor preoperative workup or low threshold to
cancel cases on DOS
• Low cancellation rate might indicate stringent preoperative workup or high
threshold to cancel cases on DOS

Anesthesiology 2006; 105:237-240


Current Opinion in Anaesthesiology 2010, 23:167-172

NPO GUIDELINES

• According to the most recent ASA practice guidelines for preoperative fasting:
• Allowing clear liquids between 2 to 4 hours prior to surgery resulted in a lower gastric
volume than >4 hours
• No specific volume limits were recommended
• Six hours recommended for a light meal
• Eight hours recommended for a meal containing fat

Anesthesiology 2011; 114: 495–511

113
Copyright © Oakstone Publishing, 2021. All Rights Reserved.

THE PATIENT IS WAKING UP!!!!!

• Movement during surgery is a spinal cord reflex – not purposeful response to


noxious stimulus
• The spinal cord is much more resistant to anesthetics than the cerebrum
• In general, it takes 3x more anesthetic to block movement than it does to prevent
recall
• Stimulation is not consistent during surgery

THE PATIENT IS WAKING UP!!!!!

Potential for movement


Stimulation

Amount of Anesthesia

Potential for hypotension

Time

114
Copyright © Oakstone Publishing, 2021. All Rights Reserved.

SAFETY

• Teamwork is now recognized


as important for safe, high-
quality perioperative care.
• A well-functioning relationship
is conducive to safe, effective
care.
• A dysfunctional relationship
can promote unsafe conditions
and contribute to an adverse
outcome
Anesthesiology 2018; 129:402-5

Surgeons’ View of Anesthesia


• Failure to communicate important changes in vital signs to the entire team
• Failure to keep the team informed of the need for vasopressor support
• Unwillingness to change anesthetic approach for the surgeon’s need to optimize surgical
technical considerations.

Anesthesiology 2018; 129:402-5

115
Copyright © Oakstone Publishing, 2021. All Rights Reserved.

Surgeons’ View of Anesthesia


• More concerned with finishing their day on time than serving their patients’ needs
• Unreasonable eagerness to cancel a procedure based on unjustified concerns
• Unappreciative of the need to maintain schedule
• Unreasonably long turnover times
• Distraction and inattention during surgery

Anesthesiology 2018; 129:402-5

Anesthesia’s View of Surgeons


• Failure to be knowledgeable about “medical” or “anesthesia-related” (as opposed to
surgical) issues
• Failure to perceive or acknowledge the extent of blood loss
• Underestimation of surgical time
• Failure to be forthcoming to patients and families about the likelihood of success and
magnitude of difficulty in recovery after surgery
• Failure to adequately consider patient health conditions and patient desires
• Discouraging speaking up by others about safety concerns.
Anesthesiology 2018; 129:402-5

116
Copyright © Oakstone Publishing, 2021. All Rights Reserved.

CONCLUSION

• Safety is paramount
• We are all doing our best for the patient
• It is important to understand the needs of all parties in the
operating room

117
Copyright © Oakstone Publishing, 2021. All Rights Reserved.

Comprehensive Review of General Surgery

Dr. James Vogel

Perioperative Care
Medical Director, Center for Preoperative
Evaluation, Brigham and Women’s Faulkner
Hospital

Instructor in Anesthesia, Harvard Medical School

Disclosures

❖ None

118
Copyright © Oakstone Publishing, 2021. All Rights Reserved.

Preoperative Risk Assessment

119
Copyright © Oakstone Publishing, 2021. All Rights Reserved.

Incidence
❖ Adverse cardiac outcome related to baseline risk
❖ A 1995 review found perioperative myocardial infarction (MI) in 1.4% and
cardiac death in 1.0% of patients over 40
❖ With selection for baseline risk, preoperative MI in 3.2%, cardiac death in
1.7%
❖ A 2016 study using data from 2004-2013 found a 3% incidence of major
adverse cardiovascular and cerebrovascular events
❖ Despite increased risk of population, rates of MI and death have decreased
over time

Initial Preprocedure Evaluation

❖ Risk Factor Assessment


❖ Physical Exam (blood pressure measurements, heart and lung exam,
abdominal palpation, examination of extremities)
❖ According to 2014 ACC/AHA Guidelines, electrocardiogram (ECG) in patients
with known cardiovascular disease, significant arrhythmia, or significant
structural heart disease unless low-risk surgery
❖ Functional Status Evaluation (Metabolic Equivalents or Duke Activity Status
Index)

120
Copyright © Oakstone Publishing, 2021. All Rights Reserved.

Very High-Risk Patients

❖ MI within 60 days or unstable angina


❖ Decompensated heart failure
❖ High-grade arrhythmias
❖ Hemodynamically important valvular heart disease
❖ Emergency Surgery

121
Copyright © Oakstone Publishing, 2021. All Rights Reserved.

Revised Cardiac Risk Index (RCRI)

❖ History of Ischemic Heart Disease


❖ History of Congestive Heart Failure
❖ Prior transient ischemic attack or cerebrovascular accident (CVA)
❖ High-risk Surgery (Thoracic, Vascular, Abdominal)
❖ Preoperative Creatinine >2 mg/dL
❖ Diabetes Mellitus Requiring Insulin

Interpretation of Risk Score

❖ 0 Points-0.4%
❖ 1 Point-0.9%
❖ 2 Points-7.0%
❖ 3+ Points-11.0%

122
Copyright © Oakstone Publishing, 2021. All Rights Reserved.

American College of Surgeons NSQIP-MICA

❖ American College of Surgeons (ACS) National Surgical Quality Improvement


Program (NSQIP) Myocardial Infarction or Cardiac Arrest (MICA) score
❖ Also referred to as “Gupta score”
❖ Type of Surgery
❖ Increased Age
❖ Abnormal Creatinine
❖ Dependent Functional Status
❖ American Society of Anesthesiologists (ASA) Class

ACS NSQIP-Surgical Risk Calculator (SRC)


❖ Age
❖ Acute Renal Failure
❖ Heart Failure
❖ Functional Status
❖ Diabetes Mellitus
❖ Procedure
❖ ASA Class
❖ Wound Class
❖ Ascites
❖ Systemic Sepsis
❖ Ventilator Dependent
❖ Disseminated Cancer

123
Copyright © Oakstone Publishing, 2021. All Rights Reserved.

ACS NSQIP-SRC (continued)


❖ Previous Cardiac Event
❖ Sex
❖ Dyspnea
❖ Smoker
❖ COPD
❖ Dialysis
❖ BMI
❖ Emergency Case
❖ Hypertension
❖ Steroid Use

Comparison of Risk Calculators


❖ RCRI
❖ Simple, validated over 20 years
❖ Only used to predict cardiac complications so does not predict all-cause mortality well
❖ ACS-SRC
❖ More complex
❖ More procedure-specific, better at predicting mortality
❖ ACS-MICA
❖ Simple, outperformed RCRI in some circumstances

124
Copyright © Oakstone Publishing, 2021. All Rights Reserved.

Management Based on Risk

❖ Further Testing
❖ Lesser Risk Procedure
❖ Non-surgical Alternative
❖ Cancelled
❖ Performed at Specialized Center

125
Copyright © Oakstone Publishing, 2021. All Rights Reserved.

Further Testing

❖ Additional testing is generally only ordered if indicated for the patient even if
they were not having surgery
❖ No evidence diagnostic or prognostic evaluation improves outcomes

12-Lead ECG
❖ Class IIa evidence that preoperative ECG is reasonable for patients with
known coronary disease, significant arrhythmia, peripheral arterial disease,
cerebrovascular disease, or other significant structural heart disease, except
for those undergoing low-risk procedure
❖ Class IIb evidence that ECG may be considered for asymptomatic patients
without known coronary artery disease, except for those undergoing low-risk
procedure
❖ Class III (no benefit) for routine preoperative ECG for asymptomatic patients
undergoing low-risk procedure

126
Copyright © Oakstone Publishing, 2021. All Rights Reserved.

Stress Testing

❖ Stress testing can stratify risk for adverse preoperative event in non-low risk
patients. Clear relationship between degree of myocardial ischemia and
long-term prognosis
❖ According to ACC/AHA guidelines, reasonable for patients who are at
elevated risk for noncardiac surgery and have poor functional status to
undergo stress testing if it will change management
❖ No study has shown that interventions performed in response to stress
testing improve outcomes

24-Hour Ambulatory Monitoring

❖ Not recommended preoperatively unless otherwise indicated


❖ No evidence of improved outcomes based on results

127
Copyright © Oakstone Publishing, 2021. All Rights Reserved.

Preoperative NT-proBNP

❖ NT-proB-type Natriuretic Peptide (BNP) secreted by the heart in response to


ventricular filling
❖ Prospective VISION cohort study found evidence that elevated preoperative
NT-proBNP was associated with increased incremental risk of vascular death
and myocardial injury or infarction within 30 days of surgery
❖ When added to RCRI score, improved cardiac risk prediction

Heart Failure
❖ Prevalence is steadily increasing due to aging population and improved survival with
newer cardiovascular therapies
❖ In Medicare claims data, elderly patients with heart failure (HF) had 50% to 100% higher
30-day mortality and readmission rates than patients without coronary artery disease
(CAD) or HF
❖ In data analysis of 38047 patients, 30-day postoperative mortality significantly higher in
patients with nonischemic HF (9.3%), ischemic HF (9.2%), and atrial fibrillation (6.4%)
than in those with CAD (2.9%)
❖ In a retrospective single-center cohort study, perioperative mortality rates for patients with
stable HF were no higher than for patients without HF

128
Copyright © Oakstone Publishing, 2021. All Rights Reserved.

Heart Failure (continued)

❖ Survival after surgery for those with Left Ventricular Ejection Fraction
(LVEF)<29% is significantly worse than for those with LVEF>29%
❖ Patients with HF and preserved LVEF had lower all-cause mortality than
patients with HF with reduced LVEF
❖ However, patients with HF with preserved LVEF had higher absolute
mortality than patients without HF

Resting Echocardiography

❖ Not indicated unless patient displays signs of left ventricular systolic or


valvular dysfunction
❖ Presence of significant systolic dysfunction or severe valvular dysfunction is
associated with worse outcomes

129
Copyright © Oakstone Publishing, 2021. All Rights Reserved.

Valvular Disease

❖ Class I evidence that patients with clinically suspected moderate or greater


degrees of valvular stenosis or regurgitation should undergo preoperative
echo if there has been either 1) no prior echo within 1 year or 2) significant
change in clinical status
❖ For adults who meet standard indications for valvular intervention
(replacement or repair) on the basis of symptoms and severity of stenosis or
regurgitation, valvular intervention before elective surgery is effective in
reducing perioperative risk

Cardiovascular Implantable Electronic Devices


(CIED)

❖ Class I evidence that before elective surgery in patients with CIED,


surgical/procedure team and clinician following CIED should communicate in
advance to formulate a plan for perioperative management
❖ ASA recommends pacemaker interrogation within one year and Automatic
Implantable Cardioverter Defibrillator (AICD) interrogation within six months

130
Copyright © Oakstone Publishing, 2021. All Rights Reserved.

Cardiac Stents

❖ Most recent ACC/AHA guidelines recommend Dual Antiplatelet Therapy


(DAPT) to continue at least six months if drug eluting stent (DES) placed in
the setting of stable ischemic disease and 12 months if placed in the setting
of acute coronary syndrome (ACS)
❖ DAPT with bare metal stent (BMS) should be continued for at least one
month if placed in the setting of stable ischemic disease and 12 months if
placed in the setting of ACS
❖ No evidence for prophylactic revascularization

Perioperative Medical Therapy-Beta Blockers

❖ Class I evidence that beta blocker should be continued in patients who are already
on chronic beta blockers
❖ Class IIb (benefit may outweigh risk) evidence that patients with intermediate or high
risk ischemia on preoperative stress testing should begin beta blocker therapy
❖ Class IIb evidence that it may be reasonable to start patients with 3 or more RCRI
risk factors on beta blocker therapy prior to surgery
❖ Uncertain perioperative benefit if patient with compelling long-term indication for beta
blocker therapy is started on beta blockers preop
❖ Class III (harm) evidence that beta blockers should not be started day of surgery

131
Copyright © Oakstone Publishing, 2021. All Rights Reserved.

Perioperative Medical Therapy-Statins

❖ Class I evidence that statins should be continued


❖ Class II evidence that perioperative initiation of statin therapy is reasonable in
patients undergoing vascular surgery
❖ Class IIb evidence that perioperative initiation of statins may be considered in
patients with clinical indications according to goal-directed medical therapy
who are undergoing elevated-risk procedures

Perioperative Medical Therapy-Alpha-2 Agonists

❖ Class III (no benefit) evidence for alpha-2 agonists preventing cardiac events
in patients undergoing noncardiac surgery

132
Copyright © Oakstone Publishing, 2021. All Rights Reserved.

Perioperative Medical Therapy-ACE Inhibitors

❖ Class IIa evidence that continuing angiotensin converting enzyme (ACE)


inhibitor or angiotensin receptor blocker (ARB) is reasonable. If they are held
on day of surgery, reasonable to restart as soon as possible
❖ Meta-analysis demonstrated hypotension in 50% of patients who took ACE
inhibitor or ARB on day of surgery, however no increase in important
cardiovascular outcomes (death, MI, stroke, kidney failure)

Post-Anesthesia Care Unit (PACU)

133
Copyright © Oakstone Publishing, 2021. All Rights Reserved.

Phases in the PACU

❖ Initial handoff
❖ Phase 1-emphasizes patient’s full recovery from anesthesia and return of
vital signs to near baseline
❖ Phase 2-prepares patient for hospital discharge

Initial Assessment and Monitoring


❖ Airway Patency
❖ Respiratory Rate
❖ Oxygen Saturation
❖ Heart Rate
❖ Blood Pressure
❖ ECG
❖ Mental Status
❖ Temperature
❖ Presence of pain, nausea, vomiting

134
Copyright © Oakstone Publishing, 2021. All Rights Reserved.

Incidence of Complications

❖ Retrospective review involving 18,473 patients noted PONV (9.8%),


complications involving the upper airway (6.9%), hypotension (2.7%),
dysrhythmias (1.4%), hypertension (1.1%), and major cardiac events (0.6%)

Postoperative Nausea and Vomiting (PONV)

❖ Five principal neurotransmitter mediators: muscarinic, histamine, dopamine,


5-HT-3 serotonin, and neurokinin 1-substance P
❖ Patient risk factors include history of PONV, preoperative nausea, female
gender, non-smoking status, younger age
❖ Anesthetic/surgical risk factors include anesthetic technique, use of nitrous
oxide, use of volatile anesthetic, amount of opioid used, and duration and
type of procedure

135
Copyright © Oakstone Publishing, 2021. All Rights Reserved.

PONV (continued)

❖ Scopolamine, dexamethasone, and ondansetron is the most commonly


administered regimen for PONV prophylaxis
❖ Dimenhydrinate, prochlorperazine, or haloperidol can be substituted if
scopolamine is contraindicated
❖ Rescue therapy is most effective from a different class of medication
❖ Neurokinin 1 receptor antagonists (aprepitant, fosaprepitant) are potentially
the most effective antiemetics with additional benefit of long half-life

Respiratory Complications
❖ Most common major complication in the immediate postoperative period, second
most common complication after PONV
❖ Patient risk factors include preexisting lung issues, obesity, heart failure,
pulmonary hypertension, tobacco use, ASA risk class
❖ Procedure-related risk factors include surgical site close to the diaphragm, ENT
procedures, neurosurgical procedures, incisional pain affecting respiration, large
resuscitation, and procedure duration greater than 3 hours
❖ Anesthetic risk factors include general anesthesia, use of neuromuscular
blockade, and administration of opioids

136
Copyright © Oakstone Publishing, 2021. All Rights Reserved.

Respiratory Complications (continued)

❖ Tachypnea, bradypnea, oxygen saturation <93%, anxiety, confusion,


agitation, tachycardia, hypertension are all signs of respiratory distress
❖ Diagnosis includes auscultation (upper vs lower airway sounds), chest
radiograph, arterial blood gas
❖ Treatment consists of supplemental oxygen and potentially ventilatory
support (high flow nasal cannula, non-invasive ventilation, reintubation)

Causes of Respiratory Distress

❖ Upper airway obstruction: pharyngeal muscle weakness, laryngospasm,


airway edema, foreign body, vocal cord paralysis, cervical hematoma, and
obstructive sleep apnea
❖ Lower airway: bronchospasm, pulmonary edema, aspiration pneumonitis,
tension pneumothorax, pulmonary embolus, atelectasis, preexisting
pneumonia
❖ Central nervous system abnormalities: anesthetic agents, neuromuscular
blockade, or cerebrovascular accident

137
Copyright © Oakstone Publishing, 2021. All Rights Reserved.

Cardiovascular Complications

❖ Third most common problem requiring treatment in the recovery room


❖ Preoperative risk factors: cardiovascular comorbidities, acute trauma, recent
CVA, urgent or emergent surgeries
❖ Intraoperative risk factors: invasiveness of procedure, effects and techniques
of anesthetic, severity of preoperative stress
❖ Postoperative risk factors: bleeding, hypoxemia, pain

Cardiovascular Complications-Hypotension

❖ Rather than targeting a specific blood pressure, maintain blood pressure


within patient’s baseline
❖ Causes include:
❖ Hypovolemia
❖ Drug Effects (antihypertensive agents, anesthetic agents)
❖ Sympathectomy due to neuraxial block
❖ Adrenal Insufficiency

138
Copyright © Oakstone Publishing, 2021. All Rights Reserved.

Cardiovascular Complications
Hypotensive Emergencies
❖ Hypovolemia
❖ Septic
❖ Anaphylactic
❖ Cardiogenic
❖ Arrhythmogenic
❖ Local Anesthetic Systemic Toxicity
❖ Tension Pneumothorax
❖ Pulmonary Embolus
❖ Left Ventricular Outflow Obstruction

Cardiovascular Complications-Hypertension

❖ Generally, blood pressure is treated if systolic blood pressure>180 mmHg or


diastolic blood pressure>110mmHg, however baseline blood pressure must
be taken into account
❖ Labetalol, hydralazine, metoprolol, or nicardipine commonly used
❖ If necessary, infusion can be started and patient transferred to appropriate
level of care

139
Copyright © Oakstone Publishing, 2021. All Rights Reserved.

Causes of Hypertension

❖ Preexisting hypertension
❖ Noxious stimuli: pain, nausea/vomiting, hypoxia or hypercarbia, hypothermia,
bladder distention, hypervolemia
❖ Drug effects: emergence delirium, alcohol withdrawal, opioid withdrawal,
recent stimulant use

Cardiovascular Complications
Cardiac Arrhythmias
❖ Atrial tachycardias
❖ Sinus Tachycardia
❖ Atrial Fibrillation
❖ AV Node Reentrant Tachycardia
❖ Ventricular Arrhythmias
❖ Premature Ventricular Contractions
❖ Ventricular Fibrillation with Pulseless Electrical Activity
❖ Polymorphic Ventricular Tachycardia (Torsades de Pointes)

140
Copyright © Oakstone Publishing, 2021. All Rights Reserved.

Cardiovascular Complications-
Bradyarrhythmias

❖ Mild sinus bradycardia that remains stable in an asymptomatic patient without


major alternations in blood pressure does not require treatment
❖ Moderate bradycardia with mild symptoms can be treated with IV
glycopyrrolate and/or ephedrine
❖ Severe bradycardia that is either symptomatic or unstable should be treated
with atropine or other chronotropic and/or temporary pacing

Causes of Bradyarrhythmias

❖ Medications (beta blockers, calcium channel blockers, digoxin, amiodarone,


anticholinesterase agents, opioids)
❖ Neuraxial anesthesia
❖ Bowel or bladder distention
❖ Respiratory problems (hypoxemia)
❖ Myocardial ischemia

141
Copyright © Oakstone Publishing, 2021. All Rights Reserved.

Neuropsychiatric Complications

❖ Intraoperative awareness and recall


❖ Delayed emergence and delirium
❖ Visual disturbance: corneal abrasion, ischemic optic neuropathy, cerebral
visual loss, retinal artery occlusion, acute angle-closure glaucoma
❖ Spinal epidural hematoma

Postoperative Urinary Retention (POUR)

❖ Incidence ranges from 5 to 70 percent


❖ Patient risk factors: older age, male, history of previous urinary retention,
neurologic disease, prior pelvic surgery
❖ Procedural risk factors: anorectal surgery, joint arthroplasty, hernia repair,
incontinence surgery
❖ Anesthetic risk factors: excessive fluid administration, opioids,
anticholinergics, beta-blockers, prolonged duration of anesthesia, and
neuraxial anesthesia

142
Copyright © Oakstone Publishing, 2021. All Rights Reserved.

POUR Evaluation and Treatment

❖ Evaluation typically performed with either ultrasound or catheterization


❖ If >600 mL is measured on ultrasound one-time bladder catheterization is
usually performed
❖ In some cases, ambulatory patients with no risk factors for POUR and <600
mL on bladder ultrasound can be discharged home without being
catheterized even if they have not voided

PACU Discharge Criteria

❖ Several scoring systems developed


❖ Postanesthetic Discharge Scoring System (PADSS) based on vital signs,
activity level, nausea or vomiting, pain, surgical bleeding, and intake and
output
❖ Aldrete score assesses activity, respiration, circulation, consciousness, skin
color. Modified Aldrete score substitutes oxygen saturation for skin color.

143
Copyright © Oakstone Publishing, 2021. All Rights Reserved.

References
Fleisher LA, Fleischmann KE, Auerbach AD, et al. 2014 ACC/AHA guideline on perioperative cardiovascular evaluation and management of patients undergoing noncardiac surgery: a report of the

American College of Cardiology/American Heart Association Task Force on practice guidelines. J Am Coll Cardiol 2014; 64:e77.

Vascular Events In Noncardiac Surgery Patients Cohort Evaluation (VISION) Study Investigators, Devereaux PJ, Chan MT, et al. Association between postoperative troponin levels and 30-day mortality

among patients undergoing noncardiac surgery. JAMA 2012; 307:2295.

Gupta PK, Gupta H, Sundaram A, et al. Development and validation of a risk calculator for prediction of cardiac risk after surgery. Circulation 2011; 124:381.

Ford MK, Beattie WS, Wijeysundera DN. Systematic review: prediction of perioperative cardiac complications and mortality by the revised cardiac risk index. Ann Intern Med 2010; 152:26.

Smilowitz NR, Gupta N, Ramakrishna H, et al. Perioperative Major Adverse Cardiovascular and Cerebrovascular Events Associated With Noncardiac Surgery. JAMA Cardiol 2017; 2:181.

Livhits M, Ko CY, Leonardi MJ, et al. Risk of surgery following recent myocardial infarction. Ann Surg 2011; 253:857.

van Klei WA, Bryson GL, Yang H, et al. The value of routine preoperative electrocardiography in predicting myocardial infarction after noncardiac surgery. Ann Surg 2007; 246:165.

Wijeysundera DN, Pearse RM, Shulman MA, et al. Assessment of functional capacity before major non-cardiac surgery: an international, prospective cohort study. Lancet 2018; 391:2631.

Lee TH, Marcantonio ER, Mangione CM, et al. Derivation and prospective validation of a simple index for prediction of cardiac risk of major noncardiac surgery. Circulation 1999; 100:1043.

Mangano DT, Browner WS, Hollenberg M, et al. Association of perioperative myocardial ischemia with cardiac morbidity and mortality in men undergoing noncardiac surgery. The Study of Perioperative

Ischemia Research Group. N Engl J Med 1990; 323:1781.

References (continued)
Rohde LE, Polanczyk CA, Goldman L, et al. Usefulness of transthoracic echocardiography as a tool for risk stratification of patients undergoing major noncardiac surgery. Am J

Cardiol 2001; 87:505.

Fortier J, Chung F, Su J. Unanticipated admission after ambulatory surgery--a prospective study. Can J Anaesth 1998; 45:612.

Horn CC, Wallisch WJ, Homanics GE, Williams JP. Pathophysiological and neurochemical mechanisms of postoperative nausea and vomiting. Eur J Pharmacol 2014; 722:55.

Apfelbaum JL, Silverstein JH, Chung FF, et al. Practice guidelines for postanesthetic care: an updated report by the American Society of Anesthesiologists Task Force on

Postanesthetic Care. Anesthesiology 2013; 118:291.

Kluger MT, Bullock MF. Recovery room incidents: a review of 419 reports from the Anaesthetic Incident Monitoring Study (AIMS). Anaesthesia 2002; 57:1060.

Baldini G, Bagry H, Aprikian A, Carli F. Postoperative urinary retention: anesthetic and perioperative considerations. Anesthesiology 2009; 110:1139.

Hines R, Barash PG, Watrous G, O'Connor T. Complications occurring in the postanesthesia care unit: a survey. Anesth Analg 1992; 74:503.

Smetana GW. Postoperative pulmonary complications: an update on risk assessment and reduction. Cleve Clin J Med 2009; 76 Suppl 4:S60.

Rose DK, Cohen MM, DeBoer DP. Cardiovascular events in the postanesthesia care unit: contribution of risk factors. Anesthesiology 1996; 84:772.

Liu JB, Liu Y, Cohen ME, et al. Defining the Intrinsic Cardiac Risks of Operations to Improve Preoperative Cardiac Risk Assessments. Anesthesiology 2018; 128:283.

144
Copyright © Oakstone Publishing, 2021. All Rights Reserved.

Thank you for your attention

145
Copyright © Oakstone Publishing, 2021. All Rights Reserved.

Head and Neck Evaluation

Rosh Sethi, MD MPH


Assistant Professor of Otolaryngology-Head and Neck Surgery
Harvard Medical School

Head and Neck Surgical Oncology and Microvascular Reconstruction


Brigham and Women’s Hospital | Dana-Farber Cancer Institute

October, 2021

No relevant financial disclosures

146
Copyright © Oakstone Publishing, 2021. All Rights Reserved.

Agenda
Examining the head and neck
Overview of general exam principles, techniques, nomenclature, and findings

Head and neck anatomy


Basic overview of critical landmarks

Head and neck radiology


Correlation between anatomy and cross-sectional imaging

Workup of a neck mass


Overview of neck masses, diagnosis, and management

Basic management of head and neck malignancies


Overview of head and neck cancer and basic management

Parotidectomy
Pearls and pitfalls

Tracheostomy
From the perspective of a head and neck surgeon

Edit entity or department by going to Insert > Header & Footer |


Confidential—do not copy or distribute

What do
otolaryngologists do?

147
Copyright © Oakstone Publishing, 2021. All Rights Reserved.

From the dura to the


pleura.
– A clever resident

Edit entity or department by going to Insert > Header & Footer |


Confidential—do not copy or distribute

Otolaryngology-Head and Neck Surgery

• 7 subspecialties
• Head and neck surgical oncology
• Neurotology-lateral skull base
• Rhinology and anterior skull base
• Facial plastics
• Laryngology
• Pediatric
• Sleep surgery

• Approximately 10,000 otolaryngologists practicing in the United States1

1 Hughes CA et al. Otolaryngology workface analysis. Laryngoscope. 2016 Dec;126 Suppl 9:S5-S11

148
Copyright © Oakstone Publishing, 2021. All Rights Reserved.

149
Copyright © Oakstone Publishing, 2021. All Rights Reserved.

Examining the head


and neck

Taking a good head and neck history

• Assess risk factors for malignancy:


• Smoking (chewing and inhaled)
• EtOH use/abuse
• Family history
• Genetic predisposition

• Thorough review of systems:


• Otalgia, dysphagia, odynophagia, hoarseness, neck mass, weight loss

150
Copyright © Oakstone Publishing, 2021. All Rights Reserved.

151
Copyright © Oakstone Publishing, 2021. All Rights Reserved.

152
Copyright © Oakstone Publishing, 2021. All Rights Reserved.

Fiberoptic nasolaryngoscopy

153
Copyright © Oakstone Publishing, 2021. All Rights Reserved.

Critical parts of the head and neck exam

• Cranial nerves

• Visual exam and palpation

• Neck mass
• Location
• Mobility
• Fixation

• Parotid mass
• Location
• Facial nerve function
• Facial pain or numbness

Head and Neck


Anatomy

154
Copyright © Oakstone Publishing, 2021. All Rights Reserved.

155
Copyright © Oakstone Publishing, 2021. All Rights Reserved.

156
Copyright © Oakstone Publishing, 2021. All Rights Reserved.

157
Copyright © Oakstone Publishing, 2021. All Rights Reserved.

Head and Neck


Radiology

158
Copyright © Oakstone Publishing, 2021. All Rights Reserved.

Workup of a neck mass

159
Copyright © Oakstone Publishing, 2021. All Rights Reserved.

Neck masses Neck mass

Congenital Inflammatory Neoplastic

Metastatic head
Branchial cleft cyst Reactive viral and neck
carcinoma

Thyroglossal duct Bacterial


Thyroid mass
cyst lymphadenopathy

Vascular anomaly Parasitic Salivary gland

Ranula Non-infectious Paraganglioma

Laryngocele Lymphoma

Dermoid Lipoma/skin cyst

Exclude malignancy: History

• Lack of obvious infection


• Duration > 2 weeks
• Age > 40 years
• Tobacco or EtOH abuse
• History of head or neck cancer
• Immunocompromised status
• COVID-vaccination

160
Copyright © Oakstone Publishing, 2021. All Rights Reserved.

Exclude malignancy: Symptoms

• Hoarseness/voice change
• Otalgia
• Epistaxis/bleeding
• Oral pain
• Persistent ulcer
• Odynophagia or dysphagia
• Hemoptysis
• Dyspnea
• Stridor/stertor
• Unexplained weight loss
• Growth/changes/progression
• B-symptoms

Exclude malignancy: Physical exam

• Size >1.5cm
• Firm texture
• Reduced mobility
• Skin changes
• Non-tender

• Full head and neck exam

161
Copyright © Oakstone Publishing, 2021. All Rights Reserved.

Physical exam pearls and pitfalls

• Localize the mass

• Characterize the mass


• Fixation
• Tenderness
• Ballotable/fluctuance
• Pulsatile or not

• Don’t focus on just the mass!


• Examine oral cavity/oropharynx
• Skin exam if indicated
• Palpate thyroid

162
Copyright © Oakstone Publishing, 2021. All Rights Reserved.

Imaging

vs US
Axial
Pathology
Core
vs
FNA

Diagnosis

163
Copyright © Oakstone Publishing, 2021. All Rights Reserved.

Imaging modalities

ULTRASOUND COMPUTED TOMOGRAPHY MAGNETIC RESONANCE IMAGING

• In-expensive • Well tolerated • Better soft-tissue definition


• Least invasive • Reformatting algorithms (e.g. assess infiltration
• Operator experience • Bone and soft tissue detail perineural spread)
• Reliability • Excellent initial diagnostic • Lack of ionizing radiation
study if malignancy suspected • Longer, more expensive
• Ionizing radiation

Fine-needle aspiration biopsy

• Accuracy of 93% regardless of location or histology1


• Safe, highly accurate
• Dependent upon skills of clinician performing procedure
• Sampling error
• Cytopathological interpretation
• Cell blocks can be sectioned and stained

1 Tandon S et al. Fine-needle aspiration cytology in a regional head and neck cancer center: comparison with a systematic review and metanalysis. Head Neck.
2008;30(9):1246

164
Copyright © Oakstone Publishing, 2021. All Rights Reserved.

Excisional
STOP
biopsy

• Paragangliomas – hemorrhage
• Schwannomas – unexpected cranial nerve deficits
STOP •

Parotid masses - -tumor spillage/recurrence
Metastatic SCCa – tumor spillage, challenges for further surgical
resection, need for adjuvant chemoradiotherapy

165
Copyright © Oakstone Publishing, 2021. All Rights Reserved.

166
Copyright © Oakstone Publishing, 2021. All Rights Reserved.

Basic management of
head and neck
malignancies

167
Copyright © Oakstone Publishing, 2021. All Rights Reserved.

Epidemiology

• 5.7% of cancer-related mortality1


• Highest incidence among males
• Tobacco (smoked and smokeless) and EtOH
• HPV infection
• Betel nut, radiation, vitamin deficiencies,
immunosuppression, vitamin deficiencies

1PattersonRH, Fischman VG, Wasserman I et al. Global Burden of Head and Neck Cancer: Economic Consequences, Health
and the Role of Surgery. Otolaryngol Head and Neck Surg. 2020;162(3):296

Anatomy

• Upper aerodigestive tract lined with


squamous mucosa
• 90-95% are SCCa
• UADT subsites:
• Oral cavity
• Oropharynx
• Hypopharynx
• Larynx
• Nasopharynx
• Nose and paranasal sinuses

168
Copyright © Oakstone Publishing, 2021. All Rights Reserved.

Treatment

Multidisciplinary approach
• Medical oncology
• Surgical oncology
• Radiation oncology
• Dentistry/oral medicine
• Speech and language pathology
• Rehabilitation therapy
• Prosthodontics
• Psychosocial oncology
• Social work

Surgery

Chemotherapy
Radiation
Immunotherapy

169
Copyright © Oakstone Publishing, 2021. All Rights Reserved.

Oral Cavity

• Surgery is favored treatment option for locoregional


disease of the oral cavity
• 7 subsites
• Lips
• Buccal mucosa
• Alveolar ridge
• Floor of mouth
• Oral tongue
• Retromolar trigone
• Hard palate

Oral tongue

170
Copyright © Oakstone Publishing, 2021. All Rights Reserved.

171
Copyright © Oakstone Publishing, 2021. All Rights Reserved.

172
Copyright © Oakstone Publishing, 2021. All Rights Reserved.

173
Copyright © Oakstone Publishing, 2021. All Rights Reserved.

174
Copyright © Oakstone Publishing, 2021. All Rights Reserved.

175
Copyright © Oakstone Publishing, 2021. All Rights Reserved.

Maxilla/Buccal

176
Copyright © Oakstone Publishing, 2021. All Rights Reserved.

177
Copyright © Oakstone Publishing, 2021. All Rights Reserved.

Mandible

178
Copyright © Oakstone Publishing, 2021. All Rights Reserved.

179
Copyright © Oakstone Publishing, 2021. All Rights Reserved.

180
Copyright © Oakstone Publishing, 2021. All Rights Reserved.

181
Copyright © Oakstone Publishing, 2021. All Rights Reserved.

182
Copyright © Oakstone Publishing, 2021. All Rights Reserved.

183
Copyright © Oakstone Publishing, 2021. All Rights Reserved.

184
Copyright © Oakstone Publishing, 2021. All Rights Reserved.

Lip

185
Copyright © Oakstone Publishing, 2021. All Rights Reserved.

186
Copyright © Oakstone Publishing, 2021. All Rights Reserved.

187
Copyright © Oakstone Publishing, 2021. All Rights Reserved.

188
Copyright © Oakstone Publishing, 2021. All Rights Reserved.

Composite resections

189
Copyright © Oakstone Publishing, 2021. All Rights Reserved.

190
Copyright © Oakstone Publishing, 2021. All Rights Reserved.

Larynx/Pharynx

191
Copyright © Oakstone Publishing, 2021. All Rights Reserved.

192
Copyright © Oakstone Publishing, 2021. All Rights Reserved.

Oropharynx

193
Copyright © Oakstone Publishing, 2021. All Rights Reserved.

194
Copyright © Oakstone Publishing, 2021. All Rights Reserved.

195
Copyright © Oakstone Publishing, 2021. All Rights Reserved.

Parotidectomy

Indications for parotidectomy

• Growing parotid mass


• Biopsy concerning for malignancy
• Biopsy proven malignancy
• Benign mass causing cosmetic deformity

196
Copyright © Oakstone Publishing, 2021. All Rights Reserved.

Parotidectomy

Preoperative evaluation Surgical technique Extent of resection

• Imaging • Incision/approach • Superficial


• CT vs MRI (favored) • Identification of facial nerve • Partial
• Biopsy • Total
• FNA • Extracapsular dissection
• Core needle biopsy • Enucleation (no longer
performed)

197
Copyright © Oakstone Publishing, 2021. All Rights Reserved.

198
Copyright © Oakstone Publishing, 2021. All Rights Reserved.

Tracheostomy

199
Copyright © Oakstone Publishing, 2021. All Rights Reserved.

200
Copyright © Oakstone Publishing, 2021. All Rights Reserved.

Bjork flap

Tracheostomy considerations in head and neck patients

• Laryngeal malignancy causing airway obstruction


• Trach HIGH (under cricoid)
• Optimize exposure
• Cricothyroidotomy hook
• Positioning
• Tracheostomy through tumor

201
Copyright © Oakstone Publishing, 2021. All Rights Reserved.

Agenda
Examining the head and neck
Overview of general exam principles, techniques, nomenclature, and findings

Head and neck anatomy


Basic overview of critical landmarks

Head and neck radiology


Correlation between anatomy and cross-sectional imaging

Workup of a neck mass


Overview of neck masses, diagnosis, and management

Basic management of head and neck malignancies


Overview of head and neck cancer and basic management

Parotidectomy
Pearls and pitfalls

Tracheostomy
From the perspective of a head and neck surgeon

Edit entity or department by going to Insert > Header & Footer |


Confidential—do not copy or distribute

Thank you

202
Copyright © Oakstone Publishing, 2021. All Rights Reserved.

203
Copyright © Oakstone Publishing, 2021. All Rights Reserved.

Thoracic Surgery for general surgeons


Jon O. Wee, MD
• Section Chief, Esophageal Surgery
• Co-Director of Minimally Invasive Thoracic Surgery
• Director of Robotics in Thoracic Surgery
• Associate Program Director
• Division of Thoracic Surgery
• Brigham and Women’s Hospital
• Assistant Professor of Surgery
• Harvard Medical School

Conflict of Interest

Consultant: Medtronic, Ethicon, Intuitive,


Boston Scientific

204
Copyright © Oakstone Publishing, 2021. All Rights Reserved.

Thoracic Surgery
• Lung Cancer
• Screening
• Staging
• Treatment
• Mediastinum
• Pleural Space
• Trauma

• Not Discussing Esophagus

Lung Nodule
• 65 yo Female presents to her PCP for routine physical. She has some
COPD and had a MI 3 years ago but has had no other symptoms. She
quite smoking following her MI after smoking a pack a day for 40
years previously. Her PCP decides to get a chest ct.

205
Copyright © Oakstone Publishing, 2021. All Rights Reserved.

Advances in Surgery for Lung Cancer

• Screening and Detection


• Diagnostic
• EBUS
• Nav Bronch
• Surgery
• VATS
• Robotic

206
Copyright © Oakstone Publishing, 2021. All Rights Reserved.

Management of Non-Small
Cell Lung Cancer
Lung Cancer
Survival

Lung Cancer
California Cancer Registry
• 1989-2003
• 101,844 NSCLC patients
• Stage I 19,702 23%
33%
• Stage II 3,753
• Stage IIIA 10,429
• Stage IIIB 22,285
• Stage IV 45,675

Chest 2007; 132:193-199

207
Copyright © Oakstone Publishing, 2021. All Rights Reserved.

Stage I NSCLC

California Cancer Registry 1989-2003

100,651 Lung Cancer pts

19,702 Stage I

1,432 pts did not receive any treatment

Not Planned (45%), Contraindicated due to patient


factors (22%), recommended but not performed (32%)

Untreated Stage I NSCLC

Median Survival 9 months

5 Year survival 7%

89% of patients who were recommended treatment but


refused died within 5 year, 78% of whom died due to cancer

208
Copyright © Oakstone Publishing, 2021. All Rights Reserved.

Untreated Stage I NSCLC


• T1 disease specific survival
• 72% 5 yr, 155 months vs 23%, 26 months
• T2 treated cancer specific survival
• 57% 5 yr, 91 months vs 12%, 10 months

International Early Lung Cancer Program


• 31567 asymptomatic pts over 40 • 484 patients developed lung cancer
yrs old at risk of lung cancer • Overall 10 year survival of all lung
• Smoking history, occupational cancer patients was 80%
exposure, second hand smoke • 85% of diagnosed patients had
• Low dose CT from 1993 to 2005 clinical stage I cancer
• Median Follow-up 40 months • Stage I patients who had resection
within 1 month of diagnosis had a
10 year survival of 92%
• Overall operative mortality 0.5% (
Board Cert Thoracic Surgeons)

209
Copyright © Oakstone Publishing, 2021. All Rights Reserved.

The National Lung Screening Trial


(NLST)

• Phase III RCT sponsored by NCI


• 53,454 participants 8/02 to 4/04
• 3 annual screens (LDCT versus CXR) followed by 4 years
observation (no screen during observation)

• Stopped early for 20% decrease in Lung Ca-specific mortality


using low-dose computed tomography (LDCT)
(30 pack-year history, ages 55–75, smoked within 15 yrs)

• Operative Mortality <1% for surgical resections

Lung Cancer Risk is a Function of Age

Incidence (per 100,000)

Age Men Women


66 – 70 years 336 248
71 – 75 years 490 350
> 75 years 517 307

http://www.who.int/gho/ncd/mortality_morbidity/cancer_test/en/index.html

210
Copyright © Oakstone Publishing, 2021. All Rights Reserved.

CT scans for Lung Cancer Screening

• To qualify for the once-per-year benefit (annual), patients


must be 55 to 77 years old.
• Additionally, Medicare beneficiaries must
• currently smoke tobacco products or have quit within the past
15 years,

• have smoked an average of one pack of cigarettes a day for 30


years, and

• have a physician or other health care professional's written


order requesting the test.

Size matters
• 10 year survival rates vs tumor size in clinical stage I patients
• 20mm or less 80% *
• 10mm or less 93% **
• SEER data
• 15mm or less 75% 8 year survival†

*Buell PE, J Surg Oncol 1971;3:539-51


**Martini N, et al. JTCVS 1995;109:120-9
†Henschke CI, et al. Lung Cancer 2003;39:327-30

211
Copyright © Oakstone Publishing, 2021. All Rights Reserved.

Lung Cancer Staging

212
Copyright © Oakstone Publishing, 2021. All Rights Reserved.

213
Copyright © Oakstone Publishing, 2021. All Rights Reserved.

Diagnostic Work-up
• PFT
• CT scan
• Pet/CT
• Head MRI
• Cervical Mediastinoscopy/EBUS
• Lung biopsy

PET/CT:

214
Copyright © Oakstone Publishing, 2021. All Rights Reserved.

Cervical Mediastinoscopy

Detection
• EBUS

215
Copyright © Oakstone Publishing, 2021. All Rights Reserved.

Navigational Bronchoscopy

216
Copyright © Oakstone Publishing, 2021. All Rights Reserved.

Lung Cancer Management


• Stage I – Surgery
• Stage II – Surgery …. Chemo/xrt
• Stage IIIa – Chemo, Chemo/xrt, (immunotherapy) followed by surgery
• Stage IIIb – Chemo, Immuno/targeted therapy
• Stage IV – Chemo, Immuno/targeted therapy

First-line Pembrolizumab
9/7/2017 2/21/2020
• 69 yr old man with metastatic
adenocarcinoma lung PD-L1 = 95%

• Received pembrolizumab
9/26/2017 – 8/13/2019

• Continue to follow without


evidence of disease >20 months
from stopping pembrolizumab

Dana-Farber Cancer Institute

217
Copyright © Oakstone Publishing, 2021. All Rights Reserved.

Immunotherapy side effect profile

• Rare but serious autoimmune toxicities


• Rash
• Colitis
• Hepatitis
• Endocrinopathies (pituitary, thyroid, adrenal)
• Pneumonitis
• Myocarditis, uveitis, myositis, type 1 diabetes, nephritis, etc…

Unresectable Stage III


• Durvalumab for 1 year after
completing concurrent
chemotherapy and radiation
improves both PFS and OS.

• Median OS:
• Durva group 47.5 months
• Placebo group 29.1 months
• 48-month PFS rate
• Durva group 35.3%
• Placebo group 19.5%

Faivre-Finn et al. PACIFIC; JTO 2021

218
Copyright © Oakstone Publishing, 2021. All Rights Reserved.

Treatment Algorithm
• Genomic Testing!!!
• PD-L1 testing

• If targetable genomic alteration  Treat with targeted therapy!


• If NO targetable genomic alterations and PD-L1 >50%
• Pembrolizumab alone
• If NO targetable genomic alterations and PD-L1 <50%
• Chemotherapy plus pembrolizumab

Surgery
• Thoracotomy
• VATS Lobectomy
• Robot Lobectomy

219
Copyright © Oakstone Publishing, 2021. All Rights Reserved.

Modern Thirty-day Operative Mortality for


Surgical Resection in Lung Cancer, 1983

• Goal: determine 30 day mortality rates


• 2,200 LCSG resections analyzed
• Mortality Rate: 3.7% at 30 day post-op
• Mortality related to age:
• 1.3% < 60 years
• 4.1% 60-69 years
• 7.1% 70-79 years
• 11% > 80 years

Ginsberg et al. JTCVS 1983

Thoracotomy
• Posterolateral thoracotomy
• Anterolateral thoracotomy
• Hemiclamshell
• Clamshell

220
Copyright © Oakstone Publishing, 2021. All Rights Reserved.

Posterolateral thoracotomy

Posterolateral thoracotomy
• Standard open incision
• 4th or 5th interspace for lung resection
• 7th interspace for lower esophageal access
• Perforation (Boerhaave syndrome)
• Incision
• Curls from behind the scapula, around below the tip of scapula, then follows the
interspace anteriorly
• Can extend superiorly behind scapula to get higher
• Divide the latissimus dorsi
• Divide tissue posterior to the serratus to get under the scapula
• Can divide serratus anteriorly
• Divide intercostal muscles to enter interspace
• Can shingle rib if needed
• Place rib spreader

221
Copyright © Oakstone Publishing, 2021. All Rights Reserved.

VATS Lobectomy

Video-Assisted Thoracic Surgery Lobectomy:


Report of CALGB 39802 – A Prospective, Multi-
Institution Feasibility Study, 2007
• Evaluate technical feasibility of VATS lobectomy with traditional hilar dissection
without rib spreading
• 111 patients underwent successful VATS lobectomies
• 30 day mortality was 2.7% (3 patients) – none related to VATS
technique
• Surgeons had QA participation requirement
• Standardized approach to VATS lobectomy was defined

Swanson et al, J Clin Oncol 2007

222
Copyright © Oakstone Publishing, 2021. All Rights Reserved.

Definition of VATS lobectomy


• Single access incision (4-8cm)
• Up to three port incisions
• Individual ligation of lobar vessels and bronchus
• Hilar lymph node sample or dissection
• NO retractor or rib spreading

Equipment
• Staplers
• Covidien
• Curved tip staplers
• Ethicon
• Powered stapler

223
Copyright © Oakstone Publishing, 2021. All Rights Reserved.

224
Copyright © Oakstone Publishing, 2021. All Rights Reserved.

225
Copyright © Oakstone Publishing, 2021. All Rights Reserved.

PRE-OP

POST-OP

Thoracoscopic Lobectomy has increasing benefit


in patient with poor pulmonary function: a STS
database analysis

Respiratory Complications
by FEV1 predicted

Ann Surg 2012 Sep 256(3):487-93

226
Copyright © Oakstone Publishing, 2021. All Rights Reserved.

Thoracoscopic Lobectomy is associated with lower


morbidity than open lobectomy: a propensity-
matched analysis from the STS database

• VATS Lobe
• Lower arrhythmias (7.3%vs11.5%, p=.0004)
• Less Reintubation (1.4% vs 3.1%, p=.0046)
• Less Transfusion (2.4% vs 4.6%, p=.0028)
• Shorter Length of Stay (4 v 6 dys, p<.0001)
• Shorted CT duration (3 v 4 dys, p<.0001)
• No difference in operative mortality

JTCVS 2010 Feb; 139(2):366-78

Video-assisted thoracoscopic lobectomy is less costly and morbid than open


lobectomy: a retrospective multi-institutional database analysis

• Comparison of Hospital Costs


• Open $21,016 v VATS $20,316 p=0.027
• Low volume surg $22,050
• High volume surg $18,133
• Cost not different for open ($21,000)
• LOS 7.83 (open) v VATS 6.15
• Risk of adverse event lower in VATS (odds ratio 1.22, p=0.019)

Ann Thor Surg 2012 Apr;93(4):1027-32

227
Copyright © Oakstone Publishing, 2021. All Rights Reserved.

Robotic Video-Assisted Lung


Surgery
• Anatomical dissection
• Trocar access only
• Individual dissection / ligation
• Vascular structures
• Bronchial structures
• Complete lymph node dissection
• No utility thoracotomy
• Lung removal performed beneath
rib cage tip of 11-12th rib

Robotic Video-Assisted Lung Surgery

228
Copyright © Oakstone Publishing, 2021. All Rights Reserved.

Long-term Survival Based on the Surgical Approach to Lobectomy For Clinical Stage I Nonsmall Cell
Lung Cancer: Comparison of Robotic, Video-assisted Thoracic Surgery, and Thoracotomy Lobectomy.

• Clinical Stage I
• 172 robot, 141 VATS, 157 Open
• LN Robot 5, Vats 3, Open 4 (P<0.001)
• LOS 4 Day Robot and VATS, 5 for Open
• 5 yr OS 77.6%, 73.5%, 77.9%
• Multivariate analysis surg approach no associated with outcome

Yang et al, Ann Surg March 2016

229
Copyright © Oakstone Publishing, 2021. All Rights Reserved.

Conclusion
• Lung Cancer screen will benefit high risk patients with earlier
detection
• Treatment of earlier stage cancers will result in better survival
• New surgical techniques will decrease morbidity and mortality

Mediastinal Compartments
Thymomas
Lymphomas
Germ cell Tumors
Neurogenic Tumors
Teratomas
Benign
Thyroid Goiters/Tumors
Malignant
Parathyroid Masses
Enteric duplications
Esophageal diverticula
Bronchogenic Cysts
Endocrine

Cysts
Pericardial
Bronchogenic
Hiatal Hernia

230
Copyright © Oakstone Publishing, 2021. All Rights Reserved.

Differential Diagnosis: Anterior/Middle Mediastinal


Mass

Common Dx possibilities by anatomic compartment


Baue, AE, et al. Glenn’s Thoracic and Cardiovascular Surgery 5th ed. Appleton & Lange 1991

Case
• 21 y.o female presents with symptoms of ptosis and diplopia.
• She is a nonsmoker
• Her speech began to be slurred the longer she spoke
• Symptoms got worse as the day progressed.

231
Copyright © Oakstone Publishing, 2021. All Rights Reserved.

• Tensilon test is positive


• Anti ACH receptor antibody levels elevated

Myasthenia Gravis
• Disease of neuromuscular junction
• Autoimmune reaction IgG directed against the nicotinic
Ach receptors at post synaptic membrane

232
Copyright © Oakstone Publishing, 2021. All Rights Reserved.

Osserman Classification

• MG with thymectomy
• 2x likely to achieve medication free remission
• 1.6x likely to become asymptomatic
• 1.7x likely to improve
• Pt undergoing surgery tended to have more severe disease, younger,
and female
• Larger improvement in pts with more severe disease (Osserman class 2b-4)
• 3.7x more likely to attain remission

Gronseth, Barohn, Am Aca Neur; 2006

233
Copyright © Oakstone Publishing, 2021. All Rights Reserved.

• Timing
• Rodriguez et al noted 10 fold increase in remission rate following thymectomy
in the 1st year
• Earlier thymectomy may increase benefit
• Mortality before 1970 – 5 to 15%
• Now < 1%
• Nerve injury 2-4%

Gronseth, Barohn, Am Aca Neur; 2006


Monden et al, Ann Thor Surg 1984, 38:287-291

Practice Guidelines
• No randomized Class I studies to validate thymectomies
• Gronseth & Barohn
• Official guideline from American Academy of Neurology
• Reviewed all studies for myasthenia gravis and thymectomies from 1966 to
1988
• MG patients with thymectomy more likely to achieve medication-free
remission, become asymptomatic, and improve
• No study described a significant negative association

Gronseth, Barohn, Am Aca Neur; 2006

234
Copyright © Oakstone Publishing, 2021. All Rights Reserved.

Thymus
• Embryolgically active

• Involutes as we age

Thymus
• Composed of multiple
lobes in neck and
mediastinum
• Can be separately
encapsulated
• Ectopic foci widely
distributed in
mediastinum

– Jaretski A, et al JTCVS, 1988; 96:711-716, Masaoka A et al, JTCVS 1975;70:747-754

235
Copyright © Oakstone Publishing, 2021. All Rights Reserved.

Surgical Goals
• Total thymectomy
• Re-operation for incomplete thymectomy and persistent symptoms
resulted in remission of MG

– Masaoka, et al Neurology 1982;32:83-85, Rosenberg, et al Am J Med 1983;74:78-82, Jaretski et al JTCVS


1988;95:747-757

Transcervical
Thymectomy

• 4cm incision, 2cm above


sternal notch
• Upper poles dissected free
• Blunt dissection with finger
into substernal plane
• Sternum lifted upwards with
retractor (Patient is nearly
off the table)
• Thymus retracted upwards
and thymic veins to
innominate taken
• Kitner and sponge dissection
to remove specimen
• Unable to see posterior to
innominate or AP window

236
Copyright © Oakstone Publishing, 2021. All Rights Reserved.

Sternal Split for Thymectomy


• Standard transsternal used by
Blalock, Keynes, and Clagett
• Partial sternotomy starts 1-2
cm below sternum to 3-4
intercostal space
• Both pleural spaces opened
and phrenics identified
• Dissection lateral to middle
• Vein taken last
• Sternum closed with wires
• Full sternotomy or Clamshell
incision for extensive tumor

Trasktek, Mayo Clinic

Transsternal Thymectomy
• Kattach et al 2005 (U.K.)
• 85 pts, f/u 4.5 yrs
• 17% complete remission
• 79% improvement, 74% asymptomatic or stage I
• Greater severity of sxs associated with greater improvement
• No deaths
• 85% extubated in 1st 24 hrs
• ICU 1 day, LOS 8 days

Kattach, et al Ann Thor Surg 2006;81:305-8

237
Copyright © Oakstone Publishing, 2021. All Rights Reserved.

VATS Thymectomy
• Supine with single lumen ETT
• Start on left anterior to
phrenic nerve
• Dissect superiorly off of
sternum
• Left horn dissected off
pericardium and innominate
vein
• Dissect toward right side

238
Copyright © Oakstone Publishing, 2021. All Rights Reserved.

239
Copyright © Oakstone Publishing, 2021. All Rights Reserved.

VATS Thymectomy

240
Copyright © Oakstone Publishing, 2021. All Rights Reserved.

Case
• 50 yo male presents with cough
• He notes that he is fatigued easily.
• Notes he has lost about 15 lbs in the past 2 months.

241
Copyright © Oakstone Publishing, 2021. All Rights Reserved.

Thymoma

242
Copyright © Oakstone Publishing, 2021. All Rights Reserved.

Thymoma
• 1/3 of patients asymptomatic
• Chest pain, cough, dyspnea most common
• SVC syndrome and weight loss occasional
• Parathymic syndromes
• Myasthenia gravis approx 45%
• 10-15% of patients with MG have a thymoma
• Red cell aplasia, hypoglammaglobulinemia
• Higher than expected incidence of second primary malignancy

Thymomas
Masaoka’s Staging System

Stage Characteristics
I No microscopic invasion
Macroscopic encapsulation
II Microscopic invasion into capsule
Pericapsular gross invasion into pleura or
mediastinal fat
III Macroscopic invasion into pericardium,
great vessels or lungs
IVA Pleural / Pericardial dissemination
IVB Lymphatic or hematogenous metastasis

243
Copyright © Oakstone Publishing, 2021. All Rights Reserved.

Thymoma
• M=F
• Age 1yr – 90 yr
• Peak 30-40 yo for those with MG
• Peak 60-70 yo for those without MG
• Presentation
• Stage I 40%
• Stage II 25%
• Stage III 25%
• Stage IVa 10%
• Stage IVb 1-2%

Stage I

244
Copyright © Oakstone Publishing, 2021. All Rights Reserved.

Stage III

245
Copyright © Oakstone Publishing, 2021. All Rights Reserved.

Stage IV

Principles of Surgical Resection


• Exploration of the entire mediastinum is mandatory
• Complete thymectomy (including upper,cervical poles)
• Avoidance of intrapleural dissection in Stage I and II
tumors may be of value in preventing pleural seeding
• Pleural implants discovered at exploration should be
resected if possible
• COMPLETE Resection
• Extracapsular
• Extended en bloc to include:
• Pleura, pericardium, lung
• Innominate vein, superior vena cava

246
Copyright © Oakstone Publishing, 2021. All Rights Reserved.

Hemiclamshell

Malignant Thymoma
Tumor SVC
Aorta
SVC

Aorta
Tumor

R Lung

247
Copyright © Oakstone Publishing, 2021. All Rights Reserved.

248
Copyright © Oakstone Publishing, 2021. All Rights Reserved.

Patterns of Recurrence
• Average rate of recurrence by stage
• I 4%
• II 14%
• III 26%
• IV 46%
• Average time to recurrence 5 yrs (reports up to 32 yrs)
• Approximately 80% recurrences are local, 20% are distant
• Pleura/lung 58%,
• Mediastinum/pericardium 41%,
• Bone 10%
• Liver 8%

Detterbeck, 2004

249
Copyright © Oakstone Publishing, 2021. All Rights Reserved.

Thymoma
• Recurrence
• Should be treated with aggressive re-resection
• 62% CR in those who underwent OR
• 10yr survival
• Completely resected 53%-72%
• Incompletely resected 0% - 11%
• 2nd recurrence seen in 16-25% of patients who underwent CR of 1st
recurrence

Case
• 40 yo female presents to PCP due to persistent cough for the past 4
weeks.
• PCP noted some wheezing so gave a presumptive diagnosis of
asthma.
• Treated with bronchodilators without benefit
• Treated with steroid without benefit
• Episode of Hemoptysis

250
Copyright © Oakstone Publishing, 2021. All Rights Reserved.

251
Copyright © Oakstone Publishing, 2021. All Rights Reserved.

Tracheal Tumors
• Primary tracheal tumors are rare.
• 1/3 squamous cell ca
• 1/3 adenoid cystic ca
• 1/3 other ( carcinoid, adeno ca, small cell )
• Cough most common presenting symptom
• Wheezing (inspiratory) and stridor prominent when narrowing
becomes more profound
• Hemoptysis common in 25% of cases

252
Copyright © Oakstone Publishing, 2021. All Rights Reserved.

• Trachea about 11cm in length


• 18 to 22 rings
• Blood supply from inferior thyroid arteries superiorly and bronchial
arteries inferiorly
• Supply in segmental and insert laterally
• Approach
• Goals

253
Copyright © Oakstone Publishing, 2021. All Rights Reserved.

254
Copyright © Oakstone Publishing, 2021. All Rights Reserved.

255
Copyright © Oakstone Publishing, 2021. All Rights Reserved.

256
Copyright © Oakstone Publishing, 2021. All Rights Reserved.

Case
• 20 yo male involved in a high speed MVA
• Unrestrained
• Pulse 110, BP 80/50, O2 sat 93% on FM
• Unresponsive

257
Copyright © Oakstone Publishing, 2021. All Rights Reserved.

Lethal six injuries:

• Airway obstruction
• Tension pneumothorax
• Pericardial tamponade
• Open pneumothorax
• Massive hemothorax
• Flail chest

258
Copyright © Oakstone Publishing, 2021. All Rights Reserved.

Pneumothorax
• Simple, open, tension

• Air enters pleural space


• Open chest wound
• Laceration of parenchyma
• Rupture of tear of alveoli

• Initial trauma CXR may miss


• Pitfall- treatment based on estimate of “down” lung

259
Copyright © Oakstone Publishing, 2021. All Rights Reserved.

Pneumothorax

Tension pneumothorax
• Air enters pleural space and cannot escape
• P/C: chest pain, dyspnea
• Dx: - respiratory distress
• tracheal deviation (away)
• absence of breath sounds
• distended neck veins
• hypotension

260
Copyright © Oakstone Publishing, 2021. All Rights Reserved.

Tension Pneumothorax

Air between lung


and chest wall

Air collapses lung


and pushes heart to
other side

Blood return to heart


restricted by kinked vessels,
heart unable to pump

Tension Pneumothorax

• Acute life threatening emergency


• Needle decompression affected side
• 2nd intercostal space mid-clavicular line -OR-
• 4th or 5th intercostal space at mid-axillary line
• Place above rib to avoid neurovascular bundle

261
Copyright © Oakstone Publishing, 2021. All Rights Reserved.

NEEDLE CHEST DECOMPRESSION

• Blood pressure improves initially but begins to fall again.


• HR 120, BP 75/45, o2 sat 95% of FM

262
Copyright © Oakstone Publishing, 2021. All Rights Reserved.

CT Insertion
• Mid-axillary line, 5-6 ICS
• Use adequate local
• 20-24 Fr. for pneumothorax
• 34-38 Fr. for hemothorax
• Not through open wounds!
• Suture in place
• Occlusive dressing

263
Copyright © Oakstone Publishing, 2021. All Rights Reserved.

• Drain 700cc of blood

• Continued hypotension and tachycardia

264
Copyright © Oakstone Publishing, 2021. All Rights Reserved.

Hemothorax

 Most frequently from lung injury.


– Upright film: 200-300 mL of blood.
– Supine film: >1000mL of blood.
– Haziness in affected hemithorax when supine
 Treat: Chest tube

Thoracic Injury
• Blunt cardiac injury
• Contusion
• Ventricular, septal or valvular rupture

• Cardiac tamponade
• Ruptured thoracic aorta
• Diaphragmatic rupture

265
Copyright © Oakstone Publishing, 2021. All Rights Reserved.

Pericardial Tamponade
• Blood in the pericardial sac
• Most frequently penetrating injuries
• Shock, ↑JVP, PEA, pulsus paradoxus
• Classically, Beck’s triad:
• distended neck veins
• muffled heart sounds
• hypotension

• Rx:
• Volume resuscitation
• Pericardiocentesis

266
Copyright © Oakstone Publishing, 2021. All Rights Reserved.

DO Not Do This

Indications for Thoracotomy


• Initial thoracostomy tube drainage is >20 mL/kg of blood
• Persistent bleeding at a rate >7 mL/kg/hr
• Increasing hemothorax seen on chest x-rays
• Patient remains hypotensive despite adequate blood replacement,
and other sites of blood loss have been ruled out
• Patient decompensates after initial response to resuscitation
• Penetrating chest/epigastric trauma associated with cardiac arrest
(any rhythm).

267
Copyright © Oakstone Publishing, 2021. All Rights Reserved.

Contraindications

• Definite loss of cardiac output for greater than 10 minutes.

• Any patient who has a cardiac output, including hypotensive patients.

• Blunt trauma.

ED Thoracotomy
• Indications
• profound shock without
response to fluids
• agonal or pending arrest upon
arrival
• loss of vital signs in transit
• not indicated for patients
without signs of life in the field
• overall survival 5% (review of 638
EDT) ; when vss, 32% stab, 15%gsw,
5% bct

268
Copyright © Oakstone Publishing, 2021. All Rights Reserved.

ED Thoracotomy (Anteriorlateral thoracotomy)

269
Copyright © Oakstone Publishing, 2021. All Rights Reserved.

270
Copyright © Oakstone Publishing, 2021. All Rights Reserved.

• Access following a left anterolateral thoracotomy is fairly limited. The


clamshell is made by performing a right sided thoracotomy in the
same interspace:

EMERGENCY "CLAM SHELL" THORACOTOMY

271
Copyright © Oakstone Publishing, 2021. All Rights Reserved.

• The clamshell thoracotomy usually starts as a standard left


anterolateral thoracotomy - often an emergency department
thoracotomy for traumatic arrest as in this case. The left thoracotomy
is placed in the 5th intercostal space (just below the nipple).

272
Copyright © Oakstone Publishing, 2021. All Rights Reserved.

• Dividing the sternum will also divide the inferior mammary arteries on both sides.
Usually these do not bleed at this stage due to profound hypotension, but will
start to bleed once blood volume and flow is restored. These will need to be
ligated at some point in the future.
• The rib retractor is placed between the cut ends of the sternum and opened. The
fibrofatty tissue between the sternum and the anterior pericardium should be
divided with scissors.

273
Copyright © Oakstone Publishing, 2021. All Rights Reserved.

• This allows the sternum to be fully elevated and provides excellent


exposure to the heart and both thoracic cavities.

274
Copyright © Oakstone Publishing, 2021. All Rights Reserved.

275
Copyright © Oakstone Publishing, 2021. All Rights Reserved.

• Closing the incision


• Don't forget to check the inferior mammary arteries and ligate both
ends if you haven't done so already.
• Large bore chest drains should be placed in both thoracic cavities, the
mediastinum and pericardium (if opened).

276
Copyright © Oakstone Publishing, 2021. All Rights Reserved.

Esophagus
• Recurrent Laryngeal Nerve
runs in tracheo-esophageal
groove
• Left is closer to esophagus
• Thoracic Duct
• Comes in through diaphragm
behind the aorta, runs dorsal
to esophagus, from 5th thoracic
vertebra up it passes left into
the neck to join L
subclavian/internal jugular
vein junction

Innervation
• RLN service cricopharyngeus and cervical esophagus
• Injury causes vocal cord paralysis and dysfunction of the
cricopharyngeus and cervical esophageal motility
• Aspiration

277
Copyright © Oakstone Publishing, 2021. All Rights Reserved.

Lymphatic Drainage
• Lymph capillaries 40-60microns
drain into collecting channels
100-200 microns that continue
through esophageal muscle
parallel to the long axis of
esophagus
• Proximal third drains into deep
cervical LN and then to thoracic
duct
• Middle third drains into superior
and posterior mediastinal nodes.
• Distal third drain into left gastric
and celiac nodes
• Bidirectional flow

Chyle Leak
• Disruption of thoracic duct or it’s branches and results from
dissection around esophagus or airways
• Results in loss of volume, nutrition, wbc
• Can have volume loss of >2000cc/dy
• Output is “creamy”
• Check triglycerides (chylomicrons), >200
• Less then 500cc/dy can be treated conservatively with TPN, NPO
• IR thoracic duct embolization effective 70%
• Operative duct ligation

278
Copyright © Oakstone Publishing, 2021. All Rights Reserved.

Thank You
Jon O. Wee, MD
[email protected]

279
Copyright © Oakstone Publishing, 2021. All Rights Reserved.

Surgical Critical Care


1. Necrotizing Pancreatitis
2. Necrotizing Soft Tissue Infections
3. Diverticulitis

About Me
Reza Afrasiabi, MD

• Board Certified General Surgeon

• Acute Care Surgery Fellow at Brigham and Women’s Hospital (2021-2022)

• Surgical Critical Care Fellowship at Brigham and Women’s Hospital (2020-2021)

280
Copyright © Oakstone Publishing, 2021. All Rights Reserved.

Financial Disclosures
None.

Necrotizing Pancreatitis

281
Copyright © Oakstone Publishing, 2021. All Rights Reserved.

Necrotizing Pancreatitis, Stats


• ~275K admissions per year in the US for acute pancreatitis, 5-10% of which are for
necrotizing pancreatitis

• Mortality rate:
• Sterile necrosis: 10-15%
• Infected necrosis: 20-30%
• Multiorgan failure: 40-50%

• Infected pancreatic necrosis = 30-45% of all pancreatic necrosis

Diagnosis: Contrast Enhanced CT

282
Copyright © Oakstone Publishing, 2021. All Rights Reserved.

2012 Atlanta Classification


• Necrotizing Pancreatitis
• Lack of pancreatic parenchymal enhancement by intravenous contrast agent OR
• Presence of findings of peripancreatic necrosis (acute necrotic collection or walled off
necrosis)

• Acute Necrotic Collection (ANC)


• Necrotic collection without a well defined inflammatory wall
• Typically less than 4 weeks from onset of necrotizing pancreatitis

• Walled Off Necrosis (WON)


• An encapsulated collection of pancreatic or peri-pancreatic necrosis that has
developed a well defined inflammatory wall.
• Typically more than 4 weeks since onset of necrotizing pancreatitis

Acute Necrotic Collection vs.Walled Off Necrosis

283
Copyright © Oakstone Publishing, 2021. All Rights Reserved.

Peripancreatic Fluid Collection vs. Pseudocyst

Infected Pancreatic Necrosis


• Median onset = 26 days after diagnosis of necrotizing pancreatitis

• Affects 30-45% of all necrotizing pancreatitis patients

• Thought to be the result of increased intestinal permeability and decreased immunity that
results from florid inflammatory response to necrotizing pancreatitis

• Bacteremia is a risk factor: 65% incidence of infected necrosis vs. 37.9% in those without
bacteremia
• ~40% match between organism isolated in blood culture and pancreas culture

• Direct correlation between extent of necrosis and risk of infected necrosis


• Risk particularly elevates once necrosis >30% of pancreas

284
Copyright © Oakstone Publishing, 2021. All Rights Reserved.

Diagnosis of Infected Necrotizing Pancreatitis


• 2013 American College of Gastroenterology guidelines: maintain high index of suspicion in
patients who clinically decline or fail to improve after 7-10 days of hospitalization.

• Confirm diagnosis by one of the following:


• Positive culture of pancreatic or peripancreatic necrosis obtained by FNA
• Presence of air in the collections on CECT

• FNA can be inaccurate in detecting infection:


• Sensitivity: 88%
• Specificity: 90%

Infected Pancreateic Necrosis: CT findings

285
Copyright © Oakstone Publishing, 2021. All Rights Reserved.

Decision Tree For Suspected Infected Necrotizing


Pancreatitis
• 2013 American College of Gastroenterology guidelines

Effect of antibiotic prophylaxis on acute necrotizing pancreatitis: Results


of a randomized controlled trial, J Gastro Hepatology, 2009
• Randomized control trial of 56 patients with necrotizing pancreatitis who were
randomized to one of two groups:
• A: imipenem-cilastatin 500 mg/day x 3 day (within 3 days of onset of symptoms)
• B: no antibiotics

286
Copyright © Oakstone Publishing, 2021. All Rights Reserved.

Infected vs. Sterile Necrotizing Pancreatitis


• Patients with necrotizing pancreatitis are sicker than those with non-necrotizing
pancreatitis

• Patients with infected necrotizing pancreatitis are sicker than those with sterile
necrotizing pancreatitis

Infected vs. Sterile Necrotizing Pancreatitis


• Higher mortality (24% vs. 3.5%), longer hospital stay (84.8 vs. 23.5 days) and higher
likelihood of surgical intervention (93% vs. 2%) in patients with infected pancreatic
necrosis vs. sterile pancreatic necrosis.

287
Copyright © Oakstone Publishing, 2021. All Rights Reserved.

Infected Necrotizing Pancreatitis: Microbiology


• Predominantly polymicrobial with Staph, Candida, Enterococcus, E. coli.

Treatment of Infected Necrotizing Pancreatitis


• In suspected or confirmed infected necrotizing pancreatitis, begin broad spectrum
antibiotics with good penetration into the pancreas:
• Carbapenems
• Quinolones
• Metronidazole
• 3rd/4th generation cephalosporins

• If no response to antibiotics, then complete FNA to determine microbiology and


respective antibiotic sensitivities.
• Use of FNA controversial due to fear of contamination of sterile necrosis

• Delay operative/endoscopic debridement for at least 4 weeks due to high associated


mortality/morbidity

288
Copyright © Oakstone Publishing, 2021. All Rights Reserved.

Timing of Surgical Intervention in Necrotizing Pancreatitis, Arch Surg, 2007

• Retrospective analysis of 53 patients with necrotizing pancreatitis at a single center


• Delayed surgical intervention correlated with decreased mortality for patients with
necrotizing pancreatitis

Timing of Surgical Intervention in Necrotizing Pancreatitis, Arch Surg, 2007

• Also completed meta-analysis of 11 total studies which also revealed negative correlation
between time to surgical intervention and mortality

289
Copyright © Oakstone Publishing, 2021. All Rights Reserved.

Pancreatitis Necrosectomy vs. sTEp up


approach (PANTER, 2005-2008)
• 88 patients with suspected/confirmed infected necrotizing pancreatitis randomized to one
of two groups:
• Group A: Laparotomy + necrosectomy + continuous postoperative lavage
• Group B: Step-Up Approach
• Drainage of necrotic collection followed by video-assisted retroperitoneal
debridement (if needed)

• Primary endpoint: proportion of patients with major morbidity or mortality

• Secondary endpoint: minor complications (fistula, abscess, strictures, pseudocyst)

• Multicenter study- 20 hospitals part of Dutch Acute Pancreatitis Study Group

PANTER Trial: Inclusion and Exlcusion Criteria


• Inclusion criteria:

• Pancreatic or peripancreatic necrosis detected on CECT

• Possibility of placing a drain (percutaneously or endoscopically)

• Exclusion criteria:

• Acute on chronic pancreatitis flare

• Previous drainage or surgical necrosectomy for infected pancreatic necrosis

• Abdominal compartment syndrome, perforated viscus, bleeding

290
Copyright © Oakstone Publishing, 2021. All Rights Reserved.

PANTER: Study Design

PANTER Trial: Details


• “Clinical improvement”
• Improved function in at least 2 of 3 organ systems (CV, pulmonary, renal) within 72
hours of placement of drain OR
• 10% improvement in 2 of 3 parameters: WBC, CRP, temperature

• If there is clinical improvement after drain placement, continue conservative management

• If there is no clinical improvement within 72 hours after drain placement, get CECT
• If undrained collection, reposition or upsize drain
• If no undrained collection remains, proceed to VARD

• If no clinical improvement after upsizing/repositioning drain, proceed to VARD


• If VARD unable to be completed, ex-lap with debridement is then completed

291
Copyright © Oakstone Publishing, 2021. All Rights Reserved.

PANTER Results (NEJM, 2010)

• Efficacy of Conservative Treatment, Without Necrosectomy, for Infected


Pancreatic Necrosis: A Systematic Review and Meta-analysis (2013)
• 64% who are treated with antibiotics and/or percutaneous drainage do not need
necrosectomy

• 12% mortality rate for those who are treated with abx and/or percutaneous drainage
• Compared to ~45% who undergo open necrosectomy

292
Copyright © Oakstone Publishing, 2021. All Rights Reserved.

Video Assisted Retroperitoneal Debridement (VARD)


1. Patient placed supine on OR table, with left side up

2. 5 cm incision is made adjacent to previously placed percutaneous drain

3. Digital exploration is used to follow drain into retroperitoneal collection

4. Retroperitoneal collection is opened and necrosectomy is completed with ring forceps


and suction device

5. Two 10 mm trocars are then placed through the incision and into the retroperitoneum

6. A videoscope and laparoscopic grasper are then used to remove any loosely adherent
necrotic material

Video Assisted Retroperitoneal Debridement

293
Copyright © Oakstone Publishing, 2021. All Rights Reserved.

Safety and Efficacy of Video-Assisted Retroperitoneal Debridement


for Infected Pancreatic Collections, Arch Surg, 2010.
• VARD = decreased fistula & pseudocyst formation, ICU time vs. open debridement

Minimal Access Retroperitoneal Pancreatic Necrosectomy


Improvement in Morbidity and Mortality With a Less Invasive
Approach, Annals Surgery, 2010.
• Retrospective analysis of outcomes of 189 patients with infected pancreatic necrosis or
non-resolving pancreatic necrosis undergoing VARD vs. open necrosectomy.

294
Copyright © Oakstone Publishing, 2021. All Rights Reserved.

Minimal Access Retroperitoneal Pancreatic Necrosectomy


Improvement in Morbidity and Mortality With a Less Invasive
Approach, Annals Surgery, 2010.
• Decrease in mortality, complications, organ failure and ICU requirement in VARD vs. open
necrosectomy.
• Undergoing VARD = independent predictor of reduced mortality

VARD vs. Transgastric Necrosectomy

295
Copyright © Oakstone Publishing, 2021. All Rights Reserved.

Endoscopic transgastric vs surgical necrosectomy for infected


necrotizing pancreatitis: a randomized trial, JAMA, 2012.
• Decreased major complications (fistula, organ failure), number of necrosectomies and need
for pancreatic enzymes when necrosectomy is completed via transgastric route rather than
via VARD/laparotomy.
• Less dissection and no general anesthesia required during transgastric route

Endoscopic Transgastric Drain Placement

296
Copyright © Oakstone Publishing, 2021. All Rights Reserved.

Resolution of Necrosis After Endoscopic Transgastric


Drainage

Treatment of necrotizing pancreatitis

297
Copyright © Oakstone Publishing, 2021. All Rights Reserved.

A Randomized Controlled Trial of Enteral versus Parenteral Feeding


in Patients with Predicted Severe Acute Pancreatitis Shows a
Significant Reduction in Mortality and in Infected Pancreatic
Complications with Total Enteral Nutrition, Dig Surg, 2006.
• Randomized control trial where patients with severe acute pancreatitis were randomized
to a minimum of 7 days of TPN or 7 days of enteral nutrition.

Early versus On-Demand Nasoenteric Tube Feeding in Acute


Pancreatitis, NEJM, 2014.
• Multicenter, randomized control trial comparing 208 patients with severe acute
pancreatitis (APACHE II >8) who were randomized to one of two groups:
• Early enteral tube via nasojejunal feeding (within 24h of admission)
• Oral feeding at 72h from admission

298
Copyright © Oakstone Publishing, 2021. All Rights Reserved.

Necrotizing Soft
Tissue Infections

“Flesh Eating Bacteria”

299
Copyright © Oakstone Publishing, 2021. All Rights Reserved.

First described by Hippocrates in 5th Century BC

“The course of the disease was the same to whatever part of the body it spread. Many lost
the arm and the entire forearm. If the malady settled in the sides there was rotting either
before or behind. In some cases the entire thigh was bared, or the shin and the entire foot.
But the most dangerous of all such cases were when the pubes and genital organs were
attacked.”

“Many were attacked by the erysipelas all over the body when the exciting cause was a
trivial accident...flesh, sinews, and bones fell away in large quantities...there were many
deaths.”

”Fever was sometimes present and sometimes absent. These symptoms were terrifying
rather than dangerous. “

A Spectrum of Conditions

300
Copyright © Oakstone Publishing, 2021. All Rights Reserved.

Necrotizing Soft Tissue Infections


• Spectrum of rapidly progressive soft tissue infectious processes

• Rare diagnosis: ~1000 cases annually in the United States, but incidence is increasing

• Pathognomonic finding: necrosis of subcutaneous tissue, fascia and/or muscle with


widespread undermining of skin

• Classified into three categories based on depth of skin and soft-tissue involvement:

1. Necrotizing cellulitis

2. Necrotizing fasciitis

3. Myonecrosis

Necrotizing Cellulitis
• Clostridial cellulitis:
• Infection begins at break in skin barrier (trauma, surgery)
• Fascia and deep muscle spared
• Causative organism: Clostridium perfringens

• Non-clostridial cellulitis:
• Predisposition in diabetics, immunocompromised patients
• Characterized by foul odor
• Causative organism: Mixed: E. coli, Enterobacter, Peptostreptococcus, Bacteroides fragilis

301
Copyright © Oakstone Publishing, 2021. All Rights Reserved.

Non-clostridial cellulitis in a diabetic patient

Necrotizing Fasciitis
• Type 1:
• Infection begins at break in skin/mucosal barrier from trauma/surgery or from erosion
via chronic disease (PVD, malignancy, anal fissures, etc).
• “Dish water fluid”
• Causative organism: Mixed: E. coli, Enterobacter, Clostridium, Peptostreptococcus, Prevotella,
Bacteroides fragilis

• Type II:
• Begins at site of non-penetrating minor trauma (bruise, muscle strain)
• Very high mortality (70-85%), increasingly frequent
• Causative organism: Group A Streptococci

302
Copyright © Oakstone Publishing, 2021. All Rights Reserved.

Abdominal Wall Necrotizing Fasciitis (s/p TAH)

Post-traumatic Necrotizing Fasciitis

303
Copyright © Oakstone Publishing, 2021. All Rights Reserved.

Myonecrosis
• Clostridial myonecrosis:
• Predisposing factors: penetrating trauma, bowel perforation, biliary disease, retained
placenta, missed abortion, improperly performed abortions
• Recurrent gas gangrene at site of previous gas gangrene

• Streptococcal myonecrosis
• ~50% begin at site of non-penetrating minor trauma (bruise, muscle strain)
• Increased expression of vimentin in skeletal muscle cells after injury thought to
facilitate binding of GAS
• Causative organism: Group A Streptococci

Uterine myonecrosis following C-section

304
Copyright © Oakstone Publishing, 2021. All Rights Reserved.

Fournier Gangrene: NSTI of the Perineum/Scrotum


• Coined in 1883 by Dr. Jean-Alfred Fournier, a French dermatologist who specialized in
venereal disease

Presentation
• PMHx: diabetic, immunocompromised, recent trauma/surgery

• Vitals: severe cases can present in septic shock

• Physical Exam: hemorrhagic bullae, skin necrosis, crepitus, tense edema, skin
discoloration

• Labs: leukocytosis, hyponatremia, elevated CRP

• Imaging: subcutaneous emphysema

305
Copyright © Oakstone Publishing, 2021. All Rights Reserved.

Imaging Findings

A Simple Model to Help Distinguish Necrotizing Fasciitis from


Non-necrotizing Soft Tissue Infection, J Am Coll Surg, 2000.

• Retrospective statistical analysis of vital signs, lab values, radiographic findings of 31 NSTI
patients vs. 328 non-NSTI patients.

• Objective: to find metrics that allow for successful differentiation of NSTI from non-NSTI

• Result: combination of WBC > 15.4 and Na+ < 135 provides a reliable way to
differentiate NSTI from non-NSTI
• Sensitivity: 90%
• Specificity: 76%
• NPV: 99%
• PPV: 26%

306
Copyright © Oakstone Publishing, 2021. All Rights Reserved.

Laboratory Risk Indicator for Necrotizing Fasciitis (LRINEC Score)


• Score developed in 2004 (Wong et al) to help differentiate NSTI from other soft tissue
infections
• Score >= 6: 50-75% probability of NSTI
• Score >= 8: >75% probability of NSTI

Necrotizing Soft Tissue Infection: Diagnostic Accuracy of Physical


Examination, Imaging, and LRINEC Score, Annals of Surgery, 2019.

• Meta-analysis of 23 studies (n= 5982) to determine accuracy of various diagnostic


markers for NSTI.

307
Copyright © Oakstone Publishing, 2021. All Rights Reserved.

Treatment
• Emergent surgical debridement
• Often with multiple takebacks to the OR

• Broad spectrum antibiotics


• Gram positive coverage + gram negative coverage + clindamycin
• Narrow coverage once cultures + sensitivities return

• Glucose control

• Wound care w/wo reconstruction

Extent of necrosis can be deceptive

308
Copyright © Oakstone Publishing, 2021. All Rights Reserved.

Reconstructed abdominal wall using ALT Flap

Anatomic Distribution and Risk Factors


• 70% occur in limbs (upper > lower)

• Obesity and diabetes = universal risk factors for NSTI

• #1 risk factor for cervical NSTI = glucocorticoid use

309
Copyright © Oakstone Publishing, 2021. All Rights Reserved.

Immunocompromised Status in Patients With Necrotizing Soft-


Tissue Infection, Askari R et al, Jama Surg. 2013.

• Factors positively correlated with mortality for all NSTI patients:


• Absence of bleeding during surgery (OR: 2.15)
• Age (OR: 0.069)
• INR (OR: 0.612)
• ASA (OR: 1.16)

• Factors negatively correlated with mortality for all NSTI patients:


• Appropriate antibiotic therapy (OR: -3.73)
• SBP (OR: -0.034)
• Albumin (OR: -0.549)

Immunocompromised Status in Patients With Necrotizing Soft-


Tissue Infection, Askari R et al, Jama Surg. 2013.

• Evaluation and Treatment of Immunocompromised Patients


• Less likely to be diagnosed correctly in first 12h (47.8% vs. 66.5%, p-value <0.001)
• Less likely to be admitted directly to the OR (4.3% vs. 61.3%, p-value 0.001)
• More likely to never be operated on (15.2% vs. 0.6%, p-value <0.001)

• Immunocompromised patients present with lower LRINEC scores:


• WBC (6.6 vs. 17.2, p-value <.001)
• Glucose level (124 vs. 134, p-value 0.03)
• CRP (124 vs. 159, p-value 0.12)

• Outcomes of Immunocompromised Patients with NSTI:


• More likely to die during NSTI admission (39.1% vs. 19.4%, p-value 0.01)
• More likely to be discharged to hospice (4.3% vs. 0.0%, p-value 0.01)

310
Copyright © Oakstone Publishing, 2021. All Rights Reserved.

To Divert or To Not Divert? (1/2)


• Comparison of Diverting Colostomy and Bowel Management Catheter Applications in Fournier
Gangrene Cases Requiring Fecal Diversion, Indian J Surg, 2015

• Retrospective analysis of patients with Fournier gangrene who received diverting


colostomies (n= 32) vs. bowel management catheters (n=16)

• Objective: to assess for differences in hospital LOS and costs

• Inclusion criteria: intact sphincters with no injury to rectum after debridement

• Exclusion criteria: urologic involvement

To Divert or To Not Divert? (2/2)


• BMC group with:
• Decreased total duration of hospital stay
• Decreased number of trips to OR
• Decreased total hospital costs

• Potential selection bias not addressed


• Why colostomy group with higher mortality (25% vs. 6.3%)?

311
Copyright © Oakstone Publishing, 2021. All Rights Reserved.

Diverticulitis
The American Society of Colon and Rectal Surgeons Clinical Practice
Guidelines for the Treatment of Left-Sided Colonic Diverticulitis

Background
• 1.9M+ per year diagnosed in ambulatory setting, 340k+ ED visits per year, 195K+
admissions per year,

• Median LOS per admission: 4 days

• Median cost per admission: $6333

312
Copyright © Oakstone Publishing, 2021. All Rights Reserved.

Rise of Elective Surgery for Diverticulitis


• Diverticulitis ED visits up 27%: 90 per 100K in 2006 to 114 per 100K in 2013

• However, admission rates dropping: 58% in 2006 to 47.1% in 2013

• And rate of emergency surgery falling: 7278 per 100K ED visits (2006) to 4827 per 100K
(2013)

CRP as a Marker of Complicated Diverticulitis


• CRP > 150: able to discriminate complicated from uncomplicated diverticulitis

• CRP > 150 + free-fluid on CT: significantly increased risk of mortality

• Lack of leukocytosis, elevated CRP and abdominal guarding = 96% NPV for complicated
diverticulitis

313
Copyright © Oakstone Publishing, 2021. All Rights Reserved.

Calprotectin and Procalcitonin


• Elevated calprotectin in stool is predictive of higher likelihood of diverticulitis recurrence

• Procalcitonin elevation also discriminates complicated vs. uncomplicated diverticulitis

Not All Need Antibiotics


• AVOD Trial: (Sweden) patients with uncomplicated diverticulitis randomized to IV
antibiotics vs. saline
• no difference in complications, rate of recurrence or time to recovery
• 31.3% recurrent rate at 11 years for both groups

• DIABOLO Trial (Netherlands): patients with uncomplicated diverticulitis randomized to


Augmentin x 10d vs. observation
• time to recovery: 12 days vs. 14 days (p = 0.15)
• no difference in mortality, recurrence, readmission, adverse events or need for
resection at 2 years
• Antibiotic group had higher rate of antibiotic-related adverse events (8.3% vs. 0.4%,
p=0.006)

• Important to emphasize: these patients were all Hinchey 1A and otherwise healthy

314
Copyright © Oakstone Publishing, 2021. All Rights Reserved.

Hinchey Classification

Predictors of Failure
• Hinchey 1A with CRP >170 significantly more likely to fail non-antibiotic therapy
(retrospective study of 565 patients by Bolkenstein et al in 2018)

• Patients with signs of systemic infection, with significant comorbidities, who are
immunocompromised were NOT included in non-antibiotic trials

315
Copyright © Oakstone Publishing, 2021. All Rights Reserved.

Method/Duration of Antibiotic Duration


• For patients with Hinchey 1 diverticulitis treated with antibiotics, there was no significant
clinical outcome difference in the following groups:

• Treatment x 4 days vs. Treatment x 7 days

• Outpatient treatment vs. Inpatient treatment

• PO antibiotics vs. IV antibiotics

Percutaneous Drainage
• 15-40% patients with acute diverticulitis have an abscess

• Abscesses > 3 cm are best treated with percutaneous drainage (34% failure rate with
antibiotics alone)

• Abscesses < 3 cm have a high rate of resolution with antibiotics alone


• If fail to resolve with antibiotics, then percutaneous drainage

• Abscesses without safe access window for percutaneous drainage can be drained
laparoscopically in select patients

316
Copyright © Oakstone Publishing, 2021. All Rights Reserved.

Diet
• Higher risk of diverticulitis for those who eat diets high in red meat, refined grains and
high-fat dairy vs. those who eat diets high in fruits, vegetables and whole grains.
• HR1.55 for men in highest quintile of Western diet consumption vs. those in lowest
quintile
• Based on prospective cohort study of 46K+ men

• 5 low-risk factors: normal BMI, fiber consumption > 23g/day, 2 hours of exercise per
week, red meat consumption < 51g/day, never smoker
• RR 0.27 when all 5 low-risk factors are present

Smoking
• Relative risk of developing diverticulitis:
• Current smoker: 1.36
• Forer smoker: 1.17

• Relative risk of developing abscess/perforation during episode of diverticulitis:


• Current smoker: 2.54
• Former smoker: 1.83

317
Copyright © Oakstone Publishing, 2021. All Rights Reserved.

BMI
• Relative risk associated with 5-unit increase in BMI:
• Diverticulitis: 1.31
• Abscess/perforation during diverticulitis: 1.20

Probiotics
• Mesalamine 1.6 g/day + Lactobacillus casei for 10 days per month for 12 months has been
shown to reduce the risk of recurrence of diverticulitis
• Should be interpreted with caution as study did not demonstrate evidence to quantify
burden of disease (imaging, inflammatory markers)

318
Copyright © Oakstone Publishing, 2021. All Rights Reserved.

Risk of Occult Malignancy


• Uncomplicated diverticulitis: 1.3% [0.1-2.0%]

• Complicated diverticulitis: 7.9% [3.9-15.3%]

• CT findings concerning for malignancy: abscess, obstruction, mesenteric/retroperitoneal


lymphadenopathy

• Clinical findings c/f malignancy: bleeding, narrowed stools

• All patients with complicated diverticulitis or uncomplicated diverticulitis with


imaging/clinical findings c/f malignancy should have colonoscopy 6+ weeks after episode

Abscess = Higher Rate of Recurrence


• Diverticular abscess
• HR of 2.02 in having recurrence
• HR of 4.08 for complicated recurrent attack
• Study of 180K patients in California hospitalized with diverticulitis

• Median time to recurrence = 278 days

• 5-year recurrence rate = 24.8%

319
Copyright © Oakstone Publishing, 2021. All Rights Reserved.

Risk Factors for Failure of Non-op Management


• Abscess > 5 cm: HR 2.96

• Abscess > 3 cm: HR 2.05

• History of prior episode of diverticulitis: HR 1.7

• Age: HR 1.03

Outcomes of Non-op Management of Complicated


Diverticulitis
• Retrospective review of 14k+ patients managed non-operatively:

• Readmission: 12.0% vs. 8.2%, p<0.001

• Future emergency surgery: 4.3 vs. 1.4%, p<0.001

• Overwhelming majority of diverticulitis treated non-operatively will not require


readmission or emergency surgery

320
Copyright © Oakstone Publishing, 2021. All Rights Reserved.

Elective Colectomy for Fistula, Stricture or


Obstruction
• Objective: to provide symptomatic relief

• Patients with complicated diverticulitis = significantly more likely to have a recurrence


that is complicated (HR 14.6)

Surgery Should Not be Based on Age

• Patients with first episode of diverticulitis prior to age 50 are more likely to be
readmitted than older patients (10.5% vs. 8.4%, p<0.001)

• However, recent data suggests no difference in need for emergency surgery (1.8% vs.
2.0%, p=0.52)
• Based on retrospective cohort study with median follow-up of 3.9 years

• Currently, 18.1% of younger patients (< 50) undergo elective colectomy after diagnosis od
diverticulitis vs. 8.5% of older patients (> 50).

• Of note, mutation in LAMB4 gene (colonic myenteric plexus) shown to predispose to


early-onset diverticulitis)

321
Copyright © Oakstone Publishing, 2021. All Rights Reserved.

Recurrence Risk Increases With Each Episode

• After 1st admission for diverticulitis, 8.7% will have a second admission for diverticulitis

• After 2nd admission for diverticulitis, 23.0% will have a third admission for diverticulitis

• After 3rd admission for diverticulitis, 36.0% will have a fourth admission for diverticulitis

Quality of Life Improvement After Sigmoidectomy

• DIRECT Trial (a multi-center randomized controlled trial of patients with recurrent


diverticulitis) revealed significantly higher quality of life scores in patients who underwent
sigmoidectomy vs. non-op (5-year follow-up)

• Based on physical, mental and pain scores (GIQLI)

• Even with higher than average rates of ostomy formation (21%) and anastomotic leak
(15%).

322
Copyright © Oakstone Publishing, 2021. All Rights Reserved.

Immunocompromised Patients

• Higher overall morbidity (OR 1.46) including wound dehiscence (OR 2.69) after
sigmoidectomy

• Equivalent mortality rates after sigmoidectomy compared to non-immunocompromised

• Equivalent rate of recurrence after medical management of 1st bout of diverticulitis


(21.5% vs. 20.5%, p=0.82)
• Median follow-up time of 81.6 months

• Some studies with small series of patients have shown higher recurrence rates- but data
should be interpreted with caution

Diverticulitis + Peritonitis:To Hartmann or Not?

• RCT completed by Binda et al over 9 years: 34 patients randomized to primary


anastomosis with DLI, 56 patients randomized to Hartmann procedure
• No difference in mortality (2.9% vs. 10.7%, p=0.24)
• No difference in morbidity (35.3% vs. 46.4%, p=0.38)
• No difference in stoma reversal (64.7% vs. 60%, p=0.65)

• Significantly higher morbidity with Hartmann reversal vs. DLI reversal (23.5% vs. 4.5%,
p=0.05)

323
Copyright © Oakstone Publishing, 2021. All Rights Reserved.

ColonPerfRCT & DIVERTI

• Randomized Controlled Trials


• Patients with Hinchey III/IV diverticulitis were randomized to primary anastomosis +
DLI or Hartmann procedure
• ColonPerfRCT (n=62), DIVERTI (n =102)

• Stoma-free rates significantly higher in anastomosis + DLI group:


• ColonPerfRCT: 90% vs. 57%, p=0.005
• DIVERTI: 96% vs. 65%, p=0.0001

• ColonPerfRCT terminated early due to significantly more serious complications in


Hartmann reversal group vs. DLI reversal group (20% vs. 0%)

Meta-Analyses of Hartmann vs. DLI studies

• No difference in mortality, OR 0.44 [0.14-1.34]

• No difference in wound infection, OR 0.75 [0.20-2.78]

• No difference in major complications, RR 0.88 [ 0.49-1.55]

• Higher likelihood to be stoma free with DLI, RR 1.40 [1.18-1.67]

• Fewer complications with DLI reversal vs. Hartmann reversal, RR 0.23 [0.07-0.70]

324
Copyright © Oakstone Publishing, 2021. All Rights Reserved.

Adjusting for Confounders

• Per NSQIP, patients who undergo Hartmann procedure are sicker than DLI patients:
• Septic shock: 11.1% vs. 5.3%, p=0.01
• COPD: 9.8% vs. 4.8%, p=0.01
• Mortality: 7.6% vs. 2.9%, p=0.01

• When these cofounders are adjusted for, there still is no postoperative morbidity
difference between those who undergo Hartmann vs. DLI, OR 0.96 [0.63-1.45]

• Of note, trend towards increased mortality/morbidity when DLI not used in setting of
primary anastomosis

Primary Anastomosis w/DLI Not Being Adopted

• Despite the data, only 3.9% of emergency surgeries for diverticulitis result in primary
anastomosis with DLI

• WHY?

325
Copyright © Oakstone Publishing, 2021. All Rights Reserved.

Lower Mortality with Colorectal Surgeons

• In setting of emergency surgery for diverticulitis, completion of primary anastomosis by a


board-certified colorectal surgeon results in lower post-operative mortality

• 7.4% vs. 15.0%, p<0.001

• Based on 10.6K+ patients who underwent urgent/emergent surgery for diverticulitis

Laparoscopic Lavage vs. Sigmoidectomy For


Purulent Peritonitis
• LOLA Trial:
• higher surgical reintervention (20% vs. 7%, p=0.12)
• higher rate of abscesses requiring drainage (20% vs. 0%, p=0.002)
• equivalent composite morbidity and mortality (67% vs. 60%, p=0.58)

• SCANDIV Trial:
• higher rate of deep surgical site infection (32% vs. 13%, p =0.006)
• higher rate of reoperation (27% vs. 10%, p =0.01)

• DILALA Trial:
• No difference in 30 day reoperation rate vs. Hartmann (13.2% vs. 17.1%, p=0.67)
• 45% risk reduction in reoperation for lavage vs. Hartmann

326
Copyright © Oakstone Publishing, 2021. All Rights Reserved.

Laparoscopic Lavage Meta-Analysis


• Increased risk of intra-abdominal abscess formation, peritonitis and future emergency
surgery reoperation

• Increased risk of sigmoid adenocarcinoma, feculent peritonitis

• 36% require sigmoidectomy within 1 year

• Lower rate of stoma reversal at 1 year (48% of 74%)

• Verdict: strong recommendation for colectomy vs. laparoscopic lavage for


purulent peritonitis

Technical Considerations
• Proximal margin: non-inflamed colon, do not need to resect all diverticula

• Distal margin: rectum

• Mobilize splenic flexure and ligate IMV as needed

• Decreased anastomotic leak with preservation of IMA?

327
Copyright © Oakstone Publishing, 2021. All Rights Reserved.

Minimally Invasive Approach


• Laparoscopic Hartmann in Emergency Surgery: lower morbidity and equivalent mortality
compared to open Hartmann

• Robotic sigmoidectomy: lower rate of conversion to open vs. laparoscopy (9.5% vs. 13.7%,
p=0.008)

• Robotic sigmoidectomy: fewer post-op complications, shorter LOS and less ileus vs.
laparoscopy (p < 0.05, even when propensity score matched)

References
1. Besselink MG, van Santvoort HC, Boermeester MA, et al. Timing and impact of infections in acute pancreatitis.
Br J Surg. 2009;96:267–73.

2. Dellinger EP, Tollado JM, Soto NE, Ashley SW, Barie PS, Dugernier T, et al. Early antibiotic treatment for severe
acute necrotizing pancreatitis: a randomized, double-blind, placebo-controlled study. Ann Surg. 2007;245:674–83.

3. Beger HG, Bittner R, Block S, Buchler M. Bacterial contamination of pancreatic necrosis a perspective clinical
study. Gastroenterology. 1986;91:433–8.

4. Rau BM, Bothe A, Kron M, Beger HG. Role of early multisystem organ failure as major risk factor for pan-
creatic infections and death in severe acute pancreati- tis. Clin Gastroenterol Hepatol. 2006;4:1053–61.

5. van Brunschot S, Bakker OJ, Besselink MG, Bollen TL, Fockens P, Gooszen HG, et al. Treatment of necrotizing
pancreatitis. Clin Gastroenterol Hepatol. 2012;10:1190–201.

6. Runzi M, Niebel W, Goebell H, Gerken G, Layer P. Severe acute pancreatitis: nonsurgical treatment of infected
necroses. Pancreas. 2005;30:195–9.

7. van Goor H, Sluiter WJ, Bleichrodt RP. Early and long term results of necrosectomy and planned re- exploration
for infected necrosis. Eur J Surg. 1997; 163:611–8.

8. Connor S, Alexakis N, Raraty MG, Ghaneh P, Evans J, Hughes M, et al. Early and late complications after
pancreatic necrosectomy. Surgery. 2005;137: 499–505.

328
Copyright © Oakstone Publishing, 2021. All Rights Reserved.

References
9. Uhl W, Warshaw A, Imrie C, Bassi C, McKay CJ, Lankisch PG, et al. IAP guidelines for the surgical management of
acute pancreatitis. Pancreatology. 2002;2:565–73.

10. Olah A, Padavi G, Belagyi T, Nagy A, Issekutz A, Mohamed GE. Early nasojejunal feeding in acute pancreatitis is
associated with a lower complication rate. Nutrition. 2002;18:259–62.

11. Petrov MS, Kukosh MV, Emelyanov NV. A random- ized controlled trial of enteral versus parenteral feed- ing in
patients with predicted severe acute pancreatitis shows a significant reduction in mortality and in infected
pancreatic complications with total enteral nutrition. Dig Surg. 2006;23:336–45.

12. Al-OmranM,AlbalawiZH,TashkandiMF,Al-Ansary LA. Enteral versus parenteral nutrition for acute pancreatitis.
Cochrane Database Syst Rev 2010;1: CD002837.

13. Hartwig W, Maksan S-M, Foitzik T, Schmidt J, Herfarth C, Klar E. Reduction in mortality with delayed surgical
therapy of severe pancreatitis. J Gastrointest Surg. 2002;6:481–7.

14. Bakker OJ, van Santvoort HC, van Brunschot S, Geskus RB, Bollen TL, van Eijck CH, et al. Endoscopic
transgastric vs surgical necrosectomy for infected necrotizing pancreatitis: a randomized trial. JAMA.
2012;307:1053–61.

15. Ross A, Gluck M, Irani S, Hauptmann E, Fotoohi M, Siegal J, et al. Combines endoscopic and percutane- ous
drainage of organized pancreatic necrosis. Gastrointest Endosc. 2010;71:79–84.

References
16. Necrotizing skin and soft-tissue infections in the intensive care unit. M. Peetermans et al. Clinical Microbiology and Infection, 26 (2020), 8-17.

17. Necrotizing soft tissue infections – a multicentre, prospective observational study (INFECT): protocol and statistical analysis plan, M. B.
Madsen et al, Acta Anaesthesiologica Scandinavica 62 (2018) 272–279.

18. Immunocompromised Status in Patients With Necrotizing Soft-Tissue Infection, Askari R et al, JAMA Surg. 2013;148(5):419-426

19. Necrotizing Soft-Tissue Infection: Diagnosis and Management, Anaya A and Dellinger E, Clinical Infectious Diseases 2007; 44:705–10

20. Necrotizing soft tissue infections: Review and current concepts in treatment, systems of care, and outcomes, Evans et al, Curr Probl Surg.
2014 August ; 51(8): 344–362.

21. Necrotizing Soft Tissue Infection: Diagnostic Accuracy of Physical Examination, Imaging, and LRINEC Score, Perry J et al, Annals of Surgery
Volume 269, Number 1, January 2019.

22. Evaluation and Management of Necrotizing Soft Tissue Infections, Kadri S and Boone S, Infect Dis Clin North Am. 2017 September ; 31(3):
497–511.

23. Comparison of Diverting Colostomy and Bowel Management Catheter Applications in Fournier Gangrene Cases Requiring Fecal Diversion,
Seydaoglu H et al, Indian J Surg, December 2015, 77:S438–S441.

24. Necrotizing Soft-Tissue Infections, Bryant A et al, N Engl J Med 377;23, December 7, 2017.

25. A Simple Model to Help Distinguish Necrotizing Fasciitis from Non-necrotizing Soft Tissue Infection, Wall et al, J Am Coll Surg, 191:3,
September 2000.

26. The LRINEC (laboratory risk indicator for necrotising fasciitis) score: a tool for distinguishing necrotising fasciitis from other soft-tissue
infections. Wong et al, Crit Care Med. 2004; 32(7):1535–1541.

329
Copyright © Oakstone Publishing, 2021. All Rights Reserved.

Ethics
Zara Cooper, MD, MSc., FACS
Kessler Director, Center for Surgery and Public Health
Director, Center for Geriatric Surgery
Brigham and Women’s Hospital

COMPREHENSIVE REVIEW OF GENERAL SURGERY


DIRECTE D BY PA RDON R. KE NNE Y, MD, MMsc, FACS

Disclosures
None

330
Copyright © Oakstone Publishing, 2021. All Rights Reserved.

Ethics: Overview
Informed Consent
Advance Directives
Topics DNR
Power of Attorney and Surrogacy
Frailty
Palliative Care in Surgery

Ethics: Overview

331
Copyright © Oakstone Publishing, 2021. All Rights Reserved.

4 Pillars of Bioethics
• In 1979, Tom Beauchamp and James Childress first published Principles of
Biomedical Ethics, now in its eighth edition (2019),1 outlining these important
moral principles to be used as guidelines when resolving ethical issues in
clinical medicine

Autonomy Beneficence Nonmaleficence Justice

1. Beauchamp TL, Childress JF. Principles of Biomedical Ethics. 2009. New York: Oxford University Press.

Autonomy
• Each patient has a fundamental right to control their body and to be
protected from unwanted intrusion, even life-saving medical care
• The patient must be included in all decisions
• A valid decision requires the following:
◦ Accurate information
◦ Mental capacity
◦ Freedom from coercion

• Surrogates: “substituted judgment”


◦ What would the patient have wanted?
◦ What is in the patient’s overall best interest?

332
Copyright © Oakstone Publishing, 2021. All Rights Reserved.

“The complete, unavoidable, albeit temporary transfer of


autonomy to the physician inherent in surgical therapy
makes it imperative that surgeons fully appreciate moral
obligations implicit in the surgeon-patient relationship.”
- David A. Axelrod, MD, MBA and Susan Dorr Goold, MD, MHSA, MA

Axelrod DA, Goold SD. Maintaining Trust in the Surgeon-Patient Relationship: Challenges for the New Millennium. Arch Surg. 2000;135(1):55–61.
doi:10.1001/archsurg.135.1.55

Beneficence
• Relieving, lessening, or preventing harm; providing benefits; balancing
benefits against risks and costs
• Health care providers have a duty to be of a benefit to the patient and take
positive steps to prevent and to remove harm from the patient as well as
prevent or treat pain and suffering

333
Copyright © Oakstone Publishing, 2021. All Rights Reserved.

Nonmaleficence
• Avoiding the causation of harm; “do no harm”
• Requires that we not intentionally create harm or injury to the patient, either
through acts of commission or omission

Justice
• Act fairly towards all patients
• Resolve dilemmas using fair and proportional means
• Distribute benefits, risks, and costs equitably

334
Copyright © Oakstone Publishing, 2021. All Rights Reserved.

Informed Consent
Complete informed consent includes all of the following:
• The patient has capacity
• The patient makes choice freely
• The patient is given the information necessary to make an informed choice; this includes:
◦ Necessity of/indication
◦ Nature of the proposed intervention
◦ Urgency
◦ Expected benefits of the intervention
◦ Risks associated with the intervention
◦ Alternative treatment options
◦ The natural course of the process for which the intervention is proposed if no intervention takes place
• The patient understands
• The patient agrees
• Document

Best Case/Worst Case framework


• Decision-making and communication in the
acute inpatient setting: shift focus from an
isolated surgical problem  discussion
about treatment alternatives and outcomes
• For surgeons: promote shared decision-
making and clarify outcomes
• For patients and families: promote
comparison of treatment outcomes, share
what outcomes matter to them

Taylor LJ, Nabozny MJ, Steffens NM, et al. A Framework to Improve Surgeon Communication in High-Stakes Surgical Decisions: Best Case/Worst Case.
JAMA Surg. 2017;152(6):531-538. doi:10.1001/jamasurg.2016.5674

335
Copyright © Oakstone Publishing, 2021. All Rights Reserved.

Informed consent: clinical vignette


76 year old woman comes in with 3 weeks of RUQ pain and
dyspepsia, and one day of fever.
She has a positive Murphy’s sign; US shows GB thickening, fluid
and a stone impacted at the neck; She has WBC of 15. Her
daughter begs her to have surgery.
How do you approach her?
• Capacity?
• Free will?
• Alternatives?
• Complications?

Advance Directives

336
Copyright © Oakstone Publishing, 2021. All Rights Reserved.

History
• 1967: first advance directive is proposed by the Euthanasia Society of America
◦ Luis Kutner, human-rights lawyer, suggested that individuals should indicate in a written
document the extent to which they would consent to treatment

• 1976: California adopts the first living will statute that creates its Directive to
Physicians, commonly referred to as a living will
◦ 1986: 41 states adopt living will laws

• 1983: California becomes the first state to enact a law regarding use of durable
powers of attorney specifically for healthcare statutes
• 1990: Patient Self-Determination Act enacted
• 1998: every state has a version of a healthcare power of attorney statute

Sabatino CP. The evolution of health care advance planning law and policy. Milbank Q. 2010;88(2):211-239. doi:10.1111/j.1468-0009.2010.00596.x

Content
• Patients may be unable to speak for
themselves when decisions to limit
treatment are considered
• Advance directives preemptively identify
certain desires and values that a patient
may have around specific issues of death
and dying. A means for patients to
indicate their wishes about the types of
interventions they expect, and which
individuals should make decisions for
them
◦ A living will is a type of advance directive

337
Copyright © Oakstone Publishing, 2021. All Rights Reserved.

• Yadav et al.: Only 37% of patients have completed advance directives,


including 29% with living wills. Healthy adults: 33%; patients with chronic
illnesses: 38%
◦ These are often poorly documented, ambiguous, inconsistent or differ from the actual
scenario
◦ No impact on hospital care or cost
• Unfortunately, advance directives do not mean that a patient’s wishes will be
followed more closely
• Barriers
◦ Not enough patients have them
◦ Too broad and too vague
◦ Difficult to find
◦ Have not discussed with HCP

Yadav KN, Gabler NB, Cooney E, et al. Approximately One In Three US Adults Completes Any Type Of Advance Directive For End-Of-Life Care. Health Aff
(Millwood). 2017;36(7):1244-1251. doi:10.1377/hlthaff.2017.0175

Law
• Legally binding document
• Specifies what medical treatment a patient does and does not want; goes into
effect when a patient is incapacitated and unable to make medical decisions
• Specifies a Health Care Proxy (HCP)
• A lawyer can help but is not required
• Some states require advance directives to be witnessed or require patient’s
signature to be notarized

https://www.nia.nih.gov/health/advance-care-planning-health-care-directives

338
Copyright © Oakstone Publishing, 2021. All Rights Reserved.

Practical Applications
MOLST/POLST forms
• In 1995, Oregon experimented with a Physician
Orders for Life-Sustaining Treatment protocol, or
POLST, which targeted seriously chronically ill
patients.1 Subsequently adopted by other states in
various versions
1. Requires a discussion between the treating health care
practitioner and the patient (or the patient's authorized
surrogate), about key end-of-life care treatment options with
the objective of discerning the wishes of the patient
2. Patient’s wishes are incorporated into doctor’s orders, recorded
on a specific form, and reviewed periodically/as needed
3. Ensure POLST form travels with patient as they move from one
setting to another, promoting continuity of care decision
making Original 1995 version of the Oregon POLST form
https://oregonpolst.org/
• MOLST: Medical Orders for Life-Sustaining Treatment

1. Tolle SW, Tilden VP, Nelson CA, Dunn PM. J Am Geriatr Soc. 1998 Sep; 46(9):1097-102.

How to start the conversation:


• I’d like to talk with you about possible health care decisions in the future. This
is something I do with all my patients so I can be sure that I know and can
follow your wishes. Have you ever completed an Advance Directive?
• What do you understand about your health situation?
• If you were unable to make your own medical decisions, who would like to
make them for you? Have you spoken to this person? Have you discussed your
wishes with your family?

339
Copyright © Oakstone Publishing, 2021. All Rights Reserved.

Do not resuscitate (DNR)

History
• 1960: cardiopulmonary resuscitation (CPR) is shown to be an effective
emergency procedure to resuscitate patients undergoing cardiac arrest;
performed under the presumption of patient consent
• Mid 1970s: hospitals begin implementing policies around writing DNR orders
• 1983: The President's Commission for the Study of Ethical Problems in
Medicine supports protocols for DNR orders based on self-determination,
well-being, and equity1
• 1995: SUPPORT Study2 Investigators found that half of hospitalized, seriously
ill patients who did not want CPR, did not have a written DNR order; 46% of
DNR orders were written within 2 days of death

1. Yuen JK, Reid MC, Fetters MD. Hospital do-not-resuscitate orders: why they have failed and how to fix them. J Gen Intern Med. 2011;26(7):791-797. doi:10.1007/s11606-011-1632-x
2. A controlled trial to improve care for seriously ill hospitalized patients. The study to understand prognoses and preferences for outcomes and risks of treatments (SUPPORT). The SUPPORT Principal
Investigators [published correction appears in JAMA 1996 Apr 24;275(16):1232]. JAMA. 1995;274(20):1591-1598.

340
Copyright © Oakstone Publishing, 2021. All Rights Reserved.

Content
• DNR order only applies to CPR
• It does not instruct other aspects of end-of-life care:
◦ Vasopressors
◦ Intubation
◦ Pain control
◦ Spiritual advisement
• Sulmasy et al. found overall quality of care plans for these patients was fairly
low
◦ Lack of explicit delineation of limits on other life-sustaining treatments
• DNR should not equal abandonment

Sulmasy DP, Sood JR, Ury WA. The Quality of Care Plans for Patients With Do-Not-Resuscitate Orders. Arch Intern Med. 2004;164(14):1573–1578.
doi:10.1001/archinte.164.14.1573

Legal Issues: DNR orders and medical


futility
• Medical futility refers to a physician’s determination that therapy will be of no
benefit to a patient and should not be prescribed
◦ Physicians use various methods to make these determinations and may not always arrive at
the same conclusion

• Futility is ill defined, yet providing futile treatment is ethically undesirable


• Case law in the United States does not provide clear guidance; state legislatures have
addressed it more directly
• A procedural approach can be effective in resolving disputes about DNR orders based
on medical futility; these policies should include clear communication with all parties
(patient, surrogate decision maker), consultations from medical experts and ethics
advisors, and documentation in the medical record

Cantor MD, Braddock III CH, Derse AR, et al. Do-Not-Resuscitate Orders and Medical Futility. Arch Intern Med. 2003;163(22):2689–2694.
doi:10.1001/archinte.163.22.2689

341
Copyright © Oakstone Publishing, 2021. All Rights Reserved.

Goals of care in DNR conversations


• Key components of the discussion:
◦ Check yourself
◦ Be well informed
◦ Understand the patient’s values related to life
sustaining treatment and goals of care
◦ Risks and benefits
◦ Summarize the discussion and answer questions

Power of Attorney and Surrogacy

342
Copyright © Oakstone Publishing, 2021. All Rights Reserved.

Definitions
• Durable Power of Attorney (DPOA): a legal document that assigns a health
care proxy/surrogate
• Surrogate: Individual assigned during that medical event if an HCP or DPOA
has not been assigned who can make medical decisions for the patient when
they are unable to do so for themselves

Legal Issues
• There is broad ethical
consensus that if a patient
lacks decisional capacity,
other persons may make
medical decisions
• What if the patient lacks
advance directives?
◦ Most states have legislation
to delineate decision-making
authority, but they vary
widely

DeMartino ES, Dudzinski DM, Doyle CK, et al. Who Decides When a Patient Can't? Statutes on Alternate Decision Makers. N Engl J Med.
2017;376(15):1478-1482. doi:10.1056/NEJMms1611497

343
Copyright © Oakstone Publishing, 2021. All Rights Reserved.

Surrogates
• Surrogates and caregivers experience
anxiety, depression, post traumatic stress
disorder (PTSD)1
• Factors affecting surrogate decision-making2:
◦ Surrogate characteristics and life circumstances
◦ Surrogates’ social networks
◦ Surrogate–patient relationships and communication
◦ Surrogate–clinician communication and relationship

• Clinician-family EOL communication:


increased proportion of family speech
significantly associated with increased
satisfaction, decreased conflict3
1. Azoulay E, Pochard F, Kentish-Barnes N, et al. Risk of post-traumatic stress symptoms in family members of intensive care unit patients. Am J Respir Crit Care Med. 2005;171(9):987-994. doi:10.1164/rccm.200409-1295OC
2. Vig EK, Starks H, Taylor JS, Hopley EK, Fryer-Edwards K. Surviving surrogate decision-making: what helps and hampers the experience of making medical decisions for others. J Gen Intern Med. 2007;22(9):1274-1279. doi:10.1007/s11606-007-0252-y
3. McDonagh JR, Elliott TB, Engelberg RA, et al. Family satisfaction with family conferences about end-of-life care in the intensive care unit: increased proportion of family speech is associated with increased satisfaction. Crit Care Med. 2004;32(7):1484-1488.
doi:10.1097/01.ccm.0000127262.16690.65

Surrogate Education

https://theconversationproject.org/wp-content/uploads/2017/03/ConversationProject-ProxyKit-English.pdf
https://www.vitaltalk.org/topics/reset-goals-of-care/

344
Copyright © Oakstone Publishing, 2021. All Rights Reserved.

Frailty

Definition of frailty
Frailty is a physiological cycle that starts
with neuroendocrine dysregulation,
leading to anorexia, under nutrition,
loss of muscle mass, reduced strength,
slower walking, decreased activity and
the cycle continues.
• Clinical result:
◦ Falls
◦ Delirium
◦ Functional impairment
◦ Physical dependence
◦ Social isolation
◦ Increased care needs

Singh M, Alexander K, Roger VL, Rihal CS, Whitson HE, Lerman A, Jahangir A, Nair KS. Frailty and its potential relevance to cardiovascular care. Mayo
Clin Proc. 2008 Oct;83(10):1146-53. doi: 10.4065/83.10.1146.

345
Copyright © Oakstone Publishing, 2021. All Rights Reserved.

Diagnosis
• Comprehensive Geriatric Assessment (CGA): an evidence-based, multidimensional
and interdisciplinary assessment of medical, psychological and functional
capabilities; aim is to develop an integrated plan for treatment and care of older
persons
• Clinical Frailty Scale (CFS)
◦ Multidisciplinary assessment of physical, psychosocial,
functional and environmental factors
◦ Score of 1 (very fit) to 9 (terminally ill)
• Modified Frailty Index (mFI)
◦ 5 or 11 items
• Surrogates of frailty measures:
◦ Gait speed
◦ Hand grip strength
◦ Timed Up and Go test

Woolford SJ, Sohan O, Dennison EM, et al. Approaches to the diagnosis and prevention of frailty. Aging Clin Exp Res. 2020;32:1629–1637. doi:10.1007/s40520-020-01559-3
Farhat JS, Velanovich V, Falvo AJ, et al. Are the frail destined to fail? Frailty index as predictor of surgical morbidity and mortality in the elderly. J Trauma Acute Care Surg. 2012;72(6):1526-1531.
doi:10.1097/TA.0b013e3182542fab

Frailty in surgery
• mFI strongly correlates with higher
rates of1:
◦ Postoperative complications (major and
wound)
◦ Readmission
◦ Unplanned reoperation
◦ Discharge to skilled care
◦ Longer hospitalization
◦ Mortality

• ACS NSQIP and AGS best practice


guidelines2 recommend formal
preoperative assessment of frailty

1. Panayi AC, Orkaby AR, Sakthivel D, et al. Impact of frailty on outcomes in surgical patients: A systematic review and meta-analysis. Am J Surg. 2019;218(2):393-400. doi:10.1016/j.amjsurg.2018.11.020
2. Chow WB, Rosenthal RA, Merkow RP, et al. Optimal preoperative assessment of the geriatric surgical patient: a best practices guideline from the American College of Surgeons National Surgical Quality
Improvement Program and the American Geriatrics Society. J Am Coll Surg. 2012;215(4):453-466. doi:10.1016/j.jamcollsurg.2012.06.017

346
Copyright © Oakstone Publishing, 2021. All Rights Reserved.

Mitigating the surgical and hospital risks


of frailty
• Prehabilitation1
◦ Exercise, nutritional interventions aimed at increasing
physiological reserve to improve post-surgical outcomes

• Delirium Prevention2
• Interventions targeting sleep deprivation, disorientation, immobility, dehydration, visual and
hearing impairment3-4
• Family member involvement5
• Early geriatrics consultation6
• Pharmacologic interventions: prophylactic antipsychotics

• Mobility
• Resistance- and balance-based physical exercises; also improves cognitive function

1. Santa Mina D, Clarke H, Ritvo P, et al. Effect of total-body prehabilitation on postoperative outcomes: a systematic review and meta-analysis. Physiotherapy. 2014;100(3):196-207. doi:10.1016/j.physio.2013.08.008
2. Brummel NE, Girard TD. Preventing delirium in the intensive care unit. Crit Care Clin. 2013;29(1):51-65. doi:10.1016/j.ccc.2012.10.007
3. Inouye SK, Bogardus ST Jr, Charpentier PA, et al. A Multicomponent Intervention to Prevent Delirium in Hospitalized Older Patients. N Engl J Med. 1999;340(9):669-676. doi:10.1056/NEJM199903043400901
4. Inouye SK, Bogardus ST Jr, Baker DI, Leo-Summers L, Cooney LM Jr. The Hospital Elder Life Program: a model of care to prevent cognitive and functional decline in older hospitalized patients. Hospital Elder Life Program. J Am Geriatr Soc. 2000;48(12):1697-1706. doi:10.1111/j.1532-5415.2000.tb03885.x
5. Martinez FT, Tobar C, Beddings CI, Vallejo G, Fuentes P. Preventing delirium in an acute hospital using a non-pharmacological intervention. Age Ageing. 2012 Sep;41(5):629–634. doi:10.1093/ageing/afs060
6. Marcantonio ER, Flacker JM, Wright RJ, Resnick NM. Reducing delirium after hip fracture: a randomized trial. J Am Geriatr Soc. 2001;49(5):516–522. doi:10.1046/j.1532-5415.2001.49108.x

Palliative Care in Surgery

347
Copyright © Oakstone Publishing, 2021. All Rights Reserved.

Definition of Palliative Care


• An interdisciplinary specialty focused on improving quality of
life for seriously ill patients and their families
• Palliative care is a surgical discipline
• Term first used in 1975 by surgeon Balfour Mount, MD
◦ Wanted surgeons to consider their impact on quality of life
◦ Goal was to anticipate, prevent, and treat the suffering patients
experienced during life threatening illness

WHO definition of Palliative Care: Palliative care is an approach that improves the quality of life
of patients and their families facing the problems associated with life-threatening illness,
through the prevention and relief of suffering by means of early identification and impeccable
assessment and treatment of pain and other problems, physical, psychosocial and spiritual.

Strand JJ, Billings JA. Integrating palliative care in the intensive care unit. J Support Oncol. 2012;10(5):180-187. doi:10.1016/j.suponc.2012.06.001

348
Copyright © Oakstone Publishing, 2021. All Rights Reserved.

If palliation is taken to apply solely to care near the time of death, or


"comfort measures only," it fails to include the life-affirming quality
of active, symptomatic efforts to relieve the pain and suffering of
individuals with chronic illness and injury. In this respect, palliative
care is required in the management of a broad range of surgical
patients and is not restricted to those at the end of life.”
American College of Surgeons, Statement of Principles of Palliative Care, 2005

As they decline, patients focus on


different outcomes

1. Expected quality of life

2. Emotional and financial costs of treatment

3. Likelihood of treatment success

4. Expected effect on longevity

Rodriguez KL, Young AJ. Perceptions of patients on the utility or futility of end-of-life treatment. J Med Ethics. 2006;32(8):444-449.
doi:10.1136/jme.2005.014118

349
Copyright © Oakstone Publishing, 2021. All Rights Reserved.

• Palliative care principles generally focus on prolonged chronic illness. But


surgical patients are different:
◦ Unanticipated nature of their illness
◦ Little time
◦ Many can’t participate
◦ Little or no relationship with the surgeon or intensivist
• Surgeons and patients are optimistic
• Surgery is often expected to be curative
• Difficult to define goals – underlying assumption is that patient is willing to
endure suffering
• Mortality is still the common endpoint

Elements of Palliative Care


1. Symptom management
2. Psychosocial support
3. Spiritual support
4. Care transitions
5. Decision support/advance care planning
6. End-of-life care
7. Bereavement support

350
Copyright © Oakstone Publishing, 2021. All Rights Reserved.

• Systematic review of 22 interventions, 8575 patients


• Outcomes:
◦ Decreased mortality (4)
◦ Improved quality of communication (3)
◦ Improved symptom management (4)
◦ Lower healthcare cost and utilization (7)

2 1 7 4 6 4 1
Surgical Decision Patient Decision EOL Decision Symptom Communication Processes of Palliative Care
Making Making Making Management Care Surgical
Education

Lilley EJ, Khan KT, Johnston FM, et al. Palliative Care Interventions for Surgical Patients: A Systematic Review. JAMA Surg. 2016;151(2):172-183.
doi:10.1001/jamasurg.2015.3625

Goals of care
Fundamental questions:
• What do you understand about your illness?
• What do you hope to achieve from this treatment?
• What are your healthcare priorities?
• What matters most to you?
• What health states are intolerable?

351
Copyright © Oakstone Publishing, 2021. All Rights Reserved.

Symptom management
• Illness summary
• Physical symptoms
• Psychological
• Spiritual
• Social/Cultural
• Communication preferences
• Decision making
• Anticipatory planning

Prognostication
• Palliative Performance Scale1
◦ Palliative Performance Scale < 80:
 3x higher odds of death
 13x higher odds of poor functional outcome
 8x higher odds of discharge to dependent care

• Surprise Question2: “Would you be


surprised if the patient did not survive in
the next 12 months, even with surgery?”
◦ Lilley EJ, et al prospective cohort study
◦ Respondants: EGS surgeons; patients: older
patients with acute surgical conditions (n=119)
◦ SENS: 81%, SPE C:51%, PPV: 52%, NPV: 82%,
ACC: 64%, Mortality: 38% at 12m
1. Anderson F, Downing GM, Hill J, Casorso L, Lerch N. Palliative performance scale (PPS): a new tool. J Palliat Care. 1996;12(1):5-11.
2. Lilley EJ, Gemunden SA, Kristo G, et al. Utility of the "Surprise" Question in Predicting Survival among Older Patients with Acute Surgical Conditions. J Palliat Med. 2017;20(4):420-423.
doi:10.1089/jpm.2016.0313

352
Copyright © Oakstone Publishing, 2021. All Rights Reserved.

Hospice
• Hospice is a formal system of
interdisciplinary care; primary
goals are to improve quality of life
and relieve suffering for the dying
in the last months of life
• Concept first developed in 1967
by Dame Cicely Saunders as a
model of care for patients dying
from advanced cancer

Kelley AS, Morrison RS. Palliative Care for the Seriously Ill. N Engl J Med. 2015;373(8):747-755. doi:10.1056/NEJMra1404684

Care of the dying patient


• For the patient:
◦ Symptom management
 Pain
 Nausea
 Constipation
 Wasting
 Dyspnea
 Anxiety

• For the family:


◦ Communication improves quality of dying and is associated with less aggressive care1
◦ Increased risk of PTSD and depression when family members do not feel they had the
right role in decision making2
◦ Lower risk of complicated grief when families find physicians comforting3

1. Wright AA, Zhang B, Ray A, et al. Associations between end-of-life discussions, patient mental health, medical care near death, and caregiver bereavement adjustment. JAMA. 2008;300(14):1665-1673. doi:10.1001/jama.300.14.1665
2. Barry LC, Kasl SV, Prigerson HG. Psychiatric disorders among bereaved persons: the role of perceived circumstances of death and preparedness for death. Am J Geriatr Psychiatry. 2002;10(4):447-457.
3. Siegel MD, Hayes E, Vanderwerker LC, et al. Psychiatric illness in the next of kin of patients who die in the intensive care unit. Criti Care Med. 2008;36(6):1722–8. doi:10.1097/CCM.0b013e318174da72.

353
Copyright © Oakstone Publishing, 2021. All Rights Reserved.

• For the family, letting go is a process


◦ Cognitive
◦ Affective
◦ Interpersonal
• Bereavement
◦ Patients and families need psychological preparation for death
 Support dignity
 Maximize patient control
 Minimize patient symptoms

• Complicated grief
◦ Feeling unprepared for a loved one’s death is the biggest predictor of complicated grief,
major depressive disorder, PTSD

• Improving end-of-life care:


1. Identify decision-maker
2. Explore patient’s values and goals
3. Provide information to the family
4. Optimize symptom management through regular
assessment
5. Offer social and spiritual support
6. Interdisciplinary family meetings

354
Copyright © Oakstone Publishing, 2021. All Rights Reserved.

Multiple Choice Question


Do not resuscitate orders:
A. Can only be changed annually
B. Must be placed if the patient’s family insists and even if the medical team does not think
they will benefit the patient
C. Should be entered based on the order from patient’s most recent hospital admission
D. Should be revisited during every hospital admission and are an opportunity to explore
patient’s health goals and priorities that can inform medical care

Multiple Choice Question – Answer and


Explanation
Do not resuscitate
Explanation: orders: when correctly implemented, support patient autonomy by
DNR orders,
allowing
A. Canpatients
only be or surrogates
changed annuallyto make informed decisions and prevent non-beneficial or
unwanted resuscitation efforts. However, many of today’s challenges with using DNR
B. Must be placed if the patient’s family insists and even if the medical team does not think
orders include: too infrequent
they will benefit the patientor delayed DNR discussions, lack of adequate information
for patients
C. Should be entered based ondecisions,
to make informed or inappropriate
the order from extrapolation
patient’s most recent by physicians
hospital admission
leading to the limiting of other treatments. This signifies a need for better
D. Should be revisited during every hospital admission and are an opportunity to explore
communication with, goals
patient’s health and involvement of, patients
and priorities that can informinmedical
their CPR
care preferences and other
goals of care discussions.

Citation: Yuen JK, Reid MC, Fetters MD. Hospital do-not-resuscitate orders: why they
have failed and how to fix them. J Gen Intern Med. 2011;26(7):791-797.
doi:10.1007/s11606-011-1632-x

355
Copyright © Oakstone Publishing, 2021. All Rights Reserved.

Organ Transplantation:
Principles of Immunosuppression
Alloimmunity
Basic Concepts of
kidney – liver – pancreas transplantation

Stefan G. Tullius, MD, Ph.D., FACS


Joseph E Murray MD Distinguished Chair in Transplant Surgery
Professor of Surgery, Harvard Medical School
Chief, Division of Transplant Surgery
Director, Transplant Surgery Research Laboratory
Brigham and Women‘s Hospital

Executive Editor, Transplantation

Boston, Ma. USA


October, 2021

No conflict of interest to declare

356
Copyright © Oakstone Publishing, 2021. All Rights Reserved.

Historical Background:

• 1902: First successful experimental kidney transplantation:


Autotransplantationen (anastomosis to carotid artery
and jugular vene (Ullmann/Wien; Carrel, Lyon)
Xenotransplantationen (Decastello/Wien)

• 1906: Xenotransplantation (pig -> human) with few hours of


function (Jaboulay/Lyon)

• 1933: First human kidney graft:


Anastomosis to femoral artery and vene using the technique
of Carrel (Voronoy/Ukraine)

• 1945: Human ‚Cadaver‘ kidney grafts


(Landsteiner, Hufnagel, Hume/Boston)

• 1950: First human kidney grafts (Paris/Boston)

• 1954: First successful kidney graft between identical twins


(Murray/Boston)

357
Copyright © Oakstone Publishing, 2021. All Rights Reserved.

Historical Background (cont.):

• 1958: First 'successful' renal allotransplantation following


whole body irradiation
(Murray, Boston/ Hamburger, Paris)

• 1967: First successful liver transplantation


(T. Starzl, Denver, Pittsburgh)

• 1967: First succesful heart transplantation


(C. Bernard, Kapstadt)

• 1981: First successful combined Heart- Lung


transplantation (Reitz, Stanford)

• 1970: Reports on successful pancreas transplantation


(Lillehei, Minneapolis)

• 1977: First successful islet transplantation

• 1985: First successful small bowel transplantation

Organ Transplants:
Treatment of Choice for End-Stage Organ Failure

358
Copyright © Oakstone Publishing, 2021. All Rights Reserved.

JAMA 1968

JAMA 1984

Brain dead diagnosis

NEJM, 2001

359
Copyright © Oakstone Publishing, 2021. All Rights Reserved.

360
Copyright © Oakstone Publishing, 2021. All Rights Reserved.

DCD Donation

Thuong, M. and others, TI 2016

DCD Donation

Controlled DCD Un-Controlled DCD

Thuong, M. and others, TI 2016

361
Copyright © Oakstone Publishing, 2021. All Rights Reserved.

Algorithm on Organ Donation

Dominguez-Gil, B, Tullius, SG, Delmonico, F. and others. Int Care Med, 2021

Paired Kidney Exchanges

362
Copyright © Oakstone Publishing, 2021. All Rights Reserved.

Paired Kidney Exchanges

5,374 transplants facilitated through NKR as of 10/27/21

Immunological examinations
prior to transplantation:

• Blood group

• HLA Typing

• Specific HLA-Antibodies

• Preformed Antibodies (Panel Reactive


Antibodies)

363
Copyright © Oakstone Publishing, 2021. All Rights Reserved.

ABO blood group compatibility for SOT

Recipient CDC and flow x-match

Mulley, WR, Nephrology, 2011

364
Copyright © Oakstone Publishing, 2021. All Rights Reserved.

Human HLA

Direct (a) and indirect (b) allorecognition

365
Copyright © Oakstone Publishing, 2021. All Rights Reserved.

CD-8 (a) and CD-4 (b) restriction to HLA I and II

Principles of Alloimmunity

Halloran PF, NEJM, 2004

366
Copyright © Oakstone Publishing, 2021. All Rights Reserved.

Allorecognition and Immunosuppression

Halloran PF, NEJM, 2004

Targets of Immunosuppression

Courtesy: Michelle Long, Harvard

Jolissaint, M.D., Stefan G. Tullius, M.D., Ph.D. in : Abdominal Organ Transplantation, Current Diagnosis and Treatment: Surgery 15th
Edition, Editor Gerald Doherty, MD

367
Copyright © Oakstone Publishing, 2021. All Rights Reserved.

Co-stimulatory molecules

Graft Rejection:
• Acute Rejection:
– Central role of T-cells
– Upregulation of IL-2, TNF, Interferon-gamma
– Morphology: Cellular Infiltrates, Edema, Necrosis

• Accelerated rejection:
– Sensitization based on existing or induced antibodies directed against
donor-specific cells:
– Prophylaxis: PRA, Crossmatch
– Morpholgy: Endothelial Accumulation of Immunglobulins, C3, Fibrin,
Infiltration of Neutrophils Granulozyten

• Hyperacute Rejection:
– Interaction between Xenoantibodies (anti-Gal Antibodies) and
Xenoendothel (Galactose-anti-1,3 Galactose) with activation of the
complement cascade
– Morphology: Intravascular coagulation,Necrosis

• Chronic graft dysfunction:


– Chronic organ dysfunction
– Etiology is multifactorial (immunological specific and unspecific
processes)
– Morpholgy: Progressive obliteration of luminal structures
(Glomerulosclerosis, Arteriosclerosis, Ductopenia)

368
Copyright © Oakstone Publishing, 2021. All Rights Reserved.

Accelerated/humoral rejection:

Courtesy: H. Rennke, MD

Acute/Cellular Rejection

Courtesy: H. Rennke, MD

369
Copyright © Oakstone Publishing, 2021. All Rights Reserved.

Chronic graft failure/Chronic graft rejection:

Courtesy: H. Rennke, MD

Consequences of unspecific immune activation on transplant


outcome

Brain Death Age


IRI

Innate Immunity

? Adaptive Immunity
Immunosuppression

Foreign Antigen
Rejection

Acute Rejection Chronic allograft dysfunction

370
Copyright © Oakstone Publishing, 2021. All Rights Reserved.

Link between innate and adaptive immunity

Infection/Injury
Unspecific injury:
•Brain death, age
•Organ procurement
Pathogen-associated molecular patterns (PAMPs)
•IRI

• Capacity to cope with


Toll cellular stress
•APC activation/migration

APC
•Increase Immunogenicity

MHC/peptide Co-stimulator
•Accelerate Immune Response
TCR CD28

Incresaed incidence of:


•DGF
•Acute rejections
•Chronic graft failure
Activation of the adaptive immune response
Adapted after: Medzhitov R, Janeway CA Jr: Science, 2002

The Dilemma of Supply and Demand in Transplantation

Tullius, SG & Rabb, H: NEJM, 2018

371
Copyright © Oakstone Publishing, 2021. All Rights Reserved.

Untapped potential: Older donors

D.K. Klassen, AJT, 2016

Organ Aging and Transplantation

Tullius, SG & Rabb, H: NEJM, 2018

372
Copyright © Oakstone Publishing, 2021. All Rights Reserved.

Improving Organ Quality:

P. Friend, and others, Nature, 2018

Improving Organ Quality

Lau, A, Tullius, SG and others, JCI, 2019

Iske, J., Elkhal, A., Tullius, S.G. et al, Nature Communications, 2020

373
Copyright © Oakstone Publishing, 2021. All Rights Reserved.

Organ Repair Centers

Untapped deceased donor potential:


Hep C positive donors

Woolley, AE, and others, NEJM, 2019

374
Copyright © Oakstone Publishing, 2021. All Rights Reserved.

Kidney Transplantation

Jolissaint, M.D., Stefan G. Tullius, M.D., Ph.D. in:


Abdominal Organ Transplantation, Current Diagnosis and Treatment: Surgery 15th Edition,
Editor Gerald Doherty, MD

Kidney Transplantation

Surgical technique:
• Iliac approach:
– Exception: pediatric transplantation,
Re-transplants

• Vessel anastomosis:
– Renal Artery:
• Common/External iliac
artery, E-S anastomosis
• Alternative: Internal iliac Art,
E-E Anastomosis
– Renal Vene:
• External Iliac Vene, E-S
anastomosis
– Position:
• Contralateral
• Urethral anastomosis:
– Technique of Lich-Gregoire
• Ureteroneocystostomy
– Alternative: Leadbetter-Politano
• Implantation of a long length
donor ureter via cystostomy
– Uretheral stent placement

Jolissaint, M.D., Stefan G. Tullius, M.D., Ph.D. in:


Abdominal Organ Transplantation, Current Diagnosis and Treatment: Surgery 15th Edition,
Editor Gerald Doherty, MD

375
Copyright © Oakstone Publishing, 2021. All Rights Reserved.

Postoperative complications:

Differential: Diagnosis: Treatment:

DGF Thrombosis/ Duplex US/ Thrombosis:


ATN BX Immediate
Intervention

Acute Hypervolemia JP output/US Immediate


Risks: previous PP Intervention
Hemorrhage Anticoagulation

Hematuria Late: Rejection/UTI Urine output/US Irrigation/Cysto


s-copy
Mechanical/Acute Clinical Nephrostomy/Foley/
Urine Leak/ Rejection signs/Immediate and Stent/Surgical
quick rising SCrea/
Obstruction intervention
Drop of urine output
US/CT urogram
Hemorrhage/urin JP drain Drainage/Intervention
Lymphocele al radiology/Surgical:
e leak output/Lab./US/C Marsupialization
T
Rejection Rising Interventional
Art./Venous Radiology/Surgery
(CAN)/Positionin SCrea/Duplex
stenosis g US

Thrombosis
• Arterial (1%).
• Venous (1-4%)
• Usually technical
• Hypotension/Hypercoagulable state,
Compression, Kinking.

376
Copyright © Oakstone Publishing, 2021. All Rights Reserved.

Renal Vein Thrombosis

Renal Artery Stenosis

377
Copyright © Oakstone Publishing, 2021. All Rights Reserved.

Seroma

Lymphocele
• Incidence: 0.6-18%.
• Presentation: Limb swelling, rise in creatinine.
• Diagnosis with Ultrasound.
• Percutaneous Aspiration.
• Sclerosing Agents.
• Surgery: Creation of a Peritoneal window.

378
Copyright © Oakstone Publishing, 2021. All Rights Reserved.

Urological Complications
• 2-10%.
• Urine leak, stenosis/obstruction.
• Asymptomatic.
• Fever, pain over graft, decreased UO.
• Elevated Serum Creatinine.
• Diagnosis: Ultrasound.

Urological Complications
Hydronephrosis

379
Copyright © Oakstone Publishing, 2021. All Rights Reserved.

Urological Complications
Hydronephrosis Ureteral Stenosis

Urological Complications
Hydronephrosis Ureteral Stenosis

Urine Leak

380
Copyright © Oakstone Publishing, 2021. All Rights Reserved.

Urological Complications
Hydronephrosis Ureteral Stenosis

Urine Leak Subcapsular Hematoma

Algorithm of assessment and management of renal allograft rejection

381
Copyright © Oakstone Publishing, 2021. All Rights Reserved.

Live-donor nephrectomy

Hand-Assisted Lap. Retroperitoneal Donor Nephrectomy

Kumar, S, Witt RG, Tullius, SG, Malek, SK: Clin Transplant 2018

382
Copyright © Oakstone Publishing, 2021. All Rights Reserved.

Kidney transplants
Indications - Outcomes

Waitlisted

SRTR 2019

Kidney transplants
Indications - Outcomes

Waitlisted Transplanted by Indication

SRTR 2019

383
Copyright © Oakstone Publishing, 2021. All Rights Reserved.

Kidney transplants
Indications - Outcomes

Waitlisted Transplanted by Indication

Deceased donor transplants

SRTR 2019

Kidney transplants
Indications - Outcomes

Waitlisted Transplanted by Indication

Deceased donor transplants Living donor transplants

SRTR 2019

384
Copyright © Oakstone Publishing, 2021. All Rights Reserved.

Pancreas

Pancreas

• SPK: Simultaneous Pancreas Kidney


Transplantation.

• PAK: Pancreas after kidney


Transplantation.

• PTA: Pancreas Transplant Alone

385
Copyright © Oakstone Publishing, 2021. All Rights Reserved.

Bladder Drainage

• Early detection of
rejection by monitoring
urinary amylase

• Increased urological
and metabolic
complications.

Enteric Drainage

• More physiologic
• Increased risk of
anastomotic leak, intra-
abdominal abscess

Jolissaint, M.D., Stefan G. Tullius, M.D., Ph.D. in:


Abdominal Organ Transplantation, Current Diagnosis and Treatment: Surgery 15th Edition,
Editor Gerald Doherty, MD

386
Copyright © Oakstone Publishing, 2021. All Rights Reserved.

Portal Vein Drainage

Advantages:
Normal insulin levels
Possible immunologic benefit

Disadvantages:
Higher thrombosis rate
no bladder drainage
Less amenable to biopsy

Portal versus systemic venous drainage

Morbidity

60 % 52% 52%
P-D
50 %
40 % 33% 33%
S-D
% 30 % 26% 30%

20 %
10 %
0%
Infection
Rejection

Revision
Acute

Stratta, et al.: Ann Surg, 2001 (n = 54)

387
Copyright © Oakstone Publishing, 2021. All Rights Reserved.

Improved outcomes over time

Sutherland, DER, and others: Ann Surg 2001

CAUSES OF GRAFT LOSS

Humar, et al.: Transplantation 2004


Humar A,et al. Transplantation. 2004;78:1188

388
Copyright © Oakstone Publishing, 2021. All Rights Reserved.

Pancreas transplantation: Long-term survival

Drognitz et al. Transplantation • 2004

Pancreas transplantation for Diabetes Typ II:

389
Copyright © Oakstone Publishing, 2021. All Rights Reserved.

Secondary complications:
Improvement of Nephropathy following PTA:

A B C

Fioretto,Mauer NEJM 1998

Live Donor Pancreas Transplant

Boggi, U, Clin Transplant Sutherland, DER: Current Opinion in Organ Transplant, 2012
2011

390
Copyright © Oakstone Publishing, 2021. All Rights Reserved.

Pancreas Transplant Volume

Gruessner, AC and Gruessner RWG: The Review of Diabetes Study, 2016

Pancreas transplantation: Now and then:


- Adults listed for pancreas transplantation -

SRTR 2015: (Active) Patients listed on the pancreas waiting list

391
Copyright © Oakstone Publishing, 2021. All Rights Reserved.

Advancement in Insulin replacement treatment:

Russel, SJ, NEJM, 2014

First World Consensus Conference on Pancreas Transplantation

Boggi, U and others, AJT, 2021

392
Copyright © Oakstone Publishing, 2021. All Rights Reserved.

First World Consensus Conference on Pancreas Transplantation

• Surgical Techniques
• Exocrine Drainage:
• One study with a higher rate of surgical complications
in bladder-drained transplants
• > 40 % require enteric conversion
• Bladder drainage increased urological and metabolic
complications

• Venous drainage:
• No study shows a clear benefit of portal venous drainage

• Immunosuppression:
• State of the art: Induction with Depleting antibodies
• CNI avoidance: inferior immunologic outcomes
• MMF improves immunologic outcomes
• mTOR: no clear advantages; higher rates of surgical
complications immediate post-operatively

Boggi, U and others, AJT, 2021

First World Consensus Conference on Pancreas Transplantation

• Impact of SPK:
– SPK improves both quality and long-term survival
– More strictly applied to patients with Typ 1 compared to Typ 2
– For Typ II it is not clear if SPK conveys and advantage over live donor renal transplant

• Impact of PAK:
– Increases the risk of early post-surgical mortality but improves life expectancy
– For Typ 2 patients, it is unclear if PAK has an advantage

• Impact of PTA
– Does not increase the long-term risk of death
– Careful patient selection with GFR > 60 ml/min/1.73m2:
– May stabilize/improve diabetic neuropathy; may slow progression of diabetic
neuropathy

Boggi, U and others, AJT, 2021

393
Copyright © Oakstone Publishing, 2021. All Rights Reserved.

Liver Transplantation

Historical developments

Zarrinpar, A Busuttil, RW: Nat Rev Gastroenterol Hepatol, 2013

394
Copyright © Oakstone Publishing, 2021. All Rights Reserved.

Liver Transplantation: Technical Aspects

Jolissaint, M.D., Stefan G. Tullius, M.D., Ph.D. in:


Zarrinpar, A Busuttil, RW: Nat Rev Gastroenterol Hepatol, 2013
Abdominal Organ Transplantation, Current Diagnosis and Treatment: Surgery 15th Edition,
Editor Gerald Doherty, MD

Liver Transplantation: Outcomes – Single Center

Busuttil, RW, and others, Ann Surg 2005

395
Copyright © Oakstone Publishing, 2021. All Rights Reserved.

Liver Transplantation: Changing Indications

Vyhmeister, R, Enestvedt, K: Clin Liver Dis, 2021

Liver Transplantation: Volume & Outcomes – SRTR

396
Copyright © Oakstone Publishing, 2021. All Rights Reserved.

Liver Transplantation: Volume & Outcomes – SRTR

Liver Transplantation: Volume & Outcomes – SRTR

397
Copyright © Oakstone Publishing, 2021. All Rights Reserved.

Summary:

• Organ transplantion has established itself as


the treatment of choice for patients with End-
Stage-Organ-Failure

• The Field is uniquely positioned at the cutting


edge between pioneering research and clinical
excellence

• Meeting the increasing demand and


understanding the complexity of alloimmunity
remain critical issues

398
Copyright © Oakstone Publishing, 2021. All Rights Reserved.

Pediatric Surgery Review

Prathima Nandivada MD
Assistant Professor of Surgery, Harvard Medical School
Boston Children’s Hospital, Boston, MA

Disclosures
• None

399
Copyright © Oakstone Publishing, 2021. All Rights Reserved.

OBJECTIVES
• Causes of infant bowel obstructions
• Necrotizing Enterocolitis
• Pediatric Hernias
• Toddler emergencies
• Pediatric Solid Tumors
• Pediatric Surgery Potpourri

Infant Bowel Obstruction


• You are called to the newborn nursery to evaluate a
newborn full term baby boy who is vomiting with feeding
attempts…

• You arrive in the nursery and see a well perfused, active


baby boy in no distress

• What else do you want to know?

400
Copyright © Oakstone Publishing, 2021. All Rights Reserved.

Thinking about infant vomiting…


• Is the emesis bilious or non-bilious?
• Non-bilious = proximal to ampulla

• Is there a proximal obstruction?


• Abdomen non-distended, AXR with no dilated loops beyond duodenum

• Is there a distal obstruction?


• Abdomen distended, AXR with many dilated loops

• Can differentiate between most of these problems with a good history,


exam and a plain abdominal film!

Differential Diagnoses
• Non-bilious emesis with exam/AXR • Bilious emesis with exam/AXR
showing proximal obstruction showing distal obstruction
• Pyloric atresia (very very rare) • Jejunoileal atresia
• Pyloric stenosis • Meconium ileus
• GET AN ABDOMINAL US • Hirschsprung disease
• Imperforate anus
• Bilious emesis with exam/AXR
showing proximal obstruction • GET A CONTRAST ENEMA FOR
DIAGNOSIS and/or THERAPY
• Midgut volvulus
• Duodenal atresia
• GET AN UGI TO RULE OUT MIDGUT
VOLVULUS UNLESS CLEARLY DA

401
Copyright © Oakstone Publishing, 2021. All Rights Reserved.

Case #1
• 5 week old baby boy presents to ED with vomiting with exhausted parents

• Emesis is non-bilious, but parents tell you it’s forceful vomiting, “projectile”

• And yet, no matter how much the baby vomits, he’s hungry and crying for milk…

• On exam, baby is well appearing, non-distended, non-tender

Pyloric Stenosis
• Presentation: Non-bilious, projectile
emesis

• Work up:
• Labs showing hyperchloremic, hypokalemia,
metabolic alkalosis
• US abdomen: > 4 mm thick, >15 mm long
• UGI: elongated pyloric channel

402
Copyright © Oakstone Publishing, 2021. All Rights Reserved.

Management
• RESUSCITATE!!
– Isotonic fluids until labs normalize
• Pyloromyotomy
– Laparoscopic vs. open
• Longitudinal incision through submucosa from distal stomach to
proximal duodenum
• Non-operative
– NJ feeding until improved
– Atropine

403
Copyright © Oakstone Publishing, 2021. All Rights Reserved.

Complications

• While you’re spreading pyloric incision, you enter mucosa…


– Repair mucosa + muscle with myotomy on other side

• Baby initially improved but starts vomiting everything again.


UGI done and shows persistent pyloric stenosis…
• INCOMPLETE MYOTOMY  Back to OR

Case #2:

• Newborn with sudden onset,


multiple episodes of bilious emesis

• On exam, baby is lethargic with non-


distended abdomen

MIDGUT VOLVULUS WITH


MALROTATION UNTIL PROVEN
OTHERWISE! URGENT UGI

404
Copyright © Oakstone Publishing, 2021. All Rights Reserved.

13

14

405
Copyright © Oakstone Publishing, 2021. All Rights Reserved.

15

Intestinal Rotation in Utero


• Herniation and growth of midgut
supplied by SMA
• Viscera return to abdominal cavity
and rotate around SMA:
• The proximal duodenojejunal loop
rotates 270º posterior and to the left
• The distal cecocolic loop moves
anterior and to the right.
• By the tenth week, the midgut has
retracted into the abdomen and
intestinal fixation secures the
duodenojejunal junction at the
ligament of Treitz in the left upper
quadrant.
• Disruption at any point can lead to
malrotation

406
Copyright © Oakstone Publishing, 2021. All Rights Reserved.

Intestinal Malrotation
• Two general considerations:
• Narrow mesenteric base
allows small bowel to
twist about the
mesentery resulting in
volvulus
• Ladd bands crossing
duodenum can lead to
obstruction

SURGICAL EMERGENCY!! • Detorse bowel (usually


counterclockwise rotation)
• Ladd’s Procedure
• Divide peritoneal
attachments between
cecum and abdominal
wall
• Kocher to free up
duodenum (especially
anteriorly)
• Lyse Ladd’s bands
(above maneuvers will
do that)
• Broaden mesenteric
vascular pedicle
• +/- Appendectomy
18

407
Copyright © Oakstone Publishing, 2021. All Rights Reserved.

Surgical Considerations
• Non-emergent scenarios:
• What if malrotation incidentally found?
• What if malrotation found during work up of chronic symptoms of intermittent
obstruction?
• Consider Ladd’s procedure, especially if patient is young
• Open vs. laparoscopic?
• Some data that higher rate of recurrent volvulus with lap Ladds…
• Long term issues?
• 5-10% SBO and 1-2% recurrent volvulus
• Major cause of intestinal failure

Case#3

Newborn with bilious emesis

Baby well appearing

No abdominal distension

Prenatal double bubble with


polyhydramnios

Trisomy 21

408
Copyright © Oakstone Publishing, 2021. All Rights Reserved.

Duodenal Atresia

Failure to recanalize segment


of duodenum

Duodenoduodenostomy

409
Copyright © Oakstone Publishing, 2021. All Rights Reserved.

Case #4

Newborn with bilious emesis

Baby well appearing

Mild abdominal distension

No prenatal concerns

Jejunoilieal Atresia
Vascular incident with loss of
segment of jejunum or ileum

Classification of intestinal atresia. Type I, mucosal (membranous)


atresia with intact bowel wall and mesentery. Type II, blind ends
are separated by a fibrous cord. Type IIIa, blind ends are
separated by a V-shaped (gap) mesenteric defect. Type IIIb,
apple-peel atresia. Type IV, multiple atresias (string of sausages)

410
Copyright © Oakstone Publishing, 2021. All Rights Reserved.

Techniques of anastomosis. A, The proximal atretic segment is resected at a 90-degree angle and the
distal segment at a 45-degree angle. B, End-to-oblique anastomosis is carried out by the techniques of
Benson (one-layer) or Nixon (two-layer) with fine interrupted
sutures. The Nixon anastomosis depicts a Cheatle slit on the antimesenteric border of the distal limb.

Case #5

Newborn with bilious emesis

Baby looks uncomfortable and


has marked abdominal
distension

Sibling with cystic fibrosis

411
Copyright © Oakstone Publishing, 2021. All Rights Reserved.

Meconium Ileus
Seen in babies with cystic
fibrosis

Simple meconium ileus - can


treat with water soluble contrast
enemas as long as making
progress… but sometimes you
don’t!

Complicated meconium ileus –


no progression with enemas or
perforation

412
Copyright © Oakstone Publishing, 2021. All Rights Reserved.

Case #6

• Small amount of bilous emesis


but hasn’t passed meconium
for 2 days

• Well appearing, distended


abdomen

• No known CF family history

29

Contrast enema
The Recto-Sigmoid Index

Abnormal rectosigmoid ratio Normal contrast enema


Marked sigmoid dilation and with normal rectosigmoid
rectal mucosal irregularity ratio

30

413
Copyright © Oakstone Publishing, 2021. All Rights Reserved.

Hirschsprung’s Disease
• Functional bowel obstruction due to abnormal development of the
enteric nervous system

• Higher risk in males, trisomy 21

• 80% have transition zone in rectum or rectosigmoid colon.


• 10% more proximal colonic involvement
• 5-10% total colonic aganglionosis

• Diagnosis made by rectal biopsy (~1.5 cm above dentate line)


• Absence of ganglions cells, hypertrophic nerves

Pull through Procedures

SOAVE SWENSON DUHAMEL

less pelvic dissection no aganglionic bowel least pelvic dissection


variable cuff length (1-5 cm) remains reservoir
less stricture

414
Copyright © Oakstone Publishing, 2021. All Rights Reserved.

Pull through Procedure

Pull through Procedure

415
Copyright © Oakstone Publishing, 2021. All Rights Reserved.

Case #7

• 27 week, BW 1000g, DOL 12

• Sudden abdominal distention,


hypotension, apneic spells

• Intubated, NPO, NG tube placed,


and started on pressors

416
Copyright © Oakstone Publishing, 2021. All Rights Reserved.

Necrotizing Enterocolitis
• Associated with low birth weight and
prematurity

• Present with abdominal distension,


sepsis, bloody stool

• AXR with dilated loops, pneumatosis


intestinalis, portal venous gas

• Can treat with bowel rest, antibiotics,


and support

A few hours later…

Ex lap (damage control vs. resection + stoma) or peritoneal drain for source control

417
Copyright © Oakstone Publishing, 2021. All Rights Reserved.

PEDIATRIC HERNIAS

Pediatric hernias: Inguinal hernia


• 4 week old, former 30
week GA baby boy in
NICU

• Nurse calls due to


concern for bulge in
right groin…

40

418
Copyright © Oakstone Publishing, 2021. All Rights Reserved.

Pediatric Inguinal Hernia Management


• Reducible hernias are elective. Incarcerated hernias can be reduced
and repaired on that admission or reduced operatively.

• For preterm inpatient babies, often fix before discharge (once


otherwise stabilized – O2, feeding, etc) due to high risk of newborn
incarceration

• Considerations:
• Is there a hydrocele? If hydrocele, can wait until age 1 yr, might close on its
own
• Are the testicles down? If not, wait to repair hernia in a newborn
41

Inguinal hernia repair in infants


• Why do we repair hernias differently in babies than in adults?

42

419
Copyright © Oakstone Publishing, 2021. All Rights Reserved.

High ligation: laparoscopic vs. open

43

Pediatric hernias: Umbilical hernia

• Most close on their own by age


5, no need to repair unless
incarcerated!

• After age 5, repair with sutures,


no mesh needed.

420
Copyright © Oakstone Publishing, 2021. All Rights Reserved.

TODDLER EMERGENCIES

Case #1

• 2 year old girl presents to ED with trouble swallowing

• Parents say she was playing on the floor with some plastic toys and
then started crying

• Has been unable to eat and is very fussy, complains that something is
stuck

• What do do??
46

421
Copyright © Oakstone Publishing, 2021. All Rights Reserved.

Esophageal Foreign Bodies


• Rigid esophagoscopy performed
under general anesthesia

• Careful to remove in an
orientation that doesn’t injure
esophagus

• +/- completion esophagoscopy

47

Button Battery Ingestion


• Button batteries are very dangerous!

• Cause necrosis of esophageal tissue that


can progress even after battery removed

• On route to hospital or in ED  honey

• Call button battery hotline & report case

• During esophagoscopy  acetic acid

• Postop MRI vs. CT to assess aorta

48

422
Copyright © Oakstone Publishing, 2021. All Rights Reserved.

Case #2

• 18 month old boy presents with cough and wheezing after sneezing
while eating crushed up peanut M&Ms… mom is concerned he
aspirated peanuts

• VS normal, mild wheezing on exam

• What to do?

49

50

423
Copyright © Oakstone Publishing, 2021. All Rights Reserved.

Airway Foreign Bodies


• Rigid bronchoscopy to remove the foreign bodies (careful not to crush the
peanut further!)

• Consider flex bronchoscopy as well

• Admit for monitoring  these kids can get sick and have bronchospasms
postop!
• Augmentin +/-
• Inhalers and steroids

51

Toddler with abdominal pain


• 18 mo old boy presents with abdominal pain and vomiting
• Episodic pain, drawing knees in during episodes
• Bilious emesis

• On exam, fussy and uncomfortable but stable VS. Abdomen, soft, nondistended,
very mildly tender in RLQ

• Labs wnl

• What is the diagnostic test of choice?

424
Copyright © Oakstone Publishing, 2021. All Rights Reserved.

Ileocolic intussusception

Ileocolic intussusception
• Most frequent cause of bowel obstruction in infancy and the second most
common cause of abdominal pain and obstruction
• Usually < 2 yo, but can be in any age
• 85% ileocolic, less common: appendiceal, colocolic
• Why does it happen?
• Ten percent of cases have a pathologic lead point (Meckel diverticulum, polyps or
lymphoma, duplication.)
• Treatment: hydrate, air contrast enema as many times as needed
• Feared complication – tension pneumoperitoneum

425
Copyright © Oakstone Publishing, 2021. All Rights Reserved.

Ileocolic intussusception
• Surgery reserved for patients with:
• Peritonitis, perforation
• Pathologic lead point
• Failure of air contrast enema
reduction

• Recurrence is possible and hard to


predict
• 5-20% of cases, most occur within
first 24 hours, but can be longer
intervals

Case #3

• 13 month old male presents to ER with bright red blood per rectum

• On exam, well appearing baby with normal VS and exam

• Labs notable for Hct of 28, otherwise wnl

• AXR and abdominal US normal.

• What is next step in work up?

426
Copyright © Oakstone Publishing, 2021. All Rights Reserved.

Meckel’s Diverticulum

• Persistent omphalomesenteric duct


• Presents with bleeding, pain due to diverticulitis, or obstruction
(volvulus, intussusception, or internal hernia)
• For bleeding, can use a Technetium 99m pertechnetate scintiscan
to identify Meckel diverticulum.
• The isotope is taken up by gastric mucosal cells
• The administration of H2 blockers inhibits the release of radiotracer out
the cells.
• CAN HAVE FALSE NEGATIVES! (consider diagnostic laparoscopy)

427
Copyright © Oakstone Publishing, 2021. All Rights Reserved.

Meckel’s Diverticulum
• Rule of twos
• two cm long
• two types of heterotopic mucosa
• within two feet of the ileocecal valve

• True diverticula and antimesenteric

• On pathologic exam, usually have heterotopic gastric mucosa, sometimes


pancreatic tissue

428
Copyright © Oakstone Publishing, 2021. All Rights Reserved.

The incidentally found MD…


• Removing it is low risk and can clearly cause problems down the road…
• 6% lifetime risk of a complication from MD (data???)
• Theoretical risk of malignancy within it (data??)
• Some argue to remove selectively…
• age younger than fifty years
• male gender
• diverticular length greater than two centimeters (which isn’t obvious until you dissect it
out…)
• ectopic or abnormal features within the diverticulum
• Dealer’s choice… just be able to justify it

PEDIATRIC APPENDICITIS

429
Copyright © Oakstone Publishing, 2021. All Rights Reserved.

Pediatric Appendicitis – Key Differences


• Imaging
• Diagnosis often made by ultrasound
• When non-diagnostic, MRI is preferred

• Non-operative management of acute appy?


• In a 2016 JAMA Surgery prospective study of simple acute appendicitis in
children, the success of nonoperative management (antibiotics alone) was 89%
at 30 days and 75.7% at 1 year.
• Recurrent appendicitis may be up to 20% and more likely when a fecalith is
present
• Impact of fecaliths + longer lifetime risk of recurrent appendicitis + nervous
parents = 

Pediatric Appendicitis – Key Differences


• Management of complicated appendicitis?
• Early, non-organized perf appy  lap appy!
• Later, organized abscess perf appy  harder decision
• Hard to send kids home with drains… and their course is LONG
• Prospective data have shown that in patients with perforated appendicitis and a well
developed abscess outcomes following initial operation or initial nonoperative management
are essentially equivalent.
• Other studies have demonstrated an improved quality of life in patients with an abscess who
are treated with initial appendectomy

• Technical considerations
• In smaller children, domain limited and may be too small for staplers!
• For appendix base, consider endoloop, externalizing through umbo, or 5-mm staplers
• For mesentery, can use hook cautery or maryland cautery

• When perforated, keep an eye out for fecoliths (can cause recurrent abscess later)

430
Copyright © Oakstone Publishing, 2021. All Rights Reserved.

PEDIATRIC TUMORS

Breaking down the pediatric abdominal mass


• Look at the age!
• <2 years  Neuroblastoma, hepatoblastoma (rare)
• >3 years  Wilm’s
• Teenagers  RCC, HCC, teratomas
• Any genetic syndromes? Premature birth?
• Consider general and tumor specific labs
• Imaging is essential
• US
• CT abdomen/pelvis vs. MRI
• Chest CT

431
Copyright © Oakstone Publishing, 2021. All Rights Reserved.

68

432
Copyright © Oakstone Publishing, 2021. All Rights Reserved.

Neuroblastoma
• Most common abdominal malignancy of childhood
• Adrenal > paraspinal > mediastinum

• 80%<4yrs, and most <2yrs

• Male >Female (13:1)

• Derived from neural crest and tumor can develop anywhere along
sympathetic ganglia

Neuroblastoma Work Up
• LABS:
• Serum and urinary metanephrines
• Catecholamines
• Homovanillic acid (HVA) & Vanillylmandelic acid (VMA)
• NSE, ferritin
• Imaging:
• US, CT vs. MRI, Bone Scan, MIBG Scan
• Biopsy
• Bone marrow +/- primary tumor
• Associated with key clinical findings:
• Racoon Eyes
• Myoclonus-opsiclonus
• Blueberry muffin skin

433
Copyright © Oakstone Publishing, 2021. All Rights Reserved.

Neuroblastoma Staging
• Based on pre-treatment imaging
• L1- localized (one body compartment), no high risk features
• L2- local-regional (ipsilateral), +high risk features
• M- distant mets, contralateral disease
• MS- localized primary tumor, <18mo, bone marrow neg but distant mets
• Age < 18 mo = better prognosis
• MYCN amplification = worse prognosis

Surgical Management
• L1 - upfront surgery with completely resection

• L2, M - chemo first then resect as able with goal of gross total
resection ~90%
• Can bivalve tumor around important structures, like visceral vessels

• MS – can observe within COG trial

• Postop treatment is multimodal: more chemo, myeloablative


tx, I-MIBG therapy

72

434
Copyright © Oakstone Publishing, 2021. All Rights Reserved.

74

435
Copyright © Oakstone Publishing, 2021. All Rights Reserved.

Wilms Tumor
• Most frequent renal tumor in children

• Second most common abdominal tumor

• Male:Female ratio 1:1 (varies with specific race)

Wilms Tumor
• LABS:
• CBC with anemia
• Coags if abnormal could suggest vWD
• UA often shows hematuria (gross or micro)
• US, CT vs. MRI, Chest CT
• Biopsy if unresectable
• Resect upfront if able, even in cases with lung mets!
• Neoadjuvant treatment if unresectable
• Tumor in RA, involving other organs, b/l tumors,
extensive pulmonary mets

436
Copyright © Oakstone Publishing, 2021. All Rights Reserved.

Staging Wilms Tumor


• Staging is based on:
• Tumor histology (favorable 90% vs. unfavorable 10% --
anaplastic)
• Patient age (<2 yrs)
• Tumor weight (<550 grams)
• Local tumor characteristics
• 1- within capsule
• 2- beyond capsule but all resected
• 3- incomplete resection, biopsy, rupture, lymph nodes
• 4- metastatic

Surgical Management
• Complete resection without rupture

• Radical nephroureterectomy

• Sample lymph nodes

437
Copyright © Oakstone Publishing, 2021. All Rights Reserved.

Hepatoblastoma
• Most common liver tumor in children
• Occur most commonly between 1-3 years
• Male>Female (1.5:1)
• Seen with prematurity and certain congenital
syndromes
• Beckwith-Weidmann (1K – 10K inc risk)
• Screen with AFP and US every 3 mo
• FAP (less tightly linked)
• Trisomy 18, Trisomy 13

438
Copyright © Oakstone Publishing, 2021. All Rights Reserved.

Hepatoblastoma
• LABS:
• May have anemia, LFTs usually normal
• AFP!

• US, MRI>CT, Chest CT


• Imaging is key in determining pre-treatment staging and
resectability

Hepatoblastoma

82

439
Copyright © Oakstone Publishing, 2021. All Rights Reserved.

Hepatoblastoma
• Gross total resection needed to improve outcome

• Biopsy if unresectable for neoadjuvant treatment

• Upfront resection for PRETEXT 1&2

• Can consider transplantation in extensive tumors


after neoadjuvant treatment

PEDIATRIC SURGERY POTPOURRI

440
Copyright © Oakstone Publishing, 2021. All Rights Reserved.

Congenital diaphragmatic hernia


• Often prenatally diagnosed, but can
present with respiratory distress or
abnormal CXR in newborn
• Principles:
• Stabilize, intubate (CPAP can cause bowel
dilation and obstruction)
• May need ECMO
• Repair hernia
• Supportive therapy for pulmonary
hypoplasia and pulmonary hypertension
• Mortality is high for more severe pulmonary
hypoplasia

Omphalocele vs. Gastroschisis

• Omphalocele: associated with other anomalies & pulmonary hypoplasia, can be


repaired in infancy or “paint and wait”
• Gastroschisis: usually isolated, silo + reduction and repair defect, can atresias
and/or have long term dysmotility

441
Copyright © Oakstone Publishing, 2021. All Rights Reserved.

Esophageal Atresia

• Principles: Need to ligate fistula if present to stop aspiration, and


establish esophageal continuity. Longer gaps required staged
operations

Choledochal Cysts
• Can cause biliary obstruction and
are associated with risk of
cholangiocarcinoma
• Type 1 most common

• Can often be repaired with


resection and RnY HJ, but
complex lesions may require
additional recon or transplant

442
Copyright © Oakstone Publishing, 2021. All Rights Reserved.

Biliary Atresia
• Sclerosing cholangiopathy of unclear
etiology
• Present with acholic stools, jaundice, and
moderate hyperbilirubinemia (often <6
mg/dL), high GGT
• Work up:
• Abdominal US: absent gallbladder
• HIDA scan: looking for bile secretion
• Cholangiogram to confirm diagnosis +/- liver
biopsy (bile duct proliferation)
• Treatment: Kasai Portoenterostomy
• 1/3 resolve, 1/3 late failure, 1/3 early failure
• 2/3 ultimately require transplantation

REVIEW
• Causes of infant bowel obstructions
• Necrotizing Enterocolitis
• Pediatric Hernias
• Toddler emergencies
• Pediatric Solid Tumors
• Pediatric Surgery Potpourri

GOOD LUCK!

443
Copyright © Oakstone Publishing, 2021. All Rights Reserved.

Arterial Disease

Matthew Menard, MD
Brigham and Women’s Hospital
Boston, MA

Disclosures

• BEST-CLI trial Co-Chair


– Supported by NHLBI: 1U01HL107407-01A1

• Scientific Advisory Board


– Janssen, Inc

444
Copyright © Oakstone Publishing, 2021. All Rights Reserved.

Etiology of PAD

• Atherosclerosis
• Embolization
• Thrombosis
• Buerger’s Disease
• Vasculitis
• Arterial Trauma
• Popliteal Entrapment
• Popliteal Adventitial Cystic Disease

Epidemiology
•Peripheral Arterial Disease
• Affects > 200 million people worldwide
•Chronic Limb Threatening Ischemia (CLI)
• Prevalence: ~11% of patients with PAD
• Incidence: 500 – 3,500 cases/million/year
Critical
Chronic Lim b Acute Lim b
Ischem ia Ischem ia

Stable Hirsch AT. Circulation 2012;125 (110);1449-1472


Claudication
Norgren L. Int Angiol 2007;6(2):81-157
Asym ptomatic
PAD
Biancari F. J Cardiovasc Surg (Torino) 2013;54:663-9.
Fowkes FG et al. Lancet 2013;382:1329-40.
Nehler MR. J Vasc Surg 2014;60(3):686-695

445
Copyright © Oakstone Publishing, 2021. All Rights Reserved.

Demographic Factors and Symptomatic PAD

Norgren L et al. TASC II. J Vasc Surg 2007;45:S5-67.

Relative Risk of Developing CLI

Norgren L et al. TASC II. J Vasc Surg 2007;45:S5-67.

446
Copyright © Oakstone Publishing, 2021. All Rights Reserved.

PAD risk with age

-PAD prevalence
increases with age
-World population is
aging
-Epidemic of diabetes
and obesity

447
Copyright © Oakstone Publishing, 2021. All Rights Reserved.

PAD is a Growing Worldwide Epidemic

Nat Rev Cardiol. 2017 Mar;14(3):156-170.

2013;382:1329-40

From 2000 to 2010 worldwide


prevalence of PAD increased by
23.5%

448
Copyright © Oakstone Publishing, 2021. All Rights Reserved.

PAD
• Cause: occlusion of the limb blood vessels usually
due to atherosclerosis
• Approximately 8-12 million US adults have PAD
• Often can be asymptomatic
• Characterized by functional impairment that
affects quality of life
• Can lead to major ischemic events that can cause
limb loss
• Increased risk of heart attack or stroke

PAD: Symptoms
• Intermittent claudication
(Pain in legs with exertion that subsides with rest)
• Leg pain at rest
• Slow or non-healing wounds on feet
• Ulceration/gangrene of toes/feet
• Can lead to acute limb ischemia and limb
amputation

449
Copyright © Oakstone Publishing, 2021. All Rights Reserved.

PAD: Disparities
• African-American population has higher disease
burden
• Incidence of PAD is nearly twice that of non-
Hispanic whites
• More likely to be diagnosed later with more
advanced disease
• Rate of amputation is considerably higher
compared to non-Hispanic whites

Coronary Artery Disease


• Cause: narrowing in the blood vessels that
supply the heart due to atherosclerosis
• About 18.2 million US adults have CAD
• Can lead to heart attack or heart failure
• Cause of 360,900 deaths in the US in 2019
• Approximately 40% of CAD patients also have
PAD

450
Copyright © Oakstone Publishing, 2021. All Rights Reserved.

CAD: Symptoms
• Chest pain
• Shortness of breath
• Heart attack

Mesenteric Disease
• Occlusive disease of the mesenteric
vasculature
• Acute mesenteric disease
• 6 hour time window
• Surgical emergency
• Chronic mesenteric ischemia
• Post-prandrial pain
• Food fear
• Weight loss

451
Copyright © Oakstone Publishing, 2021. All Rights Reserved.

Renal Arterial Disease


• Occlusive disease of the renal arteries
• Leads to Chronic Kidney Failure
• CKD I-V
• ESRD
• Hypertension
• Congestive heart failure

Cerebrovascular Arterial Disease


• Occlusive disease of the Great Vessels of the arch,
carotid arteries, vertebral arteries
• Innominate
• Subclavian arteries
• Embolic stroke
• Visual changes
• Speech disturbance
• Unilateral weakness or numbness

• Carotid endarterectomy
• Carotid stenting
• Trans-femoral
• Trans-carotid

452
Copyright © Oakstone Publishing, 2021. All Rights Reserved.

453
Copyright © Oakstone Publishing, 2021. All Rights Reserved.

Diagnosis of CLI is confirmed in the


Vascular Laboratory

• Physiological Studies
– Ankle Brachial Index (ABI)
– Toe pressures
– Doppler waveforms
– Pulse Volume Recordings (PVR)
– Transcutaneous oximetry (TcPO2)

Hemodynamic Definition of CLI

Patel MR et al. PARC. J Am Coll Cardiol. 2015;65:931-41

454
Copyright © Oakstone Publishing, 2021. All Rights Reserved.

More Hemodynamic Definitions of CLI

• ABIs < 0.4

Monophasic doppler waveforms Flat transtarsal PVRs

Confirmatory Imaging

Ankle-Brachial Index (ABI)


Toe-Brachial Index (TBI)
Exercise ABI
Non-Invasive Physiologic Testing
Segmental Blood Pressure
Pulse Volume Recording
Continuous Wave Doppler
Arterial Duplex Ultrasonography
Advanced Imaging
Computed Tomography Angiography
Magnetic Resonance Angiography
Digital Subtraction Angiography

ABI – Ankle-Brachial Index, TBI Toe-Brachial Index

455
Copyright © Oakstone Publishing, 2021. All Rights Reserved.

Asymptomatic PAD

Prevention of PAD

• Antithrombotic therapy
• Lipid management
• Hypertension management
• Smoking cessation
• Diabetes management

456
Copyright © Oakstone Publishing, 2021. All Rights Reserved.

Classic Grading of Symptomatic PAD

Rutherford Classification Fontaine Classification

Stage Clinical Presentation Stage Clinical Presentation


0 Asymptomatic I Asymptomatic
1 Mild claudication IIa Intermittent claudication after more than 200m of
2 Moderate claudication pain free walking
3 Severe claudication IIb Intermittent claudication after less than 200m of
walking
4 Rest pain
III Ischemic rest pain
5 Ischemic ulceration not exceeding ulcer of the
digits of the foot IV Ischemic ulceration or gangrene
6 Severe ischemic ulcers or frank gangrene

Rutherford R et al. J Vasc Surg. 1997;26:517-538;

Three Spheres Influence Outcomes

Limb Status
Wound, Ischemia, foot
Infection

Anatomy
Occlusion versus
stenosis
Lesion length
Patient Multiplicity of
Status lesions
Risk Factors and Disease Pattern
Comorbidities Calcification
Tibial and Pedal
runoff
Angiosome
Mills J et al. J Vasc Surg. 2014;59:220034.

457
Copyright © Oakstone Publishing, 2021. All Rights Reserved.

WIfI (Wound, Ischemia, and Foot Infection) Classification

Grade Wound Ischemia Foot Infection


(ABI)
• Wound: extent and depth 0 No ulcer and No ≥ .80 None
gangrene

• Ischemia: perfusion/flow 1 Small ulcers / no 0.6-0.79 Mild


gangrene

2 Deep ulceration / 0.4-0.59 Moderate


• foot Infection: presence and Gangrene limited to toes
extent

3 Extensive ulceration / ≤0.39 Severe (limb and/or


Extensive gangrene life-threatening)

Mills J et al. J Vasc Surg. 2014;59:220034

Natural History of PAD

Hirsch A et al. Circulation. 2006; 113:463-654.

458
Copyright © Oakstone Publishing, 2021. All Rights Reserved.

Natural History of CLI

>1,500 patients in 13 studies at 1 year f/u


----22% major amputation rate
----22% mortality rate

Risk of Amputation vs SVS WIfI Stage


Study (year): # limbs at risk Stage 1 Stage 2 Stage 3 Stage 4

Cull (2014): 151 37 (3%) 63 (10%) 43 (23%) 8 (40%)

Zhan (2015): 201 39 (0%) 50 (0%) 53 (8%) 59 (37%)

Darling (2015): 551 5 (0%) 111 (10%) 222 (11%) 213 (24%)

Causey (2016): 160 21 (0%) 48 (25%) 42 (21%) 49 (31%)

Beropoulis (2016): 126 29 (13%) 42 (19%) 29 (19%) 26 (38%)

Ward (2016): 98 5 (0%) 21 (14%) 14 (21%) 58 (34%)

Darling (2016): 992 12 (0%) 293 (4%) 249 (4%) 438 (21%)
N = 2279 (weighted mean) 148 (3.4% ) 628 (8.3%) 652 (10.3%) 851 (25%)

Median (% 1 yr amputation) 0% 10% 19% 34%

Number of limbs at risk in each WIfI Stage with % amputation at 1 year in parentheses
Courtesy of J Mills

459
Copyright © Oakstone Publishing, 2021. All Rights Reserved.

Relative 5-Year Mortality Rates

Armstrong DG. J Foot Ankle Res 2020;13:16

460
Copyright © Oakstone Publishing, 2021. All Rights Reserved.

Goals Of Treatment

• Medical therapy to optimize cardiovascular risk


• Limb-related soft-tissue therapy
• Revascularization (measures to improve limb perfusion)

Hirsch AT et al. J Am Coll Cardiol 2006;47:1239-131


Conte MS and Farber A. BJS 2015;102:1007-1009

Revascularization Options in CLI


Open Vascular Surgery Endovascular Therapy

461
Copyright © Oakstone Publishing, 2021. All Rights Reserved.

Open Vascular Surgery for CLI


Inflow Bypass
Infrainguinal
Bypass
Endarterectomy

Surgical Bypass

1906- Technique of vascular anastomosis


described
(Carrel A, Guthrie CC. Surg Gynecol Obstet 2:266,1906)

1948- 1st successful femoral popliteal bypass


using rGSV in a patient with PAD
(Kunlin J. Rev Chir Paris 70:206-236, 1951)

462
Copyright © Oakstone Publishing, 2021. All Rights Reserved.

Leg Bypass has a rich history…

Infrainguinal Bypass
Primary Graft Secondary Limb Salvage
Patency Graft Patency @ 5 years
@ 5 years @ 5 years
Taylor L. et al 80% 84% 90%
n 300

Shah D. et al 72% 81% 95%


n 2,048

Pomposelli FB. et al. 57% 63% 78%


n 1,032

Perioperative mortality: 1-6%

463
Copyright © Oakstone Publishing, 2021. All Rights Reserved.

Open Vascular Surgery for CLI


• Traditional treatment
• Durable outcomes
• Long follow up periods available

• Invasive
• Is associated with
– blood loss
– morbidity
– mortality
– wound complications

1964

Charles Dotter

464
Copyright © Oakstone Publishing, 2021. All Rights Reserved.

Novel Technology

Trends in PAD Therapy

Goodney et al. JAMA Surg 2015;150(1):84-86

465
Copyright © Oakstone Publishing, 2021. All Rights Reserved.

Endovascular Treatment Options for PAD


 Plain Balloon Angioplasty (PTA)
 Stenting
 Atherectomy
 Laser assisted PTA
 Brachytherapy
 Stent grafts
 Drug eluting stents
 Drug coated balloons
 Bioabsorbable stents
 ………

Stents and stentgrafts

Angioplasty

Atherectomy

466
Copyright © Oakstone Publishing, 2021. All Rights Reserved.

284 limbs
PTA Success -95%
Limb salvage- 91%

There is a lot of literature…

Publications reporting 1-yr patency following SFA stenting or stent-grafting from 2000-2009

courtesy L. Schwartz

467
Copyright © Oakstone Publishing, 2021. All Rights Reserved.

and more literature…

GORE VIABAHN® Endoprosthesis in the SFA

Drug Eluting Stents and


Drug Coated Balloons

468
Copyright © Oakstone Publishing, 2021. All Rights Reserved.

Has drug elution been successful in PAD?

OR

469
Copyright © Oakstone Publishing, 2021. All Rights Reserved.

Endovascular Therapy for CLI


• Minimally invasive
• No need for
• general anesthesia
• incisions
• hospitalization
• Lower morbidity and mortality
• Decreased durability
• Low patency rates in some vascular beds
• Expensive
• Driven by business interests

Regional variation in
prevalence of CLI,
amputation in CLI and
revascularization for CLI in
Medicare database

Goodney et al. Circulation Cardiovasc. Qual and


Eid et al, manuscript in preparation, 2021
Outcomes 2011

470
Copyright © Oakstone Publishing, 2021. All Rights Reserved.

Surgical Bypass vs. Endovascular


Therapy in the VQI:
Great Variation in Procedure Choice!
100% Bypass
100%

90%

80%

70%

60%

50%

40%

30%

20%

10%

0%
VQI Centers
0% Bypass

471
Copyright © Oakstone Publishing, 2021. All Rights Reserved.

What About an Endo First Policy?

Start with endo…. if we don’t succeed then


do bypass

472
Copyright © Oakstone Publishing, 2021. All Rights Reserved.

J Cardiovasc Surg (Torino). 2013 Dec;54(6):679-84.


Endovascular first as "preliminary approach" for
critical limb ischemia and diabetic foot.
Setacci C1, Sirignano P, Galzerano G, Mazzitelli G, Sauro
L, de Donato G, Benevento D, Cappelli A, Setacci F.

In BASIL-1 bypass after “endofailure” was highly significantly


less successful than primary bypass
Endovascular is not a “free shot”
Amputation free survival Overall survival

Bypass first
Bypass first

Bypass Bypass
after after
angioplasty angioplasty

473
Copyright © Oakstone Publishing, 2021. All Rights Reserved.

Sponsored by the National Heart Lung and Blood Institute

474
Copyright © Oakstone Publishing, 2021. All Rights Reserved.

Conclusions

• PAD and CLI are associated with morbidity and


mortality
– Prevalence is increasing
• Prompt diagnosis is critical
• Referral for revascularization assessment indicated
for ischemic rest pain or tissue loss associated with
Chronic Limb Threatening Ischemia

475
Copyright © Oakstone Publishing, 2021. All Rights Reserved.

Venous Disease
Louis L. Nguyen, MD, MBA, MPH, FACS, FSVS
Vascular and Endovascular Surgery
Associate Chair for Digital Health Systems, DOS
Associate Professor of Surgery, HMS

Disclosures
Dr. Nguyen has nothing to disclose.

476
Copyright © Oakstone Publishing, 2021. All Rights Reserved.

Acute venous thromboembolism (VTE)


&
Chronic venous disease (CVD)

Acute Venous Thromboembolism (VTE)


• Rudolf Virchow
• Stasis, endothelial injury, hypercoagulability

• Incidence (>900,000 annual cases in US)


• DVT: 56 per 100K
• PE: 66 per 100K
• Mortality
• 296,000 annual deaths in the US
• 12% In-hospital mortality
• 33% of deaths occur post discharge
• Recurrence
• 30% over 10 years

477
Copyright © Oakstone Publishing, 2021. All Rights Reserved.

Caprini Score

VTE Prophylaxis
• Graduated compression stockings (GCS)
• Effective alone in moderate risk patients (gen surg)
• Intermittent pneumatic compression (IPC)
• Mechanism = Stimulates fibrinolytic therapy
• Effective in higher risk patients
• Effective where anticoagulation is contraindicated
• Unfractionated heparin (UFH)
• 5000IU BID (moderate risk pts) or 5000IU TID (high risk pts)
• 50% RR reduction of DVT
• Low-molecular weight heparin (LMWH)
• Enoxaparin 40mg QD
• Ortho: 40mg pre-op; 30mg Q12 periop; 40mg QD post-op x 3 weeks
• 70% RR reduction of DVT

478
Copyright © Oakstone Publishing, 2021. All Rights Reserved.

Clinical Assessment
• Demographics & current issues
• See Caprini Score
• History
• Past history of DVT
• Physical exam
• Calf pain: Sensitivity 75-91%; specificity 3-87%
• Calf swelling: Sensitivity 35-97%; specificity 8-88%

Diagnostic tests
• D-dimer
• Sensitivity 60-96%, low specificity
• Venous duplex
• Sensitivity 90%, specificity 93.8%
• Sensitivity decreases in asymptomatic patients  false pos
• CT Venography
• Sensitivity 95%, specificity 95%
• Better for proximal veins, not as good for calf veins
• MR Venography
• Sensitivity 95%, specificity 95%
• Better for proximal veins, not as good for calf veins

479
Copyright © Oakstone Publishing, 2021. All Rights Reserved.

Acute or Chronic Clot?

Medical Treatment of DVT


• Leg elevation
• Mobilization with compression
• Unfractionated heparin (UFH) IV
• 80U/kg bolus and 18U/kg/hr infusion with PTT adjustment
• Low molecular weight heparins (LMWH)
• 1mg/kg SC Q12 or 1.5 mg/kg SC QD
• Warfarin
• New oral anticoagulants (DOAC)
• Same or superior safety and efficacy as warfarin

480
Copyright © Oakstone Publishing, 2021. All Rights Reserved.

Direct Oral Anticoagulants (DOAC)


• Direct Factor Xa inhibitor
• Apixaban (Eliquis)
• T-Max: 3-4 hours ; Half life: 8-15 hours
• Rivaroxaban (Xarelto)
• T-Max: 2-4 hours; Half life: 5-13 hours
• Edoxaban (Savaysa)
• T-Max: 1-2 hours; Half life: 10-14 hours
• Reversal agents
• Charcoal (within 2-6 hours)
• Prothrombin complex concentrate (Kcentra)
• Recombinant activated Factor X : Andexanet alfa (Andexxa)
• Direct Thrombin (IIa) inhibitor
• Dabigatran (Pradaxa)
• T-Max: 1-3 hours; Half life: 12-17 hours
• Reversal: Charcoal, idarucizumab (Praxbind)

Treatment of Calf DVT


• Progression from calf to proximal veins
• 15% of symptomatic patients in 2 weeks
• Risk of progression
• Positive D dimer
• Close to proximal vein, clot 5cm in length, multiple veins
• Cancer, history of VTE
• Admitted patients

• Anticoagulation OR repeat Duplex for clot extension

481
Copyright © Oakstone Publishing, 2021. All Rights Reserved.

Treatment of Proximal DVT


• Femoral popliteal segment
• 3 months of anticoagulation
• Role of flexible treatment duration unclear

Treatment of Iliofemoral DVT


• Above the inguinal ligament
• 3 months of anticoagulation
• Consider catheter-directed thrombolysis
• Evaluate for May-Thurner syndrome

482
Copyright © Oakstone Publishing, 2021. All Rights Reserved.

Cather-directed treatment
• For iliofemoral DVT and axillary-subclavian DVT
• Thrombolysis
• Mechanical
• AngioJet
• EKOS
• Suction
• Penumbra Indigo
• Capture
• Inari

Treatment of Thrombophlebitis
• Superficial thrombophlebitis
• Stockings, heat
• Consider anticoagulation if close to deep venous system (<5cm) to SFJ
• Traumatic/Iatrogenic thrombophlebitis
• Remove the catheter
• Suppurative thrombophlebitis
• Remove the catheter, give abx
• Vein excision is rarely needed
• Migratory thrombophlebitis
• Associated with cancer
• Associated with vasculitis (Behcet’s, Buerger’s, Polyarteritis nodosa)

483
Copyright © Oakstone Publishing, 2021. All Rights Reserved.

Post-thrombotic Syndrome (PTS)


• Chronic venous disease (CVD)

• Prevention
• Compression stockings
• 30-40mm Hg x 2 years
• 50% reduction in PTS

Chronic Venous Disease (CVD)

Telangiectasias / Varicose veins


Reticular veins (>3mm)
(<2mm)

Pigmentation Ulceration

Images from Bergan et al NEJM 355;5 2006

484
Copyright © Oakstone Publishing, 2021. All Rights Reserved.

The Problem
• Prevalence
• Telangiectasias : 80% in men and 85% in women
• Varicose veins : 16% in men and 40% of women
• Ankle edema : 7% in men and 16% in women
• Venous ulcers : 1%
• Estimated direct cost of CVD: $150M-$1B
• Risk factors
• Caucasians, older age, female gender, obesity, pregnancy,
prolonged standing, taller height

Symptoms
• None (cosmetic concerns only)
• Local symptoms (over veins)
• Pain, burning, tingling, itchiness
• Leg symptoms
• Throbbing, heaviness, edema, fatigue

485
Copyright © Oakstone Publishing, 2021. All Rights Reserved.

History
• Prior DVT
• Prior superficial venous complications
• Hemorrhage, thrombophlebitis, ulceration
• Multiparity
• Obesity
• Family history

Superficial Venous Anatomy

486
Copyright © Oakstone Publishing, 2021. All Rights Reserved.

Physical Exam

Venous or arterial?
Venous or arterial?

When Veins Go Bad

Reflux Stasis
Obstruction

487
Copyright © Oakstone Publishing, 2021. All Rights Reserved.

Venous Hypertension
• Etiology
• 45% Superficial reflux only
• 12% Deep reflux only
• 43 % Both
• Hydrostatic pressure
• Hydrodynamic pressure

488
Copyright © Oakstone Publishing, 2021. All Rights Reserved.

CEAP Classification
• Clinical classification
• Etiology
• Congenital
• Primary
• Secondary

• Anatomic distribution
• Superficial veins
• Deep veins
• Perforator veins

• Pathologic mechanism
• Reflux
• Obstruction
• Reflux & Obstruction

CEAP Clinical Classification

489
Copyright © Oakstone Publishing, 2021. All Rights Reserved.

Revised Vein Clinical Severity Score

VCSS better for changes in disease severity and QoL

Reflux Ultrasound
• Deep venous system
• DVT
• Congenital abnormalities
• Superficial System
• GSV
• SSV
• AASV
• Perforators

490
Copyright © Oakstone Publishing, 2021. All Rights Reserved.

> 0.5 secs = reflux

Components of a Reflux US
• Presence or absence of reflux (and duration of reflux)
• GSV, SSV, AASV
• Vein size
• Vein depth from skin
• Vein tortuosity
• Discontinuous vein
• Varicose veins

491
Copyright © Oakstone Publishing, 2021. All Rights Reserved.

Telangiectasias and Reticular Veins


Causes
 Estrogen
 Steroids
 Hereditary
Treatments
Sclerotherapy
Hypertonic saline
Sodium tetradecyl sulfate
(Sotradecol)
Polidocanol (Asclera)
Phototherapy
Intense pulse light (IPL)
Evaluate & treat
superficial venous system

Varicose veins

• Causes
• Valve failure

• Treatments
• Compression stockings
• Evaluate and treat the
GSV/SSV first
• Phlebectomy
• Sclerotherapy

492
Copyright © Oakstone Publishing, 2021. All Rights Reserved.

Elastic Compression
Grade I 8-15 mmHg Heaviness & fatigue

Grade II 15-20 mmHg DVT prevention

Grade III 20-30 mmHg Mild varicosities, mild edema

Grade IV 30-40 mmHg Varicosities, edema, pigmentation,


post-procedure

Grade V 40-50 mmHg Reversible lymphedema, ulcerations

Grade VI 50-60 mmHg Irreversible lymphedema

Stocking Tips & Tricks


• Put them on in the morning
• Techniques
• Invert the stocking; find the heel, slide over foot, roll up the
rest
• Use rubber gloves (eg dishwashing gloves)
• Wear them as a part of your work “uniform”
• Take them off at night
• Replace stockings every 3-6 months

493
Copyright © Oakstone Publishing, 2021. All Rights Reserved.

Not Your Grandmother’s Stockings

494
Copyright © Oakstone Publishing, 2021. All Rights Reserved.

When to do Ligation & Stripping?


• Large saphenofemoral junction and proximal GSV
• Percutaneous methods less effective and higher EHIT
• Presence of groin varicosities
• Prior failed endovenous techniques (maybe)
• SSV too short to treat with endovenous catheter

Endovenous Treatment
• Endovenous with heat source
• Endovenous laser treatment (EVLT) (Diomed)
• Radiofrequency ablation (RFA) (VNUS)
• Radiofrequency ablation (RFA) (ClosureFast)

495
Copyright © Oakstone Publishing, 2021. All Rights Reserved.

Non-Thermal Non-Tumescent Venous Ablation

• Tissue Adhesive Closure


• Cyanoacrylate (Venaseal)

• Mechanicochemical Ablation (MOCA)


• Catheter + Sotradecol (Clarivein)

• Injectable Foam
• Polidocanol (Varithena)

Advantages of Non-thermal
ablation
• Lower pain scores
• Faster procedure
• No collateral injury (skin, nerve)
• Can treat veins near the skin
• Can treat below the calf
• Self contained (no RFA / EVLT machine)

• Similar safety and efficacy


• Most insurance carriers are now covering

496
Copyright © Oakstone Publishing, 2021. All Rights Reserved.

Outcomes
Closure Adverse Events Advantageous in

High ligation & 85% at 1 year Cutaneous Proximal reflux


stripping neurosensory loss
Proximal varix
4-25%
EVLT 90-95% at 1 year EHIT 0.5-1% Cosmetic patients

VNUS 80-90% at 1 year EHIT 1% Cosmetic patients

US guided 76% at 1 year Cosmetic patients


sclerotherapy

Endothermal Heat Induced Thrombosis (EHIT)

• Class 1 (no Rx)


• Flush with the CFV
• Class 2 (anticoag + FU US)
• < 50% of the CFV
• Class 3 (anticoag + FU US)
• >50% of the CFV
• Class 4 (anticoag + FU US)
• Occlusion of the CFV

• Risk Factors for EHIT


• Vein diameter >10mm, procedure time of > 40 mins (+ phlebectomy)

497
Copyright © Oakstone Publishing, 2021. All Rights Reserved.

Microphlebectomy

Pigmentation
• Occurs in the gaiter distribution
• Ferritin and ferric oxide deposition

• Treatments
• Compression stockings
• Evaluate and treat the
GSV/SSV and perforators

498
Copyright © Oakstone Publishing, 2021. All Rights Reserved.

Ulceration

• Delayed tissue healing

• Treatments
• Compression stockings
• Unna boot
• Evaluate and treat the
GSV/SSV and perforators
• Linton procedure
• Subfascial Endoscopic
Perforator Surgery (SEPS)
• GSV ablation

Unna boot
• German dermatologist Paul
Gerson Unna (1850-1929)

• Contents
• Zinc oxide
• Calamine
• Glycerin

• Covered with Ace bandage


• Changed weekly

499
Copyright © Oakstone Publishing, 2021. All Rights Reserved.

Care Pathway for CVD


• Initial evaluation
• Compression stockings to knee (20-30mmHg)
• Discuss behavioral changes
• Regular walking, leg elevation, weight loss
• Obtain reflux US

• Treatment
• Treat GSV/SSV/AASV first
• Non-thermal methods preferred
• Post-closure US <1 week
• FU 1-2 months for phlebectomy/sclerotherapy

• Maintenance
• Compression stockings at work

Economics of CVD
• Intervention is more cost-effective than conservative
management
• For wound healing, QoL, recurrence
• Procedure> sclerotherapy > compression

• Insurance is resistant to pay for some venous


procedures

• Venous care is good business


• Office-based procedures
• Facility collections 10X as professional collections

500
Copyright © Oakstone Publishing, 2021. All Rights Reserved.

Vascular Access
Mohamad A Hussain, MD, PhD, FRCSC, RPVI

Division of Vascular & Endovascular Surgery and the Centre for Surgery and Public Health,
Brigham and Women’s Hospital, Harvard Medical School, Boston, MA, USA

Disclosures

• None

501
Copyright © Oakstone Publishing, 2021. All Rights Reserved.

ESRD in the US

• United States Renal Data System (USRDS) — 2018

• 131,000 incidence cases

• 785,000 prevalence

• 70% dialysis vs 30% transplant

• 80% started HD with a CVC

Johansen KL et al. US Renal Data System 2020 Annual Data Report: Epidemiology of Kidney Disease in the United
States. Am J Kidney Dis. 2021;77(4 Suppl 1):A7-A8.

ESRD Prevalence by Modality


62% In-Centre HD

29% Transplant

8% PD

1% Home HD

502
Copyright © Oakstone Publishing, 2021. All Rights Reserved.

ESKD Life-Plan

Lok et a. KDOQI. Am J Kidney Dis. 2020;75(4 Suppl 2):S1-S164

503
Copyright © Oakstone Publishing, 2021. All Rights Reserved.

Access Options

1. Non-tunneled (temporary) venous catheter

2. Tunneled venous catheter

3. AV access: Fistula

4. AV access: Graft

Dialysis Catheters

Woo and Rowe (2019). ‘Hemodialysis Access: Dialysis Catheters,’ in Sidawy A. N. and Perler B. A. (9th Ed.) Rutherford’s
Vascular Surgery. Philadelphia: Elsevier, pp. 2315-2323

504
Copyright © Oakstone Publishing, 2021. All Rights Reserved.

Dialysis Catheters
• Site

• Right IJV preferred (best patency, less kinking)

• Femoral vein alternative (longer catheter, 24-31 cm)

• Avoid subclavian vein (negatively influence AVF)

• Advantages

• Suitable for immediate dialysis (AVF or AVG not ready)

• Long-term options for those not a surgical candidate

Dialysis Catheters
• Late Complications

• Central venous stenosis/occlusion, SVC syndrome

• Infection — exit site, tunnel, or bacteremia

• KDOQI recommends <1.5 bloodstream infections / 1000 catheter days

• Catheter occlusion or embolism

• Early Complications

• Catheter misplacement (arterial)

• Pneumothorax, hemothorax, wire embolism, cardiac arrhythmia, cardiac


perforation, thoracic duct laceration, nerve injuries

505
Copyright © Oakstone Publishing, 2021. All Rights Reserved.

Dialysis Catheters

• Patients cannot shower!!

AV Access

• AV access (AVF or AVG) preferable to CVC due to:

• Lower infection rates

• Lower vascular complications (thrombotic & nonthrombotic)

• Timing of AV access creation:

• Ideally when eGFR 15-20 or rapid eGFR (>10/year) decline

Lok et a. KDOQI. Am J Kidney Dis. 2020;75(4 Suppl 2):S1-S164

506
Copyright © Oakstone Publishing, 2021. All Rights Reserved.

Types of AV Access
• AV Fistula (autogenous)

• Direct anastomosis

• Transposed

• AV Graft (prosthetic)

• Straight configuration

• Loop configuration

Guiding Principles of AV Access Surgery


• Ideal timing for surgery

• AVF: 6 months prior to dialysis

• AVG: 4-6 weeks prior to dialysis

• Autogenous > prosthetic

• Upper extremity > lower extremity

• Non-dominant > dominant hand

• As distal as possible to start (radial > brachial based)

• Perform preoperative mapping: artery 2+ mm; vein ideally 3+ mm

507
Copyright © Oakstone Publishing, 2021. All Rights Reserved.

Preoperative Physical Exam


• Arterial system

• Pulse exam, Allen test, bilateral BPs

• Venous system

• Edema, collateral veins, recent venipuncture

• Skin/soft tissues

• Skin integrity, depth of vessels

• Cardiovascular exam for heart failure

Preoperative Imaging
• Venous/arterial US mapping

• Noninvasive, easy to obtain

• Somewhat technician & patient physiology dependent

• Ideally done perioperatively by surgeon

• Venogram

• More anatomical information for AVF planning

• Invasive, need for contrast

• Arterial duplex, finger waveforms, arteriogram

• Consider if abnormal physical arterial exam

508
Copyright © Oakstone Publishing, 2021. All Rights Reserved.

AVF: Snuff Box

Heindel et al. J Vasc Surg 2021;74:947-56

AVF: Radiocephalic

509
Copyright © Oakstone Publishing, 2021. All Rights Reserved.

AVF: Anticubital
Brachiocephalic AVF
Anticubital Anatomy

Radial Artery to Deep Communicating Vein AVF

AVF: Brachiobasilic (1 vs. 2 Stages)


Stage 2: Basilic Vein Superficialization

Stage 1: Creation of
Brachiobasilic vein AVF

510
Copyright © Oakstone Publishing, 2021. All Rights Reserved.

AVG: Forearm Loop

AVG: Straight
Forearm Straight Graft Upper Arm Straight Graft

511
Copyright © Oakstone Publishing, 2021. All Rights Reserved.

AVG: Lower Extremity


A: Femoral Artery-to-Femoral Vein (or GSV) Loop Graft

B: Femoral Artery-to-Femoral Vein (or GSV) Straight Graft

Advantages
• Good patency
• Simple technique
• Patient hands are free during dialysis
• Reasonable for those with no upper arm options

Disadvantages
• Higher infections
• Lower limb ischemia (r/o PAD)
• Steal
• Not ideal for obese patients

AVG: Materials
ePTFE (GORE-TEX) Artegraft GORE ACUSEAL

6 mm straight Collagen Vascular Graft Early Cannulation


4-6 or 7 mm tapered 6 mm (dilates to 8 mm) <24 Hrs.

512
Copyright © Oakstone Publishing, 2021. All Rights Reserved.

HeRO Device
Two Components:
1. Graft — 6mm PTFE
2. Venous outflow — 19F silicone catheter

Indications
• Catheter dependent patients
• Central vein stenosis/occlusion

Contraindications
• Brachial artery < 3mm
• sBP < 100 mm Hg
• Active infection
• LVEF < 20%

Results:
1-Yr Primary patency 20-50%
1-Yr Secondary patency 60-90%
~2.5 reinterventions
Gandhi and Carsten III (2019). ‘Hemodialysis Access: Complex,’ in Sidawy A. N. and Perler B. A. (9th Ed.) Rutherford’s
Vascular Surgery. Philadelphia: Elsevier, pp. 2300-2313

Approach to AV Access Selection

Macsata and Sidway (2019). ‘Hemodialysis Access: General Considerations and Strategies to Optimize Access
Placement,’ in Sidawy A. N. and Perler B. A. (9th Ed.) Rutherford’s Vascular Surgery. Philadelphia: Elsevier, pp. 2288-2298

513
Copyright © Oakstone Publishing, 2021. All Rights Reserved.

Post AV Access Creation


• Assessment of AV access maturity “readiness for use” at 4-6 weeks

• Experienced team — surgeon, dialysis nurse, nephrologist

• Asses for large venous branches & stenosis

• Supplemental US criteria for maturation:

• ‘Rule of 6s’ — 6mm vein diameter, <6mm vein depth, >600 cc/min
flow, >6cm cannulation zone

• UAB criteria — Flow >500 cc/min and vein >4mm diameter

• AVGs — generally ready to use once incisions healed at 4-6 weeks

Lok et a. KDOQI. Am J Kidney Dis. 2020;75(4 Suppl 2):S1-S164

Median Time to RC-AVF Use


150
No HD at AVF Creation
Yes HD at AVF Creation
125 121

100 95
Days

75

50

25

0
Hussain et al. Predictors of Radiocephalic Arteriovenous Fistula Outcomes: First Look into the PATENCY-1 and
PATENCY-2 Randomized Controlled Trials. Vascular Annual Meeting 2021, San Diego, CA

514
Copyright © Oakstone Publishing, 2021. All Rights Reserved.

Patency by AV Access Type

Macsata and Sidway (2019). ‘Hemodialysis Access: General Considerations and Strategies to Optimize Access
Placement,’ in Sidawy A. N. and Perler B. A. (9th Ed.) Rutherford’s Vascular Surgery. Philadelphia: Elsevier, pp. 2288-2298

Patency Loss by Vein Diameter

Hussain et al. Predictors of Radiocephalic Arteriovenous Fistula Outcomes: First Look into the PATENCY-1 and
PATENCY-2 Randomized Controlled Trials. Vascular Annual Meeting 2021, San Diego, CA

515
Copyright © Oakstone Publishing, 2021. All Rights Reserved.

Interventions Post Snuffbox RC-AVF

Heindel et al. J Vasc Surg 2021;74:947-56

KDOQI Guidance on Interventions

Lok et a. KDOQI. Am J Kidney Dis. 2020;75(4 Suppl 2):S1-S164

516
Copyright © Oakstone Publishing, 2021. All Rights Reserved.

AVF Failure is Common


Early Identification is Key

Failing AV Access
• Mechanisms of AVF Failure

1. Venous outflow stenosis (intimal hyperplasia)

2. Arterial inflow stenosis (atherosclerosis)

3. Cannulation location issue

• Vein too small, short, deep, or aneurysmal

• AVG stenosis or aneurysm

517
Copyright © Oakstone Publishing, 2021. All Rights Reserved.

Early (<30D) Thrombosis


• 5% in Hemodialysis Fistula Maturation (HFM) Study

• Risk factors:

• Females

• Forearm > upper arm

• Smaller artery diameter

• Small vein 2-3 mm diameter

• Intraoperative protamine use


Farber et al. J Vasc Surg 2016;63:163-70

Late Thrombosis

• AVF — anastamotic stenosis, outflow vein stenosis

• AVG — venous anastomosis stenosis

• Central vein stenosis

518
Copyright © Oakstone Publishing, 2021. All Rights Reserved.

Interventions for Failing Access


• Endovascular

• Angioplasty — anastomosis, outflow vein, central vein

• Stenting — covered stent of venous anast or central vein

• Thrombectomy — acute thrombosis (AVG > AVF)

• Surgical

• Maturation procedures: side branch ligation, superficialization

• Patch angioplasty or interposition graft +/- thrombectomy

• Hybrid

• Surgical thrombectomy/revision with outflow vein angioplasty/stenting

Drug-Coated Balloon Angioplasty

Multicentre RCT
N = 330

Superior 6-month target


lesion primary patency
with DEB vs. plain balloon:

+23% (13% to 33%)

Lookstein et al. N Eng J Med. 2020; 383(8):733-742

519
Copyright © Oakstone Publishing, 2021. All Rights Reserved.

Infection Diagnosis

• AVG > AVF

• Local signs — induration, exposed


graft/sinus, erythema,
pseudoaneurysm, hemorrhage

• Remote signs — septic emboli,


sepsis

Infection Treatment
• Hemorrhage — direct pressure, BP cuff, elevation, OR

• Excision

• Total

• Subtotal

• Partial

• Temporary CVC

520
Copyright © Oakstone Publishing, 2021. All Rights Reserved.

Pseudoaneurysms
• 2-10%

• Trauma from repeated punctures or poor technique

• Treat if skin erosion (interposition graft vs. aneurysmorrhaphy)

Aneurysmorrhaphy

521
Copyright © Oakstone Publishing, 2021. All Rights Reserved.

Venous Hypertension

• Due to central venous stenosis

• CVC-related endothelial trauma

• Ligation of AVF vs. salvage

• Endovascular (angioplasty +/- stenting)

• Open (direct reconstruction, bypass, or decompression)

Lok et a. KDOQI. Am J Kidney Dis. 2020;75(4 Suppl 2):S1-S164

Neuropathy

• Mononeuropthies

• Compression & entrapment

• Carpel tunnel syndrome 10x more


common in HD patients

• Venous HTN —> congestion —>


median N compression & chronic
irritation on carpal ligament

522
Copyright © Oakstone Publishing, 2021. All Rights Reserved.

Ischemic Monomelic Neuropathy


• Rare — 0.5%

• Acute vascular compromise of


neurological structures

• Pre-existing marginal perfusion of


nerves compromised by additional
flow diversion by access

• Delayed recognition —>


irreversible neurological deficits in
median, ulnar, radial nerves

Ischemic Monomelic Neuropathy


• Presentation

• Acute pain, sensory-motor loss within hours Postop

• Warm hand with palpable pulse

• RFs: diabetes, PAD, brachial-based AVF

• Treatment

• Urgent AVF ligation

523
Copyright © Oakstone Publishing, 2021. All Rights Reserved.

Access Related Hand Ischemia (Steal)

• “Physiological steal” or reverse flow is normal after AVF

• Ischemic steal — inadequate collateral circulation

• 7% in HFM

• 4% needed intervention

• RFs:

• Female, diabetes, PAD, brachial-based AVF, AVG

Huber et al. J Vasc Surg 2016;64:1050-1058

Access Related Hand Ischemia (Steal)

Grade of Steal Presentation

Grade 0 No symtoms

Grade 1 Mild — cool extremity, minor symptoms

Moderate — claudication, intermittent


Grade 2
symptoms during HD

Grade 3 Severe — rest pain, tissue loss

524
Copyright © Oakstone Publishing, 2021. All Rights Reserved.

Access Related Hand Ischemia (Steal)

• Diagnosis

• Reduced/absent pulse; improves w/ AVF occ.

• PPG — digital waveforms reduced; improve w/ AVF occ.

• Pulse oximetry < 94% (improves w/ AVF occlusion)

• DUS — arterial waveforms reduced; improve w/ AVF occ.

• Angiogram — inflow stenosis in 20%

525
Copyright © Oakstone Publishing, 2021. All Rights Reserved.

Treatment of Steal
• Diameter reduction (banding)

• DRIL (distal revascularization interval ligation)

• RUDI (revision using distal inflow)

• DRAL (distal radial artery ligation)

• PAI (proximalization of arterial inflow)

• Treatment of inflow stenosis (angioplasty)

• AVF ligation

AVF Banding

Soo Hoo et al. Vascular Annual Meeting 2021, San Diego, CA

526
Copyright © Oakstone Publishing, 2021. All Rights Reserved.

DRIL and RUDI

Harris and Rivero (2019). ‘Hemodialysis Access: General Considerations and Strategies to Optimize Access Placement,’
in Sidawy A. N. and Perler B. A. (9th Ed.) Rutherford’s Vascular Surgery. Philadelphia: Elsevier, pp. 2335-2348

Proximalization of Arterial Inflow

Ligation of AV anastomosis

Conversion of inflow from more proximal level


using small caliber prosthetic interposition

Relies on large diameter proximal artery with


higher capacity & lower pressure drop across
AV anastomosis

Harris and Rivero (2019). ‘Hemodialysis Access: General Considerations and Strategies to Optimize Access Placement,’
in Sidawy A. N. and Perler B. A. (9th Ed.) Rutherford’s Vascular Surgery. Philadelphia: Elsevier, pp. 2335-2348

527
Copyright © Oakstone Publishing, 2021. All Rights Reserved.

Conclusion

• AV access creation and maintenance is a key component of


the ESKD Life-Plan

• Consider vascular anatomy, patient preferences,


physician/program expertise, and account for potential access
complications and remediations

Vascular Access
Mohamad A Hussain, MD, PhD, FRCSC, RPVI

Email: [email protected]
Twitter: ma_hussainMD

528
Copyright © Oakstone Publishing, 2021. All Rights Reserved.

Comprehensive Review of Surgery:


Plastic Surgery
Matthew J. Carty, MD
Staff Surgeon, Division of Plastic Surgery
Brigham and Women’s Hospital
Associate Professor of Surgery
Harvard Medical School

I have no conflicts of interest

I have research funding from PRORP, DARPA, CDMRP,


and intramural sources

529
Copyright © Oakstone Publishing, 2021. All Rights Reserved.

plas·tic
/’plastic/
adjective
1. (of a substance or material) easily
shaped or molded

“We restore, rebuild and


make whole those parts
which nature hath given,
but which fortune has
taken away. Not so much
that it may delight the
eye, but that it might buoy
up the spirit, and help the
mind afflicted.”
Gaspare
Tagliacozzi 1545-
1599

530
Copyright © Oakstone Publishing, 2021. All Rights Reserved.

“Service to society is the


rent we pay for living on
this planet.”
Joseph
Murray
1919-2012

Breast Facial Trauma

Burns Hand

Craniofacial Lower Extremity

Head and Neck Skin Neoplasm

Cosmetic Trunk

531
Copyright © Oakstone Publishing, 2021. All Rights Reserved.

Goals of Reconstruction

• Optimize function
• Minimize pain
• Restore form
• Maximize safety

532
Copyright © Oakstone Publishing, 2021. All Rights Reserved.

Factors Affecting Wound


Healing
LOCAL FACTORS GENERAL FACTORS

• Blood supply • Age


• Necrotic burden • Endocrine function
• Fluid collection • Drug therapy
• Infection • Sepsis
• Previous or • Major organ failure
concurrent radiation • Obesity
• Mechanical stress • Malignancy

533
Copyright © Oakstone Publishing, 2021. All Rights Reserved.

FREE TISSUE TRANSFER

REGIONAL FLAP COVERAGE

“LADDER”
LOCAL FLAP COVERAGE

SKIN GRAFTING

NEGATIVE PRESSURE WOUND


THERAPY

HEALING BY SECONDARY
INTENTION

PRIMARY WOUND CLOSURE

FREE TISSUE TRANSFER

REGIONAL FLAP COVERAGE

LOCAL FLAP COVERAGE

SKIN GRAFTING

NEGATIVE PRESSURE WOUND


THERAPY

HEALING BY SECONDARY
INTENTION

PRIMARY WOUND CLOSURE

534
Copyright © Oakstone Publishing, 2021. All Rights Reserved.

FREE TISSUE TRANSFER

REGIONAL FLAP COVERAGE

LOCAL FLAP COVERAGE

SKIN GRAFTING

NEGATIVE PRESSURE WOUND


THERAPY

HEALING BY SECONDARY
INTENTION

PRIMARY WOUND CLOSURE

535
Copyright © Oakstone Publishing, 2021. All Rights Reserved.

• Begins with violation of blood vessel


• Platelet aggregation and fibrin strand
adherence
• Blood transformed from liquid to gel
• Thrombus formation retains platelets
and blood cells in wound area

• Begins immediately after injury with


leakage of transudate and localized
swelling
• Fluid engorgement recruits
macrophages and fibroblasts to site
for clearance and repair
• Combined actions promote edema,
heat, pain and erythema

• Localized angiogenesis creates


environment for granulation tissue
formation
• Fibroblasts produce collagen and
extracellular matrix
• Myofibroblasts promote wound
contraction
• Epithelial cells resurface injury site

• Collagen is remodelled from type III


to type I via fibroblast
metalloproteinases
• Repair cells are cleared by apoptosis
• Full remodeling generally takes
approximately 1 year

536
Copyright © Oakstone Publishing, 2021. All Rights Reserved.

Comparison of Healing by
Primary Closure vs Secondary
Intention
SECONDARY
PRIMARY CLOSURE
INTENTION

Cleanliness • Clean • Contaminated


Margins • Surgically clean • Irregular
Sutures • Used • Not used
Granulation • Scant • Heavy
Appearance • Linear scar • Irregular scar
Complications • Infrequent • More frequent

537
Copyright © Oakstone Publishing, 2021. All Rights Reserved.

FREE TISSUE TRANSFER

REGIONAL FLAP COVERAGE

LOCAL FLAP COVERAGE

SKIN GRAFTING

NEGATIVE PRESSURE WOUND


THERAPY

HEALING BY SECONDARY
INTENTION

PRIMARY WOUND CLOSURE

538
Copyright © Oakstone Publishing, 2021. All Rights Reserved.

• Removal of
infectious material,
exudate an interstitial
fluid
• Creation of protected
wound healing
environment
• Promotion of
increased local
angiogenesis
• Inducement of
wound contracture

539
Copyright © Oakstone Publishing, 2021. All Rights Reserved.

Potential Purposes of Negative


Pressure Wound Therapy

1 2 3
DOWNGRADE
ACHIEVE FULL
RECONSTRUCTIV BUY TIME
HEALING
E REQUIREMENT

FREE TISSUE TRANSFER

REGIONAL FLAP COVERAGE

LOCAL FLAP COVERAGE

SKIN GRAFTING

NEGATIVE PRESSURE WOUND


THERAPY

HEALING BY SECONDARY
INTENTION

PRIMARY WOUND CLOSURE

540
Copyright © Oakstone Publishing, 2021. All Rights Reserved.

Skin Grafting: Definitions


Transfer of non-vascularized epidermal ± dermal
tissue from one corporeal site to another

SPLIT
FULL THICKNESS
THICKNESS

• Epidermis only • Dermis and


• Harvested with epidermis
dermatome • Sharp harvest
• Usually meshed • Usually not
to expand meshed
coverage • Small defects
• Large defects • More resistant to
• Contracture contracture
common

541
Copyright © Oakstone Publishing, 2021. All Rights Reserved.

Stages of Skin Graft Healing

542
Copyright © Oakstone Publishing, 2021. All Rights Reserved.

FREE TISSUE TRANSFER

REGIONAL FLAP COVERAGE

LOCAL FLAP COVERAGE

SKIN GRAFTING

NEGATIVE PRESSURE WOUND


THERAPY

HEALING BY SECONDARY
INTENTION

PRIMARY WOUND CLOSURE

Local Flap: Definition


Coverage option in which tissue
immediately adjacent to or near the primary
defect is utilized for closure, usually with
random blood supply

543
Copyright © Oakstone Publishing, 2021. All Rights Reserved.

544
Copyright © Oakstone Publishing, 2021. All Rights Reserved.

FREE TISSUE TRANSFER

REGIONAL FLAP COVERAGE

LOCAL FLAP COVERAGE

SKIN GRAFTING

NEGATIVE PRESSURE WOUND


THERAPY

HEALING BY SECONDARY
INTENTION

PRIMARY WOUND CLOSURE

Regional Flap: Definition


Coverage option in which tissue is
harvested from a site that is not adjacent to
or near the primary defect, usually with an
axial blood supply

545
Copyright © Oakstone Publishing, 2021. All Rights Reserved.

546
Copyright © Oakstone Publishing, 2021. All Rights Reserved.

FREE TISSUE TRANSFER

REGIONAL FLAP COVERAGE

LOCAL FLAP COVERAGE

SKIN GRAFTING

NEGATIVE PRESSURE WOUND


THERAPY

HEALING BY SECONDARY
INTENTION

PRIMARY WOUND CLOSURE

Free Tissue Transfer:


Definition

Coverage option in which tissue is


harvested from a site that is not
adjacent to or near the primary
defect and transferred via
microsurgical techniques

547
Copyright © Oakstone Publishing, 2021. All Rights Reserved.

548
Copyright © Oakstone Publishing, 2021. All Rights Reserved.

FREE TISSUE TRANSFER

REGIONAL FLAP COVERAGE

“LADDER”
“ELEVATO LOCAL FLAP COVERAGE

R”
SKIN GRAFTING

NEGATIVE PRESSURE WOUND


THERAPY

HEALING BY SECONDARY
INTENTION

PRIMARY WOUND CLOSURE

549
Copyright © Oakstone Publishing, 2021. All Rights Reserved.

Thoracobrachial Pterygoplasty Powered by


Muscle Transposition Flaps

Bone
Muscle

Nerve

Vascularity

Lymphatic
Tendo
s
n

550
Copyright © Oakstone Publishing, 2021. All Rights Reserved.

Thank You!

Brigham & Women’s


Faulkner Hospital
1153 Centre Street, Suite
2B
Jamaica Plain, MA 02186

(617) 983-4555
[email protected]

551
Copyright © Oakstone Publishing, 2021. All Rights Reserved.

Gynecologic Challenges for General Surgeons

Jim Greenberg, MD, FACOG


Brigham and Women’s Hospital
Harvard Medical School

Financial Disclosures
I have no financial conflicts of interest to disclose

552
Copyright © Oakstone Publishing, 2021. All Rights Reserved.

Goals
Recognizing common diagnoses
Emergency surgery
Unanticipated intra-operative findings
Surgery in pregnancy
Tips to avoiding trouble

Fertility
is what separates
gynecology
from general surgery

553
Copyright © Oakstone Publishing, 2021. All Rights Reserved.

Recognizing common diagnoses


Emergency surgery
Unanticipated intra-operative findings
Surgery in pregnancy
Tips to avoiding trouble

This is how smart doctors think about Ob/Gyn

554
Copyright © Oakstone Publishing, 2021. All Rights Reserved.

Pregnant
This is how I think about OB/GYN
1

4 6
7
Bleeding 2 3 Pain
5

1 Normal Pregnancy
2 Menstruation, DUB, Cervical/Endometrial CA 8
3 Ovarian Cyst, PID, Appendicitis
4 Ectopic, Miscarriage, Placenta Previa or abruption
5 Dysmenorrhea, Endometriosis
6 Ectopic, Labor
7 Ectopic, Miscarriage, Labor with placenta previa or abruption
8 Fibroids, Menopause, Pap smear, amenorrhea, discharges

Bleeding

555
Copyright © Oakstone Publishing, 2021. All Rights Reserved.

Bleeding
Pre-menopausal

HCG(+)

1st 3rd

Ectopic Previa
Miscarriage Complete
complete Partial
incomplete Vasa previa
missed Abruption
Implantation* Pre-term labor
Intercourse Incomp cervix
GTD ROM
Intercourse

Bleeding
Pre-menopausal

HCG(+) HCG(-)

1st 3rd

Ectopic Previa DUB


Miscarriage Complete anovulation
complete Partial Fibroids
incomplete Vasa previa Polyps
missed Abruption Trauma
Implantation* Pre-term labor Cancer
Intercourse Incomp cervix cervix
GTD ROM uterus
Intercourse Blood Dyscrasias

556
Copyright © Oakstone Publishing, 2021. All Rights Reserved.

Bleeding
Pre-menopausal Post-menopausal

HCG(+) HCG(-)

1st 3rd

Ectopic Previa DUB Cancer


Miscarriage Complete anovulation Cancer
complete Partial Fibroids Cancer
incomplete Vasa previa Polyps cervix
missed Abruption Trauma uterus
Implantation* Pre-term labor Cancer tubal
Intercourse Incomp cervix cervix Atrophy
GTD ROM uterus HRT
Intercourse Blood Dyscrasias Trauma

Pain

557
Copyright © Oakstone Publishing, 2021. All Rights Reserved.

Pelvic Pain

Not Pregnant Pregnant

Not Cyclic Cyclic

RLQ LLQ
Ectopic
PID* PID* Endometriosis Heterotopic
Torsion Torsion Adenomyosis Miscarriage
Appy Diverticulitis Function cysts Abruption
Everything Everything Ovarian Torsion
Appy
* Jacobson et al

Lund, Sweden – 1960-1967

All patients diagnosed by senior


Diagnosing “PID” clinicians and then underwent
Jacobson, 1980 laparoscopy within 24 hours prior to
treatment

905 patients in the study

558
Copyright © Oakstone Publishing, 2021. All Rights Reserved.

The true positive rate of clinically


diagnosed of “PID” is of only 65%

Of the false positive “PID” patients, 12%


had other stuff including appendicitis
(3%), endometriosis (2%), etc.
Diagnosing “PID”
Jacobson, 1980
Of the false positive “PID” patients, 23%
had no identifiable pelvic pathology

The false negative rate of “PID” was 15%

• At least 35% of the time you are WRONG


Diagnosing • Onset usually during or within 7 days of
menses
“PID” • Usually bilateral; rarely more than 2 weeks
• Definitively diagnose conservatively
• Treat liberally

559
Copyright © Oakstone Publishing, 2021. All Rights Reserved.

Recognizing common diagnoses


Emergency surgery
Unanticipated intra-operative findings
Surgery in pregnancy
Tips to avoiding trouble

• Acute surgical intervention needed


but no OB/GYN available

Emergency • Ectopic pregnancy


Surgery • Hemorrhagic ovarian cyst
• Ovarian torsion
• Cesarean section

560
Copyright © Oakstone Publishing, 2021. All Rights Reserved.

Ectopic Pregnancy

Ectopic • Appropriate patients can be treated


conservatively with methotrexate
Pregnancy • NOT YOUR PROBLEM – YOU
ARE A GENERAL SURGEON
• Hemodynamically unstable patients
need surgery
• Laparoscopy almost always
appropriate
• Salpingectomy vs. salpingostomy
• Recommend salpingectomy*
• Take care to avoid compromising
the ovary

*Ozcan MCH, et. al. A Systematic Review and Meta-analysis of Surgical


Treatment of Ectopic Pregnancy with Salpingectomy versus Salpingostomy.
J Minim Invasive Gynecol. 2021

561
Copyright © Oakstone Publishing, 2021. All Rights Reserved.

Ovarian Torsion
Arterial venous pressure gradient
exacerbates the problem

Ovarian Torsion
Conservative
Un-torse Oophorectomy
• Younger • Older
• Future fertility • Done with childbearing
• Absent 2nd ovary • Normal 2nd ovary
• Concern for malignancy

562
Copyright © Oakstone Publishing, 2021. All Rights Reserved.

Cesarean Section
• You can do this…it’s easy
• Trivial blood loss on entry is irrelevant
• The gravid uterus displaces the bowel
• Blunt dissection is often superior to sharp
• Don’t cut the baby

Pfannenstiel skin incision

Divide rectus fascia laterally

Enter the peritoneum – blunt or sharp

Transverse incision on the uterus

Extend the hysterotomy bluntly vertically


Cesarean Deliver the baby

Section Deliver the placenta

Exteriorize the uterus

Double layer closure of the hysterotomy

Replace the uterus

Close the abdomen

Thank God that you don’t have to do it a lot

563
Copyright © Oakstone Publishing, 2021. All Rights Reserved.

Recognizing common diagnoses


Emergency surgery
Unanticipated intra-operative findings
Surgery in pregnancy
Tips to avoiding trouble

Unanticipated • Endometriosis
intra-operative
• Ovarian cysts/excrescences
findings
• Pedunculated fibroids

564
Copyright © Oakstone Publishing, 2021. All Rights Reserved.

Endometriosis
• Comes in many different
appearances
• Biopsy it

Ovarian cysts

• Nothing good here for


general surgeons
• If possible, phone a
friend
• Premenopausal
patients are
challenging
• Remove in all
postmenopausal
patients
• Rupturing malignant
ovarian neoplasm
upstages
• Containment for
extraction
• DO NOT RUPTURE THE
OVARIAN CYST IF
POSSIBLE

565
Copyright © Oakstone Publishing, 2021. All Rights Reserved.

Ovarian
excrescences
• Biopsy peritoneal
excrescences
• Biopsy excrescences on the
outside of the ovary
• DO NOT RUPTURE THE
OVARIAN CYST

Pedunculated
Fibroids
• No value to keeping it in
• If the stalk is thin enough,
clamp it, cauterize it, cut
it…leave a pedicle if
possible.
• Contained extraction

566
Copyright © Oakstone Publishing, 2021. All Rights Reserved.

Recognizing common diagnoses


Emergency surgery
Unanticipated intra-operative findings
Surgery in pregnancy
Tips to avoiding trouble

Surgery in • Teratogenic risk – almost none


• Miscarriage risk – almost none
pregnancy • Obstetric Outcomes*
• Stillbirth – Every 287 procedures were associated with
one additional stillbirth (stillbirth frequency of 0.6
percent without surgery and 0.9 percent with surgery)
• Preterm delivery – Every 31 surgeries were associated
with one additional preterm birth (preterm delivery
frequency of 7.5 percent without and 11.1 percent with
surgery)
• Low birth weight infant – Every 39 operations were
associated with one additional low birth weight infant
(low birth weight frequency of 5.6 percent without and
8.6 percent with surgery)
• Cesarean delivery – Every 25 surgeries were associated
with one additional cesarean delivery (cesarean delivery
frequency of 23.9 percent without and 28.8 percent
with surgery)

*47K surgeries in 6.5M pregnancies


Balinskaite V, et. al. The Risk of Adverse Pregnancy Outcomes Following
Nonobstetric Surgery During Pregnancy: Estimates From a Retrospective
Cohort Study of 6.5 Million Pregnancies. Ann Surg. 2017;266(2):260.

567
Copyright © Oakstone Publishing, 2021. All Rights Reserved.

Timing:

• Early 2nd trimester optimal


• Avoids erroneous blame for losses or birth defects
in 1st trimester
Surgery in • Avoids monitoring in 3rd trimester

pregnancy Laparoscopy is safe

• Don’t hit the uterus

Monitoring:

• Pre- and post-procedure FH always


• After 22 weeks, continuous intraoperative fetal
monitoring

Pain Management:

• Opioids are safe


• NSAID’s are contraindicated
• Premature closure of the fetal ductus arteriosus

Recognizing common diagnoses


Emergency surgery
Unanticipated intra-operative findings
Surgery in pregnancy
Tips to avoiding trouble

568
Copyright © Oakstone Publishing, 2021. All Rights Reserved.

Check ALWAYS check a pregnancy test

Identify Identify the ureter

Check Always check for the unaffected


Tips to ovary
avoiding Remove Take out tube with ectopic
trouble pregnancy
Don’t Only remove ovaries as a last
Remove resort
Phone If in doubt, phone a friend

вопросов? des questions?


Fragen ?
질문 ? 問題嗎?
Questions?
maswali?
¿ preguntas?
‫؟‬ ‫ﺍﻷﺳﺋﻠﺔ‬ spurningar?

คําถาม ?

569
Copyright © Oakstone Publishing, 2021. All Rights Reserved.

570
Copyright © Oakstone Publishing, 2021. All Rights Reserved.

571
Copyright © Oakstone Publishing, 2021. All Rights Reserved.

572
Copyright © Oakstone Publishing, 2021. All Rights Reserved.

573
Copyright © Oakstone Publishing, 2021. All Rights Reserved.

574
Copyright © Oakstone Publishing, 2021. All Rights Reserved.

575
Copyright © Oakstone Publishing, 2021. All Rights Reserved.

576
Copyright © Oakstone Publishing, 2021. All Rights Reserved.

577
Copyright © Oakstone Publishing, 2021. All Rights Reserved.

578
Copyright © Oakstone Publishing, 2021. All Rights Reserved.

579
Copyright © Oakstone Publishing, 2021. All Rights Reserved.

580
Copyright © Oakstone Publishing, 2021. All Rights Reserved.

581
Copyright © Oakstone Publishing, 2021. All Rights Reserved.

582
Copyright © Oakstone Publishing, 2021. All Rights Reserved.

583
Copyright © Oakstone Publishing, 2021. All Rights Reserved.

584
Copyright © Oakstone Publishing, 2021. All Rights Reserved.

585
Copyright © Oakstone Publishing, 2021. All Rights Reserved.

586
Copyright © Oakstone Publishing, 2021. All Rights Reserved.

587
Copyright © Oakstone Publishing, 2021. All Rights Reserved.

588
Copyright © Oakstone Publishing, 2021. All Rights Reserved.

589
Copyright © Oakstone Publishing, 2021. All Rights Reserved.

590
Copyright © Oakstone Publishing, 2021. All Rights Reserved.

591
Copyright © Oakstone Publishing, 2021. All Rights Reserved.

592
Copyright © Oakstone Publishing, 2021. All Rights Reserved.

593
Copyright © Oakstone Publishing, 2021. All Rights Reserved.

594
Copyright © Oakstone Publishing, 2021. All Rights Reserved.

595
Copyright © Oakstone Publishing, 2021. All Rights Reserved.

596
Copyright © Oakstone Publishing, 2021. All Rights Reserved.

597
Copyright © Oakstone Publishing, 2021. All Rights Reserved.

598
Copyright © Oakstone Publishing, 2021. All Rights Reserved.

599
Copyright © Oakstone Publishing, 2021. All Rights Reserved.

600
Copyright © Oakstone Publishing, 2021. All Rights Reserved.

601
Copyright © Oakstone Publishing, 2021. All Rights Reserved.

602
Copyright © Oakstone Publishing, 2021. All Rights Reserved.

603
Copyright © Oakstone Publishing, 2021. All Rights Reserved.

604
Copyright © Oakstone Publishing, 2021. All Rights Reserved.

605
Copyright © Oakstone Publishing, 2021. All Rights Reserved.

606
Copyright © Oakstone Publishing, 2021. All Rights Reserved.

607
Copyright © Oakstone Publishing, 2021. All Rights Reserved.

608
Copyright © Oakstone Publishing, 2021. All Rights Reserved.

609
Copyright © Oakstone Publishing, 2021. All Rights Reserved.

610
Copyright © Oakstone Publishing, 2021. All Rights Reserved.

611
Copyright © Oakstone Publishing, 2021. All Rights Reserved.

Tumor Biology

Oakstone CME Lecture

Charles H. Yoon, PhD, MD, FACS


2021.10

Disclosures

• None

612
Copyright © Oakstone Publishing, 2021. All Rights Reserved.

Tumor Biology – Signal transduction

• Receptor tyrosine kinase (RTK) / mitogen-activated protein kinase (MAPK)


pathway.
• Evolutionarily conserved (yeast to vertebrates).
• Conveys extra-cellular signal into the cell nucleus for transcriptional
activation.

RTK / MAPK pathway


ligand

Ras cytosol
RTK
SOS GTP RAF
Y Y P
Grb2

Ras
S P
GDP MEK

T P
nucleus
MAPK
Y P

transcription

613
Copyright © Oakstone Publishing, 2021. All Rights Reserved.

Melanoma – Raf and Ras mutations

• Activating Braf mutation (Raf serine-threonine kinase) identified in 50—66%


of metastatic melanoma.
• Over 80% of the Braf activating mutations are in codon 600 (exon 15) and
exchanges a Valine (V) residue for Glutamic acid (E) or Lysine (K).

• Activating Nras mutation (Ras GTPase) identified in 15—20% of metastatic


melanoma, which hold the Ras in the active GTP bound state.
• Majority of Ras mutations are in codon 61 or codon 12, which exchanges
the native Glutamine (Q) and Glycine (G) residues, respectively.

Melanoma – Raf and Ras mutations


ligand

15-20% Ras cytosol

SOS GTP RAF 50-66%


Y Y P
Grb2
RTK
Ras
S P
GDP MEK

T P
nucleus
MAPK
Y P

transcription
Davies et al. Nature 2002 417: 949-954.
Jakob et al. Cancer 2012 118: 4014-4023.

614
Copyright © Oakstone Publishing, 2021. All Rights Reserved.

Melanoma – Braf inhibition

• Braf inhibition showed rapid initial response in majority of patients with


Braf-mutated metastatic melanoma, significantly better than traditional
chemotherapeutic agents.
• However, most tumors developed resistant to therapy after several months.
• Median progression free survival (PFS) to Braf inhibition was ~6 months.
• Very often, when the disease began to show resistance to therapy, the
recurrent growth and progression of the tumor was explosive.
• Disease progression was slowed but the overall survival in longer term
follow-up was essentially unchanged.

Melanoma – Resistance to Braf Inhibition

Chapman el al. N Engl J Med 2011 364: 2507-2516.

615
Copyright © Oakstone Publishing, 2021. All Rights Reserved.

Melanoma – Braf inhibition

• There are several mechanisms of resistance.


• One is caused by the presence of Braf orthologs which can overlap in
function and can partially compensate the loss of signaling via Braf.
• In mammals, three Raf serine/threonine kinase orthologs are present; Raf-
1, Braf, and Craf.
• In melanoma, the blockade of Braf signaling can be partially overcome by
signaling via Craf.
• In fact, activating mutations in Nras activate the downstream MEK via Craf.
• Resistance can also arise via signaling through alternate pathways beyond
the MAPK pathway that bypass Raf blockade.

Resistance – signaling via orthologs of Braf


ligand

Ras Braf cytosol


RTK
SOS GTP RAF Craf
Y Y P
Grb2 Raf-1

Ras
S P
GDP MEK

T P
nucleus
MAPK
Y P

transcription
Kaplan et al. J Biol Chem 287: 41797-41807.

616
Copyright © Oakstone Publishing, 2021. All Rights Reserved.

Melanoma – Braf inhibition

• Resistance can also arise via signaling through alternate pathways that can
bypass Raf blockade.
• In the following slide is a partial representation of signaling networks that
can interact with the MAPK pathway at multiple points. MAPK pathway is
outlined in red.
• These signaling networks have multiple non-linear interactions that regulate
cell growth/proliferation.
• Use of a single target in this network is unlikely to completely shut down
signaling that promotes tumor survival and progression.

Resistance – signaling via additional pathways.

617
Copyright © Oakstone Publishing, 2021. All Rights Reserved.

Melanoma – MEK inhibitors

• Due to the rise of resistance to Braf inhibition, MEK was seen as another
potential target of therapy, being downstream of the MAPK in the RTK
signaling pathway.
• Trametinib was developed as a small molecule inhibitor of MEK.
• MEK inhibition alone offered limited improvement in survival benefit in
comparison to chemotherapy, similar to Braf inhibition alone.

Melanoma – Braf + MEK inhibition

• Combined Braf + MEK inhibition results in longer progression-free survival


(PFS) in metastatic melanoma.
• Braf + MEK inhibitor therapies typically extend PFS to 10—12 months, in
comparison to ~6 months PFS with Braf or MEK inhibitor monotherapy.
• Combined therapy increases treatment toxicity (e.g., fevers, rash,
photosensitivity, diarrhea, myalgia, fatigue, transaminase elevation,
creatinine elevation).
• Combined Braf + MEK inhibition is the current standard for targeted therapy
in Braf V600E/K mutant melanoma (as adjuvant therapy after surgery or as
systemic therapy for unresectable metastatic disease).

618
Copyright © Oakstone Publishing, 2021. All Rights Reserved.

Melanoma – Ras mutation

• Nras mutated melanomas do not respond to therapy targeting Braf.


• MEK inhibitors exhibit limited response rates in Nras mutated melanoma.
• For Nras mutated melanoma, current targeted therapy trials include
combining MEK inhibition with with agents inhibiting the cell cycling and
the PI3K-AKT pathways (e.g., MEK inhibition + CDK4/6 inhibition).

Adenocarinoma – Lung

• In lung adenocarcinoma (non-small cell), somatic mutations in signaling


proteins are often identified. Many are considered driver mutations that
increase aggressive tumor behavior.

• Mutation frequency:
• Kras 25—30 %
• EGFR 15—20 %
• ALK ~5 %
• PI3KCA 3—4%
• Braf 2—3 %
• ROS11—2 %
• MET 2—4 %
• HER2~1 %
• RET 1—2 %

619
Copyright © Oakstone Publishing, 2021. All Rights Reserved.

Adenocarinoma – Lung

• Several targeted therapy options are currently available.


• Inhibitors of the EGFR tyrosine kinase domain, ALK (non-receptor tyrosine
kinase that activates MAPK pathway), and ROS1 (chromosomal
rearrangement of an orphan RTK) are currently approved for treatment of
metastatic tumors.
• In unresectable metastatic disease, these agents show extension of PFS by
several months in comparison to cytotoxic chemotherapy (for example,
targeted therapy against EGFR show response rates approaching 75% with
PFS between 9—13 months, as compared to response rates of ~25 % with
PFS <3—5 months for chemotherapy).
• Inhibitors targeting MET, Braf, MEK, RET, PI3KCA, HER2 are under clinical
trials.

Colorectal Carcinoma

• Colorectal cancer are frequently associated with chromosomal instability,


loss of heterozygosity in tumor suppressor genes, with concomitant
activation of signal transduction proteins.
• Genetic mutations include:
• APC (adenomatous polyposis coli) gene — tumor suppressor interacting with
beta-catenin/WNT signaling (mutation responsible for FAP and AFAP syndromes).
• MUTYH gene – tumor suppressor (hereditary colon cancer).
• MMR (DNA mismatch repair) genes (MLH1, MSH2, MSH6, PMS2).
• Kras.
• p53.
• TGFbeta receptor (RTK).
• EGFR (RTK).
• PI3KCA.

620
Copyright © Oakstone Publishing, 2021. All Rights Reserved.

Gastrointestinal Stromal Tumors (GIST)

• Gastrointestinal stromal tumors (GISTs) are the most common types of


malignant mesenchymal tumors in the gastrointestinal tract.
• 75–80% of patients have mutations in the KIT gene (RTK) in exons 9, 11, 13,
14, 17.
• KIT is activated through binding of its ligand, stem cell factor, to its
extracellular domain, which activates downstream factors, including the
JAK–STAT3, PI3K–AKT–mTOR, RAS–MAPK pathways.
• 5–10% of patients have mutations in the platelet-derived growth factor
receptor a (PDGFRA) gene in exons 12, 14, 18. This RTK has similar
downstream target proteins as KIT.
• The treatment for GISTs includes imatinib, sunitinib, and regorafenib, which
inhibits the kinase domain of the RTKs. Use of these agents improve
outcomes in adjuvant settings (in combination with surgery) and in
unresectable metastatic disease.

Gastrointestinal Stromal Tumors (GIST)

• GIST tumors with RTK mutations are somewhat unusual in that the
response to RTK inhibition is relatively prolonged (PFS ~2 years).
• ~50% of patients treated with RTK inhibitors develop resistance after 2
years of therapy.
• Mechanisms of resistance include loss of KIT expression (tumor becomes
KIT-independent), increased production of ligands for RTK, and secondary
mutations within the RTK which decrease binding of inhibitors.

621
Copyright © Oakstone Publishing, 2021. All Rights Reserved.

Thyroid Carcinoma

• Braf V600E mutation is encountered in ~35 % of patients with papillary


thyroid carcinoma.
• TERT mutation is encountered in 10—15 % patients with papillary thyroid
carcinoma.
• Disease recurrence risk and more advanced disease is associated with Braf
and/or TERT mutated papillary thyroid cancer.
• TERT (telomerase reverse transcriptase) is a subunit of the telomerase
nucleoprotein complex, required for maintenance of telomere length during
cell mitosis/proliferation. Activating promoter mutations in TERT gene are
associated with thyroid cancer.

• Other mutations seen in thyroid carcinoma include mutations include


PI3KCA, RET (RTK), Ras.

RET

• RET is a receptor tyrosine kinase that binds members of the glial cell-
derived neurotrophic factor (GDNF) family of ligands.
• Activating mutations RET are found in:
• papillary thyroid carcinoma (sporadic and familial).

• multiple endocrine neoplasia (MEN) type 2A and type 2B.

• Hirschsprung's disease (congenital lack of intestinal neurons).

622
Copyright © Oakstone Publishing, 2021. All Rights Reserved.

Multiple Endocrine Neoplasia type 2 – RET mutation

• MEN type 2A most often results from C634R mutation in RET (increased
dimerization of RET).
• MEN 2A – medullary thyroid carcinoma (parafollicular or C-cells of thyroid),
pheochromocytoma, primary hyperparathyroidism (may include
Hirshsprung’s disease, cutaneous lichen amyloidosis).

• MEN type 2B are predominantly caused by M918T mutation in RET


(increased substrate binding).
• MEN 2B – medullary thyroid carcinoma, pheochromocytoma, Marfan’s
habitus, intestinal neuromas, mucosal gaglioneuromas.

Multiple Endocrine Neoplasia type 1

• Loss of function mutations in menin1 gene causes MEN type 1.

• Menin1 is a tumor suppressor, whose loss-of-function causes oncogenesis.


• Scaffold protein that functions in histone modification and epigenetic gene
regulation, thought to regulate signaling and cellular processes by altering
chromatin structure through histone modification.
• MEN 1 — parathyroid carcinoma, anterior pituitary tumors, pancreatic
tumors (gastrinoma, insulinoma, glucagonoma, VIPoma). Also associated
with adrenal cortical tumors, neuroendocrine tumors (carcinoids), and
rarely, pheochromocytoma.
• Autosomal dominant trait (heterozygote) with nearly 100 % penetrance but
variable severity.

623
Copyright © Oakstone Publishing, 2021. All Rights Reserved.

Breast Cancer

• Genetic mutations in breast cancer:


• BRCA 1 (hereditary) — tumor suppressor (DNA repair)

• BRCA 2 (hereditary) — tumor suppressor (DNA repair)

• p53 — tumor suppressor (DNA repair, cell cycle arrest)

• EGFR — RTK

• ERBB2 (HER2/neu) — RTK

Breast Cancer

• Therapeutic options for metastatic breast cancers include:


• HER2 and kinase inhibitors (for HER2 mutated tumors).
• CDK4/6 (cyclin dependent kinase) inhibitors (for hormone receptor positive
tumors).
• mTOR inhibitors (for hormone receptor positive tumors).
• PI3K inhibitors (for hormone receptor positive tumors with PI3KCA mutation).
• PARP inhibitors (for HER2 negative tumors with BRCA1/2 mutation).

624
Copyright © Oakstone Publishing, 2021. All Rights Reserved.

Rationale for Immune Therapy

• T-cell activation and regulation are carefully controlled interplay between


antigen presenting cells and T-cells via multiple cell-cell interactive
elements.
• CTLA-4 and PD-1 are negative regulatory cell surface molecules which
down-regulate T-cells after activation as part of normal homeostasis.

Rationale for immune therapy

• Tumor cells can exploit the negative regulatory elements of the immune
system to evade immune surveillance and T-cell mediated tumor cell killing.
• Antibodies directed against the negative regulatory elements of T-cell
signaling can promote immune recognition and killing of tumor cells by
activated cytotoxic T-cells.

625
Copyright © Oakstone Publishing, 2021. All Rights Reserved.

PD-1 / PD-L1

• PD-1 is a transmembrane protein expressed on activated T-cells, B-cells, NK


cells, dendritic cells, macrophages, and monocytes.
• PD-1 inhibits both adaptive and innate immune responses.
• PD-1 normally functions in reduction of harmful/ineffective immune
responses and maintenance of immune tolerance.
• Inappropriate activation of PD-1 in T-cells, usually via the expression of
ligand PD-L1 by tumor cells, interferes with T-cell activated killing of tumor
cells and immune escape by the tumor cells.
• Inhibitory antibodies against PD-1 can reverse the T-cell anergic state in the
tumor microenvironment to promote tumor cell killing by activated T-cells.
• Signaling pathways that active PD-1/PD-L1 include MAPK, JAK/STAT3,
PI3K/AKT, WNT, NFkB.

CTLA-4

• Cytotoxic T Lymphocyte antigen 4 (CTLA-4), a homolog of CD28 co-receptor


of T-cell receptor, is expressed on CD4+ and CD8+ T cells.
• CTLA-4 and CD28 on T-cells both bind to B7 co-stimulatory ligand
(CD80/CD86) on APCs and macrophages.
• CD-28 binding to B7 activates T cells.
• CTLA-4 biding to B7 inhibits T cells.
• One of the normal functions of CTLA-4 appears to be involved in T cell
suppression, after its initial activation, as part of immune homeostasis.
• Inhibitory antibodies against CTLA-4 promotes tumor cell killing by activated
T cells (as well as generating autoimmune T cell activity).

626
Copyright © Oakstone Publishing, 2021. All Rights Reserved.

Sample Questions

1. A 20 year old man presents with intermittent rectal bleeding. On physical exam,
patient appears to have several firm, fixed soft tissue nodules on the torso and
extremities. He reports that many of his family members on his mother’s side have
cancer in their abdomen. The appropriate next step is:

A. Observation

B. Colonoscopy and genetic testing

C. PET CT

D. Biopsy of soft tissue nodules

627
Copyright © Oakstone Publishing, 2021. All Rights Reserved.

1. A 20 year old man presents with intermittent rectal bleeding. On physical exam,
patient appears to have several firm, fixed soft tissue nodules on the torso and
extremities. He reports that many of his family members on his mother’s side have
cancer in their abdomen. The appropriate next step is:

B. Colonoscopy and genetic testing


There is concern for FAP (familial adenomatous polyposis), frequently associated with
mutation in APC gene. These patients present with numerous colon adenomas in
childhood and adolescence, with inevitable progression to adenocarcinoma unless
removed. Desmoid tumors, duodenal/periampullary adenomas, papillary-type thyroid
carcinomas, osteomas, epidermoid cysts, and pancreatic carcinomas are also
associated with FAP.
PET CT scan may be useful as a secondary evaluation tool after confirmation of
diagnosis.
Soft tissue nodule biopsy would not be useful as an initial diagnostic tool.

2. Colonoscopy of the previous patient shows numerous colonic adenomas, several with
evidence of recent bleeding. Biopsies do not show adenocarcinoma. Genetic test reveal
APC gene mutation. The appropriate next step is:

A. Observation with colonoscopy

B. Total proctocolectomy with end ileostomy

C. Colectomy with ileorectal anastomosis

D. Immune therapy

628
Copyright © Oakstone Publishing, 2021. All Rights Reserved.

2. Colonoscopy of the previous patient shows numerous colonic adenomas, several with
evidence of recent bleeding. Rectum does not appear to be involved. Biopsies do not
show adenocarcinoma. Genetic test reveal APC gene mutation. The appropriate next
step is:

C. Colectomy with ileorectal anastomosis


Patients with FAP due to APC gene mutations will invariably progress to colorectal
cancer. Prophylactic colectomy with ileorectal anastomosis would be the most
reasonable choice, with ongoing endoscopic exams of the rectal remnant.
Protocolectomy with end ileostomy would be appropriate for biopsy-proven
adenocarcinoma, especially if there is rectal involvement.
Observation with colonoscopy is inadequate.
Immune therapy is not effective in treatment of pre-carcinoma FAP.

629
Copyright © Oakstone Publishing, 2021. All Rights Reserved.

Soft Tissue Sarcoma


Chandrajit P Raut, MD, MSc

Chief, Division of Surgical Oncology, Brigham and Women’s Hospital


BWH Distinguished Chair in Surgical Oncology
Surgery Director, Sarcoma Center, Dana‐Farber Cancer Institute
Professor of Surgery, Harvard Medical School

No disclosures

630
Copyright © Oakstone Publishing, 2021. All Rights Reserved.

Sarcoma Background

Soft Tissue Sarcoma (STS)


14,000 new sarcoma
diagnoses per year

Crago and Singer, ACS Surgery 2011

Sarcoma Background

Common Histologies
ADIPOCYTIC TUMORS “SO‐CALLED” FIBROHISTIOCYTIC PERIVASCULAR TUMORS
Well differentiated liposarcoma / atypical TUMORS Malignant glomus tumor
lipomatous tumor Giant cell tumor
CHONDRO‐OSSEOUS TUMORS
Dedifferentiated liposarcoma Unclassified pleomorphic sarcoma
Extraskeletal osteosarcoma
Myxoid liposarcoma NEURAL TUMORS
Extraskeletal chondrosarcoma
Pleomorphic liposarcoma Malignant peripheral nerve sheath tumors
OTHER
FIBROBLASTIC / MYOFIBROBLASTIC Malignant Triton tumor
TUMORS Angiomatoid MFH
VASCULAR TUMORS Gastrointestinal stromal tumor
Desmoid‐type fibromatosis
Epithelioid hemangioendothelioma Synovial sarcoma
Solitary fibrous tumor /
hemangiopericytoma Pseudomyogenic hemangioendothelioma Phyllodes tumor
Inflammatory myofibroblastic tumor Angiosarcoma Epithelioid sarcoma
Myxoinflammatory fibroblastic sarcoma SKELETAL MUSCLE TUMOR Alveolar soft parts sarcoma
Fibrosarcoma Embryonal rhabdomyosarcoma Clear cell sarcoma
Myxofibrosarcoma Alveolar rhabdomyosarcoma Myxoid chondrosarcoma
Fibromyxoid sarcoma Pleomorphic rhabdomyosarcoma PNET / extraskeletal Ewing sarcoma
Dermatofibrosarcoma protuberans (DFSP) Desmoplastic small round cell tumor
/ fibrosarcomatous DFSP
Perivascular epithelioid cell tumor
SMOOTH MUSCLE TUMORS
Leiomyosarcoma Importance of expert pathology re‐review

631
Copyright © Oakstone Publishing, 2021. All Rights Reserved.

Sarcoma Background

STS Histology by Site


Extremity STS RP/Intraabdominal STS

Crago and Singer, ACS Surgery 2011

Sarcoma Background

Epidemiology
Etiology Histology
Sporadic
Genetic disorders
RB1 Leiomyosarcoma
NFI MPNST, GIST
APC Desmoid
KIT GIST
P53 Leiomyosarcoma, osteosarcoma
Radiation Osteosarcoma, angiosarcoma, UPS
Chemical exposure
Herbicides
Wood preservatives
Vinyl chloride
Abattoir workers
Lymphedema Angiosarcoma
Trauma Desmoid

632
Copyright © Oakstone Publishing, 2021. All Rights Reserved.

Sarcoma Background

Pretreatment Imaging
• Extremity/trunk STS
• MRI primary site
• CT chest (CXR if low-grade)

• Retroperitoneal sarcoma
• CT abdomen/pelvis
• CT chest
• Split-function renal scan

• Breast sarcoma
• MRI breast
• CT chest/abdomen/pelvis

• Gastrointestinal stromal tumor/abdominal visceral STS


• CT abdomen/pelvis
• CT chest (if not GIST)
• MRI (liver, rectum) as indicated

• Less commonly indicated imaging


• MRI/A, CT angiogram
• CT head
• Bone scan
• PET/CT

Sarcoma Background

Pretreatment Biopsy
• Rationale
• Preoperative therapy
• Uncertainty about diagnosis

• Options
• Core-needle biopsy
• EUS-FNA/FNB (if suspecting GIST)
• Incisional biopsy – only if core-needle biopsy non-diagnostic
• Excisional biopsy – avoid if possible

633
Copyright © Oakstone Publishing, 2021. All Rights Reserved.

Sarcoma Background

Core‐Needle Biopsy

Needle track recurrence:


0‐0.5%

Berger‐Richardson and Swallow, Cancer 2017


Wilkinson et al, Ann Surg Oncol 2015
Berger‐Richardson et al, Ann Surg Oncol 2019

Sarcoma Background

Lymph Nodes
• Rare (<3%) Histologies
Epithelioid sarcoma
• No role for routine lymphadenectomies Rhabdomyosarcoma
• Limited use of SLNB Angiosarcoma
Clear cell sarcoma
• Prognostic but no therapeutic value Synovial sarcoma

634
Copyright © Oakstone Publishing, 2021. All Rights Reserved.

Local Treatment Strategies

Principles
• Surgery is the only potentially curative therapy

• Extent of surgery is determined by multiple factors


• Histology
• Site of origin

• Judicious use of neoadjuvant and adjuvant therapy


• Sequencing of therapy
• Potential side-effects

Local Treatment Strategies

Principles
• Not everyone needs or should get an operation
• Extent of disease
• Metastases
• Vascular involvement
• Goals of care
• Palliation
• Comorbidities

• Surgery cannot overcome tumor biology

635
Copyright © Oakstone Publishing, 2021. All Rights Reserved.

Local Treatment Strategies

Radiation Therapy (RT)


• Limb‐sparing surgery + RT replaced amputation as the
standard of care for extremity sarcoma1
• No difference in disease‐free or overall survival

• Addition of RT to limb‐sparing surgery2,3


• Lowered risk of local recurrence, particularly in large, higher
grade STS
• No improvement in overall survival

• IMRT superior to 3‐D conformal EBRT4

1Rosenberg et al, Ann Surg 1982


2Yanget al, J Clin Oncol 1998
3Beane et al, Ann Surg Oncol 2014
4Folkert et al, J Clin Oncol 2014

Extremity Sarcoma (ESTS)

636
Copyright © Oakstone Publishing, 2021. All Rights Reserved.

ESTS – Surgery
• Goal of surgery is achieving a negative margin while
preserving limb function

• What constitutes an appropriate margin is a matter of


debate

• Margins of resection should be personalized to


• Individual needs of patient, reflecting functional preferences
• Specific sarcoma histology, reflecting differences in tumor
biology

Extremity Sarcoma

ESTS – Radiation
Phase 3 Trial of Amputation v. Limb Preservation

Limb-sparing
Amputation P-value
+ EBRT
Patients (N) 16 27
Local Recurrence (%) 0 15 0.06
5-yr DFS (%) 78 71 0.75
5-yr OS (%) 88 83 0.99

Rosenberg et al, Ann Surg 1982

637
Copyright © Oakstone Publishing, 2021. All Rights Reserved.

Extremity Sarcoma

Preoperative v. Postoperative RT
Preoperative RT Postoperative RT
Dose 50 Gy ≥ 66 Gy

Field Volume Smaller Larger

Wound Complication ~35% ~17%


Rate
Long Term Function Better Worse

Delay in Treatment Surgery rarely RT may be delayed


delayed
Local Recurrence Same Same
Rate
Overall Survival Rate Same Same

O’Sullivan et al, Lancet 2002


Davis et al, Radiother Oncol 2005

Extremity Sarcoma

ESTS – Histologies

Crago and Singer (2011), ACS Surgery

638
Copyright © Oakstone Publishing, 2021. All Rights Reserved.

Extremity Sarcoma

Leiomyosarcoma

Extremity Sarcoma

Standard of Care: Radical Resection


48 year‐old woman with right forearm MPNST

639
Copyright © Oakstone Publishing, 2021. All Rights Reserved.

Extremity Sarcoma

Myxofibrosarcoma

Extremity Sarcoma

Myxofibrosarcoma
• Unique behavior with recurrence pattern different than
other sarcomas

• Positive margins associated with local recurrence

• Repeated local recurrences can lead to amputations

• Surgery
• Wider margins than for other sarcomas
• Attention to fascia, muscle, nerve and vessel margins
• Could be performed in stages
• Plastic surgery consultation

Gronchi et al, Ann Surg 2010


Haglund et al, IJROBP 2012

640
Copyright © Oakstone Publishing, 2021. All Rights Reserved.

Extremity Sarcoma

Extended Margins ‐ Myxofibrosarcoma

• 70‐year‐old man
• Progressively enlarging forearm mass
• Core‐needle biopsy – myxofibrosarcoma

Extremity Sarcoma

Extended Margins ‐ Myxofibrosarcoma


• Preoperative radiation therapy

• Radical resection
• Overlying skin
• Underlying muscle and fascia
• Exposed median and ulnar nerves
• Myocutaneous free flap from leg

• Negative margins

641
Copyright © Oakstone Publishing, 2021. All Rights Reserved.

Extremity Sarcoma

Single Histology, Multiple Sites


Myxofibrosarcoma

Extremity Sarcoma

Single Site, Different Histologies


Gluteus Maximus Muscle

Well-Differentiated
Myxofibrosarcoma Leiomyosarcoma
Liposarcoma/ALT

642
Copyright © Oakstone Publishing, 2021. All Rights Reserved.

Extremity Sarcoma

Histology-Specific Surgery

Desmoid Fibromatosis
• 1838 – Johannes Muller coined the term “desmoid”

• Low-grade mesenchymal neoplasm


• High degree of local infiltration and invasion
• No metastatic potential

Muller, Ueber den feinern Bau und die Formen der krankhaften Geschwulste 1838

643
Copyright © Oakstone Publishing, 2021. All Rights Reserved.

Desmoid

Treatment of Desmoid Tumors

“Watchful waiting”

Surgery

Radiation

Systemic therapy
Hormonal agents – tamoxifen, toremifene, raloxifene
Anti-inflammatory drugs – NSAIDs (sulindac)
Biologic therapy – interferon
Cytotoxic chemotherapy – liposomal doxorubicin, dox +/- dacarbazine, methotrexate +
vinblastine
Tyrosine kinase inhibitors – imatinib, sorafenib,1 pazopanib2
Gamma secretase inhibitor – nirogacestat (PF03084014)
β-catenin inhibitor – tegavivint (BC2059)

1Gounder et al, N Engl J Med 2018


2Toulmonde et al, ASCO 2018

Desmoid

Desmoid Tumor Working Group, Eur J Cancer (2020)

644
Copyright © Oakstone Publishing, 2021. All Rights Reserved.

Desmoid

Mesenteric Desmoid

Desmoid

Pregnancy‐Associated Desmoid

645
Copyright © Oakstone Publishing, 2021. All Rights Reserved.

Gastrointestinal Stromal Tumor


• Most common mesenchymal tumor of the GI tract
• <1% of all GI malignancies

• Arise from interstitial cells of Cajal

• Microscopic GISTs (< 1 cm)


• 35% of stomachs removed for gastric cancer (Japan)
• 22.5% of stomachs in adults undergoing autopsy (Germany)

• Clinically relevant incidence


• 10‐20 cases per 1 million population
• 3000‐4000 cases per year in the US

Kawanowa et al, Hum Pathol 2006


Agaimy et al, Am J Surg Pathol 2007

GIST

Distribution by Site
Stomach Esophagus
60% < 1%

Duodenum Jejunum / Ileum


4‐5% 30%

Colon / Appendix Rectum


1‐2% 4%

646
Copyright © Oakstone Publishing, 2021. All Rights Reserved.

GIST

Pathologic Mutations
• > 80% ‐ activating mutations of KIT proto‐oncogene
• KIT encodes the KIT receptor tyrosine kinase (TK)

• 8% ‐ mutations of another TK gene, platelet‐derived growth factor α


(PDGRFA)
KIT PDGFRA

Exon 9 (9%)
Exon 11 (67%) Exon 12 (2%)
Exon 13 (1%) Exon 14 (rare)
Exon 17 (1%) Exon 18 (5.5%)
Corless and Heinrich, Ann Rev Pathol 2008

GIST

Tyrosine Kinase Inhibitors


• Effective KIT tyrosine kinase inhibitors (TKI) Imatinib

• Imatinib mesylate (Gleevec) – selective


• Sunitinib malate (Sutent) – multi‐targeted
• Regorafenib (Stivarga) – multi‐targeted Sunitinib
O

• Key characteristics
N
N
H
F
• Relatively safe O
N
H
OH
• Well‐tolerated N
H HOOC COO
H
• Orally available H

Regorafenib

• FDA‐approved

647
Copyright © Oakstone Publishing, 2021. All Rights Reserved.

GIST

Molecular Subsets of GIST


Gene Exon/Mutation Frequency (%) Site
KIT 75‐80
Exon 8 rare Small bowel
Exon 9 5‐10 Small bowel, colon
Exon 11 60‐70 All sites
Exon 13 1 All sites
Exon 17 1 All sites
PDGFRA 10
Exon 12 1 All sites
Exon 14 <1 Stomach
Exon 18 D842V 5‐10 Stomach
SDHA/B/C/D Mutations/Loss 2 Stomach
of expression
BRAF V600E <7
RAS <1
NF1 1‐2 Small bowel
PI3KCA rare
“Wild‐type” 10 All sites

GIST

Molecular Subsets of GIST


Organ‐Specific Distribution

KIT Exon 11 KIT Exon 11


PDGFRA Exon 18 KIT Exon 9
SDH‐Deficient NF1

648
Copyright © Oakstone Publishing, 2021. All Rights Reserved.

GIST

Evaluation
• All patients should undergo a multidisciplinary evaluation

• EUS‐FNA rather than CT‐guided core‐needle biopsy

• Pathology re‐review

• Staging
• CT abdomen/pelvis
• No CT chest or routine PET
• MRI for rectal GISTs may be helpful

• Consider genetic testing of tumor

GIST

Surgery

Surgery is the principal treatment and


only curative therapy for localized,
resectable primary disease

649
Copyright © Oakstone Publishing, 2021. All Rights Reserved.

GIST

Surgery: Technique
• R0 resection

• Laparoscopic resections when feasible


• Data mostly for gastric GISTs
• Follow‐up < 5 yrs

• Extensive resections when necessary

• Gentle handling – soft, friable tumors

• Thorough abdominal exploration

• Lymphadenectomy not indicated

GIST

Surgery: Margins
• No apparent benefit to wide margins (unlike adenocarcinomas or
other sarcomas)*

• No role for enucleation or endoscopic mucosal resection due to


involvement of muscularis propria

• Adjacent organs
• Primary tumors may displace adjacent structures, but are rarely invasive into
surrounding organs
• En bloc multi‐organ resection may be necessary to achieve negative margins,
especially with recurrent disease

*McCarter et al, J Am Coll Surg 2012

650
Copyright © Oakstone Publishing, 2021. All Rights Reserved.

GIST

Outcomes After Resection of Primary GIST


Prior to the Era of TKIs

1.0
Percentage Surviving
0.8

0.6
n=80
0.4

0.2
5‐yr 54%
0.0
0 2 4 6 8 10 12 14 16
Years
DeMatteo et al, Ann Surg 2000

GIST

Risk for Recurrence


Tumor Parameters % Pts with Progressive Disease

Mitotic Jejunum /
Size Stomach Duodenum Rectum
Count Ileum

≤ 2 cm 0 0 0 0
> 2, ≤ 5 cm ≤ 5 per 50 1.9 8.3 4.3 8.5
> 5, ≤ 10 cm HPFs 3.6 24
> 10 cm 12 } 34 52 } 57
≤ 2 cm 0* * 50* 54
> 2, ≤ 5 cm > 5 per 50 16 50 73 52
> 5, ≤ 10 cm HPF 55 85
> 10 cm 86 } 86 90 } 71
* Too few cases

Miettinen and Lasota, Sem Diag Path 2006

651
Copyright © Oakstone Publishing, 2021. All Rights Reserved.

GIST

Case
Primary Jejunal GIST

• 65YO W with hematemesis


• EGD
• Submucosal mass eroding into
jejunum
• EUS‐FNA ‐ GIST

• OR: small bowel resection


• Pathology
• Primary jejunal GIST
• 5.8 cm
• Mitotic count 3/50 HPF

GIST

Case
Risk for Recurrence

Tumor Parameters % Pts with Progressive Disease

Mitotic Jejunum /
Size Stomach Duodenum Rectum
Count Ileum

≤ 2 cm 0 0 0 0
> 2, ≤ 5 cm ≤ 5 per 50 1.9 8.3 4.3 8.5
> 5, ≤ 10 cm HPFs 3.6 24
> 10 cm 12 } 34 52 } 57
≤ 2 cm 0* * 50* 54
> 2, ≤ 5 cm > 5 per 50 16 50 73 52
> 5, ≤ 10 cm HPF 55 85
> 10 cm 86 } 86 90 } 71
* Too few cases

Miettinen and Lasota, Sem Diag Path 2006

652
Copyright © Oakstone Publishing, 2021. All Rights Reserved.

GIST

Adjuvant Imatinib Trials


ACOSOG Z90001 ACOSOG Z90012 EORTC 620243 SSG XVIII/AIO4 PERSIST-55
Phase 2 Phase 3 Phase 3 Phase 3 Phase 2
Study design
single arm, open-label double-blind open-label open-label single arm, open-label
Duration of IM 1 year 1 year v. 0 2 years v. 0 3 years v. 1 year 5 years
Number of patients 106 713 835 397 91
Patient population Low, intermediate, or Intermediate or high Intermediate or high
High risk High risk
(risk of recurrence) high risk risk risk
RFS
1-year 96% --- --- ---
3-year 60% 98% v. 83% 84% v. 66% 87% v. 60% ---
5-year 40% 69% v. 63% 66% v. 48% 90%
87% v. 84%
Imatinib Failure-Free
--- --- 79% v. 73%, high risk --- ---
Survival, 5-year
only (trend)
OS
1-year 99% --- --- ---
3-year 97% --- 96% v. 94% ---
5-year 83% 100% v. 99% 92% v. 82% 95%
Early discontinuation 17% 27% 25% 26% v. 13% 49%

1DeMatteo et al, Ann Surg 2013


2DeMatteo et al, Lancet 2009 4Joensuu et al, JAMA 2012
3Casali et al, J Clin Oncol 2015 5Raut et al, JAMA Onc 2018

GIST

Rectal GIST

6 mo IM

9.5 cm 6.1 cm

653
Copyright © Oakstone Publishing, 2021. All Rights Reserved.

GIST

Large Gastric GIST

6 mo IM

GIST

Treatment Influenced by Mutation

Laparoscopic
6 mo IM OR wedge Adjuvant IM
gastrectomy

KIT Exon 11

Exploratory laparotomy,
OR Observation
wedge gastrectomy

PDGFRA Exon 18
D842V

654
Copyright © Oakstone Publishing, 2021. All Rights Reserved.

GIST

Metastatic GIST: TKI Therapy


Imatinib Sunitinib Regorafenib
24 months 6.8 months 4.8 months

Blanke et al, J Clin Oncol 2008 Demetri et al, Lancet 2006 Demetri et al, Lancet 2013

GIST

GIST Metastasectomy
Progressive‐Free Survival Overall Survival

Median PFS 11 mo Median OS 86 mo

323 pts, 400 operations


Primary mutation is not an independent predictor of
outcome
Fairweather et al, Ann Surg 2017

655
Copyright © Oakstone Publishing, 2021. All Rights Reserved.

Breast Sarcoma

Single Site, Different Histologies


Breast

Primary Mammary
Phyllodes Tumor
Angiosarcoma
Radiation-Associated
Cutaneous Angiosarcoma

Breast Sarcoma

Radical Surgery for RT‐Associated Breast AS

656
Copyright © Oakstone Publishing, 2021. All Rights Reserved.

Breast Sarcoma

DSS by Extent of Surgery


100 p < 0.01
86%

Percent survival
50 46%

0
0 50 100 150 200 250
Time (months)

Conservative
Radical

Li et al, Ann Surg 2017

Retroperitoneal Sarcoma (RPS)

657
Copyright © Oakstone Publishing, 2021. All Rights Reserved.

RPS

Disease‐Specific Survival
Site of Origin

Crago and Singer, ACS Surgery 2011

RPS

RPS Histology
UPS
MPNST 2%
SFT 3% Other
6% 7%
Well‐Differentiated
Liposarcoma
27%

Leiomyosarcoma
20%
Dedifferentiated
Dedifferentiated Liposarcoma G2
Liposarcoma G3 22%
13%

Gronchi et al, Ann Surg 2016

658
Copyright © Oakstone Publishing, 2021. All Rights Reserved.

RPS

Surgery
• Surgery remains the only potentially curative
treatment

• Principles
• Macroscopically complete (R0/R1) resection
• True R0 resections are rare
• Avoid tumor rupture
• Remove cuff of normal tissue

• Extent of resection is controversial

RPS

Surgery

659
Copyright © Oakstone Publishing, 2021. All Rights Reserved.

RPS

“Compartment” Resection

RPS

“Compartment” Resection
Components Nuances
Ipsilateral colectomy Mesocolon as anterior margin of compartment
Ipsilateral nephrectomy Spare in well‐differentiated liposarcoma
Distal pancreatectomy L‐sided RPS
Splenectomy L‐sided RPS
Pancreaticoduodenectomy Invasive R‐sided RPS
R hepatectomy Invasive R‐sided RPS
Sigmoid colectomy/LAR Pelvic extension
Psoas resection
Partial cystectomy If invaded
Vascular resection If invaded
Bony resection If invaded
RP fat Liposarcomas

660
Copyright © Oakstone Publishing, 2021. All Rights Reserved.

RPS

Locoregional Recurrence

Aggressive, compartmental resections are associated with lower


locoregional recurrence rates
Bonvalot et al, J Clin Oncol 2009

RPS

Multi‐Visceral Resection
Morbidity and Mortality

Morbidity increases with Increased risk of sarcoma-specific


resection of >3 organs death with resection of ≥3 organs

Bonvalot et al, Ann Surg Oncol 2010


Tan et al, Ann Surg 2015

661
Copyright © Oakstone Publishing, 2021. All Rights Reserved.

RPS

How Aggressive?

Outcomes Morbidity

RPS

Extent of Surgery

Does 1
size fit
all?

Resection of RPS with Systematic multivisceral


limited margin resection

662
Copyright © Oakstone Publishing, 2021. All Rights Reserved.

RPS

Liposarcoma

RPS

IVC Leiomyosarcoma

663
Copyright © Oakstone Publishing, 2021. All Rights Reserved.

Transatlantic Retroperitoneal Sarcoma Working Group

Mt Sinai Hospital, Princess Margaret Hospital, University of Toronto, Toronto, Canada


Brigham and Women’s Hospital/Dana‐Farber Cancer Institute, Boston, MA, USA
Royal Marsden Hospital, London, UK
Istituto Nazionale dei Tumori, Milan, Italy
Institut Gustave Roussy, Villejuif, France
Mannheim University Hospital, Mannheim, Germany
Netherlands Cancer Institute, Amsterdam, Netherlands
Maria Sklodowska‐Curie Memorial Cancer Center, Warsaw, Poland

Transatlantic Retroperitoneal Sarcoma Working Group

1007 patients with primary RPS from 8 centers


2002‐2011
Surgeons each perform > 50 major sarcoma resections per year

664
Copyright © Oakstone Publishing, 2021. All Rights Reserved.

RPS

Overall Survival by Histology


1007 Patients

5-yr OS 67% WDLPS 83%


SFT 77%

GII DDLPS 51%


LMS 43%
GIII DDLPS 30%

Gronchi et al, Ann Surg 2016

RPS

Local Recurrence by Histology


1007 Patients

GII DDLPS 49%

5-yr CCI of LRR 26%


GIII DDLPS 38%
WDLPS 35%

SFT 14%
LMS 12%

Gronchi et al, Ann Surg 2016

665
Copyright © Oakstone Publishing, 2021. All Rights Reserved.

RPS

Distant Metastases by Histology


1007 Patients

LMS 53%

5-yr CCI of DM GIII DDLPS 33%


21%

SFT 13%

GII DDLPS 9%
WDLPS <1%

Gronchi et al, Ann Surg 2016

RPS

RPS TNM Staging


American Joint Committee on Cancer, 8th ed
Category Definition
Tumor status
T0 No evidence of primary tumor
T1 T ≤ 5 cm
T2 T > 5 cm, ≤ 10 cm
T3 T > 10 cm, ≤ 15 cm
T4 T > 15 cm
Lymph node status
N0 No regional LN metastases
N1 Regional LN metastases present
Metastasis status
M0 No distant metastases
M1 Distant metastases present
Grade
GX Cannot be assessed
G1 Differentiation + mitotic count + necrosis score 2‐3
G2 Differentiation + mitotic count + necrosis score 4‐5
Pollock et al, AJCC Cancer Staging
G3 Differentiation + mitotic count + necrosis score 6‐8
Manual 2017

666
Copyright © Oakstone Publishing, 2021. All Rights Reserved.

RPS

RPS Stage Grouping


American Joint Committee on Cancer, 8th ed

Stage
Definition
Grouping
IA T1 N0 M0 GX, G1
IB T2, T3, T4 N0 M0 GX, G1
II T1 N0 M0 G2, G3
IIIA T2 N0 M0 G2, G3
T3, T4 N0 M0 G2, G3
IIIB
Any T N1 M0 Any G
IV Any T Any N M1 Any G

Pollock et al, AJCC Cancer Staging Manual 2017

RPS

Primary RPS Nomogram


Disease‐Free Survival Overall Survival

Gronchi et al, J Clin Oncol 2013


Raut et al, Cancer 2016

667
Copyright © Oakstone Publishing, 2021. All Rights Reserved.

RPS

Sarculator App

RPS

Phase 3 Preoperative Radiation Trial


EORTC 62092/“STRASS”
Surgery
Surgery arm

Max 4 *
wks
Randomization

256 Stratified by institution, PS (0‐1 vs 2)


Eligible
patients Radiotherapy Surgery
(3DCRT or IMRT)

Max 8 wks 6‐7 wks 4‐8 wks

Radiotherapy & Surgery arm

668
Copyright © Oakstone Publishing, 2021. All Rights Reserved.

RPS

STRASS: Phase 3 Preop RT + Surgery v. Surgery Alone


Abdominal Recurrence‐Free Survival – Entire Cohort

Preoperative RT 3y ARFS: 60%


Surgery 3y ARFS: 59%

Median Follow-up 43 months

Bonvalot et al, Lancet Oncol 2020

RPS

STRASS: Phase 3 Preop RT + Surgery v. Surgery Alone


Abdominal Recurrence‐Free Survival – Liposarcoma Cohort

Preoperative RT 3y ARFS: 72%


Surgery 3y ARFS: 60%

Median Follow-up 43 months

Bonvalot et al, Lancet Oncol 2020

669
Copyright © Oakstone Publishing, 2021. All Rights Reserved.

RPS

STRASS: Phase 3 Preop RT + Surgery v. Surgery Alone


Overall Survival

Preoperative RT 3y OS: 84%


Surgery 3y OS: 85%
Median Follow-up 43 months

Bonvalot et al, Lancet Oncol 2020

Conclusions
• General
• Sarcomas – rare family of cancers with 100+ different types
• Treatments, even within a particular anatomic site, are guided by histology
• Pre‐treatment core‐needle biopsy is critical
• Expert pathology review
• Sarcoma multidisciplinary team discussion

• Extremity sarcoma
• Limb‐sparing surgery + pre‐ or postoperative RT
• Extent of surgery and use of adjuvant treatments depends on histology

670
Copyright © Oakstone Publishing, 2021. All Rights Reserved.

Conclusions
• Desmoid fibromatosis
• Surgery is no longer considered first‐line, replaced by watchful waiting

• GIST
• Risk of recurrence depends on size, site of origin, and mitotic count
• Adjuvant therapy reduces rates of recurrence and improves survival

Conclusions
• Breast angiosarcoma
• Radical surgery is key

• RPS
• Aggressive, histology‐specific, multivisceral surgery
• Histology‐specific patterns of recurrence
• Limited utility for preoperative RT

671
Copyright © Oakstone Publishing, 2021. All Rights Reserved.

Thank you
[email protected]

672
Copyright © Oakstone Publishing, 2021. All Rights Reserved.

Melanoma and dysplastic nevi

Oakstone CME Lecture

Charles H. Yoon, PhD, MD, FACS


2021.10

Disclosures

• None

673
Copyright © Oakstone Publishing, 2021. All Rights Reserved.

Melanocytes and UV-damage

• Melanocytes are derived from embryonic neural crest, which migrate during
embryogenesis to the dermal-epidermal junction.
• Normal melanocytes produce melanin, a pigment which absorbs UV.
• UV radiation on earth's surface comprises long wavelength ultraviolet A
(UVA) (320 – 400 nm) and short wavelength ultraviolet B (UVB) (280 - 320
nm). Shorter wavelength UVC (200 – 280 nm) is filtered out by atmospheric
O2 and O3.
• UVA generates reactive oxygen species which induce single-strand breaks in
DNA and in DNA-protein crosslinks (indirect damage).
• UVB is absorbed by DNA and induces direct damage to base structure.
• C→T subs2tu2ons at dipyrimidine sites and CC → TT double base
substitutions are known UV-fingerprint mutations in DNA.

Melanocytes and UV-damage

• UV-damage and production of UV-exposure byproducts induce melanin


production in melanocytes.
• Dendritic melanocytes produce melanin within organelles known as
melanosomes.
• Melanin is transported via dendrites to adjacent keratinocytes and
accumulate within keratinocytes and melanocytes in the perinuclear area to
act as a shield against UV.
• Accumulation of DNA damage in melanocytes due to UV-exposure lead to
development of melanocytic neoplasms, ranging from dysplastic nevi to
invasive melanoma.

674
Copyright © Oakstone Publishing, 2021. All Rights Reserved.

Dysplastic Nevi

• Cytologic atypia in melanocytes (dysplastic nevi) is frequently associated


with increased UV-exposure.
• Some dysplastic nevi are congenital.
• Increasing degree of cytologic atypia (mild-moderate-severe) in dysplastic
nevi denote increased risk of transformation into melanoma.

• Dysplastic nevi with mild- to moderate cytologic atypia should be excised


completely (negative pathologic margins). This is normally performed by an
excision that measures 2—3 mm radial margin from the border of the lesion
to the underlying fibroadipose tissue deep to the dermis.
• Dysplastic nevi with any component of severe cytologic atypia should be
considered as a melanoma in situ precursor: these should be excised with 5
mm radial margin (from the border of the lesion and/or biopsy scar) to the
underlying deep fibroadipose tissue (or to the underlying fascia).

Staging for Primary Melanoma

• T1a: <0.8 mm invasive depth, not ulcerated.


• T1b: 0.8—1.0 mm depth and/or ulcerated.

• T2a: 1.1—2.0 mm depth, not ulcerated.


• T2b: 1.1—2.0 mm depth, ulcerated.

• T3a: 2.1—4.0 mm depth, not ulcerated.


• T3b: 2.1—4.0 mm depth, ulcerated.

• T4a: >4.0 mm depth, not ulcerated.


• T4b: >4.0 mm depth, ulcerated.

675
Copyright © Oakstone Publishing, 2021. All Rights Reserved.

Wide Excision for Primary Melanoma

• Until the mid-1970s, invasive melanomas were routinely excised with radial
margins of 4—5 cm.
• While such wide excisions provided excellent local disease control, they
were also associated with significant morbidity and cosmetic defects.

• Starting in the late 1970’s, several prospective randomized trials compared


varying margins of excision for melanomas with different invasive depths.
• The results have led to the reduction of margins of excision and improved
morbidity rates without sacrificing outcomes.

Wide Excision for Primary Melanoma

676
Copyright © Oakstone Publishing, 2021. All Rights Reserved.

Wide Excision for Primary Melanoma

Melanoma and Lymph Nodes

• The first report of improved survival of patients who received lymph node
excisions with occult metastases came in 1955 by McCune and Letterman
(Annals of Surgery 141:901-909).

• Several clinical trials followed in which patients without clinically evident


nodal disease at initial melanoma presentation were randomized to wide
local excision (WLE) alone or WLE plus elective lymph node dissection
(ELND).

• A trend towards improved survival in patients undergoing ELND was


observed. However, the magnitude of these improvements did not reach
statistical significance. In some of the studies, fewer than 20% of patients
were found to have nodal involvement at the time of ELND.

677
Copyright © Oakstone Publishing, 2021. All Rights Reserved.

Sentinel Lymph Node Biopsy for Melanoma

• In 1992, Morton described the technique of sentinel lymph node biopsy


(SLNB) in melanoma as a less invasive and less morbid procedure to identify
the subset of patients with occult nodal involvement (Archives of Surgery
127:392-399).

• Multicenter Selective Lymphadenectomy Trial (MSLT-1).


• Patients without clinically positive nodes were randomized to one of two
treatment arms near the time of primary melanoma resection:
1. SLNB and subsequent completion lymphadenectomy for positive sentinel
nodes.
2. Nodal observation by clinical exam only.

• Patients in the observation arm received delayed lymphadenectomy when


follow-up clinical exam identified nodal metastases.

Sentinel Lymph Node Biopsy for Melanoma

• Final trial report of the the MSLT-1 was published in 2014.

• No significant treatment-related difference in 10-year melanoma-specific


survival rate in the overall population (80.8 % with and 79.2 % without
nodal metastasis).

• For intermediate thickness melanoma (1.2 – 3.5 mm depth), 10-year


disease-free survival was 71.3 % for sentinel lymph node biopsy group
versus 64.7 % for the nodal observation group for melanoma depth 1.2 – 3.5
mm (P=0.01).

• For thick melanoma (>3.5 mm depth), 10-year disease-free survival was


50.7 % for sentinel lymph node biopsy group versus 40.5 % for nodal
observation group (P=0.03).

678
Copyright © Oakstone Publishing, 2021. All Rights Reserved.

Sentinel Lymph Node Biopsy for Melanoma

• Among patients with nodal metastases from intermediate thickness


melanomas, 10-year melanoma-specific survival rate was 62.1 % for the
sentinel node biopsy group versus 41.5 % for the observation group.

• For thick melanoma, there was no difference in the 10-year melanoma-


specific survival: 48.0 % in the sentinel node biopsy group versus 45.8 % in
the observation group.

• Distant disease-free survival was significantly improved for patients with


nodal metastases from intermediate thickness melanoma when they
received sentinel node biopsy and immediate lymphadenectomy in
comparison to the nodal observation group that received delayed
lymphadenectomy for clinically identified disease (hazard ratio = 0.62). No
difference for thick melanoma.

Sentinel Lymph Node Biopsy for Melanoma

• Identification of microscopic, clinically occult nodal involvement was the


strongest predictor of disease recurrence or disease death in the sentinel
node biopsy group:

• 10-year melanoma-specific survival of 85.1 % for sentinel node negative


patients versus 62.1 % in sentinel node positive patients for intermediate
thickness melanoma.

• 10-year melanoma-specific survival of 64.6 % for sentinel node negative


patients versus 48.0 % in sentinel node positive patients for thick
melanoma.

679
Copyright © Oakstone Publishing, 2021. All Rights Reserved.

Sentinel Lymph Node Biopsy for Melanoma

• MSLT-2 interim report (3 year follow-up; NEJM, 2017, 376: 2211-2222).

• Immediate completion radical lymphadenectomy for positive sentinel nodes


did not improve melanoma-specific survival in comparison to frequent
evaluation of the affected nodal basin (and delayed completion
lymphadenectomy for subsequent nodal recurrence identified clinically or
radiologically; 86% at 3 years in both groups).

• Only 15-25% of patients have additional non-sentinel lymph node


involvement found in completion lymphadenectomy.

• Prognostic benefit of sentinel lymph node biopsy in the setting of positive


microscopic metastatic disease (as noted in the MSLT-1 trial) appeared to
be derived from removal of the involved sentinel nodes alone.

Sentinel Lymph Node Biopsy for Melanoma

• No difference in distant metastatic disease free survival between immediate


completion lymphadenectomy and nodal observation after positive sentinel
node biopsy. However, the average follow-up appeared to be too short (3
years) and the rate of distant disease too rare within that follow-up period
for a definitive result.

• DeCOG-SLT trial also confirmed the lack of benefit of early completion


lymphadenectomy in the setting of positive sentinel node biopsy (J Clin
Oncol, 2019, 37(32):3000-3008).

680
Copyright © Oakstone Publishing, 2021. All Rights Reserved.

Sentinel Lymph Node Biopsy for Melanoma

• Immediate completion lymphadenectomy for positive sentinel node biopsy


showed a small improvement in disease-free survival (68% vs. 63%) and
increase in regional nodal basin disease control (92% vs. 77%), in
comparison to nodal observation following positive sentinel node biopsy
(and delayed completion lymphadenectomy for subsequent nodal
recurrence).

• Immediate completion lymphadenectomy can provide improved staging


information by identifying non-sentinel node metastasis early, which can
negatively impact prognosis and can therefore guide decision for adjuvant
treatment.

• However, risks and morbidity associated with lymphadenectomy generally


outweighs considerations of improved staging.

Sentinel Lymph Node Biopsy for Melanoma

• Nodal relapse was not separate from distant disease relapse; relapse-free
survival was not significantly different between patients with or without
early completion lymphadenectomy after a positive SLN biopsy.

• Among the patients followed by observation (without early completion


lymphadenectomy), only ~20% of patients with disease relapse was
expected to have recurrence limited to the lymph node basin (of the
positive SLN biopsy). Delayed lymphadenectomy in those patients with
disease relapse limited to the lymph node basin was expected to yield >70%
survival at 10 years, without loss of lymph node basin disease control
following surgery.

• These considerations clearly indicate that completion lymphadenectomy is


not necessary.

681
Copyright © Oakstone Publishing, 2021. All Rights Reserved.

Surgical Recommendations

• Sentinel lymph node biopsy (lymphoscintigraphy and excision) should be


performed at the time of wide excision for primary melanoma, T1b to T4.

• Immediate completion lymphadenectomy for positive sentinel lymph nodes


is no longer the treatment standard.

• Truly microscopic metastases (<0.1 mm focus) identified in sentinel lymph


nodes are of unknown clinical significance.

Lymphadenectomy for Bulky Nodal Metastases

• For clinically palpable or radiologically visible lymph node metastases


(without distant disease), regional lymphadenectomy should be performed
to remove all nodes within the involved nodal basin.
• Even prior to the advent of immune and targeted therapies, long-term
survival, approaching ~35% at 15 years and beyond, were reported
following lymphadenectomy for regional metastases to nodal basins (Young et
al 2006).

• In the current era, patients will receive adjuvant treatment following


lymphadenectomy: immune checkpoint inhibition (anti-PD-1) therapy or
targeted therapy with Braf +MEK inhibition (if Braf V600E/K mutation
identified in melanoma sample).

682
Copyright © Oakstone Publishing, 2021. All Rights Reserved.

Recurrent Melanoma or Satellite / In-Transit Metastases

• There is no clear distinction between local recurrence versus loco-regional


metastases such as in-transit disease or satellitosis.

• Many define local recurrence as relapse of disease within 2 to 5 cm of the


primary tumor site.

• Other definitions are much more stringent, defining true local recurrence as
that involving primary excisional scar site, containing an in situ component,
or other histopathologic attributes.

• Local recurrence may have much in common with in-transit metastases and
satellitosis.

Recurrent Melanoma or Satellite / In-Transit Metastases

• Intergroup Melanoma Surgical Trial: patients with local recurrence (defined


as recurrence within 2 cm of the scar) had significantly worse 5- and 10-year
survival rates (30% and 5%) compared to those patients without local
recurrence (90% and 85 %).

• The large difference in survival rates is not expected if local recurrence


occurs simply because of residual tumor in the surgical bed due to
insufficient margins of resection.

• Tumor thickness, ulceration, and location of primary tumor are the main
factors that correlate with recurrence rate.

• Of the patients who developed local recurrence, >60 % had metastatic


disease at distant sites as the next site of relapse.

683
Copyright © Oakstone Publishing, 2021. All Rights Reserved.

Recurrent Melanoma or Satellite / In-Transit Metastases

• Local recurrence may be a result of microscopic satellites some distance


away from the primary tumor that were not detected at the time of
excision. Alternatively, local recurrence may be a biological marker that
lymphatic/hematogenous spread of the melanoma had already occurred at
the time of primary excision.

Surgery for Recurrent Melanoma or Satellite / In-Transit Metastases

• If feasible, complete surgical resection is the primary management of local


recurrence and satellite / in-transit metastases.

• For multiple (unresectable) satellite / in-transit metastases, hyperthermic


isolated limb perfusion/infusion or radiotherapy were given in the past. In
the modern era, injectable (e.g, TVEC), immune, and/or targeted therapy
should be considered.

• There are established margins of excision for recurrent melanoma. 1—2 cm


radial margin is typically used to achieve negative margins.

• Sentinel node biopsy can be considered, especially if one had not been
performed with the prior excision of the primary lesion or if a regional nodal
dissection had not occurred due to a previously negative SLN biopsy.

684
Copyright © Oakstone Publishing, 2021. All Rights Reserved.

Staging for Regional Disease (N>0)

• Sentinel lymph node metastasis (clinically occult):

• N1a: 1 sentinel node.

• N2a: 2—3 sentinel nodes.

• N3a: 4+ sentinel nodes.

• Clinically palpable / radiologically visible lymph node metastasis:

• N1b: 1 lymph node.


• N2b: 2—3 lymph nodes.
• N3b: 4+ lymph nodes (N3b).
• N1c: Satellite / in-transit metastasis (any number) without lymph node metastasis.
• N2c: Satellite / in-transit + 1 lymph node metastasis.
• N3c: Satellite / in-transit + 2 (or more) lymph node metastasis.

Stage III Stratification

685
Copyright © Oakstone Publishing, 2021. All Rights Reserved.

Stage III Stratification: Melanoma-Specific Survival

5-year survival 10-year survival


Stage III-A 93 % 88 %
Stage III-B 83 % 77 %
Stage III-C 69 % 60 %
Stage III-D 32 % 24 %

Metastasectomy in Advanced Stage III and Stage IV melanoma

• Prior to immune and targeted therapy, patients with melanoma recurrences


at distant metastatic sites had a 1-year survival of ~50% and a 5-year
survival ~5 %.

• Site of distant metastases influenced survival. Patients with distant skin,


soft tissue, or nodal metastases (M1a) fared better than those with
metastases to visceral organs (M1b, M1c).

• Among the patients with visceral metastases, those with pulmonary


involvement (M1b) had better survival than those with involvement in other
visceral organs (M1c).

686
Copyright © Oakstone Publishing, 2021. All Rights Reserved.

Metastasectomy in Advanced Stage III and Stage IV melanoma

• Surgery for distant melanoma metastases has long been used as a form of
local treatment, often as palliative therapy for control of symptoms.

• In the past 25 years, reports have shown improved survival following


metastasectomy with intent to render the patient free of disease.

• 5-year survival following successful metastasectomy is reported to be


between 20% and 40% in larger, single-institution studies.

• However, there were no standard for selection of patients.

Metastasectomy in Advanced Stage III and Stage IV melanoma

687
Copyright © Oakstone Publishing, 2021. All Rights Reserved.

Metastasectomy in Advanced Stage III and Stage IV melanoma

• Resection of isolated metastases followed by adjuvant therapy (immune


checkpoint inhibition or Braf + MEK inhibition).

• Resection of immune or targeted therapy-resistant metastases in patients


with partial response to immune therapy.

• Neoadjuvant immune or targeted therapy for borderline resectable


metastases (~8 weeks) followed by complete surgical resection.

Future Trends

• For primary melanoma, surgery will remain the mainstay of treatment.

• Sentinel lymph node biopsy will remain useful as a staging tool.

• Extent of lymphadenectomy (amount of lymph node clearance), may


decrease as systemic therapy becomes more effective and less toxic.

• Metastasectomy, in combination with systemic therapy, will become more


important as an adjunct therapeutic modality, especially for patients with
partial response to systemic therapy.

688
Copyright © Oakstone Publishing, 2021. All Rights Reserved.

Sample Questions

1. A 45 year old woman presents with a lump in her Right axilla that she has noticed for
3 months. She has no constitutional symptoms and no recent infections. Mammogram
is normal. Complete physical exam reveals no other suspicious lesions. Mass is ~2cm in
diameter, firm, and non-tender. There is no surrounding erythema. The most
appropriate next step is:

A. Antibiotics

B. Core needle biopsy

C. Excisional biopsy

D. Sentinel lymph node biopsy

689
Copyright © Oakstone Publishing, 2021. All Rights Reserved.

1. A 45 yo woman presents with a lump in her Right axilla that she has noticed for 3
months. She has no constitutional symptoms and no recent infections. Mammogram is
normal. Complete physical exam reveals no other suspicious lesions. Mass is ~2cm in
diameter, firm, and non-tender. There is no surrounding erythema. The most
appropriate next step is:

B. Core needle biopsy


The lesion requires biopsy for tissue diagnosis. Of the 2 choices, core needle biopsy is
preferrable over excisional biopsy — core needle biopsy would have less impact on
potential future lymphadenectomy.
Antibiotics would be inappropriate as there are no signs of infection.
Sentinel node biopsy would be performed in the setting of primary lesion excision,
when there is no clinical evidence of nodal disease.

2. The core needle biopsy of the Right axilla in the previous patient shows that it is a
lymph node involved with metastatic melanoma. There are no skin lesions on the
patient’s body. PET CT shows FDG-uptake in the Right axilla but there are no other sites
of abnormal metabolic activity. The most appropriate next step is:

A. Formal lymph node dissection

B. Immune therapy only

C. Mastectomy

D. Chemo-XRT only

690
Copyright © Oakstone Publishing, 2021. All Rights Reserved.

2. The core needle biopsy of the Right axilla in the previous patient shows that it is a
lymph node involved with metastatic melanoma. There are no skin lesions on the
patient’s body. PET CT shows FDG-uptake in the Right axilla but there are no other sites
of abnormal metabolic activity. The most appropriate next step is:

A. Formal lymph node dissection


For clinically palpable or radiologically visible regional lymph node metastases, the
most appropriate next step is a regional lymphadenectomy. Following recovery from
surgery, patient may be a candidate for adjuvant immune or targeted therapy.
Immune therapy without surgery would be expected to have lower response rates
than surgery + immune therapy or even surgery alone.
Chemotherapy + radiation therapy are not the standard of care as response rates are
minimal.
Mastectomy would be an inappropriate surgical option for metastatic melanoma
involving axillary lymph node.

3. A 65 year old man has a 1 cm diameter dark colored lesion on his arm that he reports
to have changed in appearance. A complete physical exam shows no other lesions and
no suspicious findings. The most appropriate next step is:

A. Resection with 2 cm margin

B. Punch biopsy

C. Immune therapy

D. Shave biopsy

691
Copyright © Oakstone Publishing, 2021. All Rights Reserved.

3. A 65 year old man has a 1 cm diameter dark colored lesion on his arm that he reports
to have changed in appearance. A complete physical exam shows no other lesions and
no suspicious findings. The most appropriate next step is:

B. Punch biopsy
Biopsy is required for tissue diagnosis.
Punch biopsy is preferred for cutaneous lesions that could harbor melanoma as the
Breslow depth would be better captured.
Shave biopsy can transect a melanoma and therefore under-estimate the primary T-
stage of the lesion.
Wide excision with 2 cm margin would be premature as many pigmented lesions do
not harbor invasive melanoma.

4. The punch biopsy in the previous patient reveals malignant melanoma with invasion
to 2.5 mm depth, ulceration absent. A repeat complete physical exam shows no
clinically positive nodes. The most appropriate next step is:

A. Resection of primary with 2 cm margin and SLN biopsy

B. Resection of primary with 2 cm margin and formal axillary LN dissection

C. Resection of primary with 1 cm margin and SLN biopsy

D. PET CT scan

692
Copyright © Oakstone Publishing, 2021. All Rights Reserved.

4. The punch biopsy in the previous patient reveals malignant melanoma with invasion
to 2.5 mm depth, ulceration absent. A repeat complete physical exam shows no
clinically positive nodes. The most appropriate next step is:

A. Resection of primary with 2 cm margin and SLN biopsy


The lesion is a T3a melanoma. Clinically negative nodes indicate a provisional
diagnosis of Stage II-A. Patient should receive wide excision with 2 cm radial margin
and sentinel lymph node biopsy. T3 and T4 lesions (invasion >2 mm) should be
excised with 2 cm margins, when anatomically feasible.
Elective lymph node dissections are not the standard of care for patients presenting
with primary melanoma and clinically normal lymph nodes.
Preoperative PET CT scan would be premature for T3a melanoma; it can be considered
for T3b-T4.

693
Copyright © Oakstone Publishing, 2021. All Rights Reserved.

Adrenal Surgery and


Endocrine Dysfunction

Matthew A. Nehs, MD
Brigham and Women’s Hospital
Harvard Medical School

Disclosures
Dr. Nehs has nothing to disclose.

694
Copyright © Oakstone Publishing, 2021. All Rights Reserved.

“Anatomy is the war map for the


operations of the physician"
-Emil Zuckerkandl

695
Copyright © Oakstone Publishing, 2021. All Rights Reserved.

696
Copyright © Oakstone Publishing, 2021. All Rights Reserved.

Strategy
Begin with the End in Mind

Educational
Objectives for
Residents and Fellows

Facilitate long-term healing


and return to work

Minimize Tissue Trauma and


Perioperative Complications

Cure the Disease

697
Copyright © Oakstone Publishing, 2021. All Rights Reserved.

Adrenal Pathology
Tumor Size

Operative Approach

Patient Size
Visceral Fat
Prior Surgery
Occupation

Imaging
Characteristics
Tumor Size

Hormonal
Function

698
Copyright © Oakstone Publishing, 2021. All Rights Reserved.

Adrenal Incidentaloma
66 year old man
CT scan done for another indication
incidental 2.7 cm left adrenal mass
MRI: 2.7 cm left adrenal mass with intensity on T2 weighted images

Adrenal Incidentaloma
First – always form a differential diagnosis

1: Is this mass a cancer?


Adrenocortical Carcinoma
Metastasis (Lung, Breast, Melanoma, etc)

2: Does this mass make hormones?

699
Copyright © Oakstone Publishing, 2021. All Rights Reserved.

Adrenal Incidentaloma
First – always form a differential diagnosis

Godfrey Hounsfield

1971 – first CT images

700
Copyright © Oakstone Publishing, 2021. All Rights Reserved.

Hounsfield Scale
- 1000 = Air

- 100 = fat

0 = Water

30-45 = soft tissue

60- 90 = acute blood

100-500 = Iodinated contrast


400-800 = Trabecular Bone

>1000 = Cortical Bone

Adrenal Incidentaloma

701
Copyright © Oakstone Publishing, 2021. All Rights Reserved.

Adrenal Incidentaloma

887 patients analyzed

ACC was associated with increased


mass size,

90% of ACC were > 4 cm

>4cm criterion had a 93% sensitivity


for ACC

BUT 76% of masses >4cm were benign

CT / MRI Characteristics
Benign Malignant/Suspicious Pheo

CT Homogeneous Heterogeneous HU>20


Smooth Borders Irregular Borders Delayed washout
HU<10 (unenhanced) HU<10 <50%
Absolute washout >50% Absolute washout <50% Central
Calcifications necrosis/hemorrhage

MRI Isointense with liver No loss of signal Bright on T2


On T1 and T2 images High signal intensity on
T2
“Loss of signal on out of
phase imaging”

702
Copyright © Oakstone Publishing, 2021. All Rights Reserved.

Adrenal Incidentaloma

Benign and Functional - ZG

703
Copyright © Oakstone Publishing, 2021. All Rights Reserved.

Benign and Functional


Aldosteronoma

Benign and Functional


Aldosteronoma

Hypertension on multiple agents


Hypokalemia

Renin

Aldosterone

ARR = Aldo / Renin

ARR >20 is suggestive of primary hyperaldosteronism

If renin is high, consider renal artery stenosis (CTA, renal ultrasound, etc)

704
Copyright © Oakstone Publishing, 2021. All Rights Reserved.

Benign and Functional


Aldosteronoma

41 YF with hypertension (160s/ 90s)


Escalating medication regimen (amlodipine, enalapril)
K 3.3
Aldosterone: 31 ng/ml
Renin: < 0.6 ng/ml/h
ARR > 31/ 0.6 = 51

1.5cm L adrenal nodule ( -9 HU, thus adenoma)

Benign and Functional


Aldosteronoma

705
Copyright © Oakstone Publishing, 2021. All Rights Reserved.

Benign and Functional - ZF

Benign and Functional - ZF

706
Copyright © Oakstone Publishing, 2021. All Rights Reserved.

Benign and Functional - ZF

Check ACTH, Cortisol

Dexamethasone Suppression Test


Objective: inhibit the pituitary’s production of
ACTH

Check AM Cortisol and ACTH


If < 1.8ug/dl = normal suppression

If >1.8 ug / dl = autonomous cortisol


excess

Clinica chimica acta; international journal of clinical chemistry 2010


Cutoff values of midnight salivary cortisol for the diagnosis of overt hypercortisolism
are highly influenced by methods.

Benign and Functional - ZF

70YF with DM2, obesity, arthritis, weakness,


fatigue, hypertension

Dexamethasone Suppression Test:


Cortisol: 4 ug/dl (<1.8 ug/dl)
ACTH 6 (6-76 pg/ml)

MRI: 2.5 cm right adrenal adenoma

707
Copyright © Oakstone Publishing, 2021. All Rights Reserved.

Benign and Functional - ZR

708
Copyright © Oakstone Publishing, 2021. All Rights Reserved.

Endocrinologia y nutricion : organo de la Sociedad Espanola de


Endocrinologia y Nutricion 2009

Benign and Functional - Pheo

709
Copyright © Oakstone Publishing, 2021. All Rights Reserved.

Benign and Functional - Pheo

Palpitations, hypertension, anxiety, headache


Asymptomatic

What labs to get?

Metanephrine and normetanephrines


Plasma and/or Urinary

MRI, 68Ga DOTATATE, MIBG

Benign and Functional - Pheo

710
Copyright © Oakstone Publishing, 2021. All Rights Reserved.

R.R. – 69YF with a 4cm Left-Sided Pheochromocytoma

Pheochromocytoma

Adrenal Cortex
(normal)

Surgery - Technique

711
Copyright © Oakstone Publishing, 2021. All Rights Reserved.

Surgery - Technique

Laparoscopic Adrenalectomy

712
Copyright © Oakstone Publishing, 2021. All Rights Reserved.

Laparoscopic Adrenalectomy

713
Copyright © Oakstone Publishing, 2021. All Rights Reserved.

714
Copyright © Oakstone Publishing, 2021. All Rights Reserved.

1st Attending RP Adrenalectomy

16 YF – Weakness / Mental Status Changes

16YF with Metastatic PNET and Ectopic Cushing Syndrome

PNET Right Adrenal Gland

Left Adrenal
Right Adrenal

Abdominal Striae
GA-DOTATATE
PNET

Arm Bruising Liver Met


(Retrohepatic view)

PNET

“Casting a Wide NET” - NEJM 2021

715
Copyright © Oakstone Publishing, 2021. All Rights Reserved.

716
Copyright © Oakstone Publishing, 2021. All Rights Reserved.

Lap RP

Either RP or laparoscopy would work


well

Consider transabdominal laparoscopy

717
Copyright © Oakstone Publishing, 2021. All Rights Reserved.

718
Copyright © Oakstone Publishing, 2021. All Rights Reserved.

Cirulli et al. Cell Metabolism 2019

719
Copyright © Oakstone Publishing, 2021. All Rights Reserved.

720
Copyright © Oakstone Publishing, 2021. All Rights Reserved.

Part II
Malignant or Likely ACC

World Journal of Surgical Oncology 14(1) June 2016

Operative Approach for Possible


ACC – Pascal’s Wager

ACC Exists ACC does


not exist
Perform en bloc
oncologic
resection

“shell out”

721
Copyright © Oakstone Publishing, 2021. All Rights Reserved.

35YM Fisherman with Weakness

35YM with 13.8cm Right Adrenal Mass

Plasma Metanephrines < assay


Loss of fat plane 24h urine cortisol: 211 mcg/24h (3.5-45)
Free testosterone: 1.91 nl/dl (4.65-18.1)
DHEA-S: 9.60 ug/ml (0.89-4.27)
Androstendione: 155 ng/dl (40-150)
11-deoyxcortisol: pending

Subclinical Gynecomastia ACC Liver (retracted)

ACC

IVC

R. Kidney

722
Copyright © Oakstone Publishing, 2021. All Rights Reserved.

35YM Fisherman with Weakness

Plasma Metanephrines < assay


Loss of fat plane 24h urine cortisol: 211 mcg/24h (3
Free testosterone: 1.91 nl/dl (4.65
DHEA-S: 9.60 ug/ml (0.89-4.27)
Androstendione: 155 ng/dl (40-15
11-deoyxcortisol: pending

Subclinical Gynecomastia

ACC

723
Copyright © Oakstone Publishing, 2021. All Rights Reserved.

35YM Fisherman with Weakness

Liver (retracted)

ACC

IVC

R. Kidney

6 Weeks Post op

1 year post op
*All photos with permission of the patient

724
Copyright © Oakstone Publishing, 2021. All Rights Reserved.

36YF with Weakness &High Testosterone

36YF with Weakness &High Testosterone

725
Copyright © Oakstone Publishing, 2021. All Rights Reserved.

*All photos with permission of the patient

726
Copyright © Oakstone Publishing, 2021. All Rights Reserved.

Right Atrial Tumor Thrombus

727
Copyright © Oakstone Publishing, 2021. All Rights Reserved.

71YF with Incidental Right Adrenal Mass

*
*

Case 5: 71YF with Incidental Right Adrenal Mass


Diaphragm
Inferior Right Hepatic Vein
(variant)

R. Kidney

R. Renal Artery

IVC
R. Renal Vein

Aorta
R. Ureter
Duodenum
(Retracted)
IMA

R. Common
Iliac Artery

728
Copyright © Oakstone Publishing, 2021. All Rights Reserved.

71YF with Incidental Right Adrenal Mass

ACC

Aortocaval Nodes

62YF with 14 cm Malignant Pheochromocytoma

729
Copyright © Oakstone Publishing, 2021. All Rights Reserved.

Thoracoabdominal:

53YM Software Engineer – MEN2a Metastatic Pheo

730
Copyright © Oakstone Publishing, 2021. All Rights Reserved.

….to Zuckerkandl

731
Copyright © Oakstone Publishing, 2021. All Rights Reserved.

56YF with 10 cm Recurrent Para-Aortic Paraganglioma

732
Copyright © Oakstone Publishing, 2021. All Rights Reserved.

56YF with 10 cm Recurrent Para-Aortic Paraganglioma

60 YM with 8 cm Retrohepatic / Retro-IVC Para-aortic paraganglioma

733
Copyright © Oakstone Publishing, 2021. All Rights Reserved.

Surgical Strategy
Educational
Objectives for
Residents and Fellows

Facilitate long-term healing


and return to work

Minimize Tissue Trauma and


Perioperative Complications

Cure the Disease

734
Copyright © Oakstone Publishing, 2021. All Rights Reserved.

Diagnostic Strategy

Choose the Right


Operation

CT, MRI, PET, Ga DOTATATE

Biochemical Workup

Generate A Differential

Adrenal Surgery and Endocrine


Dysfunction

Matthew A. Nehs, MD
Brigham and Women’s Hospital
Harvard Medical School
Disclosures: None

735
Copyright © Oakstone Publishing, 2021. All Rights Reserved.

Thyroid and Parathyroid Surgery

Nancy L. Cho, MD
Assistant Professor of Surgery
Harvard Medical School
Associate Surgeon
Brigham & Women’s Hospital

Disclosures

• Veracyte - Consultant

736
Copyright © Oakstone Publishing, 2021. All Rights Reserved.

Blame the Thyroid

737
Copyright © Oakstone Publishing, 2021. All Rights Reserved.

Surgical Indications

Thyroid Cancer Incidence

738
Copyright © Oakstone Publishing, 2021. All Rights Reserved.

Thyroid Cancer Incidence

Thyroid Cancer Incidence

739
Copyright © Oakstone Publishing, 2021. All Rights Reserved.

Types of Thyroid Cancer

Prognosis

740
Copyright © Oakstone Publishing, 2021. All Rights Reserved.

Thyroid Nodule Work-up

Ultrasound Risk of Malignancy

741
Copyright © Oakstone Publishing, 2021. All Rights Reserved.

Thyroid Nodule Work-up

Bethesda Classification

Cytology Result Likelihood of Malignancy Usual Management


Non-diagnostic (I) 1-4% Repeat US-guided FNA
Benign (II) 0-3% Surveillance
Atypia of undetermined 5-15% Surveillance vs. repeat FNA
significance (AUS) or with molecular/genetic
Follicular lesion of testing vs. surgery
undetermined significance
(FLUS) (III)
Follicular or Hurthle cell 20-30% Repeat FNA with
neoplasm (IV) molecular/genetic testing vs.
surgery
Suspicious for malignancy (V) 60-75% Generally treat as for
malignancy
Malignant (VI) 98-99% Treat for malignancy (total
thyroidectomy vs. thyroid
lobectomy)

Cibas et al (2009) Am J Clin Pathol

742
Copyright © Oakstone Publishing, 2021. All Rights Reserved.

• Gene expression classifier to stratify indeterminate


nodules – Bethesda category III and IV – and
improve preoperative risk assessment
• Expression pattern of 167 genes

743
Copyright © Oakstone Publishing, 2021. All Rights Reserved.

Nikiforov et al (2016) JAMA Oncol

NIFTP Malignancy Rates

744
Copyright © Oakstone Publishing, 2021. All Rights Reserved.

NIFTP Malignancy Rates

Number
Bethesda Category % NIFTP by
(n=26)
Category
Bethesda I: Non-diagnostic 0 0% (0/0)

Bethesda II: Benign 2 2.7% (2/74)

Bethesda III: AUS/FLUS 13 15.3% (13/85)

Bethesda IV: SFN/SHCN 4 6.8% (4/58)


Bethesda V: Suspicious for
6 18.1% (6/33)
malignancy

Bethesda VI: Malignant 1 1.0% (1/99)

Malignancy Rates by Bethesda Category

745
Copyright © Oakstone Publishing, 2021. All Rights Reserved.

Extent of Surgery by Bethesda Category

Total thyroidectomy Thyroid lobectomy

ATA Guidelines 2015

746
Copyright © Oakstone Publishing, 2021. All Rights Reserved.

ATA Guidelines 2015

What is the effect of the guideline change on the rate of


completion thyroidectomy?

Rate of Completion Thyroidectomy?

Patient population: single institution, all patients


2014-2018 with initial thyroid lobectomy and
DTC on pathology
– Microcarcinomas < 1 cm excluded
– Subset analysis 1-4 cm

Pre Post
Versus
2014-2015 2016-2018

747
Copyright © Oakstone Publishing, 2021. All Rights Reserved.

Rate of Completion Thyroidectomy?

163 patients included in study: 63 pre, 100 post


– Pre and Post cohorts similar demographics, tumor size,
histologic characteristics
– Completion thyroidectomy: Pre 41, Post 43
– 34% decrease in rate of completion thyroidectomy, p<0.01

41/63 (65.1%) 43/100 (43%)

Pre Post
Versus
2014-2015 2016-2018

Rate of Completion Thyroidectomy?

Low-risk, 1-4cm subset: 35 pre, 60 post


– Completion thyroidectomy: Pre 17, Post 15
– 48.6% decrease in rate of completion thyroidectomy, p=0.02
– Factors associated with completion thyroidectomy: tumor
size, capsular invasion, multifocal disease

17/35 (48.6%) 15/60 (25.0%)

Pre Post
Versus
2014-2015 2016-2018

748
Copyright © Oakstone Publishing, 2021. All Rights Reserved.

Less is More

Welch and Doherty (2018) N Engl J Med

Future Directions

• Radiofrequency ablation
• Conservative management for small thyroid nodules

749
Copyright © Oakstone Publishing, 2021. All Rights Reserved.

Indian Rhinoceros
George Stubbs (1790)

“a small compact yellow glandular body [that] was attached to the


thyroid at the point where the veins emerge.”
Owen R (1862): Trans Zool Soc Lon.

750
Copyright © Oakstone Publishing, 2021. All Rights Reserved.

History of the Parathyroid

• Ivor Sandstrom (1880)


- Sandstrom IV (1938) Bull Hist Med.

• William MacCallum & Carl Voegtlin (1909)


- MacCallum WJ, Voegtlin C (1909) J Exp Med.

• Felix Mandl (1925)


- Mandl F (1926) Wien Klin Wochenschr Zentral.

• Isaac Olch (1928)


- Barr DP, Bulger HA, Dixon HH (1929) JAMA.

Epidemiology

• 100,000 new cases of primary hyperparathyroidism (PHPT)


diagnosed in the United States.
- 80-87% single adenoma
- 10-15% 4-gland hyperplasia
- 2-5% multiple adenomas
- <1% carcinoma

751
Copyright © Oakstone Publishing, 2021. All Rights Reserved.

Differential Diagnosis

• Primary hyperparathyroidism (PHPT)


- accounts for majority of cases among outpatient population
- elevated Calcium levels (normal range 8.8 – 10.7 mg/dL)
- inappropriately elevated PTH (normal range 15- 65 pg/mL)

• Differential
- malignancy
- drugs (thiazide diuretics, Calcium supplements, lithium)
- secondary HPT (renal failure, vitamin D deficiency, malabsorption, celiac
disease)
- familial hypocalciuric hypercalcemia (FHH)

UptoDate: Diagnosis and Differential Diagnosis of Primary Hyperparathyroidism

752
Copyright © Oakstone Publishing, 2021. All Rights Reserved.

Clinical Manifestations

• Painful bones
- fractures, bone/joint pain, back pain

• Kidney stones
- hematuria

• Abdominal groans
- GERD, constipation, pancreatitis, ulcers, nausea, anorexia

• Psychic moans
- depression, memory loss

• Fatigue overtones
- exhaustion, insomnia

Diagnosis

• Laboratory Studies
- elevated Calcium levels (normal range 8.8 – 10.7 mg/dL)
- inappropriately elevated PTH (normal range 15- 65 pg/mL)
- ionized Calcium
- 24 hour urine Calcium
- Vitamin D levels
- alkaline phosphatase

753
Copyright © Oakstone Publishing, 2021. All Rights Reserved.

NIH Criteria for Parathyroidectomy

• Markedly elevated serum calcium (1.0 mg/dL above normal)


• Creatinine clearance reduced to <60 ml/min with age-
matched normal subjects
• Age < 50
• Hypercalciuria (24hr urine calcium >400mg/dL)
• Substantially reduced bone mass as determined by direct
measurement (T score < -2.5 at any site, prior fx, or
vertebral compression fracture)

Silverberg et al. (2014): J Clin Endocrinol Metab

Imaging

• Bone Density Scan


- forearm/hip > spine
• Localization Studies
- Ultrasound (US) - Sestamibi (MIBI) - 4DCT

754
Copyright © Oakstone Publishing, 2021. All Rights Reserved.

Diagnosis of Primary Hyperparathyroidism

Ultrasound

Single parathyroid adenoma Negative, equivocal, or


Thyroid nodule > 1cm
multiple adenomas

Renal disease, prior lithium use,


Fine needle aspiration age > 70, adenoma size > 2 cm? Sestamibi, 4D CT (optional)

Yes No Unilateral exploration + IOPTH


or bilateral neck exploration
Bilateral neck Sestamibi/4DCT
exploration +/-
IOPTH
Discordant or negative
Concordant imaging

4D CT (optional)
Unilateral neck
exploration Unilateral neck exploration +
IOPTH or bilateral exploration

Minimally Invasive Parathyroidectomy (MIP)

http://endocrinesurgery.ucla.edu/surgery_mip.html

755
Copyright © Oakstone Publishing, 2021. All Rights Reserved.

• Surgical failure rate in patients with concordant imaging was


1% with IOPTH measurement and 2% without IOPTH
measurement (p = 0.50).
• Use of IOPTH measurement does not significantly increase
the success of MIP.

Gawande et al. (2006) Arch Surg


Kebebew et al. (2006) Arch Surg
Smith N et al. (2009) Arch Oto Head Neck Surg

Aim

Validate the effectiveness of a protocol for primary


hyperparathyroidism (PHPT) in which intra-operative
parathyroid hormone (IOPTH) was not routinely used
during minimally-invasive parathyroidectomy (MIP) for
patients with concordant localization by sestamibi (MIBI)
and ultrasound (US).

756
Copyright © Oakstone Publishing, 2021. All Rights Reserved.

Study Design

• Prospective cohort study.


• Academic university hospital (Brigham and Women’s
Hospital, Boston, MA) from October 2005 to September
2009.
• Patients with PHPT and concordant localization (MIBI &
US) undergoing MIP.
• Incidence of surgical cure following MIP without
utilization of IOPTH.

Study Algorithm

757
Copyright © Oakstone Publishing, 2021. All Rights Reserved.

Results

Results

• 4/4 surgical failures had ipsilateral unseen 2nd adenoma on re-operation.


• No significant difference in surgical cure rate following MIP without
IOPTH for this prospective study compared with our previously
published retrospective analysis (97% v. 98%, p=0.47).

758
Copyright © Oakstone Publishing, 2021. All Rights Reserved.

• Focused parathyroid exploration without IOPTH can be


successfully performed in patients with concordant
localization by MIBI and US.
• Identification of the 2nd ipsilateral gland is critical to
ruling out undetected multiglandular disease.
• New protocol includes frozen section biopsy of ipsilateral
gland.
Cho et al. (2011) Arch Surg

4DCT

• High resolution imaging – helpful for negative US or MIBI,


reoperative parathyroid surgery
• 4th dimension ~ time (arterial and venous phase)

759
Copyright © Oakstone Publishing, 2021. All Rights Reserved.

Case #1

• 57yF with elevated Ca 11.2 – 11.4 on routine labs.


• PTH 149.
• Mild fatigue, bone/joint pain, difficulty with concentration.
• No nausea/abd pain, constipation, depression, muscle weakness,
reflux, or pancreatitis.
• No kidney stones, bisphosphonate/calcimimetic, diuretic use.
• No FH, radiation.
• Bone density scan shows osteopenia.

Case #1

760
Copyright © Oakstone Publishing, 2021. All Rights Reserved.

Case #1

Case #1

• Surgery – focused RIGHT parathyroidectomy


• Final pathology results: 1.1cm parathyroid adenoma (0.4 grams)
• Post-operative Ca 9.5, PTH 28.
• Continue to check Ca/PTH for 6 months.
• Maintain Calcium supplementation approximately 1 year to
rebuild bone.

761
Copyright © Oakstone Publishing, 2021. All Rights Reserved.

Case #2

• 44yF with elevated Ca 10.3 – 10.9 on routine labs.


• PTH 135 - 150.
• Occasional fatigue and muscle aches. No nausea/abd pain,
constipation, depression, muscle weakness, reflux, or
pancreatitis.
• No kidney stones, calcimimetic, or diuretic use.
• No FH, radiation.
• Bone density scan shows osteopenia.
• Ultrasound negative for parathyroid adenoma.

Case #2

762
Copyright © Oakstone Publishing, 2021. All Rights Reserved.

Case #2

• Surgery – focused RIGHT parathyroid exploration with IOPTH


monitoring.
• Final pathology results: 1.7 cm RIGHT hypercellular parathyroid
(0.576 g).
• Pre-PTH 186 to post-PTH 32.
• Post-operative Ca 8.9, PTH 26.

763
Copyright © Oakstone Publishing, 2021. All Rights Reserved.

Benign Breast Disease

Anna Weiss MD
October 15, 2021

Disclosures
• Sponsored research agreement with Myriad
Laboratories, Inc.

764
Copyright © Oakstone Publishing, 2021. All Rights Reserved.

Objectives
• Breast masses (cysts, fibroadenomas, phyllodes, PASH)
• Nipple discharge (papillomas, terminal duct excision)
• Abnormal mammogram (calcifications, high risk lesions/atypias)
• Breast pain (breast density)
• Pregnancy findings and mastitis needing surgical intervention
• Chronic granulomatous mastitis

Breast masses

765
Copyright © Oakstone Publishing, 2021. All Rights Reserved.

Palpable breast mass


Age A single breast mass
is likely to be:
Under 30 Fibroadenoma or cysts,
palpable dense breast
tissue
30’s and 40’s Fibroadenoma,
fibrocystic changes, or
other benign problem

50 and older Cysts, invasive cancer

A 43-year-old female presents with a palpable mass in the


left breast at 12 o’clock. This mass is tender. What is your
next step (or options) in the management of this patient?

766
Copyright © Oakstone Publishing, 2021. All Rights Reserved.

Breast cyst
• Occur most often in women ages 35-50
• Cysts may fluctuate with the menstrual cycle
• The incidence of malignancy in cysts is reported to be 0.2-
2.2%

767
Copyright © Oakstone Publishing, 2021. All Rights Reserved.

Imaging findings
• Mammogram: round or oval mass, low-density,
circumscribed
• U/S: round or oval, avascular, anechoic lesion with
posterior enhancement. The mass may flatten with gentle
compression (unlike solid lesions)
• Complex cysts: have some internal echoes, representing debris

Imaging findings
• Features associated with cancer:
• Cysts >3cm
• Thick cyst wall
• Mural tumor
• Eccentric mass
• Bloody aspirate- send for cytologic analysis
• Biopsy if any concerning features, repeat U/S after aspiration, ? mass

768
Copyright © Oakstone Publishing, 2021. All Rights Reserved.

Cyst treatment
• Low risk features:
• No treatment/ observations- may regress spontaneously
• Symptomatic cyst:
• Aspiration
• Follow-up:
• Cyst that are persistent after multiple aspirations should be further
investigated with cytology and core biopsy

A 17-year-old female presents to the office with a left breast


lump. She noticed this lump two weeks ago while rowing. She
denies any breast pain, nipple discharge, or skin dimpling.
The patient and her mother are worried because her
grandmother had breast cancer. On exam she has a hard but
very mobile lump at 5 o’clock.

769
Copyright © Oakstone Publishing, 2021. All Rights Reserved.

Ultrasound is the best modality in this age group

Fibroadenoma
• A type of fibroepithelial lesion
• Connective tissue and proliferatory epithelium
• 10% of women
• Arise from hormone-dependent terminal duct lobular unit-
influenced by hormones
• Most common in second or third decade but can be younger
• Multiple or bilateral FA can occur in 10-20% of women

770
Copyright © Oakstone Publishing, 2021. All Rights Reserved.

Fibroadenoma
• The stromal element of these tumors defines their
classification and behavior
• Simple fibroadenoma
• Low cellularity and regular cytology

Fibroepithelial lesion
• Work up:
• U/S
• Classic fibroadenoma on ultrasound: Observation - q 6 month CBE
and U/S
• Mammogram
• Core needle biopsy
• Fibroepithelial lesion: Fibroadenoma vs phyllodes

771
Copyright © Oakstone Publishing, 2021. All Rights Reserved.

Management
• Surgical excision of a fibroadenoma-
• “Fibroepithelial lesions” should be excised for definitive diagnosis

• Increase in size

• > 3 cm
• Symptomatic

• Older women and uncertain diagnosis


NCCN guidelines

A 60-year-old woman presents with a right breast lump that


has increased in size over the past 3 months. Core needle
biopsy reveals fibroepithelial lesion.

772
Copyright © Oakstone Publishing, 2021. All Rights Reserved.

Phyllodes tumor
• <1% of all breast lesions
• 2.5% of fibroepithelial tumors
• Age of onset: 15-20 years later than fibroadenomas.
Median age at presentation 45 years

Imaging
• Mammogram: dense and
sharply demarcated mass
• U/S: circumscribed solid lesion
without acoustic shadowing

773
Copyright © Oakstone Publishing, 2021. All Rights Reserved.

Phyllodes tumor
• Usually benign but have malignant potential
• Histologically these lesions may be classified as benign,
borderline, or malignant
• Size
• Ratio of stroma and epithelium
• Border of the lesion
• Stromal cellularity
• Number of stromal mitosis
• Presence or absence or necrosis

Phyllodes

774
Copyright © Oakstone Publishing, 2021. All Rights Reserved.

Treatment
• Resection with negative margins for benign and intermediate
• >1 cm margin for malignant
• 550 patients, multicenter registry, Rosenberger et al. JCO Jan 2021
• 68.9% were benign, 19.6% borderline, and 10.5% malignant
• Margins were positive in 43% and negative in 57%
• Recurrence (all grades) was not reduced with:
• Wider negative margin width (≥ 2 mm v < 2 mm: odds ratio [OR] = 0.39; 95% CI, 0.07 to
2.10; P = .27)
• Final margin status (positive v negative: OR = 0.96; 95% CI, 0.26 to 3.52; P = .96)

• No role for axillary sampling unless clinically palpable nodes are present

Phyllodes
• High rate of local recurrence (20%), which is not correlated
with benign versus malignant classification
• Metastatic spread is similar to that of sarcomas
• <5% of all phyllodes metastasize
• 25% of malignant metastasize (JCO 3/58 at 36.7 months)

775
Copyright © Oakstone Publishing, 2021. All Rights Reserved.

A 27-year-old who is currently pregnant presents to you


because she palpated a mass in her lower inner breast. You
send her for ultrasound and core needle biopsy.

PASH
• Benign proliferation of breast stroma
• Very common incidental finding on breast biopsy
specimens; more rarely can present as a breast mass
• Resembles fibroadenoma as a well-defined lesion on
mammography and hypoechoic lesion on sonography

776
Copyright © Oakstone Publishing, 2021. All Rights Reserved.

PASH
• Biopsy is typically required to exclude malignancy
• Surgical excision is recommended for lesions that are
enlarging, symptomatic, or have concerning imaging
characteristics; otherwise, close clinical observation is
appropriate
• Consider excision in younger patients, pre- pregnancy. These will
enlarge during pregnancy
• PASH does not increase the risk for breast cancer

Nipple discharge

777
Copyright © Oakstone Publishing, 2021. All Rights Reserved.

A 44-year-old female presents to the clinic with spontaneous,


bloody, single duct nipple discharge on the right. This began 1
month ago and has occurred 6-7 times. Her last mammogram,
9 months ago, was unremarkable. She read on the internet
that this can be a sign of cancer.

Differential
• Intraductal papilloma
• Duct ectasia
• Intracystic papillary carcinoma
• Ductal Carcinoma in Situ
• Invasive cancer
• Nipple adenoma
• Paget’s

778
Copyright © Oakstone Publishing, 2021. All Rights Reserved.

Nipple discharge

• Common presentation to the breast clinic


• Affects 3-5% of women

• Physiological vs pathological

Nipple discharge

• Physiologic Aka Nipple secretion (non-spontaneous):


• Fluid present in the ducts that must be collected by nipple
aspiration or gentle massage

• Squeezing the nipple

• Pathologic nipple discharge (spontaneous):


• Fluid that escapes spontaneously from the nipple

779
Copyright © Oakstone Publishing, 2021. All Rights Reserved.

Nipple discharge
Physiologic vs. Pathologic
 Color:
– Milky Galactorrhea/Prolactinoma/Meds
– Gray-green, Duct Ectasia/fibrocystic
– Bloody Intraductal Papilloma/Malignancy
– Clear Intraductal papilloma/Malignancy

 Site: Unilateral vs Bilateral


 Onset: Spontaneous vs Induced
 Time course: Persistent vs Self-limited
 Risk of underlying malignancy is about 5% and is typically DCIS.

Nipple discharge
• Work up:
• Breast exam
• Try to identify quadrant
• A scab may be a clue
• Assess for palpable mass
• Assess lymphadenopathy
• Hemoccult can be helpful for black discharge
• Absence of blood does not mean there is not an underlying
cancer
• Cytology is of little value

780
Copyright © Oakstone Publishing, 2021. All Rights Reserved.

Nipple discharge
• Work up (Imaging):
• Mammogram
• Ultrasound
• Ductoscopy
• Ductal lavage
• Ductography
• MRI

Nipple discharge

• Who can be observed?


• No significant risk factors
• Single/few occurrences
• Normal exam
• Negative imaging including MRI
• Younger more than older
• Advanced age is a predictor of cancer in women with nipple discharge
• Fibrocystic breast disease often produces green/serous nipple discharge
• Selective surgery
• Definitive tissue diagnosis
• Symptom control

781
Copyright © Oakstone Publishing, 2021. All Rights Reserved.

Terminal duct excision

• If Normal Imaging, Is Duct Excision Necessary?


• 175 women referred for nipple discharge
• 81% (142) had pathologic discharge
• 119/142 underwent surgery
• Cancer was diagnosed in 7 patients (all DCIS) (5.8%)
• 6/7 patients had abnormal imaging
• 23 patients with normal imaging and PND were observed.
• No sign of cancer with 3 year follow up
• Trick - tie duct off with 4-0 Prolene

Sabel M et al, The Breast Journal 2011

Papillomas
• Many cases of nipple discharge are caused by intraductal
papilloma
• Imaging – mass identified close to the nipple

• Core needle biopsy

• Excisional biopsy recommended

782
Copyright © Oakstone Publishing, 2021. All Rights Reserved.

Abnormal mammogram

Abnormal mammogram
• Calcifications
• Density
• Mass
• Architectural Distortion
• Asymmetry

783
Copyright © Oakstone Publishing, 2021. All Rights Reserved.

BIRADS
(Breast Imaging and Reporting Data System)
• BIRADS 0: Incomplete Assessment
• BIRADS 1: Entirely negative (normal)
• BIRADS 2: Benign Findings
• BIRADS 3: Probably benign, 6 month follow up (<3% cancer)
• BIRADS 4: Suspicious abnormality, biopsy
recommended (% of cancer depends on A, B, or C designation)
• BIRADS 5: Highly suggestive of malignancy, biopsy
• BIRADS 6: Known biopsy-proven malignancy

784
Copyright © Oakstone Publishing, 2021. All Rights Reserved.

Benign “High Risk”


• Fibrocystic change • ADH Invasive Cancer
• Fibroadenoma • *LCIS/ALH
• Usual hyperplasia • *Radial Scar • Invasive
• Apocrine • *FEA DCIS ductal/lobular
metaplasia • *Intraductal carcinoma
• Sclerosing papilloma
adenosis
• Fat Necrosis
• Fibrosis
• Lumpectomy • Surgery
+/- XRT, AET • Radiation
• Mastectomy • Medical therapies
No special follow up Observation/ + SLNB
indicated Excisional biopsy

Excisional biopsy  Rule out sampling error

785
Copyright © Oakstone Publishing, 2021. All Rights Reserved.

Upgrade rates
• ADH 15-20%
• LCIS/ALH <3% (if benign and concordant)
• Radial Scar 0-6.9% (1-2 % if no atypia)
• FEA <3%
• Fibroepithelial lesion (discussed above) 10-15% -> benign phyllodes
• Intraductal papilloma 20-25% (with atypia)
0-2.9% (asymptomatic, no atypia)

ADH

• Mooney et al. Modern Pathology 2016


• 5750 core needle biopsies revealing different atypias reviewed
• 18% upgrade rate for ADH (26% for LCIS –however 5% incidental LCIS and
36% among discordant)
• Selective observation – Menen.. Bedrosian et al.
• Concordance review, # of involved TDLUs, % calcifications excised by CNB
• Among this selective cohort 125 women observation, 50 women surgery, equal
rates of development of cancer
• Not only a potential upgrade risk, but also a marker of high risk of
malignancy

786
Copyright © Oakstone Publishing, 2021. All Rights Reserved.

Hartmann et al NEJM 2015; 372:78-89

Lobular neoplasias
• TBCRC 020 – prospective multi-institution trial
• Patients with pure lobular neoplasias were included
• N=79
• Two cases (3%; 95% confidence interval 0.3-9) were upgraded to
cancer (one tubular carcinoma, one ductal carcinoma-in-situ)
(local review)
• Central review the tubular carcinoma was seen on core, so
upgrade rate was one case (1%; 95% CI 0.01-7)

787
Copyright © Oakstone Publishing, 2021. All Rights Reserved.

Lobular neoplasias
• LCIS/ALH are NOT a premalignant condition, it is a marker of high risk for
malignancy
• Roughly 30% of patients with lobular neoplasia develop ductal carcinoma and
can be in either breast!
• van Maaren et al – population-based analysis out of the Netherlands
• N=1890; 1989-2017; surgical treatment decreased from 100% to 41.1%
• 2.5% developed DCIS and 14.3% invasive breast cancer
• Surgery not needed but enhanced screening indicated
• MSKCC 30 year follow up
• N=1060; 2% cancer incidence per year
• 10-year cumulative risk: 7% with chemoprevention; 21% with no chemoprevention; P < .001

FEA and radial scars


• Poor data currently
• Excisional biopsy until more data
• TBCRC 034 – second arm includes FEA patients, just closed
to accrual, data forthcoming
• Radial scars/complex sclerosing lesions – increased
detection on Tomo; prospective registry at BWH currently
enrolling

788
Copyright © Oakstone Publishing, 2021. All Rights Reserved.

Asymptomatic Papillomas
• TBCRC 034 – prospective multicenter trial
• Included asymptomatic papillomas without atypia on core needle
biopsy
• Most presented as a mass on imaging, rest were incidental findings
• Statistics based on predefined rule that an upgrade rate of ≤ 3%
would not warrant routine excision
• N=116
• 2 (1.7%) cases were upgraded to DCIS

Breast pain/mastalgia

789
Copyright © Oakstone Publishing, 2021. All Rights Reserved.

Breast pain
• Moderate to severe pain in the breasts >5 days.
• Prevalence is between 45-100%.
• 2/3 report pain that interferes with daily activities.
• 1659 women/42 countries
• Cyclic pain (66%), affecting sexual activity (42%), sleep (35%), work activity (5%).
• Breast pain is rarely associated with breast cancer (0.5-3.3%)
• Ahmed (Ir J Med Sci 2016) 1014 referrals for breast pain – No cancers (0%)
• Chetlen (Acad Radiol 2017) 236 pts, 1 cancer-focal, noncyclical pain (0.4%)
• Martin-Diaz (Curr Opin Obst Gynec 2017) Systematic review, 7 studies, low prevalence
of breast cancer (0.4-3.2%)

Breast pain
• Physical Exam – In addition to breasts/axilla, examine shoulder, neck,
chest wall for extra-mammary sources of pain
• Extramammary Sources of Pain
• Musculoskeletal (muscle strain, costochondritis, shoulder pathology-Scapulothoracic Bursitis)
• Thoracic Outlet
• Infectious (Shingles, mastitis)
• Cardiac
• GERD
• Imaging
• <30 y.o. – diffuse, non-focal cyclic pain-> reassurance +/- treatment
• <30 y.o. – focal pain – breast ultrasound
• >/= 30 y.o. and focal pain – Diagnostic mammogram/ultrasound

790
Copyright © Oakstone Publishing, 2021. All Rights Reserved.

Types and factors


• Cyclic breast pain (70%)
• Premenopausal, bilateral or unilateral, UOQ
• Described as heaviness/tenderness
• Associated with normal hormone changes during menstrual cycle, pregnancy, menopause,
fibrocystic breasts
• Studies have shown no differences in circulating hormone levels in women
with and without breast pain
• Breast tissue is hypersensitive
• Caffeine, nicotine, tyramine, and stress increase catecholamine release
and can cause hypersensitivity of breast tissue.
• Cyclical breast pain resolves spontaneously within 3 months of onset in
30% of women
Sasaki & Klimberg. Ch 5. The Breast Book. 2018

Types and factors


• Non-cyclical breast pain (30%)
• All ages, bilateral or unilateral,
• Constant or intermittent
• Often described as sharp or burning
• Due to breast movement, large breast size, high BMI, may be chest wall pain
• 75% of women wear the wrong size bra
• Duct ectasia, cysts, stress, caffeine, trauma, HRT, idiopathic thrombophlebitis
• Pain responds poorly to treatment but may resolve spontaneously in about 50% of
women

791
Copyright © Oakstone Publishing, 2021. All Rights Reserved.

Treatment
• Diclofenac (a topical NSAID) effective at relieving symptoms
of cyclical and non-cyclical breast pain.
• Class 1 evidence
• Placebo controlled RCT (108 patients- 60 cyclic, 48 non-cyclic)
• Diclofenac gel to breast every 8 hours for 6 months
• Effects of decreased pain noticed at 15 days
• Should be considered as a first line treatment, benefits
outweigh the risk of adverse effects.

Goyal, et al. BMJ Clin Evid, 2011

792
Copyright © Oakstone Publishing, 2021. All Rights Reserved.

Treatment
• Decrease dietary fat intake (less than
15% of total calories)
• 21 patients, randomized to diet
intervention versus control
• Patients on low fat diet had significant
reduction in breast swelling and
tenderness at 6 months.

Boyd, Lancet, 1988

Treatment
• Women with breast pain have lower
levels of essential fatty acids
• Hypothesis that essential fatty acid
deficiencies may affect the functioning
of the breast cell membrane receptors
by producing a “supersensitive state” • Evening primrose oil =
• Free fatty acids (linoleic acid)
linoleic acid

793
Copyright © Oakstone Publishing, 2021. All Rights Reserved.

Treatment
• 159 Women with breast
pain
• Significant reduction in
pain at 2 months with
flaxseed and
chasteberry

Treatment
• Reassurance that this is not cancer
• Effective: 85% for mild, 70.8% moderate, 52.8% severe
• Firm supportive bra fitted by a professional
• 75-85% improvement with fitted bra or sports bra
• Exercise – Genc (Phys Sportsmed 2017), RCT 20 women, 3
x /week x 6 weeks- improved sensory component, physical
pain, bodily pain, and social functioning scores
• Stop or change HRT/OCPS

794
Copyright © Oakstone Publishing, 2021. All Rights Reserved.

Breast density
• Having dense breast tissue may increase risk of
developing breast cancer (low but real relative risk)
• Dense breasts also make it more difficult to spot
cancer on mammograms
• Dense tissue appears white on a mammogram as
does cancer

Breast density
• Extremely dense (D) Dense
• Heterogeneously dense (C)
• Scattered density (B)
Not dense
• Fatty (A)
• Breast density in the U.S.
• 10% of women have almost entirely fatty breasts
• 10% have extremely dense breasts
• 80% are classified into one of two middle categories
• Usually decreases with age.

795
Copyright © Oakstone Publishing, 2021. All Rights Reserved.

Sensitivity 88%
Specificity 97%

A B C D Sensitivity 62%
Specificity 90%

Pregnancy related findings

796
Copyright © Oakstone Publishing, 2021. All Rights Reserved.

Lactational Mastitis
• Up to 20% of breastfeeding women
• More common in first 6-12 weeks
• Painful, red, swollen breast plus fever, chills, malaise,
myalgia
• May progress to abscess, even septicemia
• Risk factors are improper nursing technique causing milk
stasis and disrupted skin integrity allowing entrance of
microorganisms.

Lactational Mastitis

• Image guided aspiration or IR drainage is standard of care


• Encourage women to continue breast feeding; abrupt
weaning worsens inflammation
• Encourage women to continue nursing from the affected
side; there is no harm to mom or baby

• Scheduled anti-inflammatories (Motrin), Lecithin

797
Copyright © Oakstone Publishing, 2021. All Rights Reserved.

If concerned for abscess order:


- Breast US
- Breast Cyst/Aspiration

Ultrasound with Abscess


• US guided aspiration by breast
radiology
• Send for culture
• Antibiotics as above; OTC probiotics
• Repeat aspiration is likely to be
needed
• Follow up with Breast Surgery

798
Copyright © Oakstone Publishing, 2021. All Rights Reserved.

Non-lactational abscess
• Commonly:
• Women in their 30s
• Frequently with central cleft in nipple
• Recurring subareolar abscess and lactiferous duct fistula is
associated with cigarette smoking
• Risk factors for recurrence
• Smoking, obesity, DM, trauma to the nipple

799
Copyright © Oakstone Publishing, 2021. All Rights Reserved.

Treatment
• Image-Guided Aspiration
• Needle aspiration of breast abscesses smaller than 5 cm can
accomplish healing without the cosmetic problems associated with
I&D
• Serial aspirations may be necessary to achieve abscess resolution
• Schedule with radiology~Q3 days
• Larger abscesses may fail treatment with needle
aspiration alone
• Surgery if:
• Superficial and spontaneously draining – small incision and wick
• Wound necrosis – operative debridement

Chronic granulomatous mastitis

800
Copyright © Oakstone Publishing, 2021. All Rights Reserved.

Chronic granulomatous mastitis


• Women of childbearing age (average 35 yrs)
• History of pregnancy and lactation within the past 5 years
• Can also occur in nulliparous women
• Higher incidence in non-Caucasians (Hispanic and Asian)

• Unilateral
• Painful breast mass/masses
• Abscess - peripheral
• Inflammatory symptoms
• Sinus tract formation “cobblestone”
• Nipple Inversion
• Enlarged axillary lymph nodes (40%)

801
Copyright © Oakstone Publishing, 2021. All Rights Reserved.

Imaging

• Mammography
• An ill-defined density with
spiculated margins
• Skin thickening
• Nipple retraction

Imaging
• Ultrasound
• Hypoechoic mass/abscess with tubular extensions
• Enlarged lymph nodes, concentric mild cortical thickening, preservation of the hila
Breast Axilla

802
Copyright © Oakstone Publishing, 2021. All Rights Reserved.

Diagnostic uncertainty
• Can be confused with malignancy, infection or
abscess!

Diagnosis
• Core needle biopsy
• (94-100% accuracy)
• Histopathology
• Non-caseating granulomas
• Send tissue for
• Gram stain
• Bacterial culture
• Note: Corynebacterium
• Acid-fast bacilli stain and culture – RULE OUT TB
• Fungal stain and culture
• If possible, send a core for microbiology

803
Copyright © Oakstone Publishing, 2021. All Rights Reserved.

Management
• Self-limited condition
• Resolution within 9-12 months
• NO progression to cancer

Management
• Watchful waiting
• Antibiotics
• NSAIDs
• Surgical intervention – AVOID

804
Copyright © Oakstone Publishing, 2021. All Rights Reserved.

Self-limited condition
• 37 patients over 11 yr period
• Managed primarily with observation, patient education and
reassurance (average time to resolution 7.4 months)
• Note: 22 patients treated initially with antibiotics prior to
diagnosis and presentation to the breast clinic (most had
little improvement)

Boutin MR et al, Am J Surgery, 2015

Self-limited condition
• 9 patients, 8 managed conservatively
• 4 patients (50%) had complete resolution
• Mean interval to resolution 14.5 months (2-24 months)
• No recurrence
• 4 patients (50%) had static disease (mean f/u 11
months)
Lai ECH, Breast J, 2005

805
Copyright © Oakstone Publishing, 2021. All Rights Reserved.

Abscess
• Percutaneously drain infection
• Send for culture and sensitivity
• Antibiotics (optimal length unclear, 5-14d)
• Augmentin
• Doxycycline
• Bactrim

Non-responders
• Once active infections are controlled
• Steroids (prednisone taper)
• Immunosuppressants
• Classically methotrexate
• Newer agents with increased soft tissue/fat penetrance
• Referral to rheumatology

806
Copyright © Oakstone Publishing, 2021. All Rights Reserved.

Thank you
Contact:
[email protected]

807
Copyright © Oakstone Publishing, 2021. All Rights Reserved.

Malignant Breast Disease

Laura S Dominici MD, FACS


Assistant Professor, Harvard Medical School
Associate Surgeon, Dana-Farber/Brigham and Women’s Cancer Center

Disclosures

• Nothing to disclose

808
Copyright © Oakstone Publishing, 2021. All Rights Reserved.

Outline

• DCIS
• Invasive breast cancer
• Systemic therapy considerations
• Breast Surgery
• Lymph node surgery
• Special considerations:
• Elderly
• Pregnant patients

809
Copyright © Oakstone Publishing, 2021. All Rights Reserved.

Ductal Carcinoma in Situ

DCIS

• Most commonly diagnosed cancer since screening mammograms


• Goals of treatment:
• Confirmation of diagnosis
• Prevent progression/recurrence
• Standard remains lumpectomy/radiation or mastectomy
• More interest in genomic testing of DCIS to guide local therapy
decisions
• COMET trial randomizes to observation versus standard therapy

810
Copyright © Oakstone Publishing, 2021. All Rights Reserved.

NSABP B17—radiation and DCIS

Fisher et al, Sem in Onc 2001;28:400-418

Local Recurrence in EORTC 10853

Bijker et al, J Clin Oncol, 2006; 24:3381-3387

811
Copyright © Oakstone Publishing, 2021. All Rights Reserved.

DCIS and tamoxifen

• Risk of any subsequent breast cancer


with ER+ (A) and ER- (B) DCIS with
tamoxifen versus placebo

• 42% reduction in relative risk; P =


.001 in ER+ only

©2012 by American Society of Clinical Oncology

Sentinel nodes in DCIS

Incidence positive nodes in DCIS


Higher in earlier studies (~7%) but likely due to under sampled
invasive disease
N # (% positive)
Van Nuys 0 0 (0%)
Charing Cross 37 0 (0%)
NCDB 10,946 406 (3.6%)

812
Copyright © Oakstone Publishing, 2021. All Rights Reserved.

Nodes in DCIS

• Not a routine component of the evaluation of a patient with


DCIS
• Upstaging at surgery after core biopsy can range from 10-
25%
• Patients undergoing lumpectomy can return to OR for
sentinel node if needed
• NCCN only recommends performance of sentinel node in
patients with DCIS undergoing mastectomy Bruening et al., Ann Intern Med 2010
Moran CJ, Am J Surg 2007

Invasive Breast Cancer

813
Copyright © Oakstone Publishing, 2021. All Rights Reserved.

BIOLOGIC SUBTYPE DEFINITIONS


• Luminal A: ER+ PR+ HER2- low-intermediate grade
• Luminal B: ER+ PR+ HER2- high grade or ER+ PR+ HER2+
• HER2: ER- PR- HER2+
• Triple negative: ER- PR- HER2-

SYSTEMIC THERAPY MATTERS

• Important for local regional


recurrence also
• Improving outcomes likely
reflect better multimodal
therapy
• Optimizing recommendations
and delivery of systemic therapy

Van der Leest, Cancer 2007.

814
Copyright © Oakstone Publishing, 2021. All Rights Reserved.

ONCOTYPE MATTERS
• Prognostic and predictive
• Endocrine therapy versus chemo
+ endocrine therapy
• For ER+ HER2-
• Core or on excision
• Now validated for 1-3 positive
nodes as well as node negative

Hassett JCO 2012; Cheng Front Oncol 2020


Albain Lancet 2010

Intermediate Oncotype Scores


• TAILORx
• Scores 11-25
• No difference for most with
endocrine v
chemo+endocrine
• Potential benefit of chemo in
<50 (caveat—ovarian
suppression)

Sparano NEJM 2018

815
Copyright © Oakstone Publishing, 2021. All Rights Reserved.

Role of Neoadjuvant Systemic Therapy

• Convert inoperable disease into operable


• Optimize surgery for patients with operable disease
• Breast
• Axilla
• Evaluate new therapies in clinical trials
• Prognostic information from pathologic complete response
(pCR)

NSABP B-18—Neoadjuvant therapy


Operable • 79% had clinical response
breast
cancer • pCR 13%
• Equal disease free survival and
overall survival
OPERATION AC x4 • Axillary nodes downstaged
• 58% v 40%
• Lumpectomy rates:
• 68% v 60%
AC x4 OPERATION
• 27% would have needed upfront
mastectomy Fisher JCO 1997

816
Copyright © Oakstone Publishing, 2021. All Rights Reserved.

Does a response to therapy translate into


potential for BCT?

Bansal IJRI 2016

Prognostic information

• Useful to know pCR or lack thereof for:


• Triple negative
• HER2+
• Post-neoadjuvant additional systemic therapy for triple
negative
• Alterations in HER2 directed therapy for HER2+
• No role for additional systemic therapy aside from
endocrine therapy for ER+ HER2-

817
Copyright © Oakstone Publishing, 2021. All Rights Reserved.

SYSTEMIC THERAPY IN HER2+

• 406 patients with tumor <3cm,


N0 or N1mi treated with THx12
weeks, Hx1yr and radiation after
BCS
• Enrolled 9/07-9/10
• 33% were under 50
• Median f/u 6.5y
• 98.6% (CI 97.4-99.8) local
regional recurrence free
Bellon BCRT 2019

ER+ HER2- Systemic therapy


• Oncotype, oncotype, oncotype

• Still neoadjuvant chemotherapy for locally advanced


disease
• Earlier stage but not a lumpectomy candidate
• Oncotype on core: Consider chemo if high
• Consider preoperative endocrine therapy if postmenopausal
• If premenopausal and low/intermediate Oncotype particularly if
node negative: upfront surgery

818
Copyright © Oakstone Publishing, 2021. All Rights Reserved.

Who should be having mastectomy?


• Inflammatory breast cancer or locally advanced breast
cancer
• Patients who cannot receive radiation (pregnancy, prior
treatment, connective tissue disorders)
• Diffuse suspicious calcifications
• Multicentric disease
• Inability to achieve clear margins with cosmetically
acceptable result
• High risk (hereditary, mantle radiation)

Who Should be thinking about CPM?


• Medically indicated:
• Cancer with known BRCA mutation
• Cancer with history mantle radiation

• Reasonable:
• Favorable prognosis cancer with significant anxiety or
symmetry issues
• Significant family history without mutation
• Very dense breasts, multiple biopsies, difficult to monitor

819
Copyright © Oakstone Publishing, 2021. All Rights Reserved.

Mastectomy Rates in US

NIH Consensus statement re:


BCS
Gomez SL, JCO 2010

Rising rates bilateral mastectomy

Nash, JAMA Surgery 2017

820
Copyright © Oakstone Publishing, 2021. All Rights Reserved.

Excellent clinical outcomes with BCT

Jacobson JA, N Engl J Med 1995

Excellent local control with BCT

Fisher B, NEJM 2002.

821
Copyright © Oakstone Publishing, 2021. All Rights Reserved.

Equivalent Oncologic Outcomes

• 49K Swedish
women
• BCT v Mx v Mx-RT
• Superior outcomes
with BCT even
after adjustment
for confounders

Benefits of BCT

• Improved body image


• Less short term morbidity
• Typically outpatient procedure
• Potential to avoid axillary dissection and postmastectomy radiation…
• Equivalent outcomes

Ganz P, JNCI 2004

822
Copyright © Oakstone Publishing, 2021. All Rights Reserved.

Younger patients

• No difference in overall survival


• No significant difference local regional recurrence in
modern studies
• Mastectomy alone associated with inferior result compared
to BCT or mastectomy with radiation
• Breast conservation very reasonable option in these
patients

Beadle BM, Int J Radiat Oncol Biol Phys 2009

Triple negative disease

Adkins, Ann Surg Oncol 2011

823
Copyright © Oakstone Publishing, 2021. All Rights Reserved.

Impact of biologic subtype on recurrence with


breast conservation

Arvold NA, JCO 2011

Tumor biology

• Significant factor in predicting local recurrence


• Interest in using this for prognostic purposes, but
robust data to suggest this should drive surgical
decisions at present
• Surgery choice does not appear to significantly
impact that risk
Nguyen PL, JCO 2008
Kyndi M, JCO 2008
Mamounas T, JCO 2010

824
Copyright © Oakstone Publishing, 2021. All Rights Reserved.

Purpose of Lymph Node Evaluation

• Informative
• Staging
• Treatment recommendations
• Prognostic information
• Local control
• Clinically positive axillary nodes require dissection (possibly
addition of radiation)
• Clinically negative axillary nodes not evaluated pathologically and
untreated by either radiation or dissection have a 20% risk of
local recurrence (NSABP B-04)

Axillary Lymph Node Dissection Morbidity


• Short term
• Pain
• Seroma
• Numbness/paresthesias
• Decreased range of motion
• Long term
• Lymphedema
• Numbness/paresthesias
• Decreased range of motion
• Damage to nerves to pectoralis, lattisimus or serratus anterior
muscles

825
Copyright © Oakstone Publishing, 2021. All Rights Reserved.

SLNB vs. ALND


Outcomes for Node Negative Patients
• Have now shown safety with no significant difference in survival and
local/regional recurrence (B-32)
• SLNB was associated with lower risks of short and longer term
complications
• Sentinel lymph node biopsy alone has become the standard of care for
clinically node negative breast cancer patients
• Nodes identified as positive on IHC only do not appear to carry
significant prognostic information
• SLN positive patients proceeded with axillary lymph node dissection in
these studies

Management of patients with


positive sentinel node
Clinically node negative patients with early stage breast
cancer

826
Copyright © Oakstone Publishing, 2021. All Rights Reserved.

Rationale for change in management


• Increased recognition of importance of tumor biology
• Multi-modality therapy:
• Better imaging
• Improved evaluation techniques in pathology
• Increased use of systemic therapy
• Better radiation planning
• Do all patients with positive sentinel nodes benefit from axillary
lymph node dissection?

Lymph node definitions

• Isolated tumor cells (i+): ITCs, Staged and managed


as negative in upfront surgery, single cell - <0.2mm
• Micrometastasis: 0.2mm-2mm
• Macrometastasis: >2mm

827
Copyright © Oakstone Publishing, 2021. All Rights Reserved.

Micrometastases

IBCSG 23-01 Schema

828
Copyright © Oakstone Publishing, 2021. All Rights Reserved.

IBCSG 23-01
Disease-Free Survival

Galimberti, Lancet Oncology 2013

IBCSG 23-01 Overall Survival

Galimberti, Lancet Oncology 2013

829
Copyright © Oakstone Publishing, 2021. All Rights Reserved.

Patients having Breast Conservation

ACOSOG Z0011
• Clinical T1-2N0M0
• Lumpectomy with whole breast irradiation
• 1-2 H&E, touch prep or frozen section detected positive lymph nodes
• Randomized to ALND vs. no further surgery
• No third field axillary irradiation allowed
• More recent study revealed some received more radiation than per protocol (19% in each arm
with third field)
• Adjuvant systemic therapy by choice
• Endpoints
• Primary: Overall survival
• Secondary: Disease Free Survival
Giuliano et al, JAMA 2011. Jagsi, JCO 2014

830
Copyright © Oakstone Publishing, 2021. All Rights Reserved.

ACOSOG Z0011: results


Median follow-up = 6.3 years
Regional recurrence seen in only 0.7% of the entire study
population

ALND, SLNBx,
Recurrence
N=420 N=436

Local 15 (2.6%) 8 (1.8%)

Regional 2 (0.5%) 4 (0.9%)

Local and Regional 17 (4.1%) 12 (2.8%)


Giuliano, American Journal of Surgery 2010
P = 0.11

Patients having Mastectomy

831
Copyright © Oakstone Publishing, 2021. All Rights Reserved.

EORTC AMAROS (After Mapping of the Axilla:


Radiotherapy or Surgery?) Trial
• Clinical T1-2N0M0
• 95% had 1 or 2 positive sentinel nodes on pathology
• Lumpectomy with whole breast irradiation or mastectomy (no
requirements about chest wall radiation)
• Randomized to ALND vs. Axillary radiation
• Axillary field encompassed Levels I, II, III
• Adjuvant systemic therapy by choice
• Endpoints
• Primary: 5 year axillary recurrence rate
• Secondary: Lymphedema, QOL, OS, DFS
Donker, Lancet 2015

AMAROS trial demographics


ALND AXILLARY RADIATION
n 744 681
Age 56 (48-64) 55 (48-63)
T1 82% 78%
Mastectomy 127 (17%) 121 (18%)
Chemotherapy 61% 61%
Hormonal therapy 79% 77%

832
Copyright © Oakstone Publishing, 2021. All Rights Reserved.

AMAROS - 5-Year axillary recurrence


Median Follow-up – 6.1 years

SLN positive, SLN positive, SLN negative, no


ALND AxRT further therapy

0.54% (4/744) 1.03% (7/681) 0.8% (25/3131)

Donker, Lancet Oncology 2014

AMAROS - Lymphedema

Donker, Lancet Oncology 2014

833
Copyright © Oakstone Publishing, 2021. All Rights Reserved.

Which upfront surgery patients with


positive nodes still need ALND?
• Grossly positive nodes/Biopsy proven positive nodes prior
to surgery
• Prior history radiation (ipsilateral BCT, Hodgkin’s)
• 3 or more positive sentinel nodes
• T3 or T4 lesion
• Unlikely that patient will receive radiation (after BCS or
mastectomy)

Management of patients with


positive sentinel node
Clinical N1 patients with good response to
preoperative systemic therapy

834
Copyright © Oakstone Publishing, 2021. All Rights Reserved.

Rationale for change in management


• 50-60% of patients receiving neoadjuvant chemotherapy
have complete response in nodes
• Important to determine axillary involvement after
chemo, but unlikely that all patients need/benefit from
having ALND
• Axillary recurrence rates are low, even in the face of false
negative sentinel nodes
Boughey, JAMA 2013

Axillary pCR by subtype


• Hormone positive, HER2 negative
• 0-29%

• Triple negative
• 47-73%

• HER2 positive
• 49-82%
Pilewskie Ann Surg Oncol 2017

835
Copyright © Oakstone Publishing, 2021. All Rights Reserved.

ACOSOG Z1071

• Assess feasibility of sentinel


lymph node surgery after
neoadjuvant chemotherapy
in patients with node-
positive breast cancer
• Demographics:
• 756 women
• Predominantly T2-T3
• 88% were clinical N1

ACOSOG Z1071--Findings
• Sentinel node detection rate: 92.7%
• Nodal pCR rate: 41%
• Location of residual nodal disease
• SLN only: 20.6%
• ALND only: 7.4%
• Both SLN and ALND: 31.1%
• Overall false negative rate: 12.6% (prespecified threshold
10%)

836
Copyright © Oakstone Publishing, 2021. All Rights Reserved.

Use of TAD (Tagged Axillary Dissection)

• Technique described to decrease false negative rate


• All nodes sampled before chemotherapy had clip placed
• Patients underwent sentinel node and retrieval of clipped node
followed by ALND
• False negative rate of SLN was 10.1%
• Clipped node was not a sentinel node 23% of the time
• Addition of clipped node to sentinel nodes decreased false negative rate to
2%
• Variations of this done at many institutions, technically challenging
Caudle, JCO 2016.

Negative Nodes after Chemotherapy with Initial


Clinical N1 disease
• Offer sentinel node to patients with clinically
negative axilla
• Stop at sentinel node after chemotherapy for
patients who are node negative if:
• Dual tracer
• Retrieve 3 or more sentinel nodes
• No random sampling
• Including clipped node

837
Copyright © Oakstone Publishing, 2021. All Rights Reserved.

Positive Nodes After Chemotherapy


• Patients with persistent adenopathy should undergo ALND
• Outside of clinical trial, patients with positive nodes should
undergo ALND
• SENTINA suggested this is the case even for ITCs in sentinel nodes
after chemotherapy
• Negative prognostic finding, and risk of local regional
recurrence likely outweighed by risk of distant metastasis
• Alliance clinical trial looks to minimize morbidity and provide
adequate local control
Kuehn, Lancet 2013.

Breast Cancer in the Elderly

838
Copyright © Oakstone Publishing, 2021. All Rights Reserved.

Management of the Breast

• 70+ with ER+ Stage I breast


cancer
• Randomized to Tam vs
Tam+RT
• No difference survival,
mastectomy free survival,
distant recurrence
Hughes JCO 2013

Management of the clinically negative axilla

• Sentinel node biopsy:


• Unlikely that it will change systemic therapy in ER+ HER2-
disease
• Consider for TNBC and HER2+
• Axillary surgery does not significantly impact local
failure in ER+ HER2- with small clinically node
negative cancers
• Martelli: 65yo and older
• -CALGB 9343: 70yo and older Martelli, Ann Surg Onc 2011; Hughes, JCO 2013

839
Copyright © Oakstone Publishing, 2021. All Rights Reserved.

Pregnancy Associated Breast


Cancer

Definition/Incidence

• Pregnancy associated breast cancer or “gestational” breast cancer refers


to breast cancer diagnosed during pregnancy, up to 1 year post-partum or
while lactating
• Different considerations depending on when diagnosed/treated
• Increasing incidence as women having children later
• 5% of breast cancers in women diagnosed under 50
• 1 in 3000 pregnancies….uncommon diagnosis
• Associated with more advanced stage at presentation
• Not having routine screening
• Changes in breast of pregnancy/lactation

840
Copyright © Oakstone Publishing, 2021. All Rights Reserved.

Basics

• Goal is to deliver the same treatment as to a non-pregnant patient


• Very seldom is there a compelling reason why women cannot
continue with pregnancy
• No recommendation for termination of pregnancy
• This used to be recommended routinely
• Many physicians still discuss this with patients as an option
• No survival benefit with termination seen in studies
• Every effort made to ensure delivery full term infant
• Try to avoid early induction to facilitate treatment

Breast Imaging/Biopsy
• Always get a mammogram
• Need to see calcifications
• Shield abdomen/pelvis
• Ultrasound breast and axilla (if clinically abnormal nodes)
• No breast MRI
• No gadolinium while pregnant
• Prone positioning challenging when pregnant
• Core biopsy whenever possible
• Previous case reports of milk fistula not reproducible
• Concerns for excisional biopsy/open procedures when lactating re: milk
fistula

841
Copyright © Oakstone Publishing, 2021. All Rights Reserved.

Systemic therapy
• Safe to give chemotherapy after 1st trimester
• Little data, no RCTs
• Prospective studies suggest no increase in developmental abnormalities or negative impact on
outcome of babies
• Usually given Adriamycin/Cytoxan
• Less safety data for Taxol but being given more frequently
• Anti-HER2 therapy contraindicated due to significant effects on amniotic fluid level
• Still give in adjuvant setting after deliver
• Treatment planning for HER2+ patients can be very complicated and potential for delays
depending on gestational age at diagnosis
• Time chemotherapy to avoid nadir in counts at delivery
• No endocrine therapy during pregnancy
• No breast feeding during chemotherapy or endocrine therapy

Local therapy considerations


• Generally safe to operate at any point
• Positioning to minimize pressure on IVC once mid-second trimester
• Perioperative fetal monitoring once viable fetus with surgery done in hospital with Labor and
Delivery unit
• Controversial to offer reconstruction during pregnancy, but would offer tissue expander/implant
only
• No radiation during pregnancy
• Can often still offer BCT if getting chemotherapy and timing works such that no significant delay
• Milk fistula risk if during lactation
• Also potential concerns for immediate reconstruction while lactating, particularly if implant based
reconstruction
• Some patients can breast feed from contralateral side

842
Copyright © Oakstone Publishing, 2021. All Rights Reserved.

Local therapy considerations—


Lymph Nodes
• As per usual if post-partum
• If pregnant, controversial to offer sentinel node, but
becoming increasingly common
• Limited safety data for mapping agents
• Better data for technetium, so most commonly used
• Methylene blue/isosulfan blue contraindicated as crosses
placenta
• No data to suggest higher false negative rates or higher rates of
failure to map during pregnancy

Thank you!

[email protected]
@dr_laurad

843
Copyright © Oakstone Publishing, 2021. All Rights Reserved.

Paget’s Disease and Male Breast Cancer:


Surgical Techniques

Faina Nakhlis, MD

Assistant Professor of Surgery, Harvard Medical School


Dana-Farber/Brigham and Women’s Cancer Center, Boston, MA

Disclosures

• None

844
Copyright © Oakstone Publishing, 2021. All Rights Reserved.

Paget’s Disease of the Breast

Paget’s Disease of the Breast


• 0.5-5% of all breast cancer presentations
• 1307 – first description of erythema and excoriation of
the nipple by John of Arderne
• 1856 – further observations described by Velpeau
• 1874 – Sir James Paget, a surgeon and pathologist at
St. Bartholomew Hospital in London, published “On
disease of the mammary areola preceding cancer of
the mammary gland”: 15 cases of an “eruption on the
nipple and areola” which had an appearance of a
florid, intensely red, raw surface, very finely granular,
as if nearly the whole thickness of very acute diffuse
eczema”.

Lim et al, 2011; Wong et al, 2015; Graham 1939; Velpeau 1856; Paget 1974

845
Copyright © Oakstone Publishing, 2021. All Rights Reserved.

Paget’s Disease Presentation


• 90% of patients have an underlying carcinoma:
• 50-60% - DCIS
• 30-40% - invasive carcinoma
• About 50% have a palpable mass
• Average age at diagnosis – 63 years
• Patients with Paget’s Disease are more likely to have:
• Hormone receptor-negative disease
• HER2-positive disease
• Grade 3 underlying invasive carcinoma

Trebska-McGowan 2013, Wong et al, 2015; Helme 2015

Paget’s Disease of the Breast

846
Copyright © Oakstone Publishing, 2021. All Rights Reserved.

Paget’s Disease Pathogenesis


More Plausible

Epidermotropic theory Transformation theory

• Adenocarcinomatous cells • Malignant transformation of


migrate along lactiferous ducts to keratinocytes to
reach the nipple epidermis adenocarcinomatous cells within
large caliber lactiferous ducts

Paone et al, 1981; Lagios et al, 1984; Yim et al 1997

Paget’s Disease Epidemiology


SEER Data, 2000-2011

• Median age at diagnosis –


50-60 years (range 24-90
years)
• Rare in male patients
• Incidence has been
decreasing, likely due to
more wide-spread breast
cancer screening and early
detection

Kawase et al, 2005; Gunhan-Bilgen et al, 2006; Lohsiriwat et al, 2012; Meibodi et al, 2008;
Bernardi et al, 2008; Chen et al, 2006; Wong et al, 2015

847
Copyright © Oakstone Publishing, 2021. All Rights Reserved.

Paget’s Disease of the Breast: Differential Diagnosis

• Eczema or contact dermatitis


• A reasonable consideration in the face of up-to-date negative breast imaging
• A 1-2-week trial of a topical anti-eczema medication is appropriate
• If no improvement is seen a full thickness nipple biopsy should be performed
• Duct ectasia
• Basal cell carcinoma
• Florid papillomatosis
• Melanoma
• Nipple adenoma

In the presence of up-to-date negative breast imaging, a nipple biopsy is diagnostic

Trebska-McGowan 2013

Evaluation of Paget’s Disease

Bilateral mammography

Possible US (if mammographic or


palpable mass is present)

Possible breast MRI

Image-guided core biopsy of


underlying imaging abnormality and

Full thickness incisional nipple biopsy

Add references

848
Copyright © Oakstone Publishing, 2021. All Rights Reserved.

Central Lumpectomy for Paget’s Disease

• Appropriate when the underlying malignancy,


if any, is reasonably close to the nipple-areolar
complex to enable a good cosmetic outcome

• Should incorporate the resection of the entire


nipple-areolar complex en bloc with the
underlying malignancy

• Upon 6 or more months following completion


of adjuvant radiation nipple reconstruction can
be pursued

Central Lumpectomy “Tips”

• 3:1 length to width ratio


• If the ellipse of resected skin is tapered on either side, the overall contour of the
breast is less deformed

849
Copyright © Oakstone Publishing, 2021. All Rights Reserved.

Total Mastectomy for Paget’s Disease

• NOT nipple-sparing

• Immediate reconstruction, if desired, can be pursued

Add references

Paget’s Disease Surgical Management Trends


Surveillance, Epidemiology and End Results (SEER) 2000-2011, n=2631 with PD

PD only (n=185) PD+DCIS (n=953) PD+IDC (n=1493)

Lumpectomy 98 (53%) 295 (31%) 166 (11%)

Mastectomy 87 (47%) 658 (69%) 1327 (89%)

Post-lumpectomy
33 (34%) 151 (51%) 89 (52%)
radiation
Sentinel node
38 (21%) 316 (33%) 329 (22%)
biopsy

ALND 9 (5%) 69 (7%) 183 (12%)

Wong et al, 2015

850
Copyright © Oakstone Publishing, 2021. All Rights Reserved.

Paget’s Disease Breast Conservation Trends


Surveillance, Epidemiology and End Results (SEER) 2000-2011, n=2631 with PD
N BCS (%) P value OR (95%CI)
PD +IDC 1493 (57%) 11% 1.00
Histology PD+DCIS 953 (36%) 31% <0.001 2.61 (1.97-3.47)
PD alone 185 (7%) 53% 4.50 (2.80-7.24)
Up to 59 years 1188 (45%) 24.3% 1.43 (0.79-2.60)
Age 60-79 years 1076 (41%) 25% <0.001 1.96 (1.09-3.54)
>80 years 367 (14%) 33% 3.24 (1.74-6.06)
Central/NAC 1235 (47%) 36% 1.00
Tumor location Peripheral 485 (18%) 8% <0.001 0.34 (0.23-0.49)
Overlapping/NOS 911 (35%) 9% 0.26 (0.20-0.35)

Tumor size Up to 2 cm 1185 (45%) 23% 1.00


(when 2.1-5 cm 605 (23%) 10% <0.001 0.53 (0.38-0.73)
reported) >5 cm 195 (7%) 3% 0.14 (0.06-0.34)
Year of 2000-2005 1438 (55%) 20% 1.00
0.02
diagnosis 2006-2011 1193 (45%) 23% 1.36 (1.09-1.70)

Wong et al, 2015

Prognosis for Paget’s Disease


SEER 2000-2014, n=611,703 without PD, n=3,058 with PD

Unadjusted Analysis Matched for Tumor Characteristics and Treatment

Chen et al, 2019

851
Copyright © Oakstone Publishing, 2021. All Rights Reserved.

Summary: Paget’s Disease

• Paget’s Disease of the breast is rare


• Full-thickness nipple biopsy is diagnostic
• Comprehensive up-to-date breast imaging defines the extent of
underlying disease and informs surgical treatment
• Prognosis in patients with Paget’s Disease is comparable to stage-
matched counterparts without it

Male Breast Cancer

852
Copyright © Oakstone Publishing, 2021. All Rights Reserved.

Male Breast Cancer


• Less than 1% of all breast cancer presentations
• Life-time risk of male breast cancer – 0.07%
• Male breast cancer has become somewhat more prevalent with
increased life expectancy
• Most common presentation – a palpable retroareolar mass with or
without nipple tethering or excoriation
• Most common histology – invasive ductal carcinoma, grade 2
(because there are no lobules in male breast parenchyma)
• Most male breast cancers are estrogen receptor (ER) and
progesterone receptor (PR) positive, and HER2 negative

Fox et al, 2021

Risk Factors for Male Breast Cancer


Yes No

Obesity Gynecomastia

Klinefelter’s syndrome

Exogenous hormones (including gender reassignment)

Cryptorchidism

Testicular cancer

Radiation exposure

Fox et al, 2021

853
Copyright © Oakstone Publishing, 2021. All Rights Reserved.

Familial Male Breast Cancer


Observations
• Males with breast cancer had elevated urine
estrogen levels

• A “defect in estrogen production or


metabolism” could be potentially “implicated in
the pathogenesis of male breast neoplasms”

• In family B, both of proband’s sons underwent


bilateral risk-reducing mastectomies (in the
proband’s contralateral breast and in both
sons’ risk-reducing mastectomy specimens,
ductal hyperplasia was identified)

Everson, Li, Fraumeni, et al 1976

Germline Mutations and Male Breast Cancer Risk


Relative Risk of Male Breast Cancer

BRCA2 X 80-100

BRCA1 X 3.2

ATM X 1.4-2.14

PALB2 X 6.6-11.2

CHEK2 X 1.47

RAD51D X 10.18

Gucalp et al, 2019; Rizzolo et al, 2013; Siegel et al, 2015; Siegel et al 2020

854
Copyright © Oakstone Publishing, 2021. All Rights Reserved.

Genetic Testing in Men with Breast Cancer


Mass General/Brigham Hospitals, n=100 with male breast cancer, 2000-2017

Johnson et al, 2021

Prognosis for Male Breast Cancer by Tumor Subtype


SEER, n=2389 with male breast cancer, 2010-2017, median follow-up – 43 months

Leone et al, 2021

855
Copyright © Oakstone Publishing, 2021. All Rights Reserved.

Prognosis for Male Breast Cancer by Stage


SEER, n=2389 with male breast cancer, 2010-2017, median follow-up – 43 months

Leone et al, 2021

Delay in Diagnosis and Prognosis for Male Breast Cancer


Mass General/Brigham Hospitals, n=100 with male breast cancer, 2000-2017

Johnson et al, 2021

856
Copyright © Oakstone Publishing, 2021. All Rights Reserved.

Incidence of Early-Stage Male vs Female Breast Cancer


SEER 2000-2015, Male BC n=2,254, Female BC n=390,539
Male Breast Cancer Female Breast Cancer

APC – annual percentage change

Wang et al, 2019

Early-Stage Male vs Female Breast Cancer


SEER 2000-2015, Male BC n=2,254, Female BC n=390,539
Unadjusted Analysis Propensity Score Matched

Wang et al, 2019

857
Copyright © Oakstone Publishing, 2021. All Rights Reserved.

Propensity-Score Matched Male vs Female Breast Cancer


SEER 2000-2015, Male BC n=2,254, Female BC n=9,016
Stage I Stage II

Wang et al, 2019

Propensity-Score Matched Male vs Female Breast Cancer


SEER 2000-2015, Male BC n=2,254, Female BC n=9,016
Hormone-Receptor Positive Hormone-Receptor Negative

Wang et al, 2019

858
Copyright © Oakstone Publishing, 2021. All Rights Reserved.

Matched Cases of Male vs Female Breast Cancer


Ohio State Experience, Stages I-III, 1994-2014, Male BC n=45, Female BC n=75
Males (n=45) Females (n=75)
Median age at diagnosis (range) 64 years (43-79) 63 years (42-79)
BMI at diagnosis 31 (20-46) 30 (16-48)
White race 35 (78%) 64 (85%)
Black race 7 (16%) 9 (12%)
Stage I 12 (27%) 21 (28%)
Stage II 24 (53%) 37 (49%)
Stage III 9 (20%) 17 (23%)
Grade 1 6 (13%) 23 (31%)
Grade 2 29 (64%) 35 (47%)
Grade 3 10 (22%) 14 (19%)
Lymph node positive 21 (47%) 43 (67%)
ER positive 44 (98%) 73 (97%)
PR positive 38 (84%) 64 (85%)
HER2 positive 4 (9%) 5 (7%)

Liu et al, 2020

Distant Disease and Survival, Male vs Female Breast Cancer


Ohio State Experience, Stages I-III, 1994-2014, Male BC n=45, Female BC n=75

Males (n=45) Females (n=75) P


Disease progression 9 (20%) 18 (24%) 0.318
Median time to progression NA 11 years
Death 8 (18%) 16 (22%) 0.287
Median time to death NA 13 years

Males (n=45) Females (n=75)


Bone 3 (7%) 14 (19%)
Brain 0 3 (4%)
Liver 2 (4%) 8 (11%)
Lung 3 (7%) 3 (4%)
Other 0 3 (4%)

Liu et al, 2020

859
Copyright © Oakstone Publishing, 2021. All Rights Reserved.

Treatment: Male vs Female Breast Cancer


Ohio State Experience, Stages I-III, 1994-2014, Male BC n=45, Female BC n=75
Males (n=45) Females (n=75) P
Mastectomy 44 (98%) 45 (60%)
Lumpectomy 0 27 (36%) <0.0001
None 1 (2%) 3 (4%)
Radiotherapy 11 (24%) 47 (63%) <0.0001
Postmastectomy radiation 10 (23%) 20 (44%) 0.0302
Chemotherapy 19 (42%) 49 (65%) 0.0134
Anthracyclines 15 (33%) 35 (47%) 0.1515
Tamoxifen 38 (84%) 19 (25%) <0.0001
Aromatase inhibitors 10 (22%) 64 (85%) <0.0001
SERM 1 (2%) 4 (5.3) 0.6492
GnRH agonist 4 (9%) 3 (4%) 0.4228
Median duration of hormonal therapy (range) 60 months (8-186) 60 months (0.4-122) 0.8141

Liu et al, 2020

Local Therapy for Male Breast Cancer


Metaanalysis, n=12,235
5-year OS BCT Mastectomy OR (95% CI)
Bateni 2019, n=6711 619/1511 (41%) 2101/5200 (40%) 1.02 (0.91-1.15)
Cloyd 2013, n=5425 474/718 (66%) 3299/4707 (70%) 0.83 (0.70-0.98)
Fogh 2013, n=42 7/8 (88%) 34/34 (100%) 0.07 (0.00-1.96
Hulborn 1987, n=57 11/13 (85%) 30/44 (68%) 2.57 (0.50-13.16)
Total 5-year OS 1111/2250 (49%) 5464/9985 (55%) 0.93 (0.73-1.19)

10-year OS BCT Mastectomy OR (95% CI)


Bateni 2019, n=6711 93/1511 (6%) 284/5200 (5%) 1.14 (0.89-1.45)
Cloyd 2013, n=5425 337/718 (47%) 2184/4707 (46%) 1.02 (0.87-1.20)
Fogh 2013, n=42 6/8 (75%) 23/34 (68%) 1.43 (0.25-8.29)
Hulborn 1987, n=57 7/13 (54%) 16/44 (36%) 2.04 (0.58-7.14)
Total 10-year OS 443/2250 (20%) 2507/9985 (25%) 1.06 (0.93-1.21)

Lin et al, 2021

860
Copyright © Oakstone Publishing, 2021. All Rights Reserved.

Male Breast Cancer Advocacy

Characterization of Male Breast Cancer


EORTC 10085/TBCRC/BIG/NABCG International Male Breast Cancer Program, n=1483, 1990-2010

Cardoso et al, 2018

861
Copyright © Oakstone Publishing, 2021. All Rights Reserved.

Summary: Male Breast Cancer


• Male breast cancer is a rare disease with little high-quality
data providing insights into its biology
• From a surgical standpoint, when appropriate, male breast
cancer can be safely treated with breast-conserving therapy
• Further research into systemic therapy for male breast
cancer is needed
Thank you

862
Copyright © Oakstone Publishing, 2021. All Rights Reserved.

General Abdominal Surgery

Luise Pernar, MD, MHPE

• I have nothing to disclose

863
Copyright © Oakstone Publishing, 2021. All Rights Reserved.

General Abdominal Surgery


• Broad Topic

• Goal - Focus on a few high-yield topics


• Acute abdominal pain
• Laparoscopic exploration/surgery
• Peritoneal dialysis
• Spontaneous bacterial peritonitis
• Rectus sheath hematoma

Acute Abdominal Pain

864
Copyright © Oakstone Publishing, 2021. All Rights Reserved.

Abdominal pain of sudden onset (<1week)

• History • Physical Examination


• Timing • Vital signs
• Location • Appearance
• Severity • Scars from prior surgery
• Radiation • Discoloration of abdominal
• Aggravating/relieving factors wall/back
• Associated signs/symptoms • Tenderness to palpation
• Prior surgery (type, timing) • Rebound/tap/shake tenderness
• Masses

Anatomy

Image from Netter, F. Atlas of Human Anatomy, 6th Edition. Philadelphia: Elsevier, 2014.

865
Copyright © Oakstone Publishing, 2021. All Rights Reserved.

Organs
1 2 3
Liver Liver Stomach
Gallbladder Stomach Pancreas
Biliary ducts Pancreas Spleen 1 2 3
4 5 6
Colon Colon Colon
Small intestine
Kidney
Small intestine
Aorta
Small intestine
Kidney
4 5 6

7 8 9
Appendix
Colon
Bladder
Uterus
Colon
Ovary
7 8 9
Ovary Rectum
Prostate

Image from Netter, F. Atlas of Human Anatomy, 6th Edition. Philadelphia: Elsevier, 2014.

Cause of Abdominal Pain


1 2 3
Hepatitis Gastritis Pancreatitis
Hepatic Abscess GERD Splenic infarct
Cholecystitis PUD/Gastric Splenic rupture 1 2 3
Cholangitis perforation Splenic abscess
Cholelithiasis Pancreatitis
4 5 6
Colitis SBO Colitis
Pyelonephritis
Kidney stone
Mesenteric
ischemia
Pyelonephritis
Kidney stone
4 5 6
Gastroenteritis
Ruptured AAA
Early appendicitis
7 8 9 7 8 9
Appendicitis Cystitis Colitis
Colitis PID/ STI Diverticulitis
Ovarian torsion Ectopic Ovarian torsion
Ovarian abscess Proctitis Ovarian abscess
Ectopic Prostatitis Ectopic
Image from Netter, F. Atlas of Human Anatomy, 6th Edition. Philadelphia: Elsevier, 2014.

866
Copyright © Oakstone Publishing, 2021. All Rights Reserved.

Investigation
• Laboratory • Imaging
CMP US
Evidence of dehydration? Biliary pathology
Acidosis? Portal mesenteric flow
CBC Plain radiograph
Leukocytosis? Free air
Hemoconcentration? Obstructive bowel-gas pattern
Evidence of bleeding? CT abd/pel
Amylase/Lipase Evaluate organs/ obstruction/ free air and fluid
Pancreatitis MRI
Pregnancy
Biliary pathology

Management
• Tailor to diagnosis
• Medical management (eg gastroenteritis)
• Interventional radiology (e.g hepatic abscess)
• Immediate surgery (e.g acute appendicitis)
• Stabilization followed by surgery (e.g. acute ascending cholangitis)
• Elective intervention (e.g. recurrent diverticulitis managed without surgery)

867
Copyright © Oakstone Publishing, 2021. All Rights Reserved.

Laparoscopic Exploration/Surgery

TOPICS
• The laparoscopic tower
• Ergonomics
• Proper room set up
• Equipment set up and trouble shooting
• Access techniques
• Port placement
• Planning
• Physiology
• Some words on the robot

868
Copyright © Oakstone Publishing, 2021. All Rights Reserved.

The Tower

The Tower - Monitor

869
Copyright © Oakstone Publishing, 2021. All Rights Reserved.

The Tower – Insufflator

The Tower – Light source

870
Copyright © Oakstone Publishing, 2021. All Rights Reserved.

Ergonomics
• Head straight
• Gaze down
• Shoulders relaxed
• Arms close to body
• Elbows bent with forearms slightly down
• Hands neutral
• Instruments at tips

Room set up
• Consider
• The target organ
• Where the surgeon will stand
• Where assistant will stand
• Triangulation

871
Copyright © Oakstone Publishing, 2021. All Rights Reserved.

Room Set Up
• Lap chole

Room Set Up
• Lap appy

872
Copyright © Oakstone Publishing, 2021. All Rights Reserved.

Room Set Up
• Lap ventral hernia

Room Set Up
• Lap inguinal hernia

873
Copyright © Oakstone Publishing, 2021. All Rights Reserved.

Room Set Up
• Lap foregut (sleeve, bypass, PEHR, Nissen)

Set up
• What do you need?
• Gas tank
• Light cable
• Insufflator tubing
• Camera cord and head
• Laparoscope(s)
• Ports
• ? Suction
• ? Energy devise (power cord vs. Ligasure vs. Harmonic
…)
• ? Staplers, clips, meshes, loops, suture …

874
Copyright © Oakstone Publishing, 2021. All Rights Reserved.

Camera head
Focus Ring 1

Program Buttons 2
2 5
4
Video Sensor Housing 3 3 1

Zoom 4

Coupler 5

Laparoscopes

875
Copyright © Oakstone Publishing, 2021. All Rights Reserved.

Ports

Trouble Shooting
Possible Cause
Problem Solutions
• Blank Screen • Poor connections
• Reconnect/get new
• Smeared picture • Lens, scope
• Clean/defog/warm
• Loss of working space • Leak
• Fix leak
• RP injury
• Inspect the RP and fix if found

876
Copyright © Oakstone Publishing, 2021. All Rights Reserved.

Trouble shooting
Possible Cause
Problem Solutions
• Low pressure, no/low flow • Tank empty
• New tank
• High pressure, no/low flow • Port occluded
• Open port
• Patient not relaxed
• Tell anesthesia
• Low pressure, high flow • Leak
• Check ports, instruments, suction,
insufflation of viscous
• High pressure, high flow • ??? WW

Access – Veress vs Hasson Technique

Access injury 5-30/10K


procedures
Majority (>75% bowel or
vascular injuries)

No clear advantage of
Veress over Hasson

877
Copyright © Oakstone Publishing, 2021. All Rights Reserved.

Access
• Complications in up to 1% of attempted access
• Extraperitoneal placement
• Vascular injury
• Organ injury
• Gas embolism

• Tricks for the Veress:


• “Double click test”
• “Waggle test”
• “Hanging/saline drop test”

Port placement

All secondary ports should be


placed under direct visualization

878
Copyright © Oakstone Publishing, 2021. All Rights Reserved.

Port placement

Port placement
• Appendectomy
• Cholecystectomy
• Ventral hernia
• Foregut
• Sigmoidectomy

• Total extraperitoneal inguinal hernia repair


• Transabdominal inguinal hernia repair

879
Copyright © Oakstone Publishing, 2021. All Rights Reserved.

Planning
• Prior surgery?
• Port site selection
• Access
• Body habitus?
• Port/ instrument selection
• Access

Planning
• Contraindications?
• Relative: peritonitis; large mass; severe cardio-
pulmonary disease; visceral arterial aneurysm
• Absolute: patient inability to tolerate laparotomy;
hypovolemic shock; lack of training; lack of institutional
support
• Other
• Patient position (supine, sloppy or true decubitus)
• Bed manipulation (Trendelenburg; break for flank
procedures)
• ETT
• Neuromuscular blockade

880
Copyright © Oakstone Publishing, 2021. All Rights Reserved.

Physiology
• CO2
• Metabolic effects
• Increased arterial CO2 and end tidal CO2
• Decreased serum pH
• Physiologic effects
• Increased minute ventilation to blow off CO2
• Can cause bradycardia
• Hypothermia

Physiology
• CO2
• Mechanical effects
• Impaired respiratory mechanics by elevating diaphragm
• Reduced venous return
• Decreased renal blood flow 2/2 increased intraabdominal
pressure

881
Copyright © Oakstone Publishing, 2021. All Rights Reserved.

CO2 WTT-WTD
• Anesthesia reports acute hypotension, tachycardia; they auscultate a
mill-wheel murmur
• GAS EMBOLISM!!!
• Evacuate pneumoperitoneum
• Left side down, Trendelenburg
• Fluids
• Ventilatory support
• CVL placement to aspirate gas embolus

Physiology
• Cardiovascular effects
• Increase preload
• Increase afterload
• Decreased cardiac output
• Decreased venous return
• Arrhythmias

882
Copyright © Oakstone Publishing, 2021. All Rights Reserved.

Cardiovascular WTT-WTD
• Bradycardia
• Desufflate
• Atropine
• Restart insufflation low pressure
• Hypotension
• Desufflate
• Check equipment and patient relaxation
• Check volume status
• Check for bleeding

Physiology
• Respiratory
• Increased minute ventilation
• Reduced functional residual capacity
• Increased air way pressures

• NB if significant CO2 infiltration in the subcutaneous tissue, watch for


respiratory effects even after surgery

883
Copyright © Oakstone Publishing, 2021. All Rights Reserved.

The robot

The robot Robot-assisted laparoscopic


surgery
Advantages Disadvantages
• Optics • Cost
• 3D • Time
• Degrees of motion • Train
• Credential
• Assist yourself • Operative time
• Less fatigue/strain • Benefit?
• Allows bad surgeons to do
complex surgery

884
Copyright © Oakstone Publishing, 2021. All Rights Reserved.

Peritoneal Dialysis

Peritoneal Dialysis Catheter Insertion


• Indications
• Contraindications
• Preparing the patient
• Complications

Picture credits
to John
Crabtree/ PD
University

885
Copyright © Oakstone Publishing, 2021. All Rights Reserved.

Peritoneal Dialysis Catheter


• Indications
• ESRD
The five stages of CKD are:
•Stage 1: Kidney damage with normal kidney function (estimated GFR ≥90 mL/min
per 1.73 m2) and persistent (≥3 months) proteinuria
•Stage 2: Kidney damage with mild loss of kidney function (estimated GFR 60-89
mL/min per 1.73 m2) and persistent (≥3 months) proteinuria
•Stage 3: Mild-to-severe loss of kidney function (estimated GFR 30-59 mL/min per
1.73 m2)
•Stage 4: Severe loss of kidney function (estimated GFR 15-29 mL/min per 1.73 m2)
•Stage 5: Kidney failure requiring dialysis or transplant for survival. Also known as
ESRD (estimated GFR <15 mL/min per 1.73 m2)

Peritoneal Dialysis Catheter


• Indications
• Stage 4: Severe loss of kidney function (estimated GFR
15-29 mL/min per 1.73 m2)
• Place buried or exteriorized catheter
• Stage 5: Kidney failure requiring dialysis or transplant
for survival. Also known as ESRD (estimated GFR <15
mL/min per 1.73 m2)
• Exteriorize buried catheter
• Place exteriorized catheter
• Transition from HD to PD

886
Copyright © Oakstone Publishing, 2021. All Rights Reserved.

Peritoneal Dialysis Catheter


• Contraindications
• Abdominal fibrosis
• Uncorrectable abdominal wall defects
• Inflammatory bowel disease
• (Recurrent) diverticulitis
• Malignancy/carcinomatosis
• Alcohol/substance abuse

• Relative contraindications
• Lack of space or ability to handle dialysis equipment

Peritoneal Dialysis Catheter Insertion


• Preparing the patient
• Informed consent
• Failure of catheter to function primarily vs delayed failure
• Delayed infection
• Need to remove/revise catheter
• Adjunct procedures
• Hernia repair
• Judgement call – weigh size, urgency of PD initiation, patient status, location (groin vs
ventral)
• Omentopexy

887
Copyright © Oakstone Publishing, 2021. All Rights Reserved.

Pre-operative marking/ catheter selection

Picture credits
to John
Crabtree/ PD
University

Catheter design

External Segment

External Cuff
Tunneled Segment

Internal Cuff

Intra-peritoneal Segment

Picture credits
to John
Crabtree/ PD
Single cuff catheters are available; two-cuff design provides superior
University
anchorage

888
Copyright © Oakstone Publishing, 2021. All Rights Reserved.

Peritoneal Dialysis Catheter Insertion


• OR strategy
• Minimize incision sites
• Retro-rectus tunnel

Picture credits
to John
Crabtree/ PD
University

Peritoneal Dialysis Catheter


• Complications - Early
• Bleeding from exit site – NOT AN EXIT SITE
PROBLEM!
• Bleeding from insertion site or tunnel
• Direct pressure
• Inject epinephrine
• Leak from exit site – NOT AN EXIT SITE
PROBLEM!
• Peritoneum not sealed yet
• Keep abdomen dry
• Prophylactic antibiotics
“Stitches/sutures/staples at the exit-site should be
assiduously avoided as they prevent complete healing of the exit-site and
increase risk for infections” (Crabtree)

889
Copyright © Oakstone Publishing, 2021. All Rights Reserved.

Peritoneal Dialysis Catheter


• Complications - Salvage
• Cuff extrusion - Cuff shaving
• Cuff infection - Tract unroofing; cuff shaving
• Poor exit site selection - Splice catheter and move exit site
• Peritonitis - Antibiotics; if fungal infection or no improvement in 48-
96hrs must remove catheter
• Acute abdominal emergencies (delayed diagnosis is common; mortality rate 30-
50%) - Treat like you would treat normally; if gross
contamination, should remove catheter

Peritoneal Dialysis Catheter


• Complications - Salvage
• Mechanical failure - KUB to check for constipation, tube
position; may need to reposition catheter or lyse adhesions
• Scrotal edema/ abdominal wall leak – Peritoneography to identify hernia; repair
as per your routine
• Encapsulating peritoneal sclerosis – Remove catheter; tamoxifen/steroids/
immune therapy; surgery only for severe obstruction

890
Copyright © Oakstone Publishing, 2021. All Rights Reserved.

Spontaneous Bacterial Peritonitis

Bacterial Peritonitis - Spontaneous


• Peritoneal infection developing without overt source; i.e.
no surgically treatable intra-abdominal infection is present
• Distinction critical: 80% mortality if unnecessary exploration in
SBP; 100% mortality if surgical cause is missed in secondary
bacterial peritonitis
• Common in patients with ascites (30% of cirrhotic patients with
ascites develop SBP)

891
Copyright © Oakstone Publishing, 2021. All Rights Reserved.

Bacterial Peritonitis - Spontaneous


• If patient with ascites has abdominal pain, fever,
leukocytosis
• Perform paracentesis to evaluate ascites fluid
• PMN count of >250cells/mm3 is diagnostic for SBP
• Growth of organisms helps tailor antibiotics
• If SBP suspected, start empiric antibiotics immediately AFTER
paracentesis (single dose of antibiotics can lead to no growth of
organisms in ~85% of patients)

Bacterial Peritonitis - Spontaneous


• Appropriate antibiotics
• First line: Cefotaxime
• Second line: Ceftriaxone or Cefepime (other alternatives also possible)

• NB: If a cirrhotic patient develops UGI bleeding, should also start


cefriaxone therapy (x7days) to prevent SBP
• Outcomes overall better with antibiotics during UGI bleeding in this context

892
Copyright © Oakstone Publishing, 2021. All Rights Reserved.

Rectus Sheath Hematoma

Rectus Sheath Hematoma


• Bleeding in to the rectus sheath due to strain or external trauma
• injury of the superior (above arcuate line) or
• inferior (below arcuate line) epigastric artery or perforating branches
• Bleeding may be spontaneous in patients on anticoagulation
(supertherapeutic)

893
Copyright © Oakstone Publishing, 2021. All Rights Reserved.

Rectus Sheath Hematoma


• Work up includes
• CBC
• Coags
• Imaging (US can identify but CT scan is superior)
• Physical exam
• Tender abdominal mass that
• does not cross midline and
• remains palpable when rectus is contracted

894
Copyright © Oakstone Publishing, 2021. All Rights Reserved.

Rectus Sheath Hematoma


• Treatment
• First line with cessation of anticoagulation and transfusion if necessary
• Arterial embolization can be considered if blush on CT
• Operation is very rarely needed and is reserved for intractable pain due to
space-occupying effect of hematoma, or bleeding not thought amendable to
embolization

895
Copyright © Oakstone Publishing, 2021. All Rights Reserved.

Abdominal Hernias
Incidence, Cause, Treatment

David C. Brooks, MD
Brigham & Women’s Hospital
Harvard Medical School

Disclosures

▪ None

896
Copyright © Oakstone Publishing, 2021. All Rights Reserved.

Abdominal Wall Hernias

Abdominal Wall Hernias

▪ Inguinal Hernias
▪ Femoral Hernias
▪ Ventral Hernias
▪ Primary hernias
▪ Incisional hernias

897
Copyright © Oakstone Publishing, 2021. All Rights Reserved.

Groin Hernias
▪ Inguinal Hernias
▪ Indirect through the internal ring - congenital
▪ Direct through the floor of the inguinal canal -
acquired

▪ Femoral Hernias
▪ Only 3%
▪ Female majority

Caption

Inguinal Hernia - Incidence

▪ 20,000,000 worldwide
▪ 700,000 in the US
▪ Lifetime risk: Males: 27% Females: 3%

898
Copyright © Oakstone Publishing, 2021. All Rights Reserved.

Inguinal Hernia - Indications

▪ Groin discomfort on exertion


▪ Inability to perform activities
▪ Inability to reduce
▪ Contraindications
▪ Age
▪ Co-morbid problems

Inguinal Hernia - Asymptomatic

▪ Historically repair when found but no advantage


▪ Watchful waiting with advice
▪ At 7-10 years, 70% will undergo surgery
▪ No evidence of increased morbidity with waiting
▪ Other reasons to avoid surgery
▪ Unable to tolerate GA
▪ Major comorbidities
▪ Pregnancy – do they require surgery
Fitzgibbons RJ Jr, Giobbie-Hurder A, Gibbs JO, et al. JAMA 2006; 295:285.

899
Copyright © Oakstone Publishing, 2021. All Rights Reserved.

Inguinal Hernia - Cause

▪ Indirect almost certainly congenital


▪ May not appear until later
▪ At least 2/3 of all groin hernias
▪ Direct
▪ Usually adult male
▪ Straining or heavy lifting
▪ Co-morbid issues

Inguinal Hernia - Treatment

▪ Non-surgical -Truss
▪ Efficacy unclear
▪ Inappropriate
placement may
injure hernia
contents

900
Copyright © Oakstone Publishing, 2021. All Rights Reserved.

Inguinal Hernia - Treatment

Surgical Options
▪ Open Surgery
▪ Bassini, Shouldice, Lichtenstein,
Plug & Patch, Kugel
▪ Laparoscopic (TAPP or TEP)
▪ Robotic (TAPP)

Inguinal Hernia - Treatment

▪ For unilateral hernia, there is no


consensus.
▪ For bilateral symptomatic hernias,
laparoscopic approach best
▪ Follow the advice of the surgeon
based on his comfort level

901
Copyright © Oakstone Publishing, 2021. All Rights Reserved.

Inguinal Hernia - Treatment

▪ Largest trial (VA) laparoscopic had less


pain, earlier return to work but more
complications
▪ Multiple other trials suggest lap = less
pain but at the risk of higher recurrence
rates

-Neumayer L, Giobbie-Hurder A, Jonasson O, et al. N Engl J Med 2004; 350:1819.


-Westin L, Wollert S, Ljungdahl M, et al. Ann Surg 2016; 263:240.
-Lundström KJ, Holmberg H, Montgomery A, Nordin P. Br J Surg 2018; 105:106.

Open vs Laparoscopic Inguinal Hernia


repair (NEJM)

▪ 1983/2164 randomized patients had surgery


▪ 9.8% of LH converted to open
▪ 87 of 862 (10.1%) LH recurred
▪ 41 of 834 (4.9%) OH recurred
▪ Morbidity: LH 39% vs OH 33.4%
▪ 30-day mortality: LH 2 patients, OH none
▪ LH had significantly higher rate of intra-op,
post-op and life threatening complications

902
Copyright © Oakstone Publishing, 2021. All Rights Reserved.

Open vs Laparoscopic Inguinal Hernia


repair (NEJM)

▪ LH had less pain on day of surgery & 2 wks


▪ at rest
▪ at work
▪ at exercise
▪ with ADL
▪ LH return to ADL 1 day earlier (4 vs 5)
▪ Resumption of sexual activity similar (14d)

Open vs Laparoscopic Inguinal Hernia


repair (NEJM)

▪ Recurrence more common after


laparoscopic repair
▪ Rates of recurrence after recurrent
hernia repair similar
▪ When surgeon’s laparoscopic
experience exceeded 250 cases,
recurrence rates similar

903
Copyright © Oakstone Publishing, 2021. All Rights Reserved.

Inguinal Hernia - Treatment

Robotic inguinal hernia repair


▪ Most North American lap repairs are TEP
▪ Robotic repairs require TAPP
▪ Recent comparison of lap vs robotic
▪ Robotic = no improvement, more $, longer OR
time and more surgeon frustration

Prabhu AS, Carbonell A, Hope W, et al. JAMA surgery. 2020;155(5):380-7

Inguinal Hernia -Treatment

▪ Additional robotic study compares


robotic vs lap vs open repair
▪ OR times similar except bilateral
longer for open
▪ Return to work favors robotic and lap
over open
LeBlanc K, Dickens E, Gonzalez A, et al.Hernia. 2020 Oct;24:1069-81

904
Copyright © Oakstone Publishing, 2021. All Rights Reserved.

Open Tension-Free Hernia Repair

Technique
▪ Monitored local anesthesia
▪ Standard groin approach
▪ Herniectomy or simple reduction
with patch & plug
▪ Sutured or unsutured mesh onlay

Gilbert-sutureless repair of inguinal hernia

905
Copyright © Oakstone Publishing, 2021. All Rights Reserved.

Gilbert-sutureless repair of inguinal hernia

Disadvantages of laparoscopic hernia


repair

▪ Cannot be done under local anesthesia


▪ More expensive
▪ Technique still evolving
▪ Finite conversion rate
▪ Not appropriate for all cases
▪ prior LH,
▪ massive scrotal hernia,
▪ prior midline incisions, prostatectomy

906
Copyright © Oakstone Publishing, 2021. All Rights Reserved.

Inguinal Hernia - recurrence rate

Historical recurrence rates as high than 25%


Causes of high recurrence rates:
▪ Inadequate understanding of anatomy
▪ Poor tissue
▪ Excessive tension
Recurrence as a function of type of repair
▪ Tension vs Tension-free
At present, this rate should be under 2%

Inguinal Hernia - Outcomes

▪ Acquisition of expertise
▪ 40 cases for open repair
▪ 250 laparoscopic repair
▪ Mortality
▪ 0.1% for elective repair
▪ 2.8%-3.1% for emergent
Merola G, Cavallaro G, Iorio O, et al. Hernia. 2020 Jun;24(3):651-9

907
Copyright © Oakstone Publishing, 2021. All Rights Reserved.

Pain and Functional Impairment


1 year after Inguinal Herniorrhaphy
Questionnaires returned 1166/1443 (81%)
Pain in groin area 28.7%
Interfered with work or leisure 11%
Constant pain 4%
Moderate to severe pain (rest) 3%
Moderate to severe pain (activity) 8%
Pain impairing specific daily activities 16.6%
Bay-Nielsen, Perkins, & Kehlet, Ann Surg, 2001

Neuroanatomy of the groin

908
Copyright © Oakstone Publishing, 2021. All Rights Reserved.

21st Century Hernia Recurrence

▪ 85% of all procedures are for primary hernias


▪ 15% are for recurrent hernias
▪ Risk of recurrence @24 months
▪ 1.7% for primary repairs
▪ 4.6% for recurrent repairs

Haapaniemi, et al. Ann Surg 2001;234:122-126

Inguinal Hernias: Treating the Other Guy’s Complications

There are 3 possible ways to repair a


recurrent hernia

▪ Open anterior re-repair


▪ Open pre-peritoneal re-repair
▪ Laparoscopic re-repair
▪ TEP, TAPP, ?IPOM

Inguinal Hernias: Treating the Other Guy’s Complications

909
Copyright © Oakstone Publishing, 2021. All Rights Reserved.

Risk Factors for Recurrence - Volume

▪ 86,000 Scandanavian hernia repairs


▪ Surgeons who performed < 6 hernias per
year had a recurrence rate 20% higher
▪ Fortunately, although this is a large group of
surgeons, they perform ~8% of all hernias
▪ Recurrence rates independent of volume if
more than 5 hernias/year

Norden P, van der Linden W. BMJ, 2008;336:934-937

Inguinal Hernias: Treating the Other Guy’s Complications

Risk Factors for Early Recurrence

▪ Observational Cohort Study


▪ 75 patients with at least 2 recurrences
▪ 44% had a family history, both primary &
recurrent hernias occurred earlier in these
▪ The older the patient, the earlier the
recurrence
▪ Smoking a risk factor for onset but not
recurrence
Jansen PL et al BMC Surgery, 2009;9:1471

Inguinal Hernias: Treating the Other Guy’s Complications

910
Copyright © Oakstone Publishing, 2021. All Rights Reserved.

Mortality after Groin Hernia

▪ Mortality rate not raised above controls for elective


procedures
▪ Mortality increased 7-fold for emergency and 20-fold if
bowel resection undertaken
▪ Emergent hernias
▪ 5.1% inguinal
▪ 36.5% femoral
▪ Standardized mortality ratio
▪ 1.4 for men versus 4.2 for woman
▪ Reflects incidence of emergent in women

Nilsson et al. Ann Surg 2007;245:656-660

Inguinal Hernias: Treating the Other Guy’s Complications

Re-Recurrence after Hernia Repair

▪ Danish Hernia Database 1998 – 2005


▪ Re-operation rate = proxy for recurrence
▪ Of 67,306 primary hernias:
▪ 227 reoperations (3.1%)
▪ 187 re-reoperations (8.8%)
▪ Re-op rate after primary Lichtenstein
significantly reduced if lap reop for
recurrence
Inguinal Hernias: Treating the Other Guy’s Complications

911
Copyright © Oakstone Publishing, 2021. All Rights Reserved.

Reoperation for Recurrent Hernia

▪ Reoperation rate after primary Lichtenstein


▪ Re-recurrence:
▪ 1.3% after laparoscopic recurrence repair
▪ 11.3% after open recurrence repair
▪ Non-mesh = 19.2% / mesh = 7.2%
▪ After primary non-mesh, non-Lichtenstein &
laparoscopic repairs no difference in re-reop
rates
Bisgaard, et al. Ann Surg 2008;247:707-711

Inguinal Hernias: Treating the Other Guy’s Complications

Open vs Laparoscopic Repair of Recurrent


Inguinal Hernia

▪ 52 patients with recurrent hernia after


non-mesh, anterior approach
▪ Randomized to Lichtenstein vs TAPP
▪ Zero recurrences both groups
▪ Lichtenstein: shorter (5 mins); fewer
complications
▪ TAPP: <pain (1,2 &3 yrs), <sick-leave
Demetrashavili Int Surg, 2011;96:233-238

Inguinal Hernias: Treating the Other Guy’s Complications

912
Copyright © Oakstone Publishing, 2021. All Rights Reserved.

Recurrent Groin Hernia Surgery

▪ 19,582 recurrences in Swedish Hernia Registry


▪ Hazard ratios (laparoscopic as reference):
▪ 2.55 for suture; 1.53 for Lichtenstein; 2.31
for plug; 1.36 for pre-peritoneal
▪ Laparoscopic & Pre-peritoneal best for prior
open repair, but no technique superior for
prior pre-peritoneal repair

Sevonius, et al. BJS, 201198:1489-1494

Inguinal Hernias: Treating the Other Guy’s Complications

Inguinal Hernia – Open vs Lap

▪ Most important variable is the surgeon


▪ Choice of operation is best left to the
surgeon based on his level of confidence in
his technique.
▪ If the surgeon is equally comfortable with
both, choice depends on hernia and patient
characteristics

913
Copyright © Oakstone Publishing, 2021. All Rights Reserved.

Femoral Hernias

Demographics Risk Factors


• Femoral hernia
represents between 2% • Childbirth
and 8% of all groin • Constipation
hernias
• Female:Male ratio is 5:1 • Heavy lifting
• Incidence increases with • Obesity
age
• Peak age is 50 years
• Chronic coughing

Femoral Hernia – Natural History

▪ Significant risk of strangulation


▪ 22% at 3 months
▪ 45% at 21 month
▪ Compare with inguinal hernia
▪ 2.8% at 3 months
▪ 4.5% at 21 months

914
Copyright © Oakstone Publishing, 2021. All Rights Reserved.

Femoral Hernia - Treatment

▪ Repair hernia when identified


▪ Laparoscopic vs Open
▪ Laparoscopic approach can identify
other hernias
▪ Infra-inguinal open approach
▪ Local, mesh plug, rapid recovery

Ventral Hernias

▪ Definitions:
▪ Anterior hernias
▪ Primary hernias (no prior surgery)
▪ Umbilical, epigastric, Spigelian
▪ Incisional hernias
▪ Prior operations

915
Copyright © Oakstone Publishing, 2021. All Rights Reserved.

Unusual Abdominal Hernias

▪ Pelvic hernias
▪ Obturator, perineal, sciatic
▪ Lumbar
▪ Grynfeltt - superior triangle
▪ Petit – inferior triangle
▪ Eponymic hernias
▪ Amyand, Littre, Maydl’s, Richters

Ventral Hernias - Incidence

▪ Of the 5,000,000 US hernias, 1/3


ventral
▪ Of all ventral, 1/3 incisional vs 2/3
primary
▪ Total cost between $2,500,000,000
and $3,000,000,000/year

916
Copyright © Oakstone Publishing, 2021. All Rights Reserved.

Risk Factors

▪ Obesity ▪ Peritoneal Dialysis


▪ Smoking, COPD ▪ Collagen Vascular
▪ Steroids Disease
▪ Wound Infection ▪ Genetic
▪ Pregnancy Predisposition

Ventral Hernia - Causes

▪ Most acquired from loss of muscle


integrity
▪ Genetic and acquired tissue disorders
▪ Pascal’s theory – equal pressure
▪ Increased intra-abdominal pressure
causes muscle to contract
▪ When intra-abd pressure > abd wall
pressure, herniation occurs

917
Copyright © Oakstone Publishing, 2021. All Rights Reserved.

Ventral Hernia – Evaluation

History and Physical


Differential diagnosis
 Evaluate for potential
risk factors ▪ Rectus diastasis
 Physical exam while both ▪ Rectus sheath
standing and lying down hematoma
 CT scans may be ▪ Lipomas, neurofibroma,
necessary to evaluate sarcoma
very small, or obscure
hernias ▪ Scar endometriosis
▪ Urachal anomalies

Ventral Hernias

▪ 350,000 hernias repaired yearly in US


▪ 75% primary (epigastric & umbilical)
▪ Mesh associated with reduction in
recurrence compared to suture
▪ Epigastric – weakened linea alba
▪ 3.5% of ventral hernias
▪ Risk: coughing, obesity, steroid, male

Nguyen MT, Berger RL, Hicks SC, et al. JAMA Surg 2014; 149:415.

918
Copyright © Oakstone Publishing, 2021. All Rights Reserved.

Congenital Abdominal Hernias

Epigastric hernia

Ventral Hernias - Epigastric

▪ Defects tend to be small


▪ Often contain small amount of
preperitoneal fat
▪ Repair involves resection of fat or
reduction
▪ Suture vs mesh repair

919
Copyright © Oakstone Publishing, 2021. All Rights Reserved.

Congenital Abdominal Hernias

Umbilical hernia

Umbilical Hernia

▪ Very common; routine screening identifies


hernia in 25% - 50%
▪ Female predominance 3:1
▪ In pregnant women, up to 90%
▪ Incarceration more common in men
▪ Prone to develop in cirrhotic pts

Henriksen NA, Montgomery A, Kaufmann R, et al.. Br J Surg 2020; 107:171.

920
Copyright © Oakstone Publishing, 2021. All Rights Reserved.

Umbilical Hernia

▪ Repair can be open vs lap


▪ Laparoscopic favored for >4 cm
▪ Repair suture vs mesh
▪ For mesh, sublay or IPOM preferred
over onlay
Halm JA, Heisterkamp J, Veen HF, et al. Hernia 2005; 9:334.
Venclauskas L, Silanskaite J, Kiudelis M. Medicina (Kaunas) 2008; 44:855.

Spigelian Hernia

Caption

921
Copyright © Oakstone Publishing, 2021. All Rights Reserved.

CongenitalAbdominal Hernias

▪ Spigelian hernia

Caption

Spigelian Hernia

▪ Aponeurosis of the transversus bounded by


linea semilunaris and rectus
▪ Rare, represent 0.12%-2% of all abdominal
wall hernias
▪ More common in females and over the age
of 60

922
Copyright © Oakstone Publishing, 2021. All Rights Reserved.

Acquired Abdominal Hernias

▪ Incisional or Ventral

Acquired Abdominal Hernias

▪ Parastomal

923
Copyright © Oakstone Publishing, 2021. All Rights Reserved.

Diastasis Recti

Diastasis recti
after
pregnancy

Types of Repair of Abdominal Hernias

▪ Suture - appropriate for small (<2 cm) defects


▪ Mesh
▪ Biologic - human, porcine or bovine
▪ Prosthetic
▪ Polypropylene
▪ Polyester
▪ PTFE (Goretex)
▪ Open vs Laparoscopic vs Robotic

924
Copyright © Oakstone Publishing, 2021. All Rights Reserved.

Types of Repair for Abdominal Hernias

▪ Onlay
▪ Inlay
▪ Sublay
▪ IPOM
(intraperitoneal
onlay of mesh)

Ventral Hernia - Treatment

Open repair
▪ Best for acutely incarcerated
▪ For defects > 10 cm
▪ Repairs: onlay, sublay, inlay, IPOM
▪ Rives Stoppa
▪ Isolate & excise sac; develop plane
between post-rectus & rectus; close post-
rectus, cover w/ mesh

925
Copyright © Oakstone Publishing, 2021. All Rights Reserved.

Ventral Hernia - Treatment

▪ Mesh must have a 4-5 cm overlap of


the defect
▪ If IPOM is used, must have a non-
adherent barrier

Ventral Hernia - Treatment

▪ Laparoscopic repair lowers the risk of SSI


▪ Best for repair of defects <10 cm
▪ Mesh, rather than suture, best in midline
defects 1-4 cm
▪ If defect >10 cm may require component
separation with mesh reinforcement

Luijendijk RW, Hop WC, van den Tol MP, et al. N Engl J Med 2000; 343:392.
Ramirez OM, Ruas E, Dellon AL. Plast Reconstr Surg 1990; 86:519.
Rosen MJ, Williams C, Jin J, et al. Am J Surg 2007; 194:385.

926
Copyright © Oakstone Publishing, 2021. All Rights Reserved.

Open Anterior Component Separation

Component Separation

Anterior Separation Posterior Separation

927
Copyright © Oakstone Publishing, 2021. All Rights Reserved.

TAR (Transversus abdominis release)

Ventral Hernia – Robotic Repair

▪ Dependent on surgeon experience


▪ Unclear role in ventral hernia, no prospective data
▪ Important to close fascial defect
▪ Defect closure & IPOM
▪ Defect closure & preperitoneal mesh
▪ Retrorectus placement of mesh
▪ Extend retrorectus to a transabdominis release

Olavarria OA, Bernardi K, Shah SK, et al. BMJ 2020; 370:m2457.


Belyansky I, Reza Zahiri H, Sanford Z, et al. Hernia 2018; 22:837.

928
Copyright © Oakstone Publishing, 2021. All Rights Reserved.

Treatment Strategies for Ventral Hernias

Selection of robotic technique

929
Copyright © Oakstone Publishing, 2021. All Rights Reserved.

Selection of Robotic Technique

Goals of Repair

▪ Regain abdominal wall functionality


▪ Increased mobility
▪ Minimize pain/discomfort
▪ Cosmesis
▪ Rapid recovery
▪ Low recurrence rate

930
Copyright © Oakstone Publishing, 2021. All Rights Reserved.

Asymptomatic Hernias - non-operative


management

▪ Abdominal wall hernias


▪ If wide opening, easily reducible
▪ Very small defect
▪ If no other co-morbidities
▪ Groin hernia
▪ If long-standing and not bothersome
▪ Compliant patient
▪ Incarceration most common 1-6 months

No disease of the human body, belonging to


the province of the surgeon, requires in its
treatment greater combination of accurate
anatomical knowledge with surgical skill than
hernia in all its varieties

-Sir Astley Paston Cooper, 1804

931
Copyright © Oakstone Publishing, 2021. All Rights Reserved.

Incisional Hernias
Douglas S. Smink, MD, MPH
Chief of Surgery
Brigham and Women’s Faulkner Hospital
Associate Professor of Surgery
Harvard Medical School

No disclosures

932
Copyright © Oakstone Publishing, 2021. All Rights Reserved.

This talk in one slide

Incisional hernias range from the simple to the


complex, and no 2 hernias are the same.
Multiple treatment options exist – none of them
is perfect.

Incisional hernias – A big problem


• 2 million laparotomies per year in US
• 25% will develop a hernia
• 350,000 to 500,000 ventral hernia
repairs annually
• Incisional hernias contribute over $3
billion in health care costs annually

933
Copyright © Oakstone Publishing, 2021. All Rights Reserved.

Structure of this talk


• Preoperative evaluation
• Risk modification
• Abdominal wall anatomy
• Types of repairs
• Illustrative cases

Our goals
• Restore abdominal wall
– Integrity
– Function
• Improve quality of life
• Minimize recurrence

934
Copyright © Oakstone Publishing, 2021. All Rights Reserved.

Reality
• Each patient is unique
– Medical history
– Risk factors
– Body habitus
– Quality of tissue
– Prior surgery
– Job/functional status/activity

Two key considerations


• Preoperative optimization
– Modifiable risk factors
– Set them and you up for success
• Choose an appropriate procedure
– First chance is your best chance

935
Copyright © Oakstone Publishing, 2021. All Rights Reserved.

Preoperative optimization
• Smoking
• Obesity
• Diabetes
• Medications
• Nutritional status
• Infection/fistula

Risk factors, but not modifiable


• Location and size • Ostomy
• Prior surgical history • Soft tissue loss
• Prior mesh • Prior radiation
• Prior infection • Loss of domain
• Fistula • Adhesions

936
Copyright © Oakstone Publishing, 2021. All Rights Reserved.

Abdominal wall anatomy

Earthslab.com

Abdominal wall anatomy

RadioGraphics 2017; 37:1218–1235

937
Copyright © Oakstone Publishing, 2021. All Rights Reserved.

Abdominal wall limitations


• Superiorly – costal margins
• Inferiorly – inguinal ligaments and pubic
symphysis
• Laterally – flank muscles – less fascia than
centrally

Abdominal wall limitations


• Muscles are functionally strong but
anatomically weak (so they don’t hold suture
well)
• Muscle fibers contract in a straight line
• Longer muscles are easier to bend than
shorter muscles

938
Copyright © Oakstone Publishing, 2021. All Rights Reserved.

Goals for repairing incisional hernias


• Close defect without tension
• Reapproximate the rectus muscles in the
midline
• Reinforce repair with mesh to minimize
recurrence
• Minimize mesh contact with bowel

Options for mesh placement


• Onlay
• Inlay (interposition)
• Sublay
– Retrorectus
– Intraperitoneal

Alimi, Y et al. Plast Aesthet Res 2020;7:5.

939
Copyright © Oakstone Publishing, 2021. All Rights Reserved.

Types of repair
• Laparoscopic repair

• Open repair

• Robotic repair

Laparoscopic repair
• Typically intraperitoneal mesh placement

Pros Cons
– Large mesh, wide – Poor cosmesis for
overlap large defects
– Minimal tension – Difficult to close
– Low SSI risk fascial defect
– Pain

940
Copyright © Oakstone Publishing, 2021. All Rights Reserved.

Open repair
Pros Cons
– Enables multiple mesh – Larger incision and skin
placements flaps
– Able to close fascial – Increased risk for SSI
defect – Pain
– Component separation
options

Component separation
• Release tension
• Reapproximate rectus muscles in midline
• Anterior component separation
• Posterior component separation
• Both have advantages and disadvantages

941
Copyright © Oakstone Publishing, 2021. All Rights Reserved.

Anterior component separation


• Described by Ramirez in 1990
• Can be combined with onlay, retrorectus, or
underlay mesh
• Medial release: 4-10 cm per side
• Easy to learn
• Concerns – skin flap necrosis, lateral hernia
• Need to preserve perforating vessels

Anterior component separation

Gnaneswaran, N. et al. European Journal of


Plastic Surgery 2016

942
Copyright © Oakstone Publishing, 2021. All Rights Reserved.

Posterior component separation


• Described by Rives-Stoppa in early 1970’s
• Sublay mesh placement
• Tissue layer between mesh and bowel
• Less skin flap creation, less risk for SSI
• Tissue plane can be scarred
• Less medial mobilization of rectus muscles

Posterior component separation

NGUYEN,D et al. European Surgery 2017

943
Copyright © Oakstone Publishing, 2021. All Rights Reserved.

Traversus abdominis release


• Described by Rosen and Yuritzsky in 2011
• Dissection beyond lateral border of rectus to
divide transversus abdominis muscle
• Allows for larger mesh to reinforce abd wall
• Sublay - tissue layer between mesh and bowel
• Technically harder to learn

Transversus abdominis release

NGUYEN,D et al. European Surgery 2017

944
Copyright © Oakstone Publishing, 2021. All Rights Reserved.

Robotic repair
Pros Cons
– Function of robot arms – Time
– 3D vision – Cost
– Closure of fascial defect – Question of added
– Posterior release/TAR benefit

Rapid adoption of robotics

JAMA Netw Open. 2020;3(1):e1918911. doi:10.1001/jamanetworkopen.2019.18911

945
Copyright © Oakstone Publishing, 2021. All Rights Reserved.

Growth of robotic repair

JAMA Netw Open. 2020;3(1):e1918911. doi:10.1001/jamanetworkopen.2019.18911

How I approach an incisional hernia


• What was the prior operation?
• What is the physical exam?
• What are the patient’s symptoms, concerns,
and goals?
• Are there risk factors for hernia recurrence?

946
Copyright © Oakstone Publishing, 2021. All Rights Reserved.

Case #1
• 62 yo M
• History of robotic prostatectomy 2 years ago
• Bulge at supraumbilical incision
• Mildly unsightly and intermittently
uncomfortable

Case #1
• Works as a janitor
• BMI = 31
• Vertical 3.0 cm supraumbilical incision
• Reducible supraumbilical hernia with 2.5 cm
fascial defect

947
Copyright © Oakstone Publishing, 2021. All Rights Reserved.

What are the options?


• Open repair
– With or without mesh
• Laparoscopic repair
– Intraperitoneal or preperitoneal mesh
• Robotic repair

My plan
• Laparoscopic repair with intraperitoneal mesh
• Why?
– Slightly overweight
– Relatively small hernia
– Good cosmetic result likely

948
Copyright © Oakstone Publishing, 2021. All Rights Reserved.

How I do it
• Veress entry

• Port placement

• Lysis of adhesions

How I do it
• Mesh type and size

• Additional ports

• Mesh placement and


fixation

949
Copyright © Oakstone Publishing, 2021. All Rights Reserved.

Case #2
• 53 yo F with history of right hemicolectomy
through midline incision
• 2 pregnancies in her 30’s
• 6 x 8 cm incisional hernia superior to umbilicus
• Uncomfortable but also does not like
protrusion of abdominal wall
• BMI 27

My considerations
• Close hernia defect

• Cosmetic result

• Durability

• Lap vs open repair

950
Copyright © Oakstone Publishing, 2021. All Rights Reserved.

My approach
• Midline incision
• Excise prior scar
• Enter peritoneum superior to hernia
• Dissect abdominal contents off abdominal
wall
• Excise hernia sac

Posterior component separation

NGUYEN,D et al. European Surgery 2017

951
Copyright © Oakstone Publishing, 2021. All Rights Reserved.

My approach
• Start on one side
• Kochers on linea alba
• Open posterior sheath just lateral to linea alba
• Put Allis clamps on posterior sheath
• Open posterior sheath for length of hernia
• Dissect posterior to rectus muscle to lateral
edge

Retrorectus dissection

https://basicmedicalkey.com/incisional-hernia-repair-abdominal-wall-reconstruction-options-2/

952
Copyright © Oakstone Publishing, 2021. All Rights Reserved.

My approach
• Preserve large perforating vessels entering
rectus
• Repeat process on contralateral side
• Close posterior sheath in midline with 2-0 PDS
• Important points
– Need maximum relaxation
– Can increase abdominal and lung pressures

My approach
• Place polypropylene mesh in retrorectus space
• Secure with hemostatic agent
– Others place sutures anteriorly through
abdominal wall
• Close linea alba in midline with 0 looped PDS
– Minimal skin flap needed
– Place binder, no drain needed

953
Copyright © Oakstone Publishing, 2021. All Rights Reserved.

Summary
• Individual approach
• Preoperative optimization
• Understand the abdominal wall anatomy
• Choose the best operation in your hands
• Restore abdominal wall integrity and function

Thank you

Douglas S. Smink, MD, MPH


Brigham & Women’s Faulkner Hospital

954
Copyright © Oakstone Publishing, 2021. All Rights Reserved.

Benign Gallbladder Disease


Ashley Haralson Vernon, MD
Associate Surgeon
Brigham and Women’s Hospital
Assistant Professor of Surgery
Harvard Medical School
Contact info: [email protected]

No Conflict of Interest

Division of General and GI Surgery


Brigham and Women’s Hospital

955
Copyright © Oakstone Publishing, 2021. All Rights Reserved.

Outline
Benign Gallbladder Disease

• Review of Biliary Anatomy


• Prevalence of Gallstones
• Formation of gallstones
• Types of stones
• Risk factors for stone formation

Division of General and GI Surgery


Brigham and Women’s Hospital

Outline
Benign Gallbladder Disease

Clinical Scenarios
• Asymptomatic Gallstones
• Symptomatic Gallstones = Biliary Colic
• Acute Cholecystitis
• Biliary Pancreatitis
• Biliary Dyskinesia
• Gallbladder Polyps
• Gallstones in pregnancy

Division of General and GI Surgery


Brigham and Women’s Hospital

956
Copyright © Oakstone Publishing, 2021. All Rights Reserved.

Format for clinical scenarios


Benign Gallbladder Disease

• Symptoms
• Pertinent History – HPI, PMH, PSH, SH/FH
• Physical Exam
• Easy tests - Labs
• Hard tests – Ultrasound, HIDA scan, MRCP, ERCP
• Plan:
• Medical management
• Surgery - pre-op prep & details of the operation

Division of General and GI Surgery


Brigham and Women’s Hospital

Biliary Anatomy

Division of General and GI Surgery


Brigham and Women’s Hospital

957
Copyright © Oakstone Publishing, 2021. All Rights Reserved.

Prevalence of Gallstones
Benign Gallbladder Disease

• In developed countries, 10–15% of adults have


gallstones.
• Increases with age and in women
• The prevalence is very high in some ethnic groups.
• Native American
• 29.5% men 64.1% women
• Pima Indians
• 50% men 73% women
• Mexican American
• 8.9% men 26.7% women

Division of General and GI Surgery


Brigham and Women’s Hospital

Gallstone Formation
• Cholesterol
• Bile Salts
• Bilirubin

Division of General and GI Surgery


Brigham and Women’s Hospital

958
Copyright © Oakstone Publishing, 2021. All Rights Reserved.

Types of Gallstones
Benign Gallbladder Disease

• Cholesterol stones = Yellow green stones


• Bile contains too much cholesterol
• Pigment stones = Black stones
• Bile contains too much
• Mixed type/Brown stones
• Rare
• Develop in bile ducts
• Infection

Division of General and GI Surgery


Brigham and Women’s Hospital

Cholesterol Gallstones
Benign Gallbladder Disease

• Cholesterol supersaturation
• excessive secretion of cholesterol
• deficient secretion of bile salt and lecithin, the solubilizers of
these otherwise insoluble lipids
• Cholesterol precipitation/crystallization
• Gallbladder hypomotility (contraction,
absorption, secretion)
• Impairment of enterohepatic circulation of
bile acids.
Division of General and GI Surgery
Brigham and Women’s Hospital

959
Copyright © Oakstone Publishing, 2021. All Rights Reserved.

Risk Factors
For developing gallstones

• Age >40
• Female, reproductive age, pregnancy
• Obesity, high fat diet, T2 DM
• Ethnicity (Native American, Mexican
American)
• Rapid weight loss
• Cirrhosis
• Disruption of enterohepatic circulation

Division of General and GI Surgery


Brigham and Women’s Hospital

Natural History of Gallstones


Benign Gallbladder Disease

• Majority of patients (70-80%) with gallstones are


asymptomatic.
• Low annual rate (<2%) of developing symptoms
• Very low annual rate (0.3%) of developing
complications like cholecystitis, pancreatitis,
choledocholithiasis
• Complications are rare (3-4 % in 10 years) when not
preceded by symptoms.

Division of General and GI Surgery


Brigham and Women’s Hospital

960
Copyright © Oakstone Publishing, 2021. All Rights Reserved.

Asymptomatic Gallstones
• Asymptomatic patients do not usually need surgery.
• There are some patients with asymptomatic stones
that are at a higher risk for complications
• Hereditary blood disorders
• Asymptomatic choledocholithiasis
• High risk of gallbladder cancer
• They should be counseled regarding symptoms.
• Once symptomatic, then cholecystectomy
recommended

Division of General and GI Surgery


Brigham and Women’s Hospital

Symptomatic Gallstones
• Symptoms
• Right upper quadrant pain
• Nausea vomiting
• Dyspepsia/Food intolerance
• Pertinent History
• Risk Factors for Gallstones
• Physical Exam
• Occasional tenderness
• Worrisome for cholecystitis if there is fever or Murphy’s
sign (inspiratory arrest)
Division of General and GI Surgery
Brigham and Women’s Hospital

961
Copyright © Oakstone Publishing, 2021. All Rights Reserved.

Testing & Plan


Symptomatic gallstones

• Lab Tests
• normal LFTs
• Imaging –
• Ultrasound
Specificity and Sensitivity
• Plan
• Pain control with NSAIDs & elective Cholecystectomy

Once symptoms begin, they tend to be recurrent. Patients with a history of biliary
colic have a nearly 70% chance of recurrent pain within 2 years.
Once symptoms begin, 2x higher chance of complications.

Division of General and GI Surgery


Brigham and Women’s Hospital

Elective Cholecystectomy
Symptomatic gallstones

• Timing of surgery - elective


• Laparoscopic surgery in most patients including
cirrhotics except those with Child’s C
• Patient prep – no antibiotics except high-risk pts
• No blood thinners
• Laparoscopic access
• Veress vs Hasson vs Direct Trocar vs Optical trocar

SAGES Guidelines for Lap Biliary Surgery

Division of General and GI Surgery


Brigham and Women’s Hospital

962
Copyright © Oakstone Publishing, 2021. All Rights Reserved.

Elective Cholecystectomy
Symptomatic gallstones

• Technique
• Completely expose and delineate the hepatocystic triangle
• Identify a single duct and a single artery entering the
gallbladder
• Completely dissect the lower part of the gallbladder off
the liver bed
• Routine IOC not necessary
• Do not divide any structure until you are certain what
it is.

Division of General and GI Surgery


Brigham and Women’s Hospital

Symptoms
Acute Cholecystitis

• Inflammation of the gallbladder


• Infection
• Acute cholecystitis
• RUQ Pain
• Nausea/Vomiting
• Fever
• Calculus vs Acalculus
• Pertinent History -Risk factors for BGD (Benign
Gallbladder Dx)
• Physical Exam - RUQ tenderness, Murphy’s sign,
palpable RUQ mass
Division of General and GI Surgery
Brigham and Women’s Hospital

963
Copyright © Oakstone Publishing, 2021. All Rights Reserved.

Tokyo guidelines
Acute Cholecystitis

• A: Local signs of inflammation


• Murphy’s sign
• RUQ Pain/Tenderness
• B: Systemic signs of Inflammation
• Fever
• Elevated CRP
• Elevated WBC
• C: Imaging Findings

• Suspected Diagnosis: A+B


• Definite Diagnosis: A+B+C
Yokoe M et al, Journal of Hepato-Biliary-Pancreatic Sciences, 2017

Division of General and GI Surgery


Brigham and Women’s Hospital

Testing
Acute Cholecystitis

• Lab Tests
• Leukocytosis
• Normal LFTs unless sepsis
• Imaging
• Ultrasound
• Thickened gallbladder wall (>4mm)
• Pericholecystic fluid
• Sonographic Murphy’s
• Sensitivity 88%, Specificity 80%
• Cholescintigraphy with HIDA
• non-visualization of the gallbladder
• at 30 mins without morphine and
• 60 mins with morphine
• Sensitivity 90-97% Specificity 71-90%
Division of General and GI Surgery
Brigham and Women’s Hospital

964
Copyright © Oakstone Publishing, 2021. All Rights Reserved.

Plan
Acute Cholecystitis

• Antibiotics with gram negative coverage


• Surgery
• 10-15% of all cholecystectomies performed are for
acute cholecystitis
• Surgery is best performed within 24-72 hours of
diagnosis without increased rates of complications
• Early cholecystectomy is associated with decreased
antibx, complications, LOS and cost
• Laparoscopic cholecystectomy is the preferred approach
in patients with acute cholecystitis
• Usually can be performed laparoscopically with rates of
conversion to an open procedure of 6-35%.

Division of General and GI Surgery


Brigham and Women’s Hospital

Plan
Acute Cholecystitis

• Difficult decision when the patient presents


outside of 72-hour window
• High risk of complications
• Options include
• Antibiotics
• Gallbladder drainage
• Percutaneous cholecytostomy is an effective
temporizing measure and cholecystitis will resolve
in 85% of patients

Division of General and GI Surgery


Brigham and Women’s Hospital

965
Copyright © Oakstone Publishing, 2021. All Rights Reserved.

Cholecystostomy
For Acute Cholecystitis

• Cholecystostomy
• Cannot tolerate general anesthesia
• Delayed presentation/failure of antibiotics
• Most patients will undergo cholecystectomy at
later date but can be a destination therapy in some
• Some very ill patients may not ultimately undergo
cholecystectomy if too high risk
• Management of tube with tube study
• Check for patency of cystic duct and clamp tube
• This makes subsequent surgical anatomy more typical

Division of General and GI Surgery


Brigham and Women’s Hospital

Cholecystectomy
For Acute Cholecystitis

• Difficult gallbladder
• Open cholecystectomy
• Cholecystostomy
• Fundus-first approach
• Subtotal cholecystectomy

Division of General and GI Surgery


Brigham and Women’s Hospital

966
Copyright © Oakstone Publishing, 2021. All Rights Reserved.

Subtotal Cholecystectomy
For Acute Cholecystitis

• Compared to open cholecystectomy, SC was reported to


result in greater postoperative bile leakage, but lower
common bile duct injury, postoperative complications,
reoperation, and mortality
• Subtotal vs Partial
• Fenestrating vs Reconstituting

• Strasberg et al, J AmColl Surg, 2016 Jan

Division of General and GI Surgery


Brigham and Women’s Hospital

Subtotal Cholecystectomy
Fenestrating

Strasberg et al, J AmColl Surg, 2016 Jan

967
Copyright © Oakstone Publishing, 2021. All Rights Reserved.

Subtotal Cholecystectomy
Reconstituting

Strasberg et al, J AmColl Surg, 2016 Jan

968
Copyright © Oakstone Publishing, 2021. All Rights Reserved.

Symptoms
Biliary Pancreatitis

• Symptoms
• Bandlike Pain (to back)
• Sxs of biliary colic
• Pertinent History
• Risk Factors for gallstones
• r/o EtOH
• Physical Exam
• Tenderness
• Mid-abdominal fullness

Division of General and GI Surgery


Brigham and Women’s Hospital

Testing
Biliary Pancreatitis

• Lab tests: WBC, LFTS (r/o CBD stone), Lipase, Ca


• Imaging:
• Ultrasound: Stones? (ultrasound misses CBD
stones)
• CT scan - Gold Standard
• MRCP
• ERCP to clear any biliary obstruction

Division of General and GI Surgery


Brigham and Women’s Hospital

969
Copyright © Oakstone Publishing, 2021. All Rights Reserved.

Surgery
Biliary Pancreatitis

• Timing of surgery
• Urgent cholecystectomy once labs and exam have
normalized
• Perform during index admission to avoid recurrence (25-
30%)
• Delay surgery if patient is critically ill
• IOC typical
• Patient prep - Antibiotics
• No blood thinners

Division of General and GI Surgery


Brigham and Women’s Hospital

Biliary Dyskinesia

• Symptoms –Symptoms of biliary colic, r/o atypical


symptoms like bloating, fullness, or dyspepsia
• Lab work: WBC, LFTS, Lipase
• Studies:
• Ultrasound: No sludge or stones
• EGD?
• HIDA scan + CCK injection

Division of General and GI Surgery


Brigham and Women’s Hospital

970
Copyright © Oakstone Publishing, 2021. All Rights Reserved.

Biliary Dyskinesia
• Surgery is indicated in select patients who may
benefit based on criteria
• Typical biliary pain, not atypical symptoms
• Gallbladder motility testing HIDA + CCK injection
• Significance of low GB EF unknown
• Reproduction of pain with CCK has been suggestive but
specificity is questioned.
• Greater than 3 months duration
Patients with typical biliary-type pain that is
recurrent (over at least three months) may
benefit from cholecystectomy
Division of General and GI Surgery
Brigham and Women’s Hospital

Gallbladder Polyps
• Found in >4 % of the adult population
• 4-10% are adenomas with malignant potential
• Adenomas >1cm have 37-55% chance of malignancy
• 2017 European guideline for surgery if
• >10mm or
• >6mm = 5+ year follow-up
• >2mm growth
• Kaiser Permanente study comparing group of pts
with gallbladder cancer and @35K adults with
ultrasonography. Szpakowski et al, JAMA, 2020

Division of General and GI Surgery


Brigham and Women’s Hospital

971
Copyright © Oakstone Publishing, 2021. All Rights Reserved.

Gallbladder Disease in Pregnancy


• Cholecystectomy is the second most common non-
obstetrical surgery during pregnancy
• Traditional recommendation had been to avoid
surgery in the first and third trimesters.
• It is possible for a woman to safely have surgery
during pregnancy during any trimester.
• Take into consideration nausea, food fear, weight
loss, etc
• Recent studies suggest increased maternal fetal
complications when surgery is delayed.
Division of General and GI Surgery
Brigham and Women’s Hospital

Summary
• Patients with gallstones and no symptoms will
probably never need surgery.
• Patients with gallstones with symptoms should be
referred for surgery to reduce complications
• Patients with mild to moderate cholecystitis should
undergo surgery in first 3 days if clinically stable.
• If delayed presentation or critically ill or unable to
tolerate general anesthesia, options include
antibiotics +/-cholecystostomy tube

Division of General and GI Surgery


Brigham and Women’s Hospital

972
Copyright © Oakstone Publishing, 2021. All Rights Reserved.

Summary
• In difficult gallbladder, helpful options include IOC
& top-down approach
• To avoid bile duct injury, best to perform subtotal
cholecystectomy
• All patients with gallstone pancreatitis/elevated
lipase should undergo cholecystectomy during
index hospitalization unless too ill
• Patients with biliary dyskinesia and typical biliary
colic x 3 months may benefit from surgery.

Division of General and GI Surgery


Brigham and Women’s Hospital

Summary
• Polyps are rarely “pre-cancerous.” Surveillance
should be pursued when >6mm in size.
• Gallbladder polyps are not likely to become
cancerous but guidelines are burdensome and
benefit not clear
• Pregnant patients with biliary disease should be
cared for in a timely manner if symptomatic to
prevent maternal fetal complications. No need to
delay until post partum.

Division of General and GI Surgery


Brigham and Women’s Hospital

973
Copyright © Oakstone Publishing, 2021. All Rights Reserved.

Thank you !

Contact information at Brigham & Women’s hospital


[email protected]

Division of General and GI Surgery


Brigham and Women’s Hospital

974
Copyright © Oakstone Publishing, 2021. All Rights Reserved.

Common Bile Duct Disorders

Thomas C. Tsai, MD, MPH

Assistant Professor of Surgery | Harvard Medical School


Assistant Professor in Health Policy and Management | Harvard Chan School
of Public Health

Conflict of Interest

• Seamless Mobile Health (Scientific Advisory Board)


• Medtronic (Consultant/Speaker)

• Research grants from The Commonwealth Fund, Massachusetts Consortium on


Pathogen Readiness, Arnold Ventures, William F. Milton Fund of Harvard
University

975
Copyright © Oakstone Publishing, 2021. All Rights Reserved.

Topcs
• Common Bile Duct Stones
• Benign Biliary Strictures
• Cystic Disorders of the Bile Ducts
• Primary Sclerosing Cholangitis

Choledocholithiasis
• Epidemiology
• Common bile duct stones can be identified in 10% of routine cholangiograms at
the time of cholecystectomy
• Retained stones after cholecystectomy can occur in 1-2% of cases
• Symptoms
• Asymptomatic
• RUQ pain
• Charcot’s Triad (Cholangitis)
• Fever, RUQ pain, Jaundice
• Only 15-20% of patients with cholangitis present with Charcot’s triad
• Reynold’s Pentad
• Charcot’s Triad + hypotension and mental status change
• Suggests sepsis and need for urgent biliary decompression

976
Copyright © Oakstone Publishing, 2021. All Rights Reserved.

Predictors of Choledocholithiasis
• Very Strong
• CBD stone on ultrasound
• Clinical ascending cholangitis
• Bilirubin > 4 mg/dL
• Strong
• Dilated CBD on US (>6 mm with gallbladder in situ)
• Bilirubin 1.8-4 mg/dL
• Moderate
• Abnormal liver biochemical test other than bilirubin
• Age > 55
• Clinical gallstone pancreatitis Maple JM, et al, Gastrointestinal Endoscopy, 2010

Choledocholithiasis: Imaging Modalities


• Ultrasound
• Can show dilated CBD
• Generally normal is 4 mm with 1
mm for each decade of life after
40

977
Copyright © Oakstone Publishing, 2021. All Rights Reserved.

Choledocholithiasis: Imaging Modalities


• MRCP
• Sensitivity: 90-94%
• Specificity: 95-99%
• Can miss small stones
• Can also have false positive due
to motion artifact
• Useful for defining biliary
anatomy

Choledocholithiasis: Imaging Modalities


• Endoscopic Ultrasound
• Sensitivity: 93-97%
• Specificity: 94-95%
• Very sensitive for stones smaller
than 5 mm
• Can be combined with ERCP if
EUS suggests choledocholithiasis

978
Copyright © Oakstone Publishing, 2021. All Rights Reserved.

Choledocholithiasis: Imaging Modalities


• ERCP
• Interventional procedure with
complications and best suited for
high risk of choledocholithiasis
• “Gold Standard”
• Can be therapeutic with
sphincterotomy and subsequent
duct sweeping with
balloons/baskets
• Complications
• Pancreatitis (1.3-6.7%)
• Infection
• Hemorrhage
• Perforation

Long-term outcomes of ERCP without


Cholecystectomy

Saito M, et al., Gastrointestinal Endoscopy, 2000

979
Copyright © Oakstone Publishing, 2021. All Rights Reserved.

Choledocholithiasis: Imaging Modalities


• Intraop Cholangiogram
• After critical view of safety is
achieved, ductotomy is made in
the cystic duct
• 5F cholangiogram catheter is
introduced and secured with clips
or Reddick clamp.
• Water-soluble contrast is injected
and fluoroscopy is performed

Intraoperative Near-Infrared
Cholangiography
• NIR Cholangiography uses
indocyanine green
(hepatically cleared into bile)
to provide anatomic
definition
• In RCT (Dip, Annals of
Surgery, 2019) has been
shown to be superior to
white light laparoscopy in
visualizing extrahepatic
biliary structures with no CBD
injury

Division of General and GI Surgery


Brigham and Women’s Hospital

980
Copyright © Oakstone Publishing, 2021. All Rights Reserved.

Cholecystectomy: Critical View


• Critical view of safety
• Hepatocystic triangle is cleared
of all fatty/fibrous tissue
• Lower third of gallbladder is
cleared to expose cystic plate
• Two and only two structures
entering the gallbladder

• Principles
• Aberrant anatomy
• Use cholangiography
• Critical view pause
• Consider subtotal or
cholecystostomy tube if
dissecting in zone of risk
• Intraoperative consult

Division of General and GI Surgery


Brigham and Women’s Hospital

Choledocholithiasis: Surgical Management


• Laparoscopic cholecystectomy with intraoperative cholangiogram
• Flush CBD with cholangiogram catheter
• Administer 1 mg of Glucagon
• Laparoscopic common bile duct exploration
• Open common bile duct exploration

981
Copyright © Oakstone Publishing, 2021. All Rights Reserved.

Choledocholithiasis: Laparoscopic CBD


Exploration
• Transcystic
• Favorable: Cystic duct > 4mm, small stones <10
mm, lateral entrance of cystic duct onto CBD
• Performed in conjunction with IOC
• New generation choledochoscopes and
accessory wires, balloons, baskets have made
this approach more accessible
• Choledochotomy
• More demanding
• Can close choledochotomy with 4-0 or 5-0
absorbable sutures
• Does not need routine T-tube placement

Choledocholithiasis: CBD Exploration


• Kocher maneuver to expose the
distal CBD
• Vicryl stay sutures are placed,
longitudinal 1-1.5 cm ductotomy is
made
• Stones removed by extraction of
biliary Fogarty balloons
• Completion cholangiogram
• Classically 18F T-tube is placed
(posterior wall cutoff so T-tube
“cracks” and reopposes to form a
cylinder on removal
• Removed 3-4 weeks after surgery
after a T-Tube cholangiogram

982
Copyright © Oakstone Publishing, 2021. All Rights Reserved.

Choledocholithiasis: Biliary –Enteric


Drainage
• Rare cases where stones can’t be retrieved, biliary-enteric drainage
can be considered
• Choledochoduodenostomy
• 1.5 cm longitudinal ductotomy and 1 cm duodenostomy; Side-to-side
anastomosis of duodenum to common bile duct
• Roux-en-Y Choledochojejunostomy

Maple JM, et al, Gastrointestinal Endoscopy, 2010

983
Copyright © Oakstone Publishing, 2021. All Rights Reserved.

Mirizzi Syndrome
• Common hepatic duct obstruction caused
by extrinsic compression from impacted
stone in the infundibulum or cystic duct
• Often leads to technically challenging
cholecystectomy
• My approach
• Fenestrate the gallbladder and remove the
impacted stone, this decompresses and
facilitates a completion cholecystectomy or a
near-total cholecystectomy

Acute Cholangitis
• Most commonly due to choledocholithiasis, but can be due to
stricture or malignancy
• Urgent biliary decompression needed
• Endoscopic (ERCP)
• Percutaneous Transhepatic
• Surgical

984
Copyright © Oakstone Publishing, 2021. All Rights Reserved.

Acute Cholangitis: Tokyo Guidelines 2018


• Grade 1 (Mild)
• Grade 2 (Moderate)
• At least 2 of the 5
• WBC >12,000 or <4,000
• T > 39 C
• Age > 75
• T bili > 5 mg/dL
• Albumin < lower limit of normal x 0.73
• Grade 3 (Severe)
• Grade 2 + sepsis or end organ dysfunction

985
Copyright © Oakstone Publishing, 2021. All Rights Reserved.

Benign Biliary Strictures: CBD Injury


• Strasberg-Bismuth Classification
of Lap CBD Injury
• A: cystic duct leak
• B and C: Aberrant right hepatic
duct injury
• D: lateral injury to hepatic duct
• E: Strictures to hepatic duct
• E1: > 2 cm from the bifurcation
• E2: < 2 cm from the bifurcation
• E3: at bifurcation
• E4: hilar stricture
• E5: Involves aberrant right hepatic
duct and bifurcation

Management of CBD Injury


• Management varies by type of injury
• General principles
• Define extent of injury (MRCP, CT, or cholangiogram)
• Control sepsis (draining extrahepatic bilomas or collections)
• Biliary decompression (transhepatic or ERCP stent)
• Repair
• Lateral injuries: closure over T-tube
• Roux-en-Y hepaticojejunostomy
• Choledochojejunostomy

986
Copyright © Oakstone Publishing, 2021. All Rights Reserved.

Choledochal Cysts
• Choledochal cysts are dilations of the extrahepatic or intrahepatic biliary tree
• Usually develop in children but diagnosed in adults
• Rare (1-1.5 per 100,000 in the West)
• More common in East Asia
• More common in women
• Risk of cholangiocarcinoma
• Clinical Presentation
• RUQ pain, jaundice, abdominal mass (classic, but rare)
• Adults can also present with symptomatic cholelithiasis (18%), pancreatitis (16%), early satiety (11%)
• Treatment
• Cholecystectomy
• Excision of choledochal cyst
• Biliary-enteric anastomosis to prevent reflux of pancreatic enzymes into biliary tract

Choledochal Cyst: Types


Type 1: dilation of the
extrahepatic biliary tract,
70% of cases
Type 2: saccular
diverticulum
Type 3: dilation within the
duodenal wall
Type 4: dilation of
intrahepatic and
extrahepatic biliary tree,
24% of cases
Type 5: intrahepatic cyst
(Caroli’s disease)

987
Copyright © Oakstone Publishing, 2021. All Rights Reserved.

Primary Sclerosing Cholangitis


• PSC is a chronic inflammatory disease of the biliary tree which
causes cholestasis and can progress to liver failure
• Epidemiology
• Rare disease, more common in males than females, usually in 4th or 5th
decade of life
• Idiopathic (likely genetic and environmental factors)
• Associated with Ulcerative colitis (60-75% of PSC patients)
• Risk factor for cholangiocarcinoma (10-20% of PSC patients).

PSC
• Labs
• Elevated Alk phos
• Perinuclear antinuclear cytoplasmic
antibody present in 26-94% of cases
• ERCP and MRCP are gold
standards for diagnosis
• If Elevated CA-19-9 and strictures
in biliary tree, needs ERCP and
brushings to evaluate for
cholangiocarcinoma
• Benign strictures can be treated
with dilation

988
Copyright © Oakstone Publishing, 2021. All Rights Reserved.

Primary and Secondary Liver


Neoplasms
Thomas Clancy MD

Division of Surgical Oncology


Brigham and Women’s Hospital
Harvard Medical School

Disclosures

Nothing to disclose

989
Copyright © Oakstone Publishing, 2021. All Rights Reserved.

Liver Tumors

• Anatomic considerations
• Cystic lesions
• Solid lesions: benign
• Solid lesions: malignant
• Primary (HCC, cholangiocarcinoma)
• Secondary (colorectal, neuroendocrine)
• Considerations for resection
• Biliary neoplasms

Incidental Liver Lesions

• Present in up to 20% abdominal radiographic


studies
• What is differential diagnosis?
• Is a radiologic diagnosis possible?
• Is tissue confirmation needed?
• What are treatment options (if needed)

990
Copyright © Oakstone Publishing, 2021. All Rights Reserved.

Incidental Liver Lesions

• Cystic vs. Solid


• Solitary vs. Multiple
• Primary vs. Secondary/metastatic
• Imaging characteristics (enhancing? Infiltrative?)

Incidental Liver Lesions

Consider clinical context:


• Known prior lesion ? change
• h/o malignancy ? metastasis
• Hepatitis, EtOH ? cirrhosis, ? HCC
• Estrogen/OCP use ? adenoma
• Travel ? echinococcal disease
• Fever, biliary obstruction ? abscess

991
Copyright © Oakstone Publishing, 2021. All Rights Reserved.

Liver: Segmental Anatomy

Liver Anatomy: Vascular inflow

Hepatic Arterial Arcade Portal Venous Inflow

992
Copyright © Oakstone Publishing, 2021. All Rights Reserved.

Liver anatomy: venous outflow

Hepatic Venous Anatomy

.
above the level of the left portal vein. at the level of the left portal vein

at the level of the right portal vein. at the level of the splenic vein.

993
Copyright © Oakstone Publishing, 2021. All Rights Reserved.

Use Landmarks on Imaging:


• What structures involved? What operation required?
• What healthy parenchyma will be left behind?

Cystic Liver Lesions

Consider Differential diagnosis:

• Simple biliary cyst

IF: internal echoes, septations, perceptible wall, solid


components, consider:

• Abscess
• Biliary cystadenoma/cystadenocarcinoma
• Cystic metastases
• Hydatid disease

994
Copyright © Oakstone Publishing, 2021. All Rights Reserved.

Benign Biliary Cyst

• Biliary origin. Cuboidal epithelium


• Rarely symptomatic (?other source pain)
• Dx usually clear on U/S
• CT: clearly defines (low attenuation, homogeneous, no
wall, no enhancement)
• MRI: usually not needed but will definitively
characterize

Benign Biliary Cyst - Ultrasound


Spherical
Imperceptible wall
No internal echoes

Posterior
enhancement

995
Copyright © Oakstone Publishing, 2021. All Rights Reserved.

Benign Biliary Cyst

• Asymptomatic cysts: NO TREATMENT


• Truly symptomatic cysts:
• Aspiration: most recur (symptoms resolve?)
• Sclerotherapy (ensure no bile/blood)
• Unroof cyst – laparoscopic fenestration
• Danger: unrecognized biliary cystadenoma

Symptomatic Benign Biliary Cyst


• Early satiety, partial
gastric outlet
obstruction
• Cyst fenestration:
remove anterior wall,
omental flap to cyst
cavity, leave back wall

996
Copyright © Oakstone Publishing, 2021. All Rights Reserved.

Biliary Cystadenoma
• Usually women, > 40;
rare
• Radiographic
diagnosis
• Dx – is it cyst with
recent hemorrhage
• Large mass, irregular
margins, septation
• Risk: malignant
progression
• Rx: complete excision
(not aspiration, not
fenestration)

Liver abscesses

• Often multiple low-


attenuation lesions
• ring enhancement
(arrows)
• often nonspecific on
appearance on CT
• Appearance may be similar
to metastases (consider
clinical context)

997
Copyright © Oakstone Publishing, 2021. All Rights Reserved.

Liver abscesses

• Origin: biliary or
hematogenous
• Risk: biliary stent, travel,
immunocompromise,
IVDU
• Aspirate, culture, +/-
drain

Hytadid Cyst (echinococcosis)

• cystic mass, discrete wall,


separated internal membranes,
several “daughter cysts”
• Sheep = intermediate host
• Endemic: Italy, Turkey, Greece,
N. Zealand
• Key: distinguish simple cyst,
cystadenoma
• Serology: (+) in 90%
• Unilocular or complex cyst, may
have Ca++

998
Copyright © Oakstone Publishing, 2021. All Rights Reserved.

Hytadid Cyst (echinococcosis)

• Treatment: eradicate
parasite, protect against
spillage
• systemic meds (albendazole)
• Percutaneous (no sclerosis if
bile seen)
Risk: spillage, anaphylaxis
• Open cyst
evacuation/sterilization

Solid Liver Lesions


• Benign
• Hemangioma
• Heptocellular adenoma
• Focal nodular hyperplasia (FNH)
• Focal fatty change
• Malignant
• Primary liver malignancy
• Hepatocellular carcinoma
• Intrahepatic cholangiocarcinoma
• Liver metastases

999
Copyright © Oakstone Publishing, 2021. All Rights Reserved.

Liver Hemangioma

• Most common lesion – in up to 15% pt, F>M


• Range from <1cm to > 20cm
• Symptoms rare, if large: occasional pain (usually
another source).
• Bleeding extremely rare
• Characteristic imaging
• MRI: T2 hyperintense, peripheral enhancement
• Most: observation only
• Resect? Only for symptoms (rare)

Hemangioma: Imaging Findings

• enhancement pattern
for hemangioma on w/
gradual filling by
contrast medium
• “outside-in” or
centripetal fill-in

1000
Copyright © Oakstone Publishing, 2021. All Rights Reserved.

Focal Nodular Hyperplasia

• 2nd most common benign lesion, F>M


• NOT influenced by oral contraceptives
• Characteristic: “central scar”
*Also present in fibrolamellar variant HCC
• CT/MRI: early enhancement, early “wash-out,” homogeneous
• Nearly always asymptomatic, no complications/bleeding
• Sulfur colloid scan (+) – unlike other lesions
• Rx: observe, serial imaging
• Biopsy? If uncertain
• Resect? Only inability to r/o malignancy

Focal Nodular Hyperplasia

• homogeneous except
central scar
• increased signal on T2
and low signal on T1
and after gadolinium
• transient enhancement
except scar, which
enhances late

1001
Copyright © Oakstone Publishing, 2021. All Rights Reserved.

Hepatocellular Adenoma

• Primary risk factor: OCPs (related duration)


• Complications: (1) hemorrhage (2) malignancy
• Most asymptomatic
• Hemorrhage- most > 5cm, increase with OCP/pregnancy
• Malignancy – transformation to well-diff HCC
• Higher in men - less related to size
• Higher in large adenoma in women
• Increased with Beta-catenin activation

Hepatocellular Adenoma

• Imaging: diffuse fat


in lesion – less
enhancement,
washout,
heterogeneous
• Biopsy – may not
always differentiate
from HCC

1002
Copyright © Oakstone Publishing, 2021. All Rights Reserved.

Hepatocellular Adenoma – Treatment

• Men – resect all adenoma


• Women – cease OCP, observe if < 5cm
• Growth off OCP - resect
• Women – resect if > 5cm
• Symptoms, bleeding - resect
• Consider more conservative approach if no Beta-
catenin mutation on biopsy?
• Role of transarterial embolization: bleeding,
preoperative with large lesions

Hepatocellular Adenoma s/p embolization

1003
Copyright © Oakstone Publishing, 2021. All Rights Reserved.

Hepatocellular Carcinoma (HCC)

• Underlying cirrhosis in 90% (HBV, HCV, EtOH, NAFLD)


• Characteristic imaging: arterial hypervascularization
and venous washout
• Biopsy often not needed, may benefit from non-
lesional biopsy to assess cirrhosis/fibrosis
• Diagnosis – mostly radiographic

Hepatocellular Carcinoma - Diagnosis

LI-RADS: Liver Reporting and Data System


Standardize reporting in patients at risk for HCC

1004
Copyright © Oakstone Publishing, 2021. All Rights Reserved.

HCC – Radiographic Diagnosis

HCC – Radiographic Diagnosis


3.6cm mass, arterial hyperenhancement, washout
capsule = LR5

1005
Copyright © Oakstone Publishing, 2021. All Rights Reserved.

Hepatocellular Carcinoma - Treatment

• Resection
• Transplantation
• Ablation
• Chemoemboliation (TACE, TARE)
• Systemic chemotherapy
• Immunotherapy

Hepatocellular Carcinoma - Treatment

• Treatment considerations: status of liver, #/size lesions


• Unifocal disease, limited cirrhosis = resection
• Multifocal disease, cirrhosis = transplant evaluation
• “Milan Criteria”:
(1) single tumor diameter >5 cm
(2) not more than three foci of tumor, each one not >3 cm
(3) no angioinvasion
(4) no extrahepatic involvement.

1006
Copyright © Oakstone Publishing, 2021. All Rights Reserved.

Intrahepatic Cholangiocarcinoma

• 2nd most common primary liver cancer behind HCC


• Risk Factors:
• Most: none
• Primary sclerosing cholangitis
• Parasitic infection – liver flukes (S.E. Asia)
• Hepatolithiasis/recurrent pyogenic cholangiohepatitis
• Congenital biliary cystic disease/choledochal cysts
• cirrhosis
• Often asymptomatic (jaundice if central)

Cholangiocarcinoma - Diagnosis

• Hypovascular mass on CT/MR


• Irregular, infiltrative, delayed enhancement
• Exclude metastasis
• Biopsy: CK-7, CK8/18 (+), CK-20 (-)
“pancreaticobiliary adenocarcinoma, primary or
metastatic”
• Define vascular/biliary relationships for treatment
planning

1007
Copyright © Oakstone Publishing, 2021. All Rights Reserved.

Cholangiocarcinoma – Treatment

• Hepatic resection: preferred treatment if margin-


negative resection (R0) is possible
• Determine: extent of vascular involvement, biliary
involvement, extent of surgery needed for margin-
negative (R0) resection
• Contraindications to surgery:
• Bilateral inflow/outflow
• Multifocal disease
• Metastases
• Insufficient liver reserve

Cholangiocarcinoma – Treatment

• Transplantation – limited role


• Ablation (RFA, microwave, cryoablation) – limited (size)
• Systemic chemotherapy
• Radiation/stereotactic body radiation therapy (SBRT)

1008
Copyright © Oakstone Publishing, 2021. All Rights Reserved.

Liver Metastases

Potentially liver-directed therapy


• Colorectal
• Neuroendocrine (small intestinal, pancreatic)

Limited/no role for liver-directed therapy outside


palliation:
• All others

Colorectal Liver Metastases

• 25% patients with CRC present with metastases


• Liver = most common (50% pts develop liver mets)
• Liver resection: proven survival benefit over
chemotherapy alone
• 5-yr OS rate 40-60% with resection in selected patients
(most not resectable)
• Can resection be performed? – primary obstacle is
size/health of future liver remnant (FLR)

1009
Copyright © Oakstone Publishing, 2021. All Rights Reserved.

Colorectal Liver Metastases

• Spread to liver may represent the only recognized


area of metastasis
• Surgical resection of metastases can improve
median survival to between 3 and 5 years if all
disease can be removed

Clinical Risk Score CRC Liver Metastases


Fong et al. JCO 15:938, 1997

**Clinical features inform biology and risk recurrence

• Lymph Node positive primary


• Disease-free interval between colon primary and
liver disease < 12 months
• Size of liver tumors > 5 cm
• Number of liver tumors > one
• Preoperative CEA level > 200 ng/ml

1010
Copyright © Oakstone Publishing, 2021. All Rights Reserved.

Relation of Clinical Risk Score to Outcome


Fong et al. JCO 15:938, 1997

Score 1-yr 5-yr Median


Survival Survival Survival
0 93% 60% 74 mos
1 91% 44% 51 mos
2 89% 40% 47 mos
3 86% 20% 33 mos
4 70% 25% 20 mos
5 71% 14% 22 mos

Colorectal Liver Metastases:


Criteria for resection
• No (or limited) disease outside liver Exclude
extrahepatic disease by PET scan
• Co-morbidity does not preclude major surgery
i.e. pre-existing heart or lung disease
• Need to be able to resect with negative margins
i.e. remove all disease (not most of disease)
• Sufficient liver reserve

1011
Copyright © Oakstone Publishing, 2021. All Rights Reserved.

Considerations: Colorectal Liver Metastases

• Systemic chemotherapy: neoadjuvant vs adjuvant


• Role of synchronous resection of primary tumor?
• Strategies with limited liver reserve
• Other liver-directed therapies: ablation (RFA,
microwave, cryo, IRE), TACE
• ? Role hepatic arterial infusion

Metastatic Colorectal Cancer


Considerations for preoperative chemotherapy
• Shrink tumor – achieve “resectability” of
large/invasive lesions or allow smaller resection
• Patient selection: rule out surgery for those with
spread on chemotherapy – allow other sites to
declare
• Earlier therapy for micrometastatic disease
• Guage chemoresponsiveness
• Response is prognostic factor

1012
Copyright © Oakstone Publishing, 2021. All Rights Reserved.

Metastatic Colorectal Cancer


Considerations for preoperative chemotherapy (2)
• Risk: chemotherapy-related liver damage
• May progress to unresectable status
• Response may hinder finding all sites –
“disappearing liver met”
• ? Increased postop complications

Neuroendocrine Liver Metastases

• Histology, site, size, metastasis, tumor biology are


major determinants of prognosis
• Often relatively indolent neoplasms with prolonged
survival, even in presence of metastatic disease
• Appropriate surgical goals:
• Resection for oncologic benefit (“curative intent”)
• Control/palliation of symptoms – mass effect and/or
systemic bioactive agents produce

1013
Copyright © Oakstone Publishing, 2021. All Rights Reserved.

Neuroendocrine Liver Metastases (NELM)


Local therapies to address metastatic burden

• Goals of therapy
• Oncologic benefit (though near-universal recurrence)
• Palliation (hormonal excess)
• “reset the clock” – delay disease progression
• Control of liver disease is a priority in advanced NET
• Options – “Locoregional therapy”
• Surgical resection
• Ablation
• Transarterial therapy (TACE/TARE…)
• Combinations/sequence of local options common

Neuroendocrine Liver Metastases

• Surgery
• Best reported outcomes
• Selected cases
• Ablation
• Limited disease, small # tumors
• Size limitation (< 3cm)
• Transarterial therapy
• Contraindications to surgery

1014
Copyright © Oakstone Publishing, 2021. All Rights Reserved.

649 NELM Patients – DFCI/BWH

Median Survival

• Surgery 160 m
• Ablation 123 m
• Embolization 66m
• Observation 38m

Metastatic NET to Right Liver

1015
Copyright © Oakstone Publishing, 2021. All Rights Reserved.

Metastatic NET to Right Liver


10 years later

September 2009: Mesenteric mass, 3 liver


lesions

1016
Copyright © Oakstone Publishing, 2021. All Rights Reserved.

10 years postop: 1cm Recurrence


(stable > 3 years)

Pt with metastatic NET to liver.


? Resection
• CT:
Large mass in segment
4b/5 liver, biopsy-proven
metastatic siNET

1017
Copyright © Oakstone Publishing, 2021. All Rights Reserved.

DOTATATE-PET prior to liver resection

• Gallium-68
DOTATATE-PET/CT:
Multiple avid lesions in
orbit, supraclavicular
and abdominal nodes,
bones

Surgery for NELM

• Best reported outcomes for locoregional options


• “Resectable” = can remove while preserving
sufficient functional liver remnant, with
limited/controlled disease outside liver– even if
staged operations, combined with ablation
• Recurrence common (expected?) – not failure!
• Symptomatic disease – may benefit from debulking
(i.e. entire tumor can not be removed)
• Role of debulking therapy for oncologic benefit
debated

1018
Copyright © Oakstone Publishing, 2021. All Rights Reserved.

Is Hepatectomy Possible?

Technical:
• Distribution of lesions relative to vasculature
• Preserve 2+ contiguous liver segments
• Preoperative liver function
• Volume of functional liver remnant (FLR)
• normal liver: FLR > 25% total liver volume sufficient
• Chronic liver disease w/o cirrhosis: FLR > 30% needed
• Cirrhosis without portal HTN: FLR > 40% needed

Is Hepatectomy Possible?

Strategies to increase R0 resection


• Neoadjuvant chemotherapy in most
• Staged approaches, surgery + ablation
• Interventions to increase size of liver remnant
• Preoperative dietary interventions
• Synchronous with colorectal surgery if feasible

1019
Copyright © Oakstone Publishing, 2021. All Rights Reserved.

Unresectable colorectal cancer metastasis


February 2012
Effacement IVC, Right/middle hepatic vein Extension to segment 4b, abut PV bifurcation

Reassessment of Resectability
June 2012: s/p 6 cycles FOLFOX/Avastin

1020
Copyright © Oakstone Publishing, 2021. All Rights Reserved.

“Resectability”

• Operations defined by tumor proximity to


inflow/outflow/ducts more than size
• Options:
• Hemihepatectomy (“lobe”)
• Trisegmentectomy
• Sectoral resection (left lateral, posterior right)
• Non-anatomic “wedge” resection
• Ablative procedures

1021
Copyright © Oakstone Publishing, 2021. All Rights Reserved.

Limits to Hepatectomy

• The “Future Liver Remnant” (FLR)

• Primary cause perioperative mortality: postoperative liver failure

• Minimal FLR:
• 25% in a normal liver
• >30% in the context of cholestasis/hepatitis/systemic chemotherapy
• >40% in Child A cirrhosis w/o portal hypertension

Management of Limited FLR

Strategies developed to increase FLR, enable


hepatectomy and R0 resection
• PVE: Portal Vein Embolization
• Increase FLR 40% in 3-8 weeks
• PVL: Portal Vein Ligation
• TSH: Two-Stage Hepatectomy
• ALPPS: Associating Liver Partition and Portal Vein
Ligation
• Rapid increase (7-10 days), increase in FLR 75%

1022
Copyright © Oakstone Publishing, 2021. All Rights Reserved.

Insufficient Future Liver Remnant:


Portal Vein Embolization
• Minimally invasive procedure used to increase
hepatic reserve to a safe threshold prior to major
hepatic resection in an attempt to minimize
postoperative complications (i.e., hepatic failure)
• Mechanism: redistributes portal blood flow away
from embolized liver, inducing ispilateral atrophy
and stimulating compensatory hypertrophy in
nonembolized areas, i.e. the future liver remnant
(FLR)

Portal Vein Embolization

• Increases the number of patients who are able to


undergo potentially curative major hepatic resection
• Decreases postoperative morbidity and mortality,
particularly in patients with underlying liver disease
• Response to PVE (i.e., presence or absence of
compensatory hypertrophy) indicates whether a
potential candidate is suitable for major hepatectomy

1023
Copyright © Oakstone Publishing, 2021. All Rights Reserved.

PVE: Contraindications/Risks
Contraindications:
• Poor surgical candidate
• Unikely to achieve negative margin/R0 resection
• Extrahepatic disease precludes curative treatment
• Child (C/B) status
• Severe portal hypertension, ascites
• Biliary obstruction

Major complications are uncommon (2-3%), but include:


• PVT, off-target embolization, infection, bile leak, hemobilia,
pseudoaneurysm, AV fistula, arterioportalshunt, liver
hematoma, pneumothorax

PVE: Technique

MC: percutaneous transhepatic approach (vs


transjugular/transsplenic)

Embolization agents include: polyvinyl alcohol, gel


foam, metallical spherical particles +/- coils

https://www.mskcc.org/cancer-care/patient-education/portal-vein-embolization

1024
Copyright © Oakstone Publishing, 2021. All Rights Reserved.

PVE: Time to growth of future remnant

PVE: Time to growth of future remnant

• Most commonly performed 3-6 weeks before a


planned extended R hepatectomy or lobectomy in a
pt with underlying liver dysfunction

1025
Copyright © Oakstone Publishing, 2021. All Rights Reserved.

PVE: Time to growth of future remnant

**Delay to increase in FLR: use of chemotherapy or


bevacizumab prior to resection is effective and does
not affect growth of FLR
Zorzi Ann Surg Onc 2008

Portal Vein Embolization: Axial View


Pre-PVE: Left (FLR)/TV: 643/2159cc Post-PVE: Left (FLR)/TV: 885/2252cc
= 30% = 39%

1026
Copyright © Oakstone Publishing, 2021. All Rights Reserved.

Portal Vein Embolization: Coronal View


Pre-PVE: Left (FLR)/TV: 643/2159cc Post-PVE: Left (FLR)/TV: 885/2252cc
= 30% = 39%

ALPPS

• “Associating Liver Partition and Portal Ligation”


• Novel method (described 2011) to stimulate rapid
growth of future liver remnant as part of staged
resection
• Enable resection beyond safe limit of 70% of liver
volume
• Rapid growth/interest in hepatobiliary community
• International registry – approximately 1000 cases

1027
Copyright © Oakstone Publishing, 2021. All Rights Reserved.

ALPPS

Stage 1:Exploration, right portal vein ligation, in situ splitting of liver tissue
to right of falciform. Preserves biliary and arterial structures
Stage 2: Extended right hepatectomy

ALPPS in Colorectal Cancer

• Expands treatment options in CRLM deemed


unresectable due to high tumor burden and low FLR
volume
• PVE: approx. 30% increase FLR in 4+ weeks; ? Risk
tumor growth
• Initial study: serious morbidity 68%, mortality rate 12%
• ALPPS registry CLRM: serous morbidity 32%, mortality
8%
• Robust long-term clinical and oncologic outcomes
lacking

1028
Copyright © Oakstone Publishing, 2021. All Rights Reserved.

ALPPS- PRO

• Rapid growth of FLR – avoid delays of PVE


• Increased growth of FLR relative to PVE/TSH
**47-93% increase FLR in 7-14 days
• Low rate of disease progression = increased rate of
completion to R0 resection

ALPPS - CON

• Significantly increased morbidity/mortality


• Initial report: morbidity 68%, in-hospital mortality 12%
• Refinement of techniques, improved pt selection:
recent reports morbidity 36%, mortality 0%
• Most deaths due to liver failure: volume of hypertrophy
does not equal function – may require longer delay
• Early Tumor Recurrence?
• No demonstrated improvement in outcomes

1029
Copyright © Oakstone Publishing, 2021. All Rights Reserved.

LIGRO trial
Sandstrom Ann Surg 2017

• What ideal approach for managing limited FLR in


patients undergoing surgery for colorectal liver
metastases (CRLM)?
• 100 patients CRLM with FLR < 30%
• PVE f/b resection vs ALPPS

LIGRO trial
-100 patients CRLM with FLR < 30% -
Sandstrom Ann Surg 2017
ALPPS PV

Ave % volume increase 68% 36%


% acceptable FLR day 7 87% 29%
Days 1st – 2nd intervention 11 43
Complications: 43% 43%
R0 Rate 77% 57%
90-day mortality 8.3% 6.1%
Negative margins 77% 57%
Recurrent liver met at 1 year 25% 25%

1030
Copyright © Oakstone Publishing, 2021. All Rights Reserved.

Extrahepatic Colorectal Metastases:


Contraindication to Hepatectomy?
• Lung
• 2nd most common site CRC mets: 10-15% pts
• Minimal morbidity: VATS
• Thermal ablation: mall, peripheral lesions (<3cm)
• 55% 5-y OS after liver/lung resection, chemo Brouquet JACS 2011
• Nodal
• Worse outcomes with periportal nodes (21% v 45% 5y OS)
• ? Survival benefit or prognotic information Nanji EJSO 2017

Extrahepatic Colorectal Metastases:


Contraindication to Hepatectomy?
• Lung
• 2nd most common site CRC mets: 10-15% pts
• Minimal morbidity: VATS
• Thermal ablation: mall, peripheral lesions (<3cm)
• 55% 5-y OS after liver/lung resection, chemo Brouquet JACS 2011
• Nodal
• Worse outcomes with periportal nodes (21% v 45% 5y OS)
• ? Survival benefit or prognotic information Nanji EJSO 2017

• Lung and limited periportal nodal disease do not represent


strict contraindications to hepatectomy IF R0 resection
anticipated.

1031
Copyright © Oakstone Publishing, 2021. All Rights Reserved.

Biliary Neoplasms

Many challenges of biliary neoplasms


• Definitive diagnosis often not possible preoperatively
• Extent of biliary involvement may be challenging to define
preoperatively
• Intramural tumor spread and duct
• Margins may be difficult to obtain
• Surgical planning challenging
• High rate of unresectability at hilum
• Assessment of resectability challenging intraoperatively
• Less response to neoadjuvant therapy

1032
Copyright © Oakstone Publishing, 2021. All Rights Reserved.

Hilar Cholangiocarcinoma
Bismuth-Corlette Classification

Unresectable Hilar Cholangiocarcinoma

• Main portal vein or arterial involvement not


amenable to reconstruction
• Unilateral segmental biliary extension (Bismuth
type IIIa/IIIb) with contralateral vascular
involvement
• Bilateral biliary involvement of secondary ducts
(Bismuth type IV)
• Inadequate future liver remnant
• Distant (N2) nodes: celiac, aortocaval

1033
Copyright © Oakstone Publishing, 2021. All Rights Reserved.

Extrahepatic Cholangiocarcinoma
Surgical Approach
• Remove extrahepatic bile duct, porta hepatis nodes
• May require pancreaticoduodenectomy if distal (Bismuth I)
• Ipsilateral hepatic lobe (unless Bismuth I)
• Roux-en-Y hepaticojejunostomy
• +/- Caudate lobectomy (involvement of caudate branch at
bifurcation in 40%)
• +/- extended hepatectomy for margins or for vascular
involvement - May need PVE

Cholangiocarcinoma: Surgical Approach

Hilar: will require


hepatectomy

Mid-duct: may avoid


pancreatic/liver resection

Distal: will require


Whipple

1034
Copyright © Oakstone Publishing, 2021. All Rights Reserved.

Gallbladder Cancer
• ~8,500 new cases diagnosed annually in U.S.
• More internationally (Chile, India, Japan)
• Diagnosed
• Late (30-50%), after symptomatic presentation
• Incidentally (50-70%), after routine cholecystectomy
• Cancer discovered in ~1 of every 100-150 cholecystectomy
specimens
• Associated with poor overall survival (5-yr, 5-10%)
• Prognosis following surgery improved, but variable (5-yr, 15-
100%)
• Depends on extent of resection and stage of disease

Gallbladder Cancer

• Strategy: T-stage dependent


• “extended cholecystectomy” or “radical
cholecystectomy”
• resect GB bed – partial segment 4b/5
• porta hepatis lymph node dissection
• +/- cystic duct resection
• bile duct resection - controversial

1035
Copyright © Oakstone Publishing, 2021. All Rights Reserved.

Liver Surgery: Challenges


• Hepatic Steatosis (fatty liver)

 Nonalcoholic fatty liver disease (NAFLD)


Liver manifestations of insulin resistance/obesity

 20-40% of general population US


 30-50% with hepatectomy for colorectal metastases

Increased steatosis with increased BMI

 65% with BMI 30-40


 85% with BMI > 40

Liver Surgery: Challenges


Steatohepatitis (NASH)
Severe/progressive NAFLD

- Steatosis
- Lobular inflammation
- Ballooning hepatocytes
- also proportional to BMI
- associated with Irinotecan-based chemotherapy
- up to 20% patients undergoing liver resection

1036
Copyright © Oakstone Publishing, 2021. All Rights Reserved.

Normal Liver

Fatty liver
• Firm
• Less mobile –
parenchymal fractures,
traction injury
• Challenge to expose
blood vessels, bile ducts
• Increased risk
complications

1037
Copyright © Oakstone Publishing, 2021. All Rights Reserved.

Steatosis is risk factor for complications


after major hepatectomy
• 58 patients with
steatotic livers
undergoing major
hepatectomy (Swiss
study, transplant ctr)
• Matched 1:1 with
normal livers
• Increased EBL, units
transfused in steatotic
livers
McCormack et al. Ann Surg
2007

Steatosis is risk factor for complications


after major hepatectomy
• 58 patients with steatotic
livers undergoing major
hepatectomy
• Matched 1:1 with normal
livers
• Increased rate of major
complications in steatotic
livers

McCormack et al. Ann Surg 2007

1038
Copyright © Oakstone Publishing, 2021. All Rights Reserved.

Reduction in steatosis with short-term


diet modification
• Retrospective review 111 liver
resection patients, 51 with 1-
week diet, 60 control
• No difference in BMI, surgery
• Blinded pathology review:
steatosis as % hepatocytes with
lipid accumulation,
steatohepatitis score

Reeves et al. Surgery 2013

Reduction in steatohepatitis with short-


term diet modification
• Retrospective review 111 liver
resection patients, 51 with 1-
week diet, 60 control
• No difference in BMI, surgery
• Blinded pathology review:
steatosis as % hepatocytes with
lipid accumulation,
steatohepatitis score

Reeves et al. Surgery 2013

1039
Copyright © Oakstone Publishing, 2021. All Rights Reserved.

Randomized Trial: +/- short-term restricted


diet in patients undergoing liver resection

• Previous cohort study: decreased blood loss and


steatosis on restricted diet
• Goal: prospective RCT of low-calorie, low-fat diet on
blood loss, ease of surgery, pathologic correlates
• Challenges to RCT: heterogeneity in techniques, tumors,
blood loss/case
Barth et al. Ann Surg 2018

RCT: +/- Control Diet prior to hepatectomy


Diet Arm: 5 Units Optifast Diet
800 kcal, 20g Fat, 70g protein, 100g carbo
94% compliance

Control: No specified diet. Strict food diary

Barth et al. Ann Surg 2018

1040
Copyright © Oakstone Publishing, 2021. All Rights Reserved.

Control Diet
Kcal/d 1991 805
Fat g/d 84 19
Prot g/d 83 70

Barth et al. Ann Surg 2018

Outcomes • Stopped at interim


analysis after 60
patients

• Liver easier to
manipulate

• No differences in
liver function

• No differences in
complications

Barth et al. Ann Surg 2018

1041
Copyright © Oakstone Publishing, 2021. All Rights Reserved.

Primary and Secondary Liver


Neoplasms
Thomas Clancy MD

Division of Surgical Oncology


Brigham and Women’s Hospital
Harvard Medical School

1042
Copyright © Oakstone Publishing, 2021. All Rights Reserved.

Pancreatic Neoplasms

Mark Fairweather, M.D.


Associate Surgeon
Division of Surgical Oncology
Brigham and Women’s Hospital/Dana-Farber Cancer Institute
Assistant Professor of Surgery
Harvard Medical School, Boston, MA

• I have no disclosures

1043
Copyright © Oakstone Publishing, 2021. All Rights Reserved.

Outline
• Solid pancreatic neoplasms
• Pancreatic adenocarcinoma*
• Pancreatic neuroendocrine tumors*
• Acinar cell carcinoma
• Lymphoma
• Autoimmune pancreatitis (AIP)
• Metastases
• Solid Pseudopapillary Tumor (SPT)

• Cystic neoplasms of the pancreas

Pancreatic adenocarcinoma:
Epidemiology
• >60,000 new cases anticipated in 2021
• Third leading cause of cancer-related death
• Projected to become second-leading cause of cancer death
by 2030

• Accounts for 90% of pancreatic neoplasms


• 5-year overall survival 10%
• 70% involve pancreatic head

1044
Copyright © Oakstone Publishing, 2021. All Rights Reserved.

Pancreatic adenocarcinoma:
Risk factors
• Tobacco smoking
• Alcohol
• Chronic pancreatitis
• Obesity
• Up to 10% associated with germline mutation
• BRCA (1<2)
• ATM
• Mismatch repair deficiency genes (MLH1, MSH2, MSH6,
PMS2)

Pancreatic adenocarcinoma:
Presentation
• New Jaundice
• Unexplained abdominal/lumbar back pain
• New dyspepsia
• New onset or worsening diabetes
• Unexplained or “idiopathic” pancreatitis
• Steatorrhea
• Weight loss/anorexia

1045
Copyright © Oakstone Publishing, 2021. All Rights Reserved.

Pancreatic adenocarcinoma:
Presentation contd.
• At time of presentation:
• 50% - metastatic disease
• 30-35% - locally advanced/unresectable disease due to
vascular involvement
• 10-15% - localized diseased/potentially resectable

• Common sites of metastatic disease


• Liver (90%)
• Lung (25%)
• Lymph nodes (25%)
• Peritoneum (20%)

Pancreatic adenocarcinoma:
Diagnosis and Evaluation
• Contrast-enhanced CT imaging of abdomen
• CT scan of chest
• +/- MRI abdomen
• +/- PET scan
• Full labs with tumors markers (CA 19-9)
• Endoscopic ultrasound/ERCP

1046
Copyright © Oakstone Publishing, 2021. All Rights Reserved.

Anatomy of the Pancreas

Imaging: Staging
• Metastases
• Liver/lung

• Lymph nodes
• Locoregional
• Distant/retroperitoneal

1047
Copyright © Oakstone Publishing, 2021. All Rights Reserved.

Imaging: Pancreas protocol CT


• Multiphase Spiral CT
• Early arterial phase: arterial anatomy
• Late arterial phase: venous anatomy
• Portal venous phase: pancreatic parenchyma
• Sensitivity 90% for detection
• Specificity lower (chronic pancreatitis)
• Prediction of resectability 85%
• Best to assess vascular involvement

Imaging: Anatomic variants


Anatomic variants

Accessory right hepatic artery Replaced right hepatic artery

1048
Copyright © Oakstone Publishing, 2021. All Rights Reserved.

Imaging: Pancreas protocol CT


• Pancreatic cancer
• Hypodense/infiltrative

• Pancreatic neuroendocrine tumor


• Arterial hyperenhancement

Imaging: MRI
• Some variability in protocols
• Superior soft tissue visualization
• Inferior to CT for vascular invasion
• Valuable to trouble-shoot
• < 2cm tumor
• Iso-attenuating tumor
• Focal fat
• Best for evaluation of liver

1049
Copyright © Oakstone Publishing, 2021. All Rights Reserved.

Imaging: PET
• Lower sensitivity (<75%)
• Not routinely used for staging
• Potential role with presumed metastatic disease or
indeterminate findings

Imaging: Solid tumor mimics


• Chronic pancreatitis
• Ill-defined, hypodensity, Ca++
• MRI, EUS useful
• Autoimmunie pancreatitis
• Typical “sausage-like”
• Can be focal, discrete
• Serum IgG4, steroid trial
• Metastases: RCC, lung, sarcoma (rare)

1050
Copyright © Oakstone Publishing, 2021. All Rights Reserved.

Solid Tumor?

CT 6/2013: MRI 9/2013.


“unresectable tumor” Prednisone 40mg QD for 4 weeks

Endoscopic Ultrasound (EUS)


• Advantage
• Better identification of small lesions < 2cm
• Assess vascular interface

1051
Copyright © Oakstone Publishing, 2021. All Rights Reserved.

Endoscopic Ultrasound (EUS)


• Tissue diagnosis – Fine needle biopsy
• Sensitivity > 90%
• Specificity > 95%
• Sample regional nodes

Endoscopic Retrograde
Cholangiopancreatography (ERCP)
• Primary role: therapeutic (biliary obstruction)
• Diagnosis: Imaging

• Diagnosis: brushings
• Sensitivity 35-70% (specificity >90%)

1052
Copyright © Oakstone Publishing, 2021. All Rights Reserved.

ERCP: Biliary stenting


• Routine stenting controversial
• Risk of complications: procedure related, infectious
• Indications:
• Cholangitis
• Prolonged jaundice, tbilii > 20, symptoms
• Delay to surgery
• Liver dysfunction/coagulopathy
• Prior to neoadjuvant chemotherapy

“Resectability”
? Involvement of local vessels

1053
Copyright © Oakstone Publishing, 2021. All Rights Reserved.

Resectable pancreatic head tumor

Adjuvant
Surgery
chemotherapy
Resectable
PDAC
Neoadjuvant
Surgery
chemotherapy

Borderline resectable disease


Tumors “for which resection, while possible, would likely be compromised
by positive margins in the absence of preoperative therapy.”

Resectable vs “borderline”: Need evaluation by pancreatic surgeon

Katz et al. Ann Surg Oncol. 2013;20:2787.

1054
Copyright © Oakstone Publishing, 2021. All Rights Reserved.

Borderline resectable pancreatic


head tumor

Borderline resectable: SMV


involvement

1055
Copyright © Oakstone Publishing, 2021. All Rights Reserved.

Locally advanced pancreatic


cancer (unresectable)
• SMA encased > 180 degrees
• Celiac axis encased > 180 degrees
• PV/SMV involvement precludes reconstruction
• Aorta, IVC involvement

Definition of “resectability” - requires evaluation by


experienced pancreatic surgeon

Locally advanced pancreatic head


tumor

1056
Copyright © Oakstone Publishing, 2021. All Rights Reserved.

Surgery for pancreatic tumors


Pancreatic Head (“right side”)
Pancreaticoduodenectomy (Whipple)
- Duodenum (shares blood supply with pancreas)
- Distal bile duct
- Head of pancreas

Pancreatic body/tail (“left side”)


Distal/subtotal pancreatectomy
- with splenectomy

Whipple procedure
(Pancreaticoduodenectomy)

1057
Copyright © Oakstone Publishing, 2021. All Rights Reserved.

Volume-outcome relationship

Birkmeyer et al., NEJM 2003

Pancreatic surgery: controversies

• Diagnostic laparoscopy
• Occult metastases: seen less with current imaging
• Selective use: > 3cm tumor, elevated CA 19-9,
body/tail lesions
• Pylorus preservation
• Several RCT: same gastric emptying/oncologic
outcomes

1058
Copyright © Oakstone Publishing, 2021. All Rights Reserved.

Pancreatic surgery: controversies

• Minimally invasive approaches


• Laparoscopic/robotic
• Distal body/tail masses
• Routine
• Pancreatic head (whipple)
• Safe in highly selected series
• ? overall benefit

Pancreatic neuroendocrine tumors:


Epidemiology
• Distinct biologic behavior and clinical management
compared with adenocarcinoma
• Comprise 1-2% of pancreatic neoplasms
• Incidence of <1 case per 100,000 individuals per year
• Incidence increasing

• 70% of patients present with metastatic disease


• 5-year survival of 65%

1059
Copyright © Oakstone Publishing, 2021. All Rights Reserved.

Pancreatic neuroendocrine tumors:


Risk factors
• Limited association with smoking, alcohol, pancreatitis
• Inherited genetic syndromes:
• Neurofibromastosis type 1 (NF 1)
• Multiple endocrine neoplasia type 1 (MEN 1)
• Primary hyperparathyroidism
• Pituitary adenomas
• Pancreatic neuroendocrine tumors
• Von Hippel-Lindau syndrome (VHL)
• Retinal, cerebellar, spinal, adrenal, renal, pancreatic
lesions

Pancreatic neuroendocrine tumors:


Presentation
• Non-functional tumors-60-90%
• Often found incidentally or during evaluation of non-specific
symptoms

• Functional tumors
• Insulinoma
• Gastrinoma
• Glucagonoma
• VIPoma
• Somatostatinoma

1060
Copyright © Oakstone Publishing, 2021. All Rights Reserved.

Pancreatic neuroendocrine tumors:


Insulinomas
• Most common functioning tumor
• Occur in 1-4 people per million, 1-2% of all pancreatic
neoplasms
• 90%: benign, solitary, intrapancreatic, <2 cm
• Evenly distributed throughout pancreas
• Classic findings: Whipple’s triad
• Hypoglycemia (plasma glucose <50 mg/dL)
• Neuroglycopenic symptoms
• Resolution of symptoms with intake of glucose

Pancreatic neuroendocrine tumors:


Insulinomas cont.
• Diagnosis
• Monitored 72 hour fast
• Measure plasma glucose and insulin levels every 4-6 hrs
• Inappropriately high insulin level
• Serum C-peptide
• Screen for sulfonylurea agents

1061
Copyright © Oakstone Publishing, 2021. All Rights Reserved.

Pancreatic neuroendocrine tumors:


Insulinomas cont.
• Management
• Resection or enucleation
• No regional lymphadenectomy required

Pancreatic neuroendocrine tumors:


Gastrinoma
• Second most common functioning tumor
• Sporadic in 70-80%
• Associated with MEN1 in 20-30%

• Zollinger Ellison Syndrome


• Refractory peptic ulcer disease
• Abdominal pain
• Diarrhea
• >50% malignant

1062
Copyright © Oakstone Publishing, 2021. All Rights Reserved.

Pancreatic neuroendocrine tumors:


Gastrinoma cont.
• Diagnosis
• Serum gastrin level >1000 pg/mL
• Gastric pH < 2.5
• Must stop proton pump inhibitors for 2 weeks
• Secretin stimulation test can be used if gastrin level if
equivocal
• Confirm diagnosis if gastrin level increases by >200
pg/mL

Pancreatic neuroendocrine tumors:


Gastrinoma cont.
• Localization
• Can be small and may be multiple
• 75% occur in the gastrinoma triangle
• Confluence of cystic duct and common bile duct
• Junction of 2nd/3rd portions of duodenum
• Junction of neck/body of pancreas
• 25% in pancreatic body and distal duodenum
• Duodenal location more common than pancreas

1063
Copyright © Oakstone Publishing, 2021. All Rights Reserved.

Pancreatic neuroendocrine tumors:


Gastrinoma cont.
• Management
• Surgical resection with regional lymphadenectomy
• Duodenotomy in order to rule out multifocal radiographic
occult disease

Pancreatic neuroendocrine tumors:


Glucagonoma
• Often large and solitary
• Malignant >80%
• >50% of patients present with metastatic disease
• Rarely associated with MEN1
• Often seen in patients with undiagnosed glucose
intolerance who develop necrolytic migratory erythema
that prompts evaluation

1064
Copyright © Oakstone Publishing, 2021. All Rights Reserved.

Pancreatic neuroendocrine tumors:


Glucagonoma cont.
• Diagnosis
• Serum glucagon >1000 pg/mL diagnostic
• Equivocal results can be confirmed with a secretin
stimulation test or skin biopsy of rash
• Management
• Distal pancreatectomy and splenectomy with regional
lymphadenectomy

Pancreatic neuroendocrine tumors:


VIPomas
• Verner-Morrison (or WDHA syndrome)
• Severe intermittent watery diarrhea
• Hypokalemia
• Achlorhydria
• Diagnosis
• VIP level >200 pg/mL (often >1000 pg/mL)
• Management
• Correct electrolyte abnormalities
• Subtotal/distal pancreatectomy with splenectomy and
regional lymphadenectomy

1065
Copyright © Oakstone Publishing, 2021. All Rights Reserved.

Pancreatic neuroendocrine tumors:


Somatostatinomas
• Often large at time of diagnosis
• >50% malignant
• 2/3rd located in the pancreas, most commonly in head
• Periampullary, duodenal, or small bowel

• Diagnosis
• Fasting serum somatostatinoma >100 pg/mL

• Management
• Often a whipple procedure with regional lymphadenectomy

Pancreatic neuroendocrine tumors:


Diagnosis and Evaluation
• CT contrast-enhanced imaging of abdomen
• CT scan of chest
• +/- MRI abdomen
• DOTATATE PET scan
• Full labs with tumors markers (Chromogranin A)
• Endoscopic ultrasound
• Ki-67 analysis

1066
Copyright © Oakstone Publishing, 2021. All Rights Reserved.

Pancreatic neuroendocrine tumors:


Evaluation
• Chromogranin A
• Most important tumor marker for pancreatic neuroendocrine
tumors
• Secretory glycoprotein found in neuroendocrine cell vesicles

• Ki-67
• Proliferation index determined histologically by calculating
number of positive cells among >2000 cells in >10
representative high-powered fields
• Categorized as <2 (low), 2-20 (intermediate), >20 (high)
• Independent predictor for local-regional and distant disease

Pancreatic neuroendocrine tumors:


Management
• Surgery is cornerstone of management
• Nonfunctional tumors ≤2 cm can often be observed
with active surveillance q6-12 months
• Functional tumors ≤2 cm can be considered for
enucleation
• Pancreatectomy with lymphadenectomy
• Neoadjuvant therapy can be considered for locally
advanced disease

1067
Copyright © Oakstone Publishing, 2021. All Rights Reserved.

Pancreatic neuroendocrine tumors:


Management cont.
• Surgery can play a role in patients with metastatic
disease
• Unclear role of debulking surgery outside of palliative
purposes

• Somatostatin analog therapy (Octreotide)


• Symptom control
• Disease control

• Systemic chemotherapy
• High grade/refractory tumors

Pancreatic cystic lesions


• Pseudocyst
• Serous cystadenoma
• Mucinous cystadenoma/mucinous cystic neoplasm
(MCN)
• Intraductal Papillary Mucinous Neoplasm (IPMN)
• Main duct
• Side branch
• Mixed duct
• Solid pseudopapillary neoplasms (SPN)
• Cystic adenocarcinoma
• Cystic neuroendocrine tumor

1068
Copyright © Oakstone Publishing, 2021. All Rights Reserved.

Pancreatic cystic neoplams:


Epidemiology
• Increased detection due to increased utilization of
imaging
• Present in up to 50% of patients in autopsy studies
• Most are incidentally discovered
• Malignant potential varies between the various types
• Surveillance vs. operative intervention based on
high-risk features

Pancreatic cystic neoplasms:


Mucinous cystic neoplasms
• Almost exclusively in middle-aged women
• More common in body and tail of pancreas
• Often present with symptoms typically due to their
large size
• Ovarian-type stroma lines the cyst walls
• Invasive carcinoma occurs in approximately 1/5th
• Increased risk with increased size and complexity

1069
Copyright © Oakstone Publishing, 2021. All Rights Reserved.

Pancreatic cystic neoplasms:


Intraductal papillary mucinous neoplasms
• Account for 1/3rd of all pancreatic cystic neoplasms
• Occur in patients 60-70 years old
• Slightly more common in men
• Up to 70% occur in the pancreatic head
• Three main subtypes:
• Main-duct IPMN
• Branch-duct IPMN
• Mixed-type

Pancreatic cystic neoplasms:


Intraductal papillary mucinous neoplasms
• Main-duct IPMN
• Diffuse or segmental dilation of the main pancreatic duct >5
mm without clear mass
• Account for 20-30% of all IPMNs
• 2/3rd found within pancreatic head
• 60% demonstrate high grade dysplasia
• 45% demonstrate invasive carcinoma

1070
Copyright © Oakstone Publishing, 2021. All Rights Reserved.

Pancreatic cystic neoplasms:


Intraductal papillary mucinous neoplasms
• Branch-duct IPMN
• Account for up to half of IPMNs
• Multifocal in 30-40%
• More indolent course and lower risk of malignant
transformation
• High-grade dysplasia 25%
• Invasive carcinoma 20%

Pancreatic cystic neoplasms:


Intraductal papillary mucinous neoplasms
• Mixed-type IPMN
• Involves both main duct and side branches
• 1/3rd of IPMNs
• Typically managed as a main-duct IPMN

1071
Copyright © Oakstone Publishing, 2021. All Rights Reserved.

Pancreatic cystic neoplasms:


Evaluation
• Contrast-enhanced CT imaging
• Pancreas protocol CT

• MRI/MRCP
• More sensitive than CT
• Less radiation exposure during surveillance

• Endoscopic ultrasound
• Investigate high risk features (i.e. mural nodule)
• Tissue biopsy
• Fluid analysis

IPMN-branch duct

Early Late

1072
Copyright © Oakstone Publishing, 2021. All Rights Reserved.

IPMN-main duct

Early Late

IPMN-main duct

1073
Copyright © Oakstone Publishing, 2021. All Rights Reserved.

Pancreatic cystic neoplasms:


EUS
• Fluid analysis via fine needle aspiration
• High CEA-suggests pancreatic mucinous cystic neoplasm
• MCN/IPMN
• High amylase-suggests communication with main pancreatic
duct
• IPMN/pseudocyst
• No standardized CEA cutoff
• Low cyst fluid CEA may not exclude a mucinous cyst

Pancreatic adenocarcinoma:
Key features

Van Huijgevoort et al., Nature Reviews 2019

1074
Copyright © Oakstone Publishing, 2021. All Rights Reserved.

Pancreatic cystic neoplasms:


Absolute surgical indications*
• Mucinous cystic neoplasms
• Cyst diameter ≥40 mm
• Enhancing mural nodule
• Jaundice
• Intraductal papillary mucinous neoplasms
• Positive cytology for malignancy or high-grade dysplasia
• Solid mass/enhancing mural nodule
• Jaundice
• Main pancreatic duct dilation ≥10 mm
*European Guidelines 2018, Gut 2018

Pancreatic cystic neoplasms:


Relative surgical indications*
• Intraductal papillary mucinous neoplasms
• Growth rate ≥5 mm per year
• Increased levels of serum CA 19-9 (>37)
• Pancreatic duct dilation between 5 and 9.9 mm
• Cyst diameter ≥40 mm
• New-onset diabetes mellitus
• Acute pancreatitis (caused by the cystic lesion)
• Enhancing mural nodule

*European Guidelines 2018, Gut 2018

1075
Copyright © Oakstone Publishing, 2021. All Rights Reserved.

Pancreatic cystic neoplasms:


Surveillance*
• Mucinous cystic neoplasms (<40 mm)
• EUS and/or MRI
• Every 6 months for 1 year then yearly
• Intraductal papillary mucinous neoplasms (<40 mm)
• EUS and/or MRI
• Every 6 months for 1 year then yearly
• Postoperative follow up required due to risk of recurrent
disease

*European Guidelines 2018, Gut 2018

Unusual pancreatic tumors:


Acinar cell carcinoma
• Accounts for <1% of pancreatic cancers
• Arises from acinar elements of the exocrine pancreas
• More common in males in 6th/7th decades of life
• Often very large and found in the head of pancreas
• Surgery is first line treatment in patients with
resectable disease
• Tend to be less sensitive to chemotherapy

• Better prognosis than pancreatic adenocarcinoma

1076
Copyright © Oakstone Publishing, 2021. All Rights Reserved.

Unusual pancreatic tumors:


Solid pseudopapillary tumors
• Account for 3% of all pancreatic cancers
• Occur almost exclusively in young women in 3rd
decade of life
• Often present with abdominal pain due to large size at
diagnosis
• Low malignant potential
• Surgical resection recommended
• 5 yr overall survival >90% with low risk of recurrence

Unusual pancreatic tumors:


Primary pancreatic lymphoma
• Accounts for <0.5% of pancreatic cancers
• B-cell non-Hodgkins lymphoma
• Diffuse, large cell lymphoma most common
• No involvement of lymph nodes outside of peri-
pancreatic lymph nodes
• More common in men in 5th/6th decade
• Treatment is chemotherapy alone (CHOP)
• Remission in 60-70% with treatment

1077
Copyright © Oakstone Publishing, 2021. All Rights Reserved.

Unusual pancreatic tumors:


Metastatic renal cell carcinoma
• Metastatic disease accounts for <2% of pancreatic
tumors
• Renal cell carcinoma metastases account for 40%
• Typically present after an extended disease-free interval
• No location predominance, solitary lesions most common
• Up to 1/3rd of patients with have extra-pancreatic disease
• Similar radiographic appearance to PNETs
• Surgical often recommended

Summary
• Surgery is 1st treatment of choice for most solid lesions
• CT most important to assess local invasion
• Decision re: “resectability” requires pancreatic
surgeon
• Major pancreatic surgery increasingly safe
• Evaluation/management specialized and
multidisciplinary
• Pancreatic cystic lesions common
• MRI/MRCP and EUS have primary role in diagnosis
• Most nonoperative in absence of worrisome features

1078
Copyright © Oakstone Publishing, 2021. All Rights Reserved.

Inflammatory
Pancreatic
Disease
Stan Ashley MD
Brigham and Women’s
Hospital;
Frank Sawyer Professor of
Surgery
Harvard Medical School

Conflicts of Interest

• No Disclosures

1079
Copyright © Oakstone Publishing, 2021. All Rights Reserved.

Outline

• Acute pancreatitis and its complications

• Chronic pancreatitis and its complications

Acute pancreatitis is the


most terrible of all the
calamities that occur in
connection with the
abdominal viscera.

Sir Berkely Moynihan


1925

1080
Copyright © Oakstone Publishing, 2021. All Rights Reserved.

Acute Pancreatitis (AP)


• 3rd most common GI diagnosis at admission in the US
• 275,000 admissions each year
• Cost 2.6 billion dollars
• Overall mortality 1-2%

Peery et al. Gastroentrol 2015

A 10 minute surgical discussion of acute pancreatitis should


probably include 9 minutes of silence…

C. Gardner Child
1970

1081
Copyright © Oakstone Publishing, 2021. All Rights Reserved.

Reclassification of Acute Pancreatitis


2012
● Interstitial edematous pancreatitis
Acute peripancreatic fluid collection
Pancreatic pseudocyst

● Necrotizing pancreatitis
Acute necrotic collection
Walled-off necrosis

Acute Pancreatitis
Natural History
Interstitial edematous pancreatitis (Mild acute) 80-85%
• Mortality rate 0%
• Resolves in 3 – 7 days with minimal therapy

Necrotizing pancreatitis (Severe acute) 15-20%


• Mortality rate 10% - 20%
• 30% of deaths within 72hrs
• 50% of deaths within early phase (2 weeks):
SIRS/multi-organ failure
• Remainder of deaths beyond this period: Persistent
organ failure/infected necrotizing pancreatitis

Yang A et al. Gastroenterology 2014

1082
Copyright © Oakstone Publishing, 2021. All Rights Reserved.

Never in medical history have


so many owed so much to a
single stone

Fitzgerald, on Opie’s
common channel theory

1083
Copyright © Oakstone Publishing, 2021. All Rights Reserved.

Acute Peripancreatic Fluid Collection

Pancreatic Pseudocyst

1084
Copyright © Oakstone Publishing, 2021. All Rights Reserved.

Pancreatic Pseudocysts
Intervention (Usually endoscopic)
• Symptomatic
• Rapidly enlarging
• Infected pseudocysts that do not improve
with medical management
EUS-guided transgastric or duodenal
drainage

Acute Pancreatitis
Natural History
Interstitial edematous pancreatitis (Mild acute) 80-85%
• Mortality rate 0%
• Resolves in 3 – 7 days with minimal therapy

Necrotizing pancreatitis (Severe acute) 15-20%


• Mortality rate 10% - 20%
• 30% of deaths within 72hrs
• 50% of deaths within early phase (2 weeks):
SIRS/multi-organ failure
• Remainder of deaths beyond this period: Persistent
organ failure/infected necrotizing pancreatitis

Yang A et al. Gastroenterology 2014

1085
Copyright © Oakstone Publishing, 2021. All Rights Reserved.

When Do Patients with AP Die?

Prognostic signs and the role of


operative management in acute
pancreatitis

Ranson JHC et al. Surg Gyn Obst 139,


1974

1086
Copyright © Oakstone Publishing, 2021. All Rights Reserved.

Ranson’s Criteria of Severity


At admission Initial 48 hours
• Age >55 years • Hct decrease of >10%
• WBC >16,000/mm3 • BUN increase of >5 mg/dl
• Glucose >200 mg/dl • Ca++ <8 mg/dl
• LDH >350 U/L • PaO2 <80 mm Hg
• AST >250 U/L • Base deficit >4 mEq/L
• Fluid sequestration >6 L

Morbidity of Acute Pancreatitis vs


Number of Positive Prognostic Signs

1087
Copyright © Oakstone Publishing, 2021. All Rights Reserved.

Management of Acute Pancreatitis


Treatment guidelines
• Assessment of severity (SIRS, organ failure,
markers of fluid sequestration, clinical
scoring systems)
• Supportive care – ED, ICU
• Aggressive fluid resuscitation
• Nutritional support (enteral or oral)
• No antibiotics
• Endoscopic sphincterotomy only for
cholangitis

Surgery for Acute Pancreatitis


…recovery from this
disease, apart from
operation, is so rare that
no case should be left
untreated…

Moynihan 1925

1088
Copyright © Oakstone Publishing, 2021. All Rights Reserved.

Severe Acute Pancreatitis


Indications for Operation?
• Diagnostic uncertainty

The variations of blood amylase during acute transient


disease of the pancreas

Elman R. Ann Surg 105: 379, 1937

1089
Copyright © Oakstone Publishing, 2021. All Rights Reserved.

Acute Necrotic Collection

Severe Acute Pancreatitis


Indications for Operation?
• Diagnostic uncertainty
• Intrabdominal emergency

1090
Copyright © Oakstone Publishing, 2021. All Rights Reserved.

1091
Copyright © Oakstone Publishing, 2021. All Rights Reserved.

Acute Pancreatitis
Indications for Operation?
• Diagnostic uncertainty
• Intrabdominal emergency
• Interrupt evolution

1092
Copyright © Oakstone Publishing, 2021. All Rights Reserved.

I have little doubt


that my operation
hastened the
death of the
patient.

Halsted WS. Bull J Hopkins Hosp 1901

Acosta JM.
Early surgery for acute
gallstone pancreatitis.
Surgery 1978

1093
Copyright © Oakstone Publishing, 2021. All Rights Reserved.

First description of ERCP

McCune WS et al.
Ann Surg 167, 1968

Early Endoscopic Sphincterotomy in


Biliary Pancreatitis
• 3 randomized trials – 450 patients
Neoptolemos et al. Lancet, 1988
Folsch et al N Engl J Med, 1997
Oria et al Ann Surg, 2007

• Meta-analysis
In patients without cholangitis no significant
reduction in overall complications or mortality
Petrov et al. Ann Surg 2008

1094
Copyright © Oakstone Publishing, 2021. All Rights Reserved.

Acute Pancreatitis
Indications for Operation?
• Diagnostic uncertainty
• Intrabdominal emergency
• Interrupt evolution

Acute Pancreatitis
Indications for Operation?
• Diagnostic uncertainty
• Intrabdominal emergency
• Interrupt evolution
• Early drainage/debridement

1095
Copyright © Oakstone Publishing, 2021. All Rights Reserved.

1096
Copyright © Oakstone Publishing, 2021. All Rights Reserved.

Pancreatic Debridement

1097
Copyright © Oakstone Publishing, 2021. All Rights Reserved.

1098
Copyright © Oakstone Publishing, 2021. All Rights Reserved.

1099
Copyright © Oakstone Publishing, 2021. All Rights Reserved.

Impact of Timing on Surgical Mortality

• Necrotizing pancreatitis – 167 patients


Overall mortality – 11%
Surgery before 28d – 20%
Surgery after 28d – 5% Rodriguez et al. Ann Surg 2008

• Necrotizing pancreatitis – 53 patients


Overall mortality – 36%
Surgery before 30d – 63%
Surgery after 30d – 8% Besselink et al. Arch Surg 2007

2.

1100
Copyright © Oakstone Publishing, 2021. All Rights Reserved.

Early Versus Late Necrosectomy in


Severe Necrotizing Pancreatitis

36 patients randomized to early (12 d) versus


late operation

• Trial abandoned due to higher mortality


in early (56%) versus late (27%)

• 20% of those randomized to late


improved without operation
Mier J et al. Am J Surg 2004

Time Course of Pancreatitis Progression

Interventional Window
Occurrence

0 12 24 36 48 60 72 84 90
Hours Since Onset of Pain
Denham W et al. Surgical Clinics North America 1999

1101
Copyright © Oakstone Publishing, 2021. All Rights Reserved.

Acute Pancreatitis
Indications for Operation?
• Diagnostic uncertainty
• Intrabdominal emergency
• Interrupt evolution
• Early drainage/debridement

Acute Pancreatitis
Indications for Operation?
• Diagnostic uncertainty
• Intrabdominal emergency
• Interrupt evolution
• Early drainage/debridement
• Pancreatic infection

1102
Copyright © Oakstone Publishing, 2021. All Rights Reserved.

Gerzof SG, Banks PA, et al. Early diagnosis of pancreatic


infection by computed tomography-guided aspiration

Gastroenterology 1987

1103
Copyright © Oakstone Publishing, 2021. All Rights Reserved.

Selective Debridement Only for Infected


Necrosis
• Ashley SW et al. Necrotizing pancreatitis: Contemporary analysis of 99
consecutive cases. Ann Surg 2001
• Bradley EL et al. A prospective longitudinal study of observation
versus surgical intervention in the management of necrotizing
pancreatitis Am J Surg 1991

• Büchler MW et al. Acute necrotizing pancreatitis: Treatment


strategy according to the status of infection Ann Surg 2000

• Ashley SW et al. Necrotizing pancreatitis: Contemporary analysis of


99 consecutive cases. Ann Surg 2001

of necrotizing pancreatitis Am J Surg

Selective Debridement for Infected


Necrosis
Conservative Strategy
(n = 93)

Operation for Gas 4 No FNA 35 FNA 54


Deaths 0 Deaths 2

Infection 30 No Infection 24
Deaths 4 Deaths 4

Operation for
Organized Necrosis
5

1104
Copyright © Oakstone Publishing, 2021. All Rights Reserved.

Walled-Off Necrosis

• Baron TH et al. Organized pancreatic necrosis: Endoscopic,


radiologic, and pathologic features of a distinct clinical entity.
Pancreas 1999

• Warshaw AL Editorial. Persistent unwellness. Ann Surg


2000

Walled-Off Necrosis

1105
Copyright © Oakstone Publishing, 2021. All Rights Reserved.

Operation For Organized Necrosis


BWH Series (1995-2000)

• Mean interval until operation (days) (range) 29 (23-34)


• Indications
Persistent pain 5
Inability to tolerate feeding 3
• Operations
Debridement 3
Debridement/cystogastrostomy 2
• Deaths 0

1106
Copyright © Oakstone Publishing, 2021. All Rights Reserved.

Debridement By Cystogastrostomy

Percutaneous Drainage of Infected


Necrosis

Freeney PC et al. Am J Roentgenol 1998

1107
Copyright © Oakstone Publishing, 2021. All Rights Reserved.

Impact of Radiologic Intervention on Mortality in


Necrotizing Pancreatitis

• Survived n = 12 (92%)
6 (46%) catheter necrosectomy only
6 (46%) catheter necrosectomy and surgery with a mean
delay 24 days (range: 5-120 days)

• Died n = 1 (8%) after surgery

Rocha FG et al. Arch Surg 2009

Treatment Options
• Percutaneous catheter drainage (PCD)
• Endoscopic transluminal drainage (ETD)
• Laparoscopic ( retroperitoneal, transperitoneal, transgastric)
• Sinus tract endoscopy ( nephroscope or flexible endoscope)
• Video-assisted retroperitoneal debridement (VARD)
• Endoscopic transluminal necrosectomy (ETN)

1108
Copyright © Oakstone Publishing, 2021. All Rights Reserved.

Step-Up Approach to Treatment of


Infected Necrosis
• Randomized prospective trial – 19 hospitals, 88 patients
• Primary open necrosectomy versus step-up approach
• Step-up approach
First – percutaneous or endoscopic drainage
If needed - minimally invasive retroperitoneal necrosectomy
• Advantages of step-up approach
35% did not require retroperitoneal necrosectomy
Less new-onset multiple organ failure
Less diabetes and less need for pancreatic enzymes
Less costly
• No significant difference mortality (19% vs 16%)
van Santvoort et al. NEJM 2010

Endoscopic versus Surgical Step-Up


Approach
• 19 hospitals, randomized study
• Endoscopic approach
First – transluminal drainage; if needed – endoscopic
necrosectomy
• Surgical approach
First – percutaneous catheter drainage; if needed – surgical
debridement
• Endpoints
Composite of complications or death by 6 months
• Results – 98 patients
No difference in mortality (18% vs 13%)
No difference in major complication
Endoscopic – fewer pancreatic fistulas and hernias, shorter
hospitalization
van Brunshot S et al. Lancet 2017

1109
Copyright © Oakstone Publishing, 2021. All Rights Reserved.

1110
Copyright © Oakstone Publishing, 2021. All Rights Reserved.

1111
Copyright © Oakstone Publishing, 2021. All Rights Reserved.

1112
Copyright © Oakstone Publishing, 2021. All Rights Reserved.

1113
Copyright © Oakstone Publishing, 2021. All Rights Reserved.

Indications for Surgery


Early
• Evidence of intra-abdominal emergency eg. intestinal
perforation or infarction
• Abdominal compartment syndrome
• Pseudoaneurysm/bleeding

Delayed
• Failure of endoscopic therapy
• Lack of available expertise

1114
Copyright © Oakstone Publishing, 2021. All Rights Reserved.

Algorithm for Debridement of WON

Fong ZV et al. J Gastroint Surg 2019

Immediate versus Postponed Drainage


for Infection
• Multicenter randomized trial comparing
immediate drainage when infection diagnosed
versus drainage after WON
• Comparable complications and mortality
• Immediate drainage required a mean of 4.4
interventions versus 2.6 in the postponed
drainage group
• 39% of postponed group never required
intervention
Boxhoorn L et al. NEJM 2021

1115
Copyright © Oakstone Publishing, 2021. All Rights Reserved.

Chronic Pancreatitis (CP)

• Chronic obstructive pancreatitis


• Chronic calcifying pancreatitis
• Chronic inflammatory pancreatitis

Marseilles-Rome Classification, 1988

1116
Copyright © Oakstone Publishing, 2021. All Rights Reserved.

TIGAR-0 Risk Factor Classification for Chronic


Pancreatitis
• Toxic • Autoimmune
eg. Alcohol, tobacco,
hypercalcemia,
hyperlipidemia, etc • Recurrent and severe acute
pancreatitis
• Idiopathic
• Obstructive
• Genetic eg. Pancreas divisum, SOD
dysfunction, etc
eg. CFTR, SPINK1, etc
Etemad B, Whitcomb DC. Gastroenterology
2001

Symptoms of Chronic Pancreatitis


• Pain
• Weight loss
• Steatorrhea
• Diabetes
• Biliary, intestinal
obstruction

1117
Copyright © Oakstone Publishing, 2021. All Rights Reserved.

Diagnosis of Chronic Pancreatitis

Diagnosis of Chronic Pancreatitis

1118
Copyright © Oakstone Publishing, 2021. All Rights Reserved.

Multidisciplinary Medical Management of Chronic


Pancreatitis
• Avoid alcohol, tobacco, fat
• Pancreatic enzyme supplementation
• Jejunal or parenteral feeding
• Antioxidants?
• Steroids/immunotherapy for autoimmune disease
• Pain management (NSAIDs, narcotics, gabapentin,
celiac plexus block)
• Stenting/lithotripsy

Endoscopic Management of Chronic Pancreatitis

Dite P et al. Endoscopy 2003; Cahnen DL et al. NEJM 2007

1119
Copyright © Oakstone Publishing, 2021. All Rights Reserved.

Surgical Management of Chronic Pancreatitis

• Lateral pancreaticojejunostomy (Puestow or


Partington-Rochelle procedure)
• Whipple procedure
• Duodenum-preserving pancreatic head resection
(Berne, Beger, Frey)
• Total pancreatectomy with islet autotransplantation

Lateral Pancreaticojejunostomy

1120
Copyright © Oakstone Publishing, 2021. All Rights Reserved.

Pancreaticoduodenectomy

Berne/Beger Procedure

1121
Copyright © Oakstone Publishing, 2021. All Rights Reserved.

Frey Procedure

Adapting the Surgical Management of Chronic


Pancreatitis
• Dilated duct, no head mass Lateral pancreaticojejunostomy
• Dilated duct, head mass, no BD Frey or Beger
• Nondilated duct, head mass, BD Berne or Beger
• Head mass, suspicion of Whipple
malignancy or small duct
disease
• Small duct disease, hereditary Total pancreatectomy
with islet autotransplantation

1122
Copyright © Oakstone Publishing, 2021. All Rights Reserved.

1123
Copyright © Oakstone Publishing, 2021. All Rights Reserved.

Surgical Diseases of the Spleen

Ali Tavakkoli, MD
Chief, Division of General and GI Surgery
Brigham & Women’s Hospital
Associate Professor of Surgery, Harvard Medical School

Conflict of Interest

AltrixBio: cofounder and Consultant

1124
Copyright © Oakstone Publishing, 2021. All Rights Reserved.

Spleen
• Has been an organ of mystery
• Used to think that it extracted melancholy from blood,
and purified it

Spleen

Anatomy:
• Beneath: 9 - 11th rib
• Measures: 9 - 11 cm
• Weights: 90 - 300 mg

• 5% of CO

1125
Copyright © Oakstone Publishing, 2021. All Rights Reserved.

Accessory Spleen
• Seen in 15-30% of cases
• Seen anywhere in peritoneal cavity
• Can be of varying size

Spleen

Anatomy:
• Blood supply via splenic artery. Two patterns:

Distributed: 70% Magistral: 30%

1126
Copyright © Oakstone Publishing, 2021. All Rights Reserved.

Spleen

Physiological role:

• Erythrocyte quality control – through Pitting and Culling


• Pooling – 30% of plat stored in spleen
• Hematopoiesis – in fetal life

Spleen

Physiological role:

• Erythrocyte quality control – through Pitting and Culling


• Pooling – 30% of plat stored in spleen
• Hematopoiesis – in fetal life
• Immunological –
- Filtration of encapsulated organisms by resident macrophages
- Antibody production

1127
Copyright © Oakstone Publishing, 2021. All Rights Reserved.

Spleen & Surgeons

Complicated relationship!
• Traumatic splenectomy most common
• Iatrogenic injury and splenectomy
• Indications for elective splenectomy confusing

Elective Splenectomy

1128
Copyright © Oakstone Publishing, 2021. All Rights Reserved.

Indications for Elective Splenectomy

Indications for Splenectomy

• Hematological disorders:
- Benign
- Malignant
- Autoimmune
• Tumors and cysts
• Miscellaneous disorders

1129
Copyright © Oakstone Publishing, 2021. All Rights Reserved.

Indications for Splenectomy

• Hematological disorders:
- Benign

Indications for Splenectomy

• Hereditary Hemolytic Anemias:


Membrane defects: Spherocytosis
- variable disease spectrum
- presents with anemia, jaundice and splenomegaly
- splenectomy indicated in severe cases after age 5

1130
Copyright © Oakstone Publishing, 2021. All Rights Reserved.

Indications for Splenectomy

• Hereditary Hemolytic Anemias:


Enzyme defects: G6PD, Pyruvate Kinase Deficiency
- variable presentation
- splenectomy indicated in transfusion dependent patients

Indications for Splenectomy

• Hereditary Hemolytic Anemias:


Hemoglobinopathies: Thalassemia ( β most common – Hb-F)
- defect in hemoglobin synthesis
- leads to increased red cell destruction
- splenectomy is not commonly indicated unless splenomegaly

1131
Copyright © Oakstone Publishing, 2021. All Rights Reserved.

Indications for Splenectomy

• Hereditary Hemolytic Anemias:


Hemoglobinopathies: Thalassemia ( β most common – Hb-F)
- defect in hemoglobin synthesis
- leads to increased red cell destruction
- splenectomy is not commonly indicated unless splenomegaly

• Sickle Cell Disease ( Hb-S )


- some have “autosplenectomy”

Crescent shaped RBC seen


in Sickle Cell Disese

Indications for Splenectomy

• Hematological disorders:

- Malignant

1132
Copyright © Oakstone Publishing, 2021. All Rights Reserved.

Indications for Splenectomy

Hematologic malignancies
• Extensive reclassification based on cytogenetics
• Multiple types and sub-types
• Tailored treatment with new targeted agents

Indications for Splenectomy

Hematologic malignancies
• Myeloid neoplasms:
- leukemias
- myelodysplastic syndrome
- myeloproliferative disorders

• Lymphoid neoplasms:
- Hodgkin’s lymphoma
- Non-Hodgkin’s lymphoma

1133
Copyright © Oakstone Publishing, 2021. All Rights Reserved.

Indications for Splenectomy

Hodgkin's disease
• Splenectomy was indicated in staging

Radiation Chemo + radiation

Indications for Splenectomy

Surgery for hematological malignancies:

–Treatment of splenomegaly
–Treatment of hypersplenism: cytopenia in the setting of splenomegaly
–Treatment or tissue diagnosis for localized disease to spleen:
Splenic Marginal Zone Lymphoma

1134
Copyright © Oakstone Publishing, 2021. All Rights Reserved.

Indications for Splenectomy

• Hematological disorders:

- Autoimmune

Indications for Splenectomy

Autoimmune Disorders:
• Idiopathic Thrombocytopenic Purpura (ITP)
- Common cause of elective splenectomy
- Acquired disorder in which platelets destroyed by antibodies
- Spleen is source of antibody production and red cell destruction
- 3:1 female to male presentation
- majority present with abnormal bleeding
- In children, but less so in adults, can be self-limiting

1135
Copyright © Oakstone Publishing, 2021. All Rights Reserved.

Indications for Splenectomy

Autoimmune Disorders:
• Idiopathic Thrombocytopenic Purpura (ITP)
– First line is steroids and IVIG
– Second line treatment options: Medications vs Splenectomy
– Rituximab: Good initial response in 40%
– Thrombopoietin Receptor (TPO-R) agonists increasing used as
second line
– Surgery in refractory cases or those needing chronic high dose
steroid

Indications for Splenectomy

Autoimmune Disorders:
• Idiopathic Thrombocytopenic Purpura (ITP)
– Overall response to surgery is 80%
– Lack of initial response can be due to missed splenic tissue
– Look at blood film for evidence of splenectomy: Howell-Jolly bodies
– If residual splenic tissue expected, image for and proceed with
removal
– 20% risk of relapse. Prognostic markers can include initial
response to steroids and IVIG

1136
Copyright © Oakstone Publishing, 2021. All Rights Reserved.

Indications for Splenectomy

Autoimmune Disorders:
• Autoimmune Hemolytic Anemia
– IgG mediated or IgM mediated
– Majority due to Warm antibodies
– Maybe associated with other lymphoproliferative disorders, e.g. CLL

Indications for Splenectomy

Autoimmune Disorders:
• Autoimmune Hemolytic Anemia
– Steroid first line therapy
– Second line therapy includes rituximab
– Splenectomy ONLY in warm agglutinin that is refractory to steroids

1137
Copyright © Oakstone Publishing, 2021. All Rights Reserved.

Indications for Splenectomy

• Tumors and cysts

Indications for Splenectomy

• Cysts:
- Can be true cysts or pseudocysts

- Pseudocysts:
• Do not have epithelial lining & majority of cysts in Western countries
• Secondary to trauma and resolution of hematomas
• Treat if large or symptomatic
• Options are splenectomy or laparoscopic deroofing
• Recurrence rates are 20-40% unless marsupialized

1138
Copyright © Oakstone Publishing, 2021. All Rights Reserved.

Indications for Splenectomy

• Cysts:
- True cysts can be parasitic
- Most common cause of splenic cysts worldwide are hydatid disease
- Hydatid cysts have and inner endocyst with an outer ectocyst
- Diagnosis confirmed by serology
- Care during splenectomy to avoid cyst rupture
- Can sterilize using 3% sodium chloride

8-cm cyst – diagnosis?

Indications for Splenectomy

• Tumors:
- Increasingly commonly seen during CT scans
- Most often small hemangiomas and do not require further action
- Primary malignant tumors are exceeding rare
- Metastatic tumors also seen but not common
- Splenectomy for tissue diagnosis maybe indicated

1139
Copyright © Oakstone Publishing, 2021. All Rights Reserved.

Indications for Splenectomy

• Miscellaneous disorders

Indications for Splenectomy

• Splenic Artery Aneurysms:


- More common in those with cirrhosis and portal hypertension
- Rupture in aneurysms of <2cm rare (unless pregnant)
- Treatment recommended for symptomatic or aneurysms of >2cm
- Endovascular approaches have been tried for proximal lesions
- Surgical approaches to proximal aneurysms includes proximal and
distal ligation
- Laparoscopic splenectomy recommended for distal aneurysms

1140
Copyright © Oakstone Publishing, 2021. All Rights Reserved.

Surgical Approaches

Surgical Approaches
• Open Splenectomy (OS)
• Laparoscopic Splenectomy (LS)
• Hand-Assisted Lap Splenectomy (HALS)
• Robotic-Assisted Splenectomy

1141
Copyright © Oakstone Publishing, 2021. All Rights Reserved.

Surgical Approaches
• Open Splenectomy (OS): Gold Standard
• Laparoscopic Splenectomy (LS)
• Hand-Assisted Lap Splenectomy (HALS)
• Robotic-Assisted Splenectomy

Open Splenectomy
• Midline incision
• Open lesser sac, control splenic artery above pancreas
• Proceed with mobilization

1142
Copyright © Oakstone Publishing, 2021. All Rights Reserved.

Laparoscopic Splenectomy
• First performed in 1991
Lateral Approach

Anterior Approach

Laparoscopic Splenectomy

1143
Copyright © Oakstone Publishing, 2021. All Rights Reserved.

Laparoscopic Splenectomy

Laparoscopic Splenectomy

1144
Copyright © Oakstone Publishing, 2021. All Rights Reserved.

Laparoscopic Splenectomy

Laparoscopic Splenectomy

1145
Copyright © Oakstone Publishing, 2021. All Rights Reserved.

Laparoscopic Splenectomy

Accessory Spleens

1146
Copyright © Oakstone Publishing, 2021. All Rights Reserved.

Laparoscopic vs. Open


Meta-analysis including a total of 2940 pt:
- LS longer operative time (180 min vs. 114 min)
However:
- Shorter hospital stay
- less respiratory, wound and infectious complications
- Similar retrieval rate for accessory spleen

Laparoscopic Splenectomy

Utilization Rate of Laparoscopic Splenectomy

Normal
- Using NSQIP dataset
- Compared rate of
laparoscopic approach
for different diagnosis Large
- ITP vs. splenomegaly

1147
Copyright © Oakstone Publishing, 2021. All Rights Reserved.

Laparoscopic Splenectomy

Encouraged wider utilization


of laparoscopic approach in
splenomegaly amongst
surgeons comfortable with
approach

Splenomegaly

Spleen Class Spleen size (cm) Spleen Weight (g)


Normal Up to 13 < 300
Mild splenomegaly >13-15 300-500
Moderate Splenomegaly 16-20 500-1000
Massive Splenomegaly >20 >1000

1148
Copyright © Oakstone Publishing, 2021. All Rights Reserved.

Splenomegaly

Spleen Class Spleen size (cm) Spleen Weight (g)


Normal Up to 13 < 300
Mild splenomegaly >13-15 300-500
Moderate Splenomegaly 16-20 500-1000
Massive Splenomegaly >20 >1000

Splenomegaly

• Review of 108 cases showed that LS in spleens


>1000 gm:
- Longer OR time
- Higher conversion rate (5% vs 18%)
- Higher re-operation rate
- 14x more likely to have a complication

1149
Copyright © Oakstone Publishing, 2021. All Rights Reserved.

491
elective
splenectomies

268
for
splenomegaly
(>500g)

66* 78*
Moderate Massive
splenomegaly splenomegaly
(500g-100g) (>1000g)

22 44 26 52
Laparoscopic Open Laparoscopic Open

*after 1:2 matching of LS:OS

Massive Splenomegaly
• Matched cohort with Massive Splenomegaly

Laparoscopic Splenectomy Open Splenectomy p

Number of patients 26 52

Age, years 58.5 (25-87) 60.6 (29.3-83) NS

Gender, F:M 17:9 38:14 NS

BMI, kg/m2 27.9 (20.4-39.3) 25.4 (18.8-39.9) NS

Splenic size, grams 1754.9 (1032-3800) 1755.6 (1034-3850) NS

Shin et al. Surg Endo (2019) 33:1298–


1303

1150
Copyright © Oakstone Publishing, 2021. All Rights Reserved.

Massive Splenomegaly
• Matched cohort with Massive Splenomegaly

Results Laparoscopic Splenectomy Open Splenectomy


n=26 n=52 p

Operative time, minutes 170.8 (70-291) 112.1 (45-322) <0.01*

Intraoperative blood loss, ml 606.6 (5-2400) 588.5 (5-9500) NS

Length of hospital stay, days 4.6 (2-12) 5.7 (3-27) NS

Postoperative morbidity 11 (42%) 14 (27%) NS

Infection 7 (27%) 6 (12%) NS

Thromboembolism 5 (19%) 4 (8%) NS

Reoperation 0 (0%) 1 (2%) NS

Readmission 7 (27%) 3 (6%) <0.05*

Death 0 (0%) 1 (2%) NS


Shin et al. Surg Endo (2019) 33:1298–
1303

Massive Splenomegaly
• Matched cohort with Massive Splenomegaly

Results Laparoscopic Splenectomy Open Splenectomy


n=26 n=52 p

Operative time, minutes 170.8 (70-291) 112.1 (45-322) <0.01*

Intraoperative blood loss, ml 606.6 (5-2400) 588.5 (5-9500) NS

Length of hospital stay, days 4.6 (2-12) 5.7 (3-27) NS

Postoperative morbidity 11 (42%) 14 (27%) NS

Infection 7 (27%) 6 (12%) NS

Thromboembolism 5 (19%) 4 (8%) NS

Reoperation 0 (0%) 1 (2%) NS

Readmission 7 (27%) 3 (6%) <0.05*

Death 0 (0%) 1 (2%) NS


Shin et al. Surg Endo (2019) 33:1298–
1303

1151
Copyright © Oakstone Publishing, 2021. All Rights Reserved.

Massive Splenomegaly

Although LS may be a feasible approach in moderate


and massive splenomegaly, potential advantages
require further studies

Shin et al. Surg Endo (2019) 33:1298–


1303

Other Surgical Considerations


• Pre-operative Splenic Artery Embolization:
 Not helpful in spleen sizes <20cm
 Controversial in larger spleens
 May lead to reduced blood loss but at expense of increased
risk of an additive procedure

1152
Copyright © Oakstone Publishing, 2021. All Rights Reserved.

Hand-Assisted Laparoscopic Splenectomy

• Can be used for:


- Large spleens
- Splenic Tumors
• Studies have documented safety and faster
recovery compared to open splenectomy

Spleen-Preserving Approaches

• Given important immunological role of spleen, there is


increasing interest in spleen-preserving approaches
• Particularly important tin children following trauma but also
elective procedures
• Partial splenectomy has been reported but remains
uncommon

1153
Copyright © Oakstone Publishing, 2021. All Rights Reserved.

Surgical Complications

Complications
• Bleeding
• Infection
• Iatrogenic organ injury including pancreas

• Post-Splenectomy Infection
• Post-operative Splenoportal vein thrombosis

1154
Copyright © Oakstone Publishing, 2021. All Rights Reserved.

Complications
• Post-Splenectomy Infection
- Overwhelming Post-Splenectomy Infections (OPSI)
- Initial studies suggested a 3% risk with 1.5% mortality rate
- Some of the risk is however due to underlying condition

Complications
• Post-Splenectomy Infection
- Those with asplenic state have a small life long higher risk of getting
serious infections from certain organisms
- These include pneumococcal, meningococcal, haemophilus and few
other less common pathogens
- Strategies to minimize this small risk include pre-operative
vaccination

1155
Copyright © Oakstone Publishing, 2021. All Rights Reserved.

Pre-Operative Vaccination
- Pneumococcal: 13-valent pneumococcal conjugate vaccine (PCV13) followed by the
23-valent pneumococcal polysaccharide vaccine (PPSV23) ≥8
weeks later
- Haemophilus: H. influenzae type b vaccine (Hib)
- Meningococcal: quadrivalent meningococcal conjugate ACWY vaccine series
(MenACWY); the monovalent meningococcal serogroup B vaccine
series (MenB-4C or MenB-FHbp)

Complications
• Post-Splenectomy Infection
- In young children and immunocompromised, daily antibiotic use in
the first year recommended by some
- Advice patients to seek medical help if fever >101
- Provide patients with course of antibiotics at home

1156
Copyright © Oakstone Publishing, 2021. All Rights Reserved.

Complications
• Post-Operative Splenoportal Vein Thrombosis

Complications
• Post-Operative Splenoportal Vein Thrombosis
- More common after laparoscopic approach
- Most often asymptomatic
- Symptomatic rate is 3-5% and equivalent between the 2 techniques
- Other risk factors also include splenomegaly, underlying
lymphoproliferative disorder or hemolytic anemias

- Treated with anticoagulation


- Post-operative platelet count as predictor?
- Prescribe ASA if platelet count is over 650K

Szasz et al. Surg Endo 34, 2191–2196 (2020)

1157
Copyright © Oakstone Publishing, 2021. All Rights Reserved.

Thank You!

Spleen

Anatomy:
• Red pulp (75%)
• White pulp

1158
Copyright © Oakstone Publishing, 2021. All Rights Reserved.

BWH experience

Open Laparoscopic p-value


Mean Age 55.0 48.5 < 0.05
Sex M: 101 F: 83 M: 35 F: 43 NS
Mean BMI 25.7 26.3 NS
Splenic weight 1.27 kg .32 kg < 0.05
Mean OR time 108 min 147 min < 0.05
Blood Loss 593 cc 356 cc < 0.05
Length of Stay 5.7 days 3.8 days < 0.05
Complication 14.7% 3.9% < 0.05

1159
Copyright © Oakstone Publishing, 2021. All Rights Reserved.

Benign and
Malignant
Neoplasms of the
Esophagus
M. Blair Marshall, MD
Michael A. Bell Family Distinguished Chair in Healthcare Innovation
Vice-Chief for Quality, Promotions Mentorship and Inclusion
Brigham and Women’s Hospital;
Associate Professor of Surgery
Harvard Medical School

Disclosures
• Consultant Ethicon
• Grant funding, research Xi surgical robot-Intuitive

1160
Copyright © Oakstone Publishing, 2021. All Rights Reserved.

Objectives
• Review diagnostic and management strategies for
benign tumors of the esophagus
• Review diagnostic and management strategies for
malignant tumors of the esophagus
• Highlight complications associated with
esophagectomy and management strategies

Classification

Surg Clin N Am 95 (2015) 491–514

1161
Copyright © Oakstone Publishing, 2021. All Rights Reserved.

Hyper plastic polyp Leiomyoma

World J Gastroenterol. 2015 Jan 28; 21(4): 1091–1098

Incidence and Presentation


• Overall incidence rare- roughly 0.5%
• Esophageal cancer 50 times more common (and
growing)
• 50% patients are asymptomatic-dx on endoscopy
for other reasons or other imaging modality
• Symptoms-dysphagia most common, pain less
common, respiratory symptoms

1162
Copyright © Oakstone Publishing, 2021. All Rights Reserved.

Diagnostic Work-up
• Contrast swallow/esophagram
• EGD and EUS
• Consider FNA
• CT with oral contrast
• Evaluate relationship to other structures
• Operative planning

http://www.revistagastroenterologiamexico.org/en-endoscopic-resection-giant-
esophageal-leiomyoma-articulo-S2255534X15000353

Biopsy?
• Pro-Benign versus malignant
• GIST versus benign leiomyoma

• Con-
• Results in scar leading to increased risk of perforation
during enucleation
• Sampling errors Surg Clin N Am 95 (2015) 491–514

• Contaminate? Duplication cysts

1163
Copyright © Oakstone Publishing, 2021. All Rights Reserved.

Leiomyoma
• Found throughout GI Tract
• 10% in esophagus
• Majority localized, 10% circumferential
• Distal 2/3 of esophagus
• 80% found 2nd to 6th decades and more common in
males (2:1)
• Historically considered part of the spectrum of GIST,
recent studies show they are distinct
• Leiomyoma (+) for desmin, SMA, (-) for CD 117 and CD 34
• GISTs (+) for CD 117 (c-kit protein) and CD34

Adv Anat Pathol. 2007 May;14(3):178-88. doi: 10.1097/PAP.0b013e318050aa66

Management
• Observation
• Majority low risk, asymptomatic
• Malignant transformation in adenomas, GISTs,
occasionally schwannomas
• Resection for symptomatic lesions or those greater
than 4cm
• Enucleation
• VATS, Robotic
• Laparoscopic trans-gastric for distal lesions
• Open approach-acceptable if expertise lacking
• Endoscopic- EMR and ESD
https://doi.org/10.1016/j.jamcollsurg.2003.08.015.

1164
Copyright © Oakstone Publishing, 2021. All Rights Reserved.

1165
Copyright © Oakstone Publishing, 2021. All Rights Reserved.

1166
Copyright © Oakstone Publishing, 2021. All Rights Reserved.

Jon Wee, MD

Alternative Approach
•GE junction is a complex anatomic region
•Tumors in this location typically require
resection of the GE junction
• Significant short- and long-term morbidity
•GE Junction sparring approach
•Remove the tumor
•Preserve the LES, Vagi
•Minimal morbidity

1167
Copyright © Oakstone Publishing, 2021. All Rights Reserved.

Combination Techniques for Complicated Lesions


of the GEJ

CTSnet: https://www.ctsnet.org/article/laparoscopic-
intra-gastric-resection-gastro-esophageal-leiomyoma
Marshall JTCVS 2015

Villano Ann Thorac Surg 2019

Combination Laparoscopic/Endoscopic
Techniques for Resection of Complex Gastric
Lesions

World J Gastroenterol. 2015 Nov 21;21(43):12482-97. doi:


10.3748/wjg.v21.i43.12482. PMID: 26604655; PMCID: PMC4649131.

1168
Copyright © Oakstone Publishing, 2021. All Rights Reserved.

World J Gastroenterol. 2015 Nov 21;21(43):12482-97.


doi: 10.3748/wjg.v21.i43.12482. PMID: 26604655; PMCID: PMC4649131

Combination Laparoscopic/Endoscopic Techniques

World J Gastroenterol. 2015 Nov 21;21(43):12482-97. doi:


10.3748/wjg.v21.i43.12482. PMID: 26604655; PMCID:
PMC4649131

1169
Copyright © Oakstone Publishing, 2021. All Rights Reserved.

Malignant Tumors of the Esophagus


• Esophageal cancer 8th most common cancer and
sixth most common cause of death worldwide
• Adenocarcinoma
• Frequency has increased dramatically over the last 3
decades
• Obesity, reflux, smoking
• White males 6:1
• Squamous cell cancer (SCC)
• Smoking and ETOH
• HPV-22% of SCC

Diagnosis and Staging


• Endoscopy-biopsy
• EUS-staging criteria
• 30/30/30
• CT with contrast
• PET
• Pre-induction
• Post-induction
• Brain imaging-routine not cost effective unless
clinical suspicion

1170
Copyright © Oakstone Publishing, 2021. All Rights Reserved.

AJCC Cancer Staging Manual, Eighth Edition (2017) Springer International Publishing.
2018.

Clinical Stage

1171
Copyright © Oakstone Publishing, 2021. All Rights Reserved.

AJCC 8th Edition: Represents a change in the staging system from away from
location of nodal disease to number of lymph nodes in volved

AJCC Cancer Staging Manual, Eighth Edition (2017) Springer International Publishing.
2018

AJCC Cancer Staging Manual, Eighth Edition (2017) Springer International Publishing.
2018

1172
Copyright © Oakstone Publishing, 2021. All Rights Reserved.

Clinical Stage

AJCC Cancer Staging Manual, Eighth Edition (2017) Springer International Publishing.
2018

Pathologic Stage

1173
Copyright © Oakstone Publishing, 2021. All Rights Reserved.

Post treatment Pathologic Stage

AJCC Cancer Staging Manual, Eighth Edition (2017) Springer International Publishing.
2018

Management of Esophageal
Cancer
• Early stage
• T1A-
• Invasion into the mucosa
• Endoscopic management-EMR
• T1B-
• Invasion into sub-mucosa
• 22-26% will have lymph node metastases
• EMR or ESD considered
• Risk factors to stratify patients at risk of nodal metastases
• Tumor > 2cm
• Poor differentiation, lymphovascular invasion, invasion
beyond 500 micrometers
• Patients with multiple risk factors have up to 50% risk of
LN metastases

1174
Copyright © Oakstone Publishing, 2021. All Rights Reserved.

Advanced Esophageal Cancer


• Neoadjuvant concurrent chemoradiation therapy
• CROSS regimen-Carbo Taxol
• Targeted and immuno-therapies are playing a greater
roll- currently under investigation
• PD-L1, HER-2, VEGFR, EGFR, among others
• Esophagectomy
• Minimally invasive
• Robotic
• Open

Controversies in the management of


esophageal cancer
• Histology
• Location-GEJ-esophageal or gastric?
• Neoadjuvant? Chemo versus chemo/XRT
• Immunotherapy and targeted therapy

World J Clin Oncol. 2021 Jul 24;12(7):557-564.

1175
Copyright © Oakstone Publishing, 2021. All Rights Reserved.

Esophagectomy
• Surgical technique varies
• Anatomic regions for the conduct of the operation
• Abdominal phase-used for conduit preparation and
hiatal dissection
• Celiac lymph node dissection
• Left neck-esophageal dissection
• ? anastomosis
• Right chest-esophageal dissection
• +/- anastomosis
• Mediastinal lymph node dissection
• Left Chest
• Dissection, rarely anastomosis

Postoperative Complications
• Varies widely from 20-80%
• Risk factors associated with increased
complications:
• Increased age, poor pulmonary function, malnutrition,
hepatic dysfunction, emergency surgery
• Mortality
• In hospital mortality- 0-22%

1176
Copyright © Oakstone Publishing, 2021. All Rights Reserved.

Systemic Complications
• Pulmonary
• 16-67% of patient s undergoing esophagectomy
• Pneumonia, bronchospasm, ARDS, acute exacerbation
COPD
• Cardiac
• Atrial arrythmias- up to 20% of patients undergoing
esophagectomy
• MI-roughly 1-4 % of patients

https://www.uptodate.com/contents/complications-of-esophagealresection?search=esophagectomy&topicRef=2530&source=see_link

Procedure Specific Complications


• Conduit
• Anastomotic leak
• Incidence differs depending on location of anastomosis
• Large STS database study-
• Intrathoracic 9.3 %
• Cervical 12.3%
• Conduit ischemia-minor to major
• Anastomotic stricture: 9-40%
• Recurrent laryngeal nerve injury
• Chylothorax

Dig Surg. 2011;28(1):29-35.

1177
Copyright © Oakstone Publishing, 2021. All Rights Reserved.

Anastomotic leak
• Investigated with EGD
• Determine extent of involvement
• Consider stent placement, revision, separation, etc.
• Endoluminal VAC (EVAC)
• Not FDA approved
• Changed the current strategy
• NGT with VAC sponge on the end
• Placed endoscopically and changed intermittently

Endosc Int Open. 2015 Dec; 3(6): E554–E558

Long-term complications
• Dysphagia-65%
• Delayed gastric emptying-50%
• Reflux- 20-80%
• Dumping-1-5%
• Hiatal hernia- 15% but only 2% requiring surgical
repair
• More common after MIE

1178
Copyright © Oakstone Publishing, 2021. All Rights Reserved.

Summary
• Management of benign and malignant tumors of
the esophagus vary widely
• Preservation of the esophagus and its function is
ideal when possible
• Esophagectomy through a variety of approaches is
part of the standard approach to stage II or above
cancers
• Neoadjuvant concurrent chemoradiation is
indicated in the majority of patients with advanced
disease

Thank you

1179
Copyright © Oakstone Publishing, 2021. All Rights Reserved.

David Spector, MD
Director of Bariatric and Reflux Surgery, FH
Brigham and Women’s Hospital
Harvard Medical School

Boston, Massachusetts

1180
Copyright © Oakstone Publishing, 2021. All Rights Reserved.

Outline

Introduction

1181
Copyright © Oakstone Publishing, 2021. All Rights Reserved.

Introduction

Introduction

1182
Copyright © Oakstone Publishing, 2021. All Rights Reserved.

Introduction

95%
5%

Type I Hiatal Hernia

1183
Copyright © Oakstone Publishing, 2021. All Rights Reserved.

Introduction

1184
Copyright © Oakstone Publishing, 2021. All Rights Reserved.

1185
Copyright © Oakstone Publishing, 2021. All Rights Reserved.

1186
Copyright © Oakstone Publishing, 2021. All Rights Reserved.

1187
Copyright © Oakstone Publishing, 2021. All Rights Reserved.

1188
Copyright © Oakstone Publishing, 2021. All Rights Reserved.

1189
Copyright © Oakstone Publishing, 2021. All Rights Reserved.

1190
Copyright © Oakstone Publishing, 2021. All Rights Reserved.

The difference: routine robust repair of hiatal hernias

1191
Copyright © Oakstone Publishing, 2021. All Rights Reserved.

1192
Copyright © Oakstone Publishing, 2021. All Rights Reserved.

1193
Copyright © Oakstone Publishing, 2021. All Rights Reserved.

1194
Copyright © Oakstone Publishing, 2021. All Rights Reserved.

1195
Copyright © Oakstone Publishing, 2021. All Rights Reserved.

Hiatal hernia reduces the effectiveness of the lower


esophageal sphincter

Sleeve gastrectomy => reduced compliance of stomach

=> Post prandial intra-gastric pressure elevation


increasing the pressure gradient driving the reflux

New onset reflux

Prevalence of hiatal hernia in obese patients:


20% - 40%

1196
Copyright © Oakstone Publishing, 2021. All Rights Reserved.

Prevalence of hiatal hernia in obese patients:


20% - 40%

90% of surgeons will repair this when doing a


sleeve gastrectomy

Hiatal Hernia Repair

1197
Copyright © Oakstone Publishing, 2021. All Rights Reserved.

Hiatal Hernia Repair

Introduction

1198
Copyright © Oakstone Publishing, 2021. All Rights Reserved.

Introduction

Indication for surgery

1199
Copyright © Oakstone Publishing, 2021. All Rights Reserved.

Indications

Pre-Operative Work Up

1200
Copyright © Oakstone Publishing, 2021. All Rights Reserved.

Pre-Operative Work Up

Surgical Approach

1201
Copyright © Oakstone Publishing, 2021. All Rights Reserved.

Surgical Approach

Surgical Approach

1202
Copyright © Oakstone Publishing, 2021. All Rights Reserved.

Surgical Approach

Surgical Approach

1203
Copyright © Oakstone Publishing, 2021. All Rights Reserved.

Surgical Approach

Surgical Approach

360°

1204
Copyright © Oakstone Publishing, 2021. All Rights Reserved.

Surgical Approach

Surgical Approach

270°

1205
Copyright © Oakstone Publishing, 2021. All Rights Reserved.

Surgical Approach

Surgical Approach

180°

1206
Copyright © Oakstone Publishing, 2021. All Rights Reserved.

Choice of Fundoplication

Choice of Fundoplication

1207
Copyright © Oakstone Publishing, 2021. All Rights Reserved.

Choice of Fundoplication

Choice of Fundoplication

1208
Copyright © Oakstone Publishing, 2021. All Rights Reserved.

Choice of Fundoplication

Choice of Fundoplication

1209
Copyright © Oakstone Publishing, 2021. All Rights Reserved.

Patients BMI>35

Post Operative - Early

1210
Copyright © Oakstone Publishing, 2021. All Rights Reserved.

Post Operative – Long Term

Post Operative – Long Term

1211
Copyright © Oakstone Publishing, 2021. All Rights Reserved.

Post Operative – Long Term

Post Operative – Long Term

1212
Copyright © Oakstone Publishing, 2021. All Rights Reserved.

Post Operative – Long Term

Post Operative – Long Term


Slipped Nissen

Transdiaphragmatic
herniation of the wrap

1213
Copyright © Oakstone Publishing, 2021. All Rights Reserved.

Para-Esophageal Hernia

95%
5%

Para-Esophageal Hernia

95%
5%

5% 95%

1214
Copyright © Oakstone Publishing, 2021. All Rights Reserved.

Introduction

Manifestation

1215
Copyright © Oakstone Publishing, 2021. All Rights Reserved.

Acute presentation

Acute presentation

Type III

1216
Copyright © Oakstone Publishing, 2021. All Rights Reserved.

Acute presentation

Type III

Acute presentation

Type II Type III

1217
Copyright © Oakstone Publishing, 2021. All Rights Reserved.

Acute presentation

Type II Type III

Chronic presentation

1218
Copyright © Oakstone Publishing, 2021. All Rights Reserved.

Preoperative Work Up

Preoperative Work Up

1219
Copyright © Oakstone Publishing, 2021. All Rights Reserved.

General Principles

General Principles

1220
Copyright © Oakstone Publishing, 2021. All Rights Reserved.

General Principles

General Principles

1221
Copyright © Oakstone Publishing, 2021. All Rights Reserved.

General Principles

General Principles

1222
Copyright © Oakstone Publishing, 2021. All Rights Reserved.

General Principles

General Principles

1223
Copyright © Oakstone Publishing, 2021. All Rights Reserved.

General Principles

General Principles

1224
Copyright © Oakstone Publishing, 2021. All Rights Reserved.

General Principles

General Principles

1225
Copyright © Oakstone Publishing, 2021. All Rights Reserved.

General Principles

General Principles

1226
Copyright © Oakstone Publishing, 2021. All Rights Reserved.

360° Nissen
General Principles

270°
Toupet
General Principles

1227
Copyright © Oakstone Publishing, 2021. All Rights Reserved.

General Principles

1228
Copyright © Oakstone Publishing, 2021. All Rights Reserved.

Slipped Nissen

Transdiaphragmatic
herniation of the wrap

1229
Copyright © Oakstone Publishing, 2021. All Rights Reserved.

1230
Copyright © Oakstone Publishing, 2021. All Rights Reserved.

Gastric Cancer Treatment

Jiping Wang, MD, PhD, FACS


Brigham and Women’s Hospital

Disclosures
Dr. Wang has nothing to disclosure.

1231
Copyright © Oakstone Publishing, 2021. All Rights Reserved.

GBD 2017 Stomach Cancer Collaborators

Gastric Cancer Treatment in the last two


decades
ADJ CRT Peri-op CT ADJ CT D2 ADJ CT D2 Peri-op CT
Classic

2001 2006 2007 2012 2018 2019 ????


McDonald Magic ACTS-GC Artist Critics FLOT4
ADJ RT Asia ADJ RT West
1999

Dutch Safer
D1 vs D2 2008 2018 2019 2020 Function
Preservation
JCOG9501 JCOG1001 Class 01 SENORITA Smaller
D2 vs. D2+ Bursectomy Klass 01 SENORITA 2
Klass 02

1232
Copyright © Oakstone Publishing, 2021. All Rights Reserved.

As compared with D2 lymphadenectomy


alone, treatment with D2 lymphadenectomy
plus PAND does not improve the survival
rate in curable gastric cancer.

1233
Copyright © Oakstone Publishing, 2021. All Rights Reserved.

Bursectomy did not provide a survival advantage over


non-bursectomy. D2 dissection with omentectomy
Our findings will accelerate the trend toward less-invasive
alone should be done as a standard surgery for
surgery in the treatment of gastric cancer worldwide.
resectable cT3–T4a gastric cancer.

Laparoscopic sentinel node navigation surgery versus


laparoscopic gastrectomy with lymph node dissection for
early gastric cancer: short‐term outcomes of a multicentre
randomized controlled trial (SENORITA). Br J Surg.
2020;107(11):1429–39.
A multicenter non‐randomized phase III study of sentinel
node navigation surgery for early gastric cancer. Jpn J Clin
Oncol. 2020. 10.1093/jjco/hyaa179
Sentinel node mapping for post‐endoscopic resection
gastric cancer: multicenter retrospective cohort study in
Japan. Gastric Cancer. 2020;23(4):716–24.

1234
Copyright © Oakstone Publishing, 2021. All Rights Reserved.

mOS: CRT group 36 months


surgery-only 27 months

Postoperative chemoradiotherapy
should be considered for all patients at high
risk for recurrence of adenocarcinoma of the
stomach or gastroesophageal junction who
have undergone curative resection.

5 y OS: CT group 36.3%


surgery-only 26.0%

Perioperative regimen of
ECF decreased tumor size and stage
and significantly improved progression-
free and overall survival.

1235
Copyright © Oakstone Publishing, 2021. All Rights Reserved.

Was the systemic therapy the salvage of poorer quality of surgery?

3 y OS: S1 group 80.1%


surgery-only 70.1%

S-1 is an effective adjuvant treatment


for East Asian patients who have undergone
a D2 dissection for locally advanced
gastric cancer.

3 y DFS: CT group 74%


surgery-only 59%

Adjuvant capecitabine plus


oxaliplatin treatment after curative
D2 gastrectomy should be considered as a
treatment option for patients with operable gastric
cancer.

1236
Copyright © Oakstone Publishing, 2021. All Rights Reserved.

The addition of XRT to XP chemotherapy


did not significantly reduce recurrence after
curative resection and D2 lymph node dissection in
gastric cancer

Postoperative chemoradiotherapy did


not improve overall survival compared with
postoperative chemotherapy in patients with resectable gastric
cancer treated with adequate preoperative chemotherapy and
surgery. In view of the poor postoperative patient
compliance in both treatment groups, future studies
should focus on optimising preoperative treatment
strategies.

1237
Copyright © Oakstone Publishing, 2021. All Rights Reserved.

mOS: FLOT group 50 months


ECF/ECX group 35 months

In locally advanced, resectable gastric or


gastro-oesophageal junction adenocarcinoma,
Perioperative FLOT improved
overall survival compared with perioperative
ECF/ECX.

The current treatment outcome for


resectable gastric cancer

1238
Copyright © Oakstone Publishing, 2021. All Rights Reserved.

Recurrence
Recurrence Median RFS (Months)
INT 0116
Surgery 64.40% 19
Surgery+CRT 42.70% 30
MAGIC
Surgery 57.40% 24
ECF+surgery+ECF 38.80% 24

CALGB 80101
FU+LV 46.10% 29
ECF 48.10% 27
CRITIC
CT 49.10% 28
CRT 51.90% 25

Potential Correctable Factors

Poor quality of surgery: Tumor cell spillage due to surgery: Microscopic


positive margin, Bleeding, distant
D1/D0 surgery, lymphatic tract metastasis
In-transit disease Interruption
Tumor manipulation: CTC

1239
Copyright © Oakstone Publishing, 2021. All Rights Reserved.

Evidence of tumor cell spillage

Lavage Surgery Lavage

49 gastric cancer patients underwent curative resection


No tumor cell before but has tumor cell detected after D2 lymph node dissection:
SM: 14.3% MP: 26.7% SS: 46.2%

Evidence of tumor cell spillage Before Surgery:


102 patients with peritoneal washing
57 showed no CEA or CK20 mRNA amplification.
61%
After gastrectomy:
CEA or CK20 mRNA was detected in 35 of these 57 samples

Viable cancer cells were identified in 24. The viable cancer


cells in all 24 cases showed Ki-67 positivity, indicating
proliferative activity. Cultured viable cancer cells generated
peritoneal nodules after spilling over the peritoneal cavity in
NOD/ ShiJic-scid mice in 4 cases.

1240
Copyright © Oakstone Publishing, 2021. All Rights Reserved.

NEO Adjuvant
CT/Target
NEO Adj CT Adjuvant
RT RT Immunotherapy

Local
Poor quality of surgery: TumorRegional
cell spillage: Distant
Microscopic
positive margin, Bleeding, distant
D1/D0 surgery, lymphatic tract interruption metastasis
In-transit disease
Tumor manipulation: CTC

HIPEC

TOPGEAR CRITICS II PILGRIM

1241
Copyright © Oakstone Publishing, 2021. All Rights Reserved.

TOPGEAR: Phase III, international, intergroup trial for resectable gastric cancer

≥18 years, IB-IIIC Primary Endpoint: OS

752
age, tumor site, T, N, ECX/ECFx2 ECX/ECFx2 Surgery ECX/ECFx3
institution, and
staging investigations
ECX/ECFx2 45 Gy+5FU/X Surgery ECX/ECFx3

23
Jiping Wang, MD PhD

CRITICS-II: a multicenter randomized phase II trial of neo-adjuvant chemotherapy


versus Chemo+CRT versus CRT only in resectable gastric cancer

stage IB-IIIC resectable gastric cancer 1 y Event Free Survival 60 % STOP 75% Phase III
Slagter et al. BMC Cancer (2018) 18:877
24
Jiping Wang, MD PhD

1242
Copyright © Oakstone Publishing, 2021. All Rights Reserved.

A phase II study of SHR-1210 combined with neoadjuvant chemoradiation in


patients with locally advanced proximal stomach adenocarcinoma (Neo-PLANET).

25

PILGRIM: Phase III Multicenter Adjuvant Hyperthermic Intraperitoneal


Chemotherapy for Resectable Gastric Cancer

Primary Endpoint: OS
≥18 years, Karnofsky 70+
cT3/4, NxM0
317
Surgery HIPEC XELOX/SOX
Medical Center
Surgery No HIPEC XELOX/SOX

331
First HIPEC (<=48 post op): 43℃, 60 min,Paclitaxel 75mg/m2
2nd HIPEC: Within 1st week 43℃, 60 min,Paclitaxel, 100mg/m2

26

1243
Copyright © Oakstone Publishing, 2021. All Rights Reserved.

Prevention

+Cyt
Site of Recurrence
Study Implant
Local Peritoneal Distant
Schwart 2002 40% 54% 54% T3,T4
M0 Carcinomatosis
Marrelli 2002 42-48% 21-52% 25-46%
HIPEC
MacDonald 2001 29% 72% 18%
Gunderson 2002 38-93% 30-43% 49%

27

Mismatch Repair Deficiency, Microsatellite Instability, and


Survival
Patients with operable gastroesophageal cancer with MSI high have
superior survival compared with low MSI/MSS tumors when treated with
surgery alone. However, patients with low MSI/MSS tumors have superior
survival compared with patients with MSI high when treated with
perioperative chemotherapy plus surgery.

Surgery MSI high

Surgery MSS

JAMA Oncol. 2017 Sep; 3(9)

1244
Copyright © Oakstone Publishing, 2021. All Rights Reserved.

Microsatellite Instability and Programmed Cell Death-Ligand


1
The CAPOX adjuvant studyExpression
in stomach cancerin Stage
(CLASSIC) trialII/III Gastric
demonstrated Cancer
the benefit of adjuvant chemotherapy
(capecitabine plus oxaliplatin) after D2 gastrectomy over D2 gastrectomy alone for stage II/III gastric cancer [5-year
disease-free survival (DFS), 68% vs 53%; hazard ratio (HR) 0.66, 95% confidence interval (CI) 0.51–0.85, P ¼ 0.0015).

Jiping Wang, MD, PhD, FACS


Brigham and Women’s Hospital

Choi, Annals of Surgery, 270(2), pp. 309–316

Individual Patient Data Meta-Analysis of the Value of


Microsatellite Instability As a Biomarker in Gastric Cancer

Jiping Wang, MD, PhD, FACS


Brigham and Women’s Hospital

1245
Copyright © Oakstone Publishing, 2021. All Rights Reserved.

CONCLUSION
In patients with resectable primary GC, MSI is a
robust prognostic marker that should be
adopted as a stratification factor by clinical
trials. Chemotherapy omission and/or immune
checkpoint blockade should be investigated
prospectively in MSI-high GCs according to
clinically and pathologically defined risk of
relapse.

Final results and subgroup analysis of the PETRARCA randomized phase II


AIO trial: Perioperative trastuzumab and pertuzumab in combination with
FLOT versus FLOT alone for HER2 positive resectable esophagogastric
adenocarcinoma
Treatment A Treatment B
A: FLOTx4 Surgery FLOTx4
pCR 12% 35%
HER2+ pLN- 39% 68%
EGA (>cT2 or cN+) DFS (2y) 54% 70%
B: FLOTx4 FLOTx4
Trastuzumab Surgery Trastuzumab OS (2y) 77% 84%
Pertuzumab Pertuzumab Toxicity 75% 85%

Resectable vs. Mets

1246
Copyright © Oakstone Publishing, 2021. All Rights Reserved.

Gastric Cancer Treatment in the last two


decades
ADJ CRT Peri-op CT ADJ CT D2 ADJ CT D2 Peri-op CT
Classic Neoadjuvant CRT,
Immunotherapy
2001 2006 2007 2012 2018 2019 Target therapy
HIPEC
McDonald Magic ACTS-GC Artist Critics FLOT4 ctDNA
ADJ RT Asia ADJ RT West
1999

Dutch Safer
D1 vs D2 2008 2018 2019 2020 Function
Preservation
JCOG9501 JCOG1001 Class 01 SENORITA Smaller
D2 vs. D2+ Bursectomy Klass 01 SENORITA 2
Klass 02

1247
Copyright © Oakstone Publishing, 2021. All Rights Reserved.

Scott A. Shikora, MD, FACS


Director, Center for Metabolic and Bariatric Surgery
Brigham and Women’s Hospital
Professor of Surgery
Harvard Medical School

Scott A. Shikora, MD, FACS


Director, Center for Metabolic and Bariatric Surgery
Brigham and Women’s Hospital
Professor of Surgery
Harvard Medical School

1248
Copyright © Oakstone Publishing, 2021. All Rights Reserved.

Company Name (s) Nature of Relationship (s)


Medtronic Consulting
Obesity Surgery Journal Editor-in-Chief

Introduction
• The incidence of severe obesity continues to increase
world-wide
• There is also an increase in the number of patients
who suffer from the obesity-associated conditions
such as type 2 diabetes
• Bariatric surgery is currently the only treatment that
can provide meaningful and sustainable weight loss
and improvements in the related-conditions
• This presentation will review its current status

1249
Copyright © Oakstone Publishing, 2021. All Rights Reserved.

The Obesity Epidemic – By 2030


BMI > 30 kgm2 BMI > 35 kgm2

Ward ZJ, et al, NEJM 2019

The Global Obesity Epidemic

1250
Copyright © Oakstone Publishing, 2021. All Rights Reserved.

Obesity Adversely Affects Every Organ System in the Body


Pulmonary Disease
Abnormal Function Idiopathic Intracranial Hypertension
Obstructive Sleep Apnea Stroke
Hypoventilation Syndrome
Cataracts
Nonalcoholic Fatty
Coronary Heart Disease
Liver Disease Diabetes
Steatosis Dyslipidemia
Steatohepatitis Hypertension
Cirrhosis

Gall Bladder Disease Severe Pancreatitis

Gynecologic Abnormalities Cancer


Abnormal Menses Breast, Uterus, Cervix,
Infertility Colon, Esophagus, Pancreas,
Polycystic Ovarian Syndrome Kidney, Prostate

Osteoarthritis
Phlebitis
Skin Venous Stasis
Gout
http://www.obesityonline.org/slides/

The Global Obesity Epidemic

1251
Copyright © Oakstone Publishing, 2021. All Rights Reserved.

The Obesity Epidemic – By 2030


Overall Obesity (BMI >30 kg/2) Severe Obesity (BMI >35 kg/2)

Subgroups Overall Men Women Overall Men Women


(%) (%) (%) (%) (%) (%)
U.S. overall 48.7 (47.7-50.1) 48.2 (46.8-49.6) 49.9 (48.5-51.4) 24.2 (22.9-25.5) 21.1 (19.6-22.6) 27.6 (26.1-29.2)

Mass 42.3 (40.2-44.3) 43.1 (40.4-45.7) 41.7 (39.1-44.2) 20.0 (18.2-22.1) 18.7 (16.3-21.4) 21.5 (19.3-24.0)

Oklahoma 58.4 (56.4-60.2) 59.5 (56.9-61.9) 57.5 (54.9-59.8) 31.7 (29.7-33.9) 29.0 (26.1-32.0) 34.9 (32.6-29.7)

Colorado 38.2 (36.3-40.3) 37.5 (34.8-40.0) 39.2 (36.7-42.0) 16.8 (15.2-18.6) 14.3 (12.1-16.6) 19.8 (17.6-22.2)

Ward ZJ, et al, NEJM 2019

The Global Obesity Epidemic

Sturm R, Hattori A. Int J Obes, 2012

1252
Copyright © Oakstone Publishing, 2021. All Rights Reserved.

Obesity = Poor Health


BMI 40-<50 BMI 50-<60 BMI 60+
70 68

59 60
60
53 52
% With Comorbidity

50
42 42
40 35
33 31
30 27
24 22
20
9
10
4
0
Walk <200 ft. Asthma DM Sleep Apnea HTN

LABS Writing Group, SOARD 2008

Obesity is a Killer

Obesity Hyperglycemia Hypertension

21.29 million deaths in 2017

1253
Copyright © Oakstone Publishing, 2021. All Rights Reserved.

Weight Loss Options


• Lifestyle changes (diet, exercise, behavioral
modifications) are the cornerstone of
therapy
• Pharmacotherapy can be useful in properly
selected patients
• Bariatric surgery is the most effective long-
term therapy for severe or morbid obesity

What is Metabolic Surgery?

1254
Copyright © Oakstone Publishing, 2021. All Rights Reserved.

What is Metabolic Surgery?


“The operative manipulation of a normal organ or
organ system to achieve a biological result for a
potential health gain”

Buchwald & Varco: Metabolic Surgery, Grune and Stratton, 1978

Metabolic Surgery - Examples


• Gastrectomies and vagotomies for peptic ulcer disease
• Splenectomy for idiopathic thrombocytopenia purpura
• Portal diversion for glycogen storage disease
• Endocrine ablations for malignancies
• Transplantation for metabolic diseases (i.e. pancreas
for diabetes mellitus)
• Partial ileal bypass for hyperlipidemia

Courtesy of Henry Buchwald, MD, PhD

1255
Copyright © Oakstone Publishing, 2021. All Rights Reserved.

Obesity is a Metabolic Disease


Pulmonary Disease
Abnormal Function Idiopathic Intracranial Hypertension
Obstructive Sleep Apnea Stroke
Hypoventilation Syndrome
Cataracts
Nonalcoholic Fatty
Coronary Heart Disease
Liver Disease Diabetes
Steatosis Dyslipidemia
Steatohepatitis Hypertension
Cirrhosis

Gall Bladder Disease Severe Pancreatitis

Gynecologic Abnormalities Cancer


Abnormal Menses Breast, Uterus, Cervix,
Infertility Colon, Esophagus, Pancreas,
Polycystic Ovarian Syndrome Kidney, Prostate

Osteoarthritis
Phlebitis
Skin Venous Stasis
Gout
http://www.obesityonline.org/slides/

Bariatric/Metabolic Surgery
• Currently reserved for BMI > 35
• Achieves meaningful and sustainable
weight loss
• Improves or “cures” most obesity-related
comorbid conditions
• Improves life expectancy
• Improves quality of life

1256
Copyright © Oakstone Publishing, 2021. All Rights Reserved.

Goals of Surgery Past and Present


Massive
Bariatric Surgery Weight
Loss

Treat
Metabolic Surgery Metabolic
Diseases

The Obesity Treatment Continuum


OBESITY TREATMENT OPTIONS
Gastric
Bypass
WEIGHT LOSS

Gastric
Band Duodenal
Switch
Endoscopic Sleeve
Surgery Gastrectomy
Diet and
Devices
Exercise

Pharmaceuticals

COMPLEXITY - RISK

Adapted from Choi HS, Chun HJ, Clin Endosc 2017

1257
Copyright © Oakstone Publishing, 2021. All Rights Reserved.

Current Treatment
BMI > 30 kg/m2 BMI = 30 - 35 kg/m2 BMI > 35 kg/m2

Pharmacotherapy Bariatric/Metabolic
& Lifestyle Modification Surgery

Risk
Lower Higher
Weight Loss

Bariatric Surgery is Metabolic Surgery


Comorbidity % Improved % Resolved
Osteoarthritis/Joint Pain 47 41
Hyperlipidemia 33 63
Gastroesophageal Reflux 24 72
Hypertension 18 70
Sleep Apnea 19 74
Depression 47 8
Peripheral Edema 55 41
Urinary Incontinence 39 44
Asthma 69 13
Diabetes 18 82
Coronary Artery Disease 75 25

Schauer P, et al, Ann Surg 2002

1258
Copyright © Oakstone Publishing, 2021. All Rights Reserved.

The Operative Procedures


• 1950s - Intestinal Bypasses
• 1960s - Gastric Bypass
• 1970s - Gastroplasties
• 1970s - The Adjustable Gastric Band
• 2000s - Sleeve Gastrectomy, Switch
• 2010s - SILS, NOTES, Robotics
• 2020s - Endoscopic, OAGB, SADI-S, etc

Intestinal Bypasses
• Richard Varco 1953
• Bypassed 90% of the small intestine and its absorptive
capacity
• Referred to as “Controlled Malabsorption”
• Resulted in significant late complications
• Protein malnutrition
• Arthralgias, myalgias, metabolic bone disease
• Intractable diarrhea, steatorrhea
• Liver failure Intestinal Bypass
• Abandoned in favor of gastric procedures

1259
Copyright © Oakstone Publishing, 2021. All Rights Reserved.

The Gastric Bypass


• 1967 - Developed by Edward Mason, MD
• Divided horizontal staple line and a loop
gastrojejunostomy
• High morbidity including bile
gastritis/esophagitis, anastomotic leaks
• Excellent weight loss (restriction and
malabsorption
• Safer than the intestinal bypass Mason
Gastric Bypass

Mason EE, Ito C, Surg Clin North Am. 1967

Finding the Perfect Gastric Bypass


• Pouch size and location
• Horizontal or vertical staple lines
• Roux-en-Y or loop
• Limb lengths
• Anastomoses

Mason Greenville Current


Gastric Bypass Gastric Bypass Gastric Bypass

1260
Copyright © Oakstone Publishing, 2021. All Rights Reserved.

Currently Popular Procedures

Adjustable Sleeve Roux-en-Y Duodenal


Gastric Band Gastrectomy Gastric Bypass Switch

Currently Popular Procedures

Adjustable Sleeve Roux-en-Y Duodenal


Gastric Band Gastrectomy Gastric Bypass Switch

1261
Copyright © Oakstone Publishing, 2021. All Rights Reserved.

North American Surgical Trends

Angrisani, L, et al, Obesity Surgery, 2018

Bariatric Surgery Numbers, 2012-2018


2012 2013 2014 2015 2016 2017 2018

Total 173,000 179,000 193,000 196,700 215.666 228,005 252,565


Sleeve 33.00% 42.10% 51.70% 53.61% 58.11% 59.39% 61.4%

RYGB 37.50% 34.20% 26.80% 23.02% 18.69% 17.80% 17.0%

Band 20.20% 14.00% 9.50% 5.68% 3.39% 2.77% 1.1%

BPD-DS 1.00% 1.00% 0.40% 0.60% 0.57% 0.70% 0.8%

Revision 6.00% 6.00% 11.50% 13.55% 13.95% 14.14% 15.4%

Other 2.30% 2.70% 0.10% 3.19% 2.63% 2.46% 2.3%

Balloons - - - 0.36% 2.66% 2.75% 2.0%

English WJ, et al, SOARD 2019

1262
Copyright © Oakstone Publishing, 2021. All Rights Reserved.

Roux-en-Y Gastric Bypass


• One limb of the small intestine is anastomosed to a
small gastric pouch
• There is a second anastomosis between the bowel
limbs
• Limits ingestion of food and changes hunger
signals
Average weight loss: 30-35% of total body weight or
60 -70% of excess body limb
Biliopancreatic weight
Roux limb
Common channel

Roux-en-Y Gastric Bypass


Beneficial effects are not all secondary to weight loss
Meal Responses
GBP GBP

GLP-1 PYY
Le Roux CW et al, Ann Surg 2006

1263
Copyright © Oakstone Publishing, 2021. All Rights Reserved.

Sleeve Gastrectomy
• Resection of the greater curvature
• Removes about 85% of the stomach
• Surgery takes about 1 hour
• Limits ingestion of food and changes hunger
signals
• Average weight loss: 25% of total weight or
50-60% of excess weight
• Irreversible Sleeve
Gastrectomy

Sleeves vs Gastric Bypasses

Sleeves Bypasses
• 25% total weight loss • 35% total weight loss
• Simpler and safer • Complicated/greater risk
• No dumping syndrome • Dumping syndrome
• Better for meds • Better for GERD
• Better for adhesions • Better for Barrett’s
• Convertible to GBP or DS • Less revisional options

1264
Copyright © Oakstone Publishing, 2021. All Rights Reserved.

Duodenal Switch
• 1998 - First described by Hess and Hess
• Modification of the Scopinaro BPD
• Retains the antrum and the pylorus
• 1998 – Marceau et al
• 1999 – Gagner – first lap DS
• Restrictive and malabsorptive Duodenal Switch
1998

• The best weight loss, lowest recidivism, and best


metabolic results
• Potential for nutritional deficiencies

Gastroplasties (1970s to 1980s)


• Stomach partitioning without the risks of an
anastomosis
• Many variants – all proven to be inferior
• Mason in 1982 -Vertical Banded Gastroplasty
• Purely restrictive

VBG
Mason EE, et al, Arch Surg 1982

1265
Copyright © Oakstone Publishing, 2021. All Rights Reserved.

VBG is Inferior to the RYGB


Randomized patients to RYGB or VBG

Sugerman H, et al, Am J Surg 1987

1990s – The Age of Laparoscopy


• The game-changer
• Dramatically reduced the incidence of wound
complications, metabolic stress, etc
• More cosmetic, faster return to usual activities

1266
Copyright © Oakstone Publishing, 2021. All Rights Reserved.

Diabetes is a Metabolic Disease

International Diabetes Federation, IDF Diabetes Atlas,9th Ed, 2019

Obesity and Diabetes = Diabesity

1267
Copyright © Oakstone Publishing, 2021. All Rights Reserved.

Obesity and Diabetes

“With an excess of fat, diabetes


begins and from an excess of fat,
diabetics die”

Elliott P. Joslin, MD 1927

Type 2 Diabetes Mellitus


• Individuals with severe obesity are 3x more
likely to be diabetic
• 30% of patients undergoing bariatric surgery
are diabetic or glucose intolerant
• Type II diabetes is put into remission in up to
75% of patients after surgery
• Improvements are seen long before significant
weight loss

1268
Copyright © Oakstone Publishing, 2021. All Rights Reserved.

Diabetes is a Killer
• Sixth leading cause of death in the U.S.
• 2002
• 71,000 deaths in the U.S.
• Another 186,000 deaths from related
conditions
• Increases the risk of heart disease six-fold
and the risk of stroke four-fold
Diabesity, Dr. Katherine Kaufman, former ADA president, Bantam Books, 2005

Diabetes is a Crippler
In a single year in the U.S……..
• 82,000 amputations
• 12,000-24,000 people lost their
eyesight
• 41,000 people began treatment for end
stage renal failure
American Diabetes Association National Diabetes Factsheet, Accessed April, 2005

1269
Copyright © Oakstone Publishing, 2021. All Rights Reserved.

Medical Therapy for Diabetes


• Never “cures” the patient
• May have significant side effects
• Compliance issues
• 1 in 5 patients still has poor glycemic control*
• Many patients still suffer progression of the
disease
• Significant cost

*Saaddine JB, et al, Ann Int Med 2006

Best Medical Therapy for Diabetes


• Look Ahead Trial
• Multicenter randomized trial comparing intensive
lifestyle intervention (ILI) and diabetes support on CVD
events in 5145 patients with T2DM
• Results
• ILI improved HgA1c, BS, Meds, BP, HDL, and TG
• Only half of the study patients achieved glycemic
control
• 33% of patients in the control group needed
medications by 12 months

Wing RR, et al, Arch Int Med 2010

1270
Copyright © Oakstone Publishing, 2021. All Rights Reserved.

Optimal BS Control Still Eludes Us


2006 2013
HbA1c > 7.0 HbA1c > 7.0
43.3% 45.8%

Lipska KJ, et al, Diabetes Care 2017

Surgical Therapy May…..


• Put T2DM into remission (?cure)
• Reduce progression of the disease
• Prevent the onset of T2DM
• Decrease the likelihood of development
• Improve (cures) other conditions
• Cost effective

1271
Copyright © Oakstone Publishing, 2021. All Rights Reserved.

Gastric Bypass Improves Diabetes


• 608 patients undergoing gastric bypass
• 271 of the 298 (91%) became euglycemic
• Diabetic 121 of 146 (83%)
• Glucose intolerant 150 of 152 (99%)
• Success was less likely in
older patients and those
who were diabetic longer

Pories et al, Ann Surg 1995

Intervene Sooner Than Later

Pories et al, Ann Surg 1995

1272
Copyright © Oakstone Publishing, 2021. All Rights Reserved.

The Effects are Sustainable


• Retrospective study of 217 patients with T2DM
who underwent bariatric surgery (2004-2007)
• Results - Mean follow up = 6 years (5-9)
• 162 GB, 32 Bands, 23 Sleeves
• 24% and 26% = complete and partial remission
• 34% = improved
• 19% = recurrence of T2DM

Brethauer SA, et al, Ann Surg 2013

Surgery May Prevent T2DM


Swedish Obesity Subject Study (SOS)
• Prospective, nonrandomized
• 1658 surgery patients
Mean follow up = 10
• 1771 matched controls years (0-15)
• None had T2DM at onset
• Results
• 392 (22.1%) in control vs 110 (6.6%) in surgery groups
developed T2DM (p < 0.001)

Carlsson LMS et al, N Engl J Med 2012

1273
Copyright © Oakstone Publishing, 2021. All Rights Reserved.

Over 15 RCTs Show Better Weight Loss


and Diabetes Control With G.I. Surgery
than Medical-Lifestyle Interventions

Bariatric Surgery vs. Best Medicine


• STAMPEDE Trial
• Prospective, randomized comparing RYGB and
LSG to intensive medical therapy for T2DM
• 150 patients with obesity and uncontrolled
T2DM (HbA1c = 9.2+1.5%)
• Primary endpoint = % of patients with HbA1c <
6.0%
• 12 month follow up
Schauer PR et al, N Engl J Med 2012

1274
Copyright © Oakstone Publishing, 2021. All Rights Reserved.

STAMPEDE Trial Results


12 month f/u=93% Medical Gastric Bypass Sleeve P

P=0.002
Primary endpoint (%) 12 42 37 P=0.008
P<0.001
HgA1c (%) 7.5+1.8 6.4+0.9 6.6+1.0 P=0.003
Weight loss (kg) 5.4+8.0 29.4+9.0 25.1+8.5 P<0.001
P<0.001
EWL (%) 13 88 81 P<0.001
P<0.002
Triglycerides -44 -44 -42 P=0.08
P<0.001
C-Reactive Protein -33.2 -84 -80 P<0.001

Schauer PR et al, N Engl J Med 2012

STAMPEDE Trial – 5 Year Follow-Up


• Results (n = 134)
• Maintained primary endpoint
1 Year 5 Years
• Medical 12% 5%
• Gastric bypass 42% 29%
• Sleeve 37% 23%

Schauer PR et al, N Engl J Med 2017

1275
Copyright © Oakstone Publishing, 2021. All Rights Reserved.

STAMPEDE Trial – 5 Year Follow-Up


HgbA1c(%) Change in BMI

Schauer PR et al, N Engl J Med 2017

STAMPEDE Trial – 5 Year Follow-Up

Schauer PR et al, N Engl J Med 2017

1276
Copyright © Oakstone Publishing, 2021. All Rights Reserved.

When to Operate?

What is the Risk of Waiting?


• Further deterioration of the islet cells
• Multiple studies have concluded that
waiting only reduces remission and
increases the likelihood of relapse.
• Potential to suffer an untoward event from
the patient’s diabetes
• Potential to suffer an untoward event
related to other health issues

1277
Copyright © Oakstone Publishing, 2021. All Rights Reserved.

Intervene Sooner than Later

Pories WJ, et al, Ann Surg 1995

Predictors of Control and Relapse


• Retrospective review of patients with T2DM
who underwent RYGB (n = 4,434 patients)
• Results (5 years)
• 68.2% of patients reached complete remission
• 35.1% of patients in remission relapsed within 5 years

Poor preoperative glycemic control, insulin use,


older age, and longer duration of T2DM

Arteburn, DE, et al, Obes Surg, 2012

1278
Copyright © Oakstone Publishing, 2021. All Rights Reserved.

Predictors of Control and Relapse


• STAMPEDE Trial – 5 year F/U
• PE - % of patients with HbA1c < 6.0% off meds
• Results (n = 134)
• Made PE – 5% med, 23% LSG, 29% LGB
• Predictors of reaching glycemic control
Duration of T2DM less than 8 years
Glycemic control relapse was not related to weight gain

Schauer PR et al, N Engl J Med 2017

Predictors of Control and Relapse


• Retrospective study of 217 patients with T2DM who
underwent bariatric surgery (2004-2007)
• Results - Mean follow up = 6 years (5-9)
• 24% = Complete remission
162 bypass
• 26% = Partial remission 32 band
• 34% = improved 23 Sleeve
• 19% = recurrence of T2DM
Shorter duration of T2DM, higher %EWL predicted
remission, and surgery in < 5years
Brethauer SA, et al, Ann Surg 2013

1279
Copyright © Oakstone Publishing, 2021. All Rights Reserved.

Obesity Cardiomyopathy Spectrum

End-stage Left
Asymptomatic
Ventricular Dilatation
Diastolic
With Reduced
Dysfunction Systolic Function

Kindel T, et al, Surg Obes Relat Dis, 2018

Heart Failure Secondary to Severe Obesity


• Risk of HF increases 30-100% in the obese
• Multiple mechanisms
• Increased body mass
• Inflammation
• Impaired cardiac metabolism
• Complicated by the other co-morbid
conditions

Kindel T, et al, Surg Obes Relat Dis, 2018

1280
Copyright © Oakstone Publishing, 2021. All Rights Reserved.

Improved Cardiac Function


• Follow up of the Utah Obesity Study
• 423 GBP patients compared with 733 severely
obese – 2 year f/u
• Results
• Significant reductions in BPs, HR, triglycerides,
LDL, insulin resistance
• Reductions in LV mass index, RV cavity size
• Improvements in myocardial function

Owan T, et al, J Am Coll Cardiol 2011

Bariatric Surgery and Heart TXP


• Heart transplantation is contraindicated
in patients with morbid obesity
• Bariatric surgery
• May improve cardiac function
• Enable patients to be eligible for
transplantation
• Limited published data
Jeng E, et al, J Card Surg 2016

1281
Copyright © Oakstone Publishing, 2021. All Rights Reserved.

Obesity-Associated Cancers
• Esophagus (adeno) • Multiple Myeloma
• Breast (postmenopause) • Ovarian
• Colorectal • Pancreatic
• Endometrial • Liver
• Gallbladder • Thyroid
• Gastric (cardia) • Meningioma
• Kidney

Lauby-Secretan B, et al, NEJM 2016

New Cancer Cases - 2018

Breast and Uterus = 34%

American Cancer Society, Cancer Statistics Center

1282
Copyright © Oakstone Publishing, 2021. All Rights Reserved.

Obesity and Cancer Risk


• Overweight and obesity-associated cancers
• 40% of all cancers diagnosed in 2014
• 55% of all cancers diagnosed in women
• 24% of all cancers diagnosed in men
• Endometrial, ovarian, and postmenopausal breast
cancer accounted for 42% of overweight and obesity
associated cancers
• High BMI is associated with cancer risk

Steele CB, et al, MMWR 2017

Bariatric Surgery and Cancer Risk


• Any cancer HR = 0.67, p < 0.001
• Obesity-assoc cancer HR = 0.59, p < 0.001
• Breast cancer HR = 0.58, p < 0.001
• Colon cancer HR = 0.59, p = 0.04
• Endometrial cancer HR = 0.46, p < 0.001
• Pancreatic cancer HR = 0.46, p = 0.04

Schauer DP, et al, Ann Surg 2017

1283
Copyright © Oakstone Publishing, 2021. All Rights Reserved.

All Cancers

Schauer DP, et al, Ann Surg 2017

Bariatric Surgery and Breast Cancer


Bariatric Surgery Control
No. of No. of No. of No. of
Patients Cancers (%) Patients Cancers (%)
Adams 2009 6596 73 (1.1) 9442 107 (2.3)
Christou 2008 1035 12 (1.2) 5746 362 (6.3)
Sjostrom 2009 1420 21 (1.5) 1447 28 (1.9)
McCawley 2009 1482 8 (0.54) 3495 19 (5.4)
Total 10,533 114(1.1) 20,130 516 2.6)

Adapted from Winder AA, et al, Obes Surg 2017

1284
Copyright © Oakstone Publishing, 2021. All Rights Reserved.

Bariatric Surgery and Endometrial CA


• Systematic review – 5 controlled studies
• 113,032 patients had bariatric surgery
• 462 developed endometrial cancer (0.4%)
• 848,864 controls
• 11,997 developed endometrial cancer (1.4%)
• OR = 0.317 (95% CI 0.161 to 0.627)

Winder AA, et al, Obes Surg 2018

Bariatric Surgery and Endometrial CA


Bariatric Surgery Control
No. of No. of No. of No. of Cancers
Patients Cancers (%) Patients (%)
Adams 2009 6596 14 (0.2) 9442 98 (1.0)
Christou 2008 1035 3 (0.3) 5746 20 (0.3)
Sjostrom 2009 1420 21 (1.5) 1447 25 (1.7)
Ward 2014 103,797 423(0.4) 832,372 11,728(1.4)
McCawley 2009 1482 1 (0.07) 3495 126 (3.6)
Total 114,330 462(0.4) 852,502 11,997(1.4)

Adapted from Winder AA, et al, Obes Surg 2018

1285
Copyright © Oakstone Publishing, 2021. All Rights Reserved.

Bariatric Surgery Is Safe


Sleeve Gastrectomy Gastric Bypass
• Leak 0.5% Stricture 4.2%
• Stricture 0.4% Ulcer 2.8%
• Mortality 0.1% Obstruction 1.4%
Bleed 1.2%
National MBSAQIP Leak 0.6%
DATA 2008-2014 Mortality 0.2%

Conclusions
• Unchecked obesity and T2DM will cause
significant damage to health care systems across
the world
• Bariatric/metabolic surgery currently provides
the best treatment for both diseases
• Bariatric/metabolic surgery should at least be
considered for obese patients with comorbid
conditions such as T2DM

1286
Copyright © Oakstone Publishing, 2021. All Rights Reserved.

Thank you !!!

“My daddy is a doctor and he treats


diabetes.”
“My daddy is a surgeon and he
cures it.”

Courtesy of Walter Pories, MD

Scott A. Shikora, MD, FACS


Director, Center for Metabolic and Bariatric Surgery
Brigham and Women’s Hospital
Professor of Surgery
Harvard Medical School

1287
Copyright © Oakstone Publishing, 2021. All Rights Reserved.

Company Name (s) Nature of Relationship (s)


Medtronic Consulting
Obesity Surgery Journal Editor-in-Chief

Introduction
• Peptic ulcers are injuries of the digestive tract, usually
the stomach or proximal duodenum
• Improved diagnosis and treatment have reduced the
prevalence of the disease
• However, the disease has not been irradicated and
patients may still present for treatment of the primary
disease or the complications of it
• This presentation will review the diagnosis and
treatment of peptic ulcer disease.

1288
Copyright © Oakstone Publishing, 2021. All Rights Reserved.

Causes of PUD
• Was traditionally thought to be due to abnormally
high levels of gastric acid production
• Currently thought to be to:
• Bacterium Helicobacter pylori infection
• Nonsteroidal anti-inflammatory drugs
• Zollinger-Ellison syndrome
• Other medications, i.e., serotonin-re-uptake inhibitors,
corticosteroids, aldosterone antagonists, and anticoagulants
• Smoking and poor socioeconomic status?
• Idiopathic

Zollinger-Ellison Syndrome
• Extreme form of PUD
• Due to the release of excessive gastrin
secondary to islet cell tumors of the pancreas
• Only accounts for 1% of the cases of PUD
• Is diagnosed with the secretin stimulation test
• Ulcers are atypical in location
• Total gastrectomy or removal of all tumor

1289
Copyright © Oakstone Publishing, 2021. All Rights Reserved.

ZE Syndrome Symptoms
• 70-95% - Ulcer pain
• 30% - Diarrhea
• 29% - Melena
• 29% - Vomiting
• 23% - Hematemesis
• 8% - Abdominal cramps

Blalock JB, AM J Surg 1981

Symptoms
• Burning stomach pain
• Heartburn
• Gastric and/or duodenal ulcers
• Feeling of fullness, bloating or belching
• Intolerance to fatty foods
• Pain improved by eating or taking acid reducing
mediations
• Two-thirds of patients are asymptomatic

1290
Copyright © Oakstone Publishing, 2021. All Rights Reserved.

Declining Prevalence
• The lifetime prevalence of PUD in the general
population is 5-10%
• The incidence has decreased dramatically in the
last few decades that to be due to:
• Availability of accurate testing for H. pylori
• Aggressive treatment with antibiotics and
antisecretory drugs (proton pump inhibitors)
• Better understanding of the mechanism PUD

Lanas A and Chan FKL, The Lancet 2017

Complications of PUD
• Intractability – 50%
• Hemorrhage – 33% - fatality rate 5 - 10%
• Perforation – 8-10% - fatality rate - 20%
• Obstruction – 8-10%
• Intractability

Nyhus LM and Kraft R, Hardy’s Textbook of Surgery 1977

1291
Copyright © Oakstone Publishing, 2021. All Rights Reserved.

NSAIDs and Aspirin


• Must be used judiciously
• Compared with non-users:
NSAIDs – increases the risk of
complications of PUD by 4X
Aspirin – increases the risk of
complications of PUD by 2X
Huang JQ, et al, Lancet 2002

Medical Treatment

Fallone CA, et al, Gastroenterol 2016

1292
Copyright © Oakstone Publishing, 2021. All Rights Reserved.

Medical Treatment

Fallone CA, et al, Gastroenterol 2016

Treatment of Bleeding Ulcers


• 40-60% of all causes of acute upper GI bleeding
• Acid suppression to maintain a neutral pH (IV PPI) more
effective than oral
• Prevents platelet disaggregation and clot lysis
• Faster onset of gastric acid suppression
• Endoscopy
• Prokinetic agents - improve endoscopic view and
reduce the need for a second look endoscopy
• Embolization
• Surgery
Lanas A and Chan FKL, The Lancet 2017

1293
Copyright © Oakstone Publishing, 2021. All Rights Reserved.

Recurrent Bleeding Ulcers


•High mortality
•Higher rate of complications for patients
who underwent surgery vs endoscopy
•Shock and ulcer size (diameter > 2 cm)
predictive of endoscopic treatment failure

Lanas A and Chan FKL, The Lancet 2017

Endoscopy for Bleeding Peptic Ulcers


• Early endoscopy (within 24 hours) is the treatment
of choice
• Meta-analysis of RCTs – Endoscopy reduces
• Rebleeding
• Need for surgery
• Mortality
• Identifies low risk patients

Sung JJ, et al, Gut 2007

1294
Copyright © Oakstone Publishing, 2021. All Rights Reserved.

Endoscopy for Bleeding Peptic Ulcers


• Injection
• Thermal
• Mechanical
• Combination therapy
• Effective for achieving primary hemostasis, reducing re-
bleeding, need for transfusion
• If bleeding recurs
• Interventional radiology embolization
• Surgery
Kavitt, et al, AM J Med, 2019

Bleeding on Anticoagulants or Antiplatelets

Witt DM, et al, Arch Intern Med 2012

1295
Copyright © Oakstone Publishing, 2021. All Rights Reserved.

Peptic Ulcer Perforation


• Usually occurs in the anterior wall of the
first portion of the duodenum
• Graham omental patch is the treatment of
choice
• 1/3 – no further ulcer issues
• 1/3 – successfully managed medically
• 1/3 – continued to have complications of
ulcer disease ulcer

Perforated PU Initial Treatment


• Hemodynamically stable patient
• NPO, NGT
• IV fluids
• Antibiotics
• Proton pump inhibitors
• Prompt surgery

1296
Copyright © Oakstone Publishing, 2021. All Rights Reserved.

Peptic Ulcer Perforation


• PUD triad
• Sudden onset of abdominal pain
• Tachycardia
• Abdominal rigidity
• Free air seen in the abdomen on x-ray
• Leukocytosis
• Fever

Peptic Ulcer Prevention


• Avoid NSAIDs (COX-2 if you must)
• Avoid aspirin (start early if you must)
• Antibiotics to eradicate H.pylori
• Proton pump inhibitors

1297
Copyright © Oakstone Publishing, 2021. All Rights Reserved.

Surgical Treatment (Historical)


• It was commonly believed that PUD was due to
excessive gastric acid production
• Prior to the introduction of histamine blocking
medications, surgery was the mainstay treatment
for PUD
• Intractable PUD
• Complications of PUD
• Prevention

Nyhus LM and Kraft R, Hardy’s Textbook of Surgery 1977

Surgical Options
• Truncal vagotomy with drainage
(pyloroplasty or gastrojejunostomy
• Selective vagotomy with drainage
• Highly selective vagotomy
• Truncal vagotomy and antrectomy
• Subtotal gastrectomy with BI or BII
reconstruction

1298
Copyright © Oakstone Publishing, 2021. All Rights Reserved.

Gastrojejunostomy (GJ)
• 1881 - Wolfler performed the first in 1881
to bypass an obstructing cancer
• 1881 - Billroth performed the first
successful Gastric resection with
gastroduodenostomy
• 1882 - Von Rydiger performed the first
gastric resection for ulcer disease
Blalock JB, AM J Surg 1981

Gastrojejunostomy (GJ)
• 1925 – Lewisohn identified 92 cases that gastric
resection (but not a GJ) resulted in anacidity
• Supported partial or subtotal gastrectomy (66-75%)
to significantly reduce gastric acidity
• 1930s – 66-75 % gastrectomy with GJ was the
procedure of choice for PUD

Gastrojejunostomy
Lewisohn R, Surg Gynecol Obstet 1925

1299
Copyright © Oakstone Publishing, 2021. All Rights Reserved.

The Vagus Nerve


• Dragstedt demonstrated that a transthoracic
truncal vagotomy resulted in a marked reduction
of the volume of gastric output and an increase in
the pH of the gastric fluid
• Nearly all of the patients had improvement of
their symptoms
• Many patients developed gastric hypotonicity and
a GJ was added to the vagotomy
Dragstedt LR, Ann Surg 1945

PUD Definitive Treatment


• Reserved for patients with recurrent
disease
• Only performed if shortly after the
perforation (< 6 hours) to prevent
infection from getting into the
mediastinum.

1300
Copyright © Oakstone Publishing, 2021. All Rights Reserved.

Truncal Vagotomy and Pyloroplasty


• 10-13% of patients developed recurrent
ulcers (thought to be from antral stasis)
• A modified Heinecke-Mikulicz pyloroplasty
was then added to the procedure to improve
drainage
• 1022 patients results:
• Mortality – less than 1%
• Recurrent ulcers – 5%
Weinberg JA, Am J Surg, 1963

Truncal Vagotomy and Antrectomy


• 1946 – Farmer and Smithwick performed a
truncal vagotomy and hemigastrectomy
• 1947 – Edwards and Herrington perform
truncal vagotomy and antrectomy
• Results – 3771 patients
• 94% minimal nutritional issues
• 0.5% recurrent ulcers
Blalock JB, AM J Surg 1981

1301
Copyright © Oakstone Publishing, 2021. All Rights Reserved.

Highly Selective Vagotomy


• Also known as a parietal cell vagotomy
• Griffith and Harkins were the first to denervate
the parietal cell mass without injuring the nerves
to the antrum and pylorus
• 1967 – Holle and Hart, performed the first in
man highly selective vagotomy (with drainage)
• 1970’s – HSV - Johnston and Wilkinson,
Amdrup and Jensen
Blalock JB, AM J Surg 1981

Subtotal Gastrectomy with BI or BII

Construction Complications
Subtotal gastrectomy Mortality = 3-4%
Antrectomy Dumping = 5-50%
BI or BII GJ Microcytic anemia
Vagotomy Vitamin B12 def
? Drainage procedure SIBO

Blalock JB, AM J Surg 1981

1302
Copyright © Oakstone Publishing, 2021. All Rights Reserved.

Conclusions
• The incidence of peptic ulcer disease is
declining as the diagnostic tools and
treatments are improving
• Once a surgical disease, PUD is now
essentially non-surgical
• However, the complications of PUD are still
serious and there is still a role for surgical
management

1303
Copyright © Oakstone Publishing, 2021. All Rights Reserved.

Small Bowel:
Management Challenges
Jason S. Gold

Chief of Surgical Oncology, VA Boston Healthcare System


Associate Professor of Surgery, Harvard Medical School
(Brigham and Women’s Hospital)

Nothing to disclose

1304
Copyright © Oakstone Publishing, 2021. All Rights Reserved.

Small Bowel: Management


Challenges

Acute mesenteric ischemia

Short bowel syndrome

Enterocutaneous fistula

Small Bowel: Management


Challenges

Acute mesenteric ischemia

Short bowel syndrome

Enterocutaneous fistula

1305
Copyright © Oakstone Publishing, 2021. All Rights Reserved.

Small Bowel: Management


Challenges
Mesenteric ischemia

• Classified:
– Acute vs. chronic
• Acute by degree (i.e. perforation, full thickness
necrosis)
– Territory affected
– Arterial vs. venous
• Arterial by embolic, thrombotic, nonocclusive
• Following limited to acute small intestinal
mesenteric ischemia

Small Bowel: Management


Challenges
Mesenteric ischemia: Mechanisms

• Arterial embolus – left heart or aorta


– Venous/right heart with shunting
– Oblique orientation of SMA to aorta
– Distal to middle colic and proximal jejunals
• Spares proximal jejunum and transverse colon
• Arterial thrombosis – exacerbation of chronic
– Predisposing event: dissection, infection, trauma
• Arterial nonocclusive – hypoperfusion and
vasoconstriction
– Watershed in colon

1306
Copyright © Oakstone Publishing, 2021. All Rights Reserved.

Small Bowel: Management


Challenges
Mesenteric ischemia: Mechanisms

• Arterial in older patients with


(atherosclerotic) vascular disease
• Venous thrombosis – hypercoaguable state
– External process: trauma/surgery, mass,
inflammatory process

Small Bowel: Management


Challenges

Mesenteric ischemia: Presentation

“Abdominal pain out of proportion to


the physical examination”

1307
Copyright © Oakstone Publishing, 2021. All Rights Reserved.

Small Bowel: Management


Challenges
Mesenteric ischemia: Presentation

• Arterial embolus – sudden, severe, periumbilical


– Often accompanied by nausea and vomiting
• Arterial thrombosis – worsening of chronic
symptoms
– Postprandial pain
• Arterial nonocclusive – variable symptoms
– May be overshadowed by signs and symptoms related
to precipitating disorder
• Hypotension, shock, heart failure, arrythmia
• Venous thrombosis – insidious onset of pain
– Can wax and wane for a period of time

Small Bowel: Management


Challenges

Mesenteric ischemia: Presentation

• Often mild tenderness and distention


initially until transmural necrosis/perforation
• Laboratory evaluation – nonspecific
– Leukocytosis, metabolic acidosis common
– Can ✓ lactate, amylase, phosphate (elevated)
• All not particularly sensitive or specific

1308
Copyright © Oakstone Publishing, 2021. All Rights Reserved.

Small Bowel: Management


Challenges
Mesenteric ischemia: Approach
• Peritonitis  OR
• Plain X-ray useful to exclude perforation or gross
pneumatosis/portal venous air
– When CT scan cannot practically be obtained (quickly)
• Shock
• CT angiography without oral contrast
– Exclude other causes of abdominal pain
– Clear vascular causes – SMA/SMV occlusion
– Clear signs of hollow viscus perforation – free air/fluid
– Suggestive findings
• Pneumatosis, portomesenteric venous gas, pneumobilia,
focal/segmental bowel wall thickening or lack of enhancement,
mesenteric stranding

Small Bowel: Management


Challenges
Mesenteric ischemia: Approach

• Angiography can be required for diagnosis of


nonocclusive mesenteric ischemia
– Dynamic and spastic etiology
– Dilation and narrowing – “string of sausages” or “chain
of lakes”
• Time to diagnosis and treatment is paramount
• Initial management – pain control, antibiotics
(bacterial translocation), supportive care
(fluids/monitoring), correction of electrolyte (as
needed), decompression (as needed)
• Anticoagulation for embolic/thrombotic disease

1309
Copyright © Oakstone Publishing, 2021. All Rights Reserved.

Small Bowel: Management


Challenges
Mesenteric ischemia: Surgery

• Exploration
– Resect gross or impending perforation to limit spillage
– Assess extent and severity of ischemia
• Color, peristalsis, pulsation in mesentery
• Questionable viability should be left until revascularization
– Assess for other pathology and cause of ischemia
• Revascularization – embolectomy, bypass, hybrid
• Reexamine bowel for clear irreversible injury
– Doppler, fluorescein, indocyanine green (ICG) adjuncts
• No clear superiority over visual inspection

Small Bowel: Management


Challenges
Mesenteric ischemia: Surgery

• Restoration of intestinal continuity in patients


– Well demarcated area of necrosis with remaining
bowel clearly viable in hemodynamically stable patient
• Liberal use of damage control approach
– 80% require second look to assess unresected bowel
– Temporary abdominal closure
– Bowel left in discontinuity
– Resuscitation in ICU
• Second look in 24-48 hours
• Palliation poor surgical risk and/or extensive
bowel resection

1310
Copyright © Oakstone Publishing, 2021. All Rights Reserved.

Small Bowel: Management


Challenges

Mesenteric ischemia: Treatment

• Embolus – SMA thromboembolectomy


– Transverse arteriotomy and balloon
embolectomy
– Catheter-directed aspiration or thrombolysis
• Cannot visualize bowel

Small Bowel: Management


Challenges
Mesenteric ischemia: Treatment

• Thrombosis – various approaches


– Tailored based on need to directly evaluate bowel
– Observation in selected patients with good collaterals
– Pure endovascular
• Catheter-directed thrombolysis
• Stenting (antegrade)
– Hybrid
• Stenting (including retrograde)
– Open
• Bypass
– Antegrade – supraceliac aorta
– Retrograde – infrarenal aorta or iliacs

1311
Copyright © Oakstone Publishing, 2021. All Rights Reserved.

Small Bowel: Management


Challenges
Mesenteric ischemia: Treatment

• Venous thrombosis – usually anticoagulation only


– Patients at risk for variceal bleeding due to increased
mesenteric venous pressure
• Can be from “ectopic” sites in small bowel or colon
– Transcatheter thrombolysis and/or thrombectomy
• Venous or arterial thrombolysis
• Non-occlusive – resuscitation (avoid inotropes
with vasoconstriction), correction of underlying
problem, eliminate vasoconstricting medications
– Catheter-directed vasodilator infusion unproven
– Role of anticoagulation unknown

Small Bowel: Management


Challenges

Mesenteric ischemia: Results

• Patients treated surgically have ~50%


mortality
• Mean length of stay 23 days
• Prompt diagnosis and treatment associated
with better results

1312
Copyright © Oakstone Publishing, 2021. All Rights Reserved.

Small Bowel: Management


Challenges

Acute mesenteric ischemia

Short bowel syndrome

Enterocutaneous fistula

Small Bowel: Management


Challenges
Short bowel syndrome
• Disabling malabsorptive condition caused by
inadequate bowel length
• Most common cause of intestinal failure
– Intestinal failure – insufficient absorption of fluid,
electrolytes and/or macronutrients required for health
• In adults usually results from surgical resection
– Massive resection – 75% mesenteric occlusion
• Primary acute mesenteric ischemia, volvulus, trauma
– Multiple sequential resections
• Crohn’s disease
– Prevention

1313
Copyright © Oakstone Publishing, 2021. All Rights Reserved.

Small Bowel: Management


Challenges

Short bowel syndrome: Risk

• Length of bowel, presence of ileum and ileocecal


valve, presence of colon, continuity
– Adults with residual small intestine <180 cm are at risk
• Measure what is left not (just) what is taken out
– 1/2 of the colon is approximately equivalent to
additional 50 cm of small bowel
– Loss of ileum/ileocecal valve causes more intestinal
dysfunction than loss of similar amount of jejunum

Small Bowel: Management


Challenges

Short bowel syndrome: Timeline

• Acute management
• Adaptation phase
– Structural and functional changes of the
remaining bowel to increase nutrient
absorption and slow gastrointestinal transit
• Lasts 1-2 years
• Management of persistent intestinal failure

1314
Copyright © Oakstone Publishing, 2021. All Rights Reserved.

Small Bowel: Management


Challenges
Short bowel syndrome: Acute phase
• For initial 3-4 weeks after lengthy intestinal
resection – management of large volume fluid
and electrolyte losses
• IVF – NS + K + Mg
– Replace losses on top of maintenance/nutrition
• Gastric acid suppression – PPI or H2 blockade
– Usually PPI BID
– Gastric acid hypersecretion for first several months
• Acid-peptic disease
• Increased fluid/electrolyte secretion
• pH-related decreased absorption
– Deactivation of pancreatic enzymes and direct reduction of fat absorption

Small Bowel: Management


Challenges
Short bowel syndrome: Acute phase
• Nutrition – TPN typically required initially
– Should be started when patient stabilized
– Enteral feeding should be trialed
• Even if cannot completely meet goals enterally can help
minimize TPN needed
– Promotes adaption
• Continuous feeding allows for continuous saturation of
absorption system
– Better net absorption of lipids, protein, total calories
– Better favors intestinal adaptation
– Better tolerated
• Using gut at night to allow for normal activities during day
• Generally standard formula
• Fiber may slow output but does not increase absorption

1315
Copyright © Oakstone Publishing, 2021. All Rights Reserved.

Small Bowel: Management


Challenges

Short bowel syndrome: Acute phase

• Anticathartics – slow transit and ↑ absorption


– Loperamide (Imodium), diphenoxylate-atropine
(Lomotil) usual first-line agents
– Opiates – tincture of opium and codeine may be added
– Octreotide in cases with profound fluid losses
• Increases small bowel transit time and reduces fluid losses
• Discontinue if no effect
– Clonidine (oral or transdermal) – modest benefit in
treating high output stool losses
• Presumably via effects on intestinal motility and secretion
• Clinical benefit yet to be determined

Small Bowel: Management


Challenges
Short bowel syndrome: Adaptive phase
• Diet –transition to oral feedings in a slow and
stepwise manner over weeks to months
– Avoid hypertonic fluids and carbohydrates (dumping)
• Complex carbohydrates over sugars
– If colon in continuity, restrict dietary oxalate
• Calcium oxalate nephrolithiasis
– Moderate fat restriction if colon in continuity
• Reduces fecal calorie loss, ↑ energy absorption, ↓ Mg/Ca loss,
↓ oxalate absorption
• Medium-chain triglyceride (MCT) supplementation
– Supplement essential FAs in low-fat diet or steatorrhea
– Fat restriction may impair adequate caloric intake and
weight stability when colon not in continuity

1316
Copyright © Oakstone Publishing, 2021. All Rights Reserved.

Small Bowel: Management


Challenges

Short bowel syndrome: Adaptive phase

• Hypotonic fluids ↑ output without colon in


continuity
• Supplementary fiber may be helpful
• Nutritional supplementation
– Fat-soluble vitamins
– B12 in terminal ileum resection
– Trace elements
• Zinc, copper, selenium

Small Bowel: Management


Challenges
Short bowel syndrome: Adaptive phase
• Avoid enteric-coated drugs and timed/delayed-
release medications
• Discontinue PPI and H2 blockade if possible after
6 months
– Gastric acid suppresses bacterial overgrowth
• Empiric antibiotics for typical symptoms of small
intestinal bacterial overgrowth
– Gas-bloat, diarrhea, discomfort
• Avoid/eliminate octreotide if possible
– Diminishes splanchnic protein synthesis
• Theoretically interferes with adaptation
– Predisposes to gallstones (already high risk)

1317
Copyright © Oakstone Publishing, 2021. All Rights Reserved.

Small Bowel: Management


Challenges

Short bowel syndrome: Adaptive phase

• Wean TPN proportionate to the amount of


oral and/or enteral
– No optimal weaning protocol
– Likely better to ↓ proportionally throughout
week than ↓ number of days given
– Monitor symptoms, hydration, electrolytes,
weight

Small Bowel: Management


Challenges

Short bowel syndrome: Adaptive phase

• Various hormones evaluated to ↓ TPN and


↑/maintain weight
– GLP-1, GLP-2, growth hormone, IGF-1
– Tedeglutide approved for this indication
• Long-acting GLP-2 analogue
• Initiates and maintains adaption and improves absorption
– 20 percent reduction in TPN compared with placebo
– 11 percent were able to be completely weaned
• Reserve for those unable to wean from TPN
• Particularly those with significant complications or severe
impairment in quality of life

1318
Copyright © Oakstone Publishing, 2021. All Rights Reserved.

Small Bowel: Management


Challenges

Short bowel syndrome: Complications

• Gallstones from disruption of the enterohepatic


circulation
• Nephrolithiasis from hyperoxaluria
– Increased absorption from fat/bile in colon and
decreased bacterial breakdown
• Complications from TPN
– Catheter-related
• Thrombosis, infection, mechanical
– Hepatic steatosis
• Esophagitis/peptic ulcer disease from gastric
hypersecretion

Small Bowel: Management


Challenges

Short bowel syndrome: Complications

• Bacterial overgrowth from adaptive dilation/↓


peristalsis and lack of IC valve
• Metabolic bone disease from
electrolyte/nutritional alterations
• D-lactic acidosis
– Rare neurologic syndrome characterized by altered
mental status
– From bacterial fermentation of unabsorbed
carbohydrates
• Primarily in patients with an intact colon

1319
Copyright © Oakstone Publishing, 2021. All Rights Reserved.

Small Bowel: Management


Challenges

Short bowel: Persistent intestinal failure

• Complete reversal in ~50% adults within 2 years


– Minimal chance of resolution after
– Length, location, and health of the remaining intestine
predict ability to adapt
• Proximal bowel resection is tolerated better than distal
– Ileum adapts/↑ absorptive capacity more efficiently than the jejunum.

– High chance of TPN independence with small bowel


length ≥75 cm when colon connected

Small Bowel: Management


Challenges
Treatment of persistent intestinal failure

• Restoration of intestinal continuity


• Relief of strictures
• Many surgical procedures tried/limited success
– Procedures to delay intestinal transit time
– Procedures to increase absorptive area
– More commonly done in children
– Experienced surgeons in carefully selected patients

1320
Copyright © Oakstone Publishing, 2021. All Rights Reserved.

Small Bowel: Management


Challenges
Treatment of persistent intestinal failure
• Poor motility and dilated bowel
– Serial transverse enteroplasty procedure (STEP)
– Longitudinal intestinal lengthening and tailoring (LILT,
Bianchi procedure)
• Fast intestinal transit without bowel dilatation
– Insertion of antiperistaltic segments of small intestine
• Intestinal transplantation
– Failure to wean from TPN with inability to manage
nutrition/hydration or irreversible complications
• Overt/impending liver failure, central venous thrombosis,
recurrent line-related sepsis
– 1- and 5-year survival 77% and 58%

Small Bowel: Management


Challenges

Acute mesenteric ischemia

Short bowel syndrome

Enterocutaneous fistula

1321
Copyright © Oakstone Publishing, 2021. All Rights Reserved.

Small Bowel: Management


Challenges
Enterocutaneous fistula
• Fistula – abnormal connection between 2
epithelialized hollow spaces or organs
– Classified by:
• Origin and point of termination
• Output volume
– Low-output fistula drains <200 ml/day
– Moderate-output fistula drains 200-500 ml/day
– High-output fistula drains >500 ml/day
• Etiology
• Annual cost >$500 million in US
• Mortality historically 70% now 10-20%

Small Bowel: Management


Challenges

Enterocutaneous fistula: Etiology

• 75-85% EC fistulas are iatrogenic or


postoperative
– 1.5% trauma surgery, 3.6% general surgery,
15-35% Crohn’s surgery results in EC fistula
• Up to 21% open abdomen cases develop EC fistula
• 15-25% develop spontaneously
– Crohn’s disease most common

1322
Copyright © Oakstone Publishing, 2021. All Rights Reserved.

Small Bowel: Management


Challenges

Enterocutaneous fistula: Etiology

Foreign body
Radiation
Inflammation/Infection
Epithelialization
Neoplasia
Distal obstruction

Small Bowel: Management


Challenges
Enterocutaneous fistula: Presentation
• Most commonly presents after abdominal surgery
– Patient fails to recover normally
– Initially abdominal symptoms
• Pain, nausea and vomiting, obstipation, fullness/induration of
abdominal wall
– Fever and leukocytosis
– Wound infection usually recognized POD 7-10
• Ultimately drains enteric/bilious material
• Diagnosis usually clinical
– Exception small/very low output fistulas
• CT/MR enterography, small bowel follow-through, fistulogram
• Dye (indigo carmine, methylene blue, charcoal) can be
ingested or instilled via endoscopy and output visualized

1323
Copyright © Oakstone Publishing, 2021. All Rights Reserved.

Small Bowel: Management


Challenges

Enterocutaneous fistula: Management

• Initial phase
– Assess for sepsis and control if present
• Chronic phase
– Support patient, allow for spontaneous closure
• Definitive phase
– Repair

Small Bowel: Management


Challenges
Enterocutaneous fistula: Acute phase

• Initial management focuses on identification


control and treatment of sepsis
– Etiology/anatomy not important
• Infection/sepsis responsible for 70% of mortality
– Goal is to reduce mortality/organ dysfunction
– Low suspicion for intraabdominal sepsis
– Broad-spectrum antibiotics
– Source control
• Principles equivalent to other intra-abdominal emergencies
– Diffuse peritonitis  OR
– Otherwise evaluate with imaging (usually CT)

1324
Copyright © Oakstone Publishing, 2021. All Rights Reserved.

Small Bowel: Management


Challenges

Enterocutaneous fistula: Source control

• Early definitive repair


– Rare exceptions
– Patients who present very early (POD 1-2), typically
with inadvertent enterotomy without fistulation,
significant intraabdominal inflammation or
hemodynamic compromise
• Image-guided percutaneous drainage
– Usually transabdominal but also transgluteal or even
transgastric, -rectal, -vaginal

Small Bowel: Management


Challenges
Enterocutaneous fistula: Source control
• Operation for peritonitis or inability to completely
drain percutaneously
– Goal is expeditious control of sepsis
– Usually drainage with diversion or exteriorization
– Avoid repairing or redoing anastomosis/creating new
one in infected field
• Antibiotic management and resuscitation per
Surviving Sepsis
– Based on STOP-IT, short fixed course of antibiotics
after source control definitive established
• Drains left in place until closure of fistula
– Output <10 ml/day and/or no longer enteric

1325
Copyright © Oakstone Publishing, 2021. All Rights Reserved.

Small Bowel: Management


Challenges
Enterocutaneous fistula: Chronic phase

• Focus is on fluids/electrolytes, nutrition, wound


management
– Resuscitate patient, attempt to reduce output/promote
closure, protect wound/skin
– Early aggressive correction of hypovolemia and
electrolyte loss
• Up to ml per ml replacement
• Hyponatremia, hypokalemia, hypomagnesiumia most common
• Replacement fluid should mimic fluid lost as much as possible
– Better nutrition = lower mortality
• Healing rate of the fistula improves with nutritional optimization

Small Bowel: Management


Challenges
Enterocutaneous fistula: Chronic phase

• Enteral feeding, TPN or combination


– Depends upon fistula output
– Energy requirement up to 2.5 times ↑
• Higher with ongoing infection, large open abdominal wounds,
high-output fistulas
– Doses of vitamins and trace elements required for
healing ↑
• Vitamin C, zinc, copper, selenium
– Attention to vitamins/elements not absorbed based on
anatomy or length of bowel in continuity
• Principles similar to short bowel

1326
Copyright © Oakstone Publishing, 2021. All Rights Reserved.

Small Bowel: Management


Challenges
Enterocutaneous fistula: Chronic phase

• Generally patients managed with a short period


of NPO after initial diagnosis
– Allow for investigation and treatment of sepsis
– Establish output and behavior of fistula
– May not require TPN
• If fistula closing rapidly, may hold PO to allow
closure
• If output stable or decreasing, may trial PO
– Assess if output/behavior changes
– Monitor ability to control sepsis and prevent excoriation
of skin/damage to wound

Small Bowel: Management


Challenges
Enterocutaneous fistula: Chronic phase

• Output of fistula and length of bowel proximal to


fistula correlate with ability to tolerate full enteral
intake
• Enteral feeding always preferred to TPN
– Preserves intestinal mucosal barrier
– Effects on immunologic and hormonal gut functions
• TPN added for failure to tolerate enteral or to
supplement

1327
Copyright © Oakstone Publishing, 2021. All Rights Reserved.

Small Bowel: Management


Challenges
Enterocutaneous fistula: Chronic phase

• Protect skin/wound from corrosive enteric


contents (acid/pancreatic enzymes)
– Very low output may need nothing special
– Try to convert into a stable functional stoma
– Many techniques utilized
• Pouching
• Protective creams
– Barrier, topical treatment of cellulitis/fungal infection
• Various creative approaches to isolate fistula from wound
• NPWT (e.g. V.A.C.®) widely used
– Theoretical concerns of keeping fistula open, increasing output
– Concern of creating new fistulas with bowel in close proximity
– Minimizes contact with output, may make it easier to manage
– Data show accelerated fistula closure without increased rate of closure

Small Bowel: Management


Challenges
ECF: Output reduction

• Gastric acid blockade – PPI or H2


– Standard therapy
– Reduces output, does not facilitate closure
• Anticathartics
– Similar principles to short bowel syndrome
– Somatostatin and analogues can ↓ fistula duration and
hospital stay without ↑ closure rate
– Given when output cannot be lowered beyond 1L
• Tried for a few days to see if decreases output
• When successful, continued – depot form can be used

1328
Copyright © Oakstone Publishing, 2021. All Rights Reserved.

Small Bowel: Management


Challenges

ECF: Spontaneous closure

• 1/3 of fistulas close in 5-6 weeks


– 90% of closures within month of sepsis
resolution
– 10% during the second month
– None after 2 months

Small Bowel: Management


Challenges
Enterocutaneous fistula: Definitive repair

• Various endoscopic/non-surgical options for


fistula closure have been attempted
– Endoscopic clipping, fistula plugs, stents, fibrin sealant
• Timing of definitive surgery a matter of judgment
– Should be delayed as long as fistula output is ↓ and
the wound or tract healing
– Generally postponed at least 3-6 months
• Improvement in inflammation/adhesions – may ↓ bowel injury
• Post-operative fistulas ↑ with earlier surgery
• Specialized centers often delay 6-12 months

1329
Copyright © Oakstone Publishing, 2021. All Rights Reserved.

Small Bowel: Management


Challenges

Enterocutaneous fistula: Definitive repair


• Patient needs to be nutritionally replete, free of
infection, and have supple, soft tissues adjacent
to the fistula
– Serum albumin correlates with perioperative mortality
– Ability to pinch and raise incision/skin graft
• Workup to exclude ongoing intraabdominal
collections and to determine anatomy
– CT for abscesses, fluid collection
– CTE/MRE, small bowel series, fistulograms can be
helpful in determining anatomy
– Anatomy determined any time after patient stabilizes
• In acute phase it usually does not change management

Small Bowel: Management


Challenges

Enterocutaneous fistula: Definitive repair


• Work-up for other relevant pathologies as needed
– Neoplasia, distal obstruction…
– When Crohn’s suspected endoscopy usually indicated
• Patient needs to be mentally prepared and
expectations managed
– Repair can be long, complex procedure
– Difficult recovery
– Risk of complications such as SSI, hernia
– Recurrence rate as high as 20-36%
– Diverting stoma may be required

1330
Copyright © Oakstone Publishing, 2021. All Rights Reserved.

Small Bowel: Management


Challenges

Enterocutaneous fistula: Definitive repair

• Goals of procedure:
– 1st do no harm
– Eliminate the fistula
– Reestablish gastrointestinal continuity
– Tension-free closure of the abdomen

Small Bowel: Management


Challenges
Enterocutaneous fistula: Definitive repair
• Incision
– Placement where less adhesions expected
– Must be able to provide adequate exposure
• Significant adhesiolysis and running of bowel to be expected
– Must allow for closure with well vascularized tissue
– Must accommodate procedures needed for abdominal
closure when fascia not intact preoperatively
• Adhesiolysis
– Meticulous
• Be prepared to spend hours
– Try to work from areas least involved first
• Goal is not to create further injury/fistula
– All manipulated bowel will need to be inspected

1331
Copyright © Oakstone Publishing, 2021. All Rights Reserved.

Small Bowel: Management


Challenges
Enterocutaneous fistula: Definitive repair
• Clear segment of bowel with fistula so that
pathology can be defined and addressed
– Resection of fistula with restoration of GI continuity
• Segmental resection has ↓ recurrence than wedge
• In Crohn’s resect adjacent diseased bowel
• Address other pathologies relevant to fistula
– Foreign body, infection, neoplasm, distal obstruction
• Reconstruct abdominal wall with durable, well-
vascularized tissue
– Sometimes requires advanced flap techniques
• Large defects/loss of domain, prior stomas/fistulas
– Consider use of a two-team approach
• Well-rested plastic surgeon for complex closures

Small Bowel: Management


Challenges
Key points

Acute mesenteric ischemia


• Abdominal pain out of proportion to physical examination
• Time to diagnosis and treatment is paramount
Short bowel syndrome
• Supportive care early on
• Allow for adaptation, which can last up to 2 years
Enterocutaneous fistula
• Think of FRIENDS
• Initially assess for sepsis and control if present; then
support patient and allow for spontaneous closure;
generally postpone definitive repair for 3-6 months

1332
Copyright © Oakstone Publishing, 2021. All Rights Reserved.

Inflammatory Bowel Disease

Joel Goldberg, MD, MPH


Assistant Professor of Surgery
Division, Colon and Rectal Surgery
Brigham and Women’s Hospital
Harvard Medical School

Disclosures
Dr. Goldberg is a Consultant for Medtronic.

1333
Copyright © Oakstone Publishing, 2021. All Rights Reserved.

Overview

• Crohn’s Disease and Ulcerative Colitis


– Background
– Diagnosis and Evaluation
– Medical therapy
– Surgical therapy

Background
• Crohn’s disease and ulcerative colitis
collectively referred to as IBD
• Also Indeterminate Colitis
• Present in different distributions in the GI tract
• Present with different clinical symptoms.

1334
Copyright © Oakstone Publishing, 2021. All Rights Reserved.

IBD Etiology: Associations


• Genetic predisposition (150+ genes)
• NOD2 gene most common
• Bacterial recognition
• Foreign antigen causes immune dysregulation
• Overlap genes between UC and Crohn’s
• Smoking:
– UC: protective
– Crohn’s: exacerbates
• NSAID’s and family history predictive in both
• Above factors lead to mucosal inflammation and
loss of mucosal integrity

IBD Etiology: Associations

• Increased prevalence industrialized environment


– Urban
– Socioeconomic: high more common
– Geography: northern climates
– Ashkenazi Jews
• Viruses: EBV, CMV, Norovirus
• Bacteria: Bacteroides, E. Coli et. al.
• Fungi: Candida, Aspergillosis, Saccharomyces

1335
Copyright © Oakstone Publishing, 2021. All Rights Reserved.

IBD Etiology: Associations


• Microbiome/Hygiene Hypothesis:
– Urban areas more sanitary less exposures
– Exaggerated immune response when exposed
• Family member with IBD #1 risk factor for IBD
– Concordance for age of onset, location, behavior
• Gene associations: MS and RA

IBD Etiology: Cancer Risk


• Increased risk in both UC and Crohn’s
– 15% of IBD patients die from colorectal cancer
– Frequent surveillance with chromoendoscopy
– p53 and APC mutations in affected tissue
– Faster and not normal progression as in non-IBD pts
• APC first step in normal cancer, late in IBD
• p53 mutations increased in IBD

1336
Copyright © Oakstone Publishing, 2021. All Rights Reserved.

Ulcerative Colitis

• Classic presentation bloody diarrhea


• Important to take a good history and
physical
– Rapid onset
– Bloody diarrhea
– Weight loss
– Urgency/tenesmus
– Crampy abdominal pain

Ulcerative Colitis

• Physical exam: evaluate for signs of toxicity


• Abdominal exam, tender, distended
• Vitals: fever, tachycardia
• Low urine output
• Anal exam to exclude perianal Crohn’s disease
• R/o infectious causes of bloody diarrhea
• Colonoscopy extent of disease extent

1337
Copyright © Oakstone Publishing, 2021. All Rights Reserved.

Ulcerative Colitis

• Classic presentation: begins in distal rectum


• Extends proximally confluent and continuous
inflammation
• Only affects the mucosa, no fissuring or skip
lesions
• No Granuloma’s (except in crypt rupture)
• Medically treated can have skip areas

Ulcerative Colitis

Steele at al. ASCRS Textbook of Colon and Rectal Surgery. Third Edition

1338
Copyright © Oakstone Publishing, 2021. All Rights Reserved.

Macroscopic features: UC vs Crohn’s

Steele at al. ASCRS Textbook of Colon and Rectal Surgery. Third Edition

Microscopic Features: UC vs Crohn’s

Steele at al. ASCRS Textbook of Colon and Rectal Surgery. Third Edition

1339
Copyright © Oakstone Publishing, 2021. All Rights Reserved.

Inflammatory Markers
• Serology --elevated in other inflammatory diseases
• pANCA - perinuclear antineutrophil cytoplasmic Ab’s
• ASCA- anti-Saccharomyces cerevisiae antibodies
• Common measures of acute inflammation
• CRP
• ESR
• Nonspecific if pt has other inflammatory disease:
• Rheumatoid Arthritis
• SLE
• Can help exclude IBD from functional disorders

Crohn’s Disease

• Can affect entire digestive tract:


• Mouth to anus
• Discontinuous
• Three major patterns
• Colonic: 30%
• Ileocolonic: 50%
• Small Intestine: 15-20%
• Anorectal alone: 1%

1340
Copyright © Oakstone Publishing, 2021. All Rights Reserved.

Crohn’s Disease

• Full thickness inflammation


• Strictures
• Fistulas
• Abscesses
• Bleeding
• Pathology/Histology
• Granuloma’s
• Full thickness
• Creeping fat
• Aphthous ulcers

Indeterminate Colitis

• Only after colectomy


• No definitive diagnosis
• No small bowel involvement
• Worse outcome with IPAA
– Sepsis
– Fistula
– Pouch failure

1341
Copyright © Oakstone Publishing, 2021. All Rights Reserved.

Colonoscopy in Ulcerative Colitis


• Risk of colorectal cancer increases at 8-10 yrs
• After 8-10 years colonoscopy recommended every 1-2 years
• Evaluate for dysplasia
• Chromoendoscopy: 1.8-3.5 times more likely to diagnose dysplasia
• 20% of cancers occur within first 8 years
• Cumulative risk of cancer in UC
• 10 yrs: 2.1%
• 20 yrs: 8.5%
• 30 yrs: 17.8%
• Strictures: 24% malignant
• PSC yearly colonoscopy due to higher cancer risk
• Four quadrant sampling every 10 cm

Steele at al. ASCRS Textbook of Colon and Rectal Surgery. Third Edition

Colonoscopy in Crohn’s Disease


• Colonoscopy recommended 6-12 months for Crohn’s
after surgery to evaluate for anastomotic recurrence

• Strictures: 6% malignant

• Indicated in change in symptoms

• Prior to surgery if not done recently

1342
Copyright © Oakstone Publishing, 2021. All Rights Reserved.

Calprotectin

• Calcium binding protein


• Presence signifies mucosal inflammation
• Useful in UC and colonic Crohn’s
• Elevated level predict endoscopic
recurrence in Crohn’s
• Better than CRP, ESR

Ulcerative Colitis: Medical Management

• Mild to moderate disease


– Bottom up manner: less expensive/effective added until
desired effect
• Oral 5-ASA agents
• Steroids for flares
• Purine analogues
• Biologics
• Moderate to severe disease
– Top down matter: Aggressive therapy then wean once
remission
• Biologic first
• Immune modulator: purine analogues

1343
Copyright © Oakstone Publishing, 2021. All Rights Reserved.

Ulcerative Colitis: Medical Management

• When considering surgery rule out


– CMV: biopsy
– C. Diff: culture
• 75% affected never need surgery
– 50% in remission
– 50% active disease
• Dysplasia in flat lesions
– Colectomy due to 20-30% cancer risk
– >50% cancer risk in 5 years

Ulcerative Colitis: Medical Management

• Goals of treatment
– Induce remission
– Avoid steroids
– Improve QoL
– Prevent cancer

1344
Copyright © Oakstone Publishing, 2021. All Rights Reserved.

Ulcerative Colitis: Medical Therapies

• 5-ASA agents: mild to mod disease\


– Side effects: sunburn, GI upset
• Purine analogues (6-MP and Azathioprine)
– Side effects: pancreatitis, hepatitis, leukopenia

Ulcerative Colitis: Biologics


• Infliximab (Remicade, Inflectra)
• mouse human mAb
• Anti-TNF agents: 5mg/kg at 0,2,6 weeks
• Then 5mg/kg q8 wks.
• Max dose 10mg/kg q4 weeks
• Need to r/o TB and HepB prior to starting
• Adalimumab (Humira) anti-tnf
• Human mAb
• Not as effective as infliximab
• Certolizumab (Cimzia): anti-tnf
• Golimumab (Simponi): humanized anti-tnf

1345
Copyright © Oakstone Publishing, 2021. All Rights Reserved.

Ulcerative Colitis: Biologics

• Anti-Integrin Antibodies
– Vedolizumab (Entyvio)
• mAb against alpha4Beta7 (aka LPAM-1)
• Risk: PML (progressive multifocal
leukoencephalopathy
– Natalizumab (Tysabri)
• Off label for CUC
• Risk: PML
• JC virus activation

Ulcerative Colitis: Miscellaneous

• Steroids
– IV solumedrol
– Budesonide
• Cyclosporine/Tacrolimus (calcineurin inhibitors)
• Bind to T-cell receptors inhibit cytokine release
• Methotrexate (anti-metabolite)
• Inhibits folate metabolism
• Teratogenic (so not in women contemplating
Pregnancy

1346
Copyright © Oakstone Publishing, 2021. All Rights Reserved.

Crohn’s: Medical Therapy


• Therapy guided by several factors
– Age
– Extent of disease
– Disease behavior
– Symptom severity
– Response to therapy
– Risks of therapy
– Prior number of surgeries

Crohn’s: Indications for surgery

• Failed medical management


• Complications of disease
– Bleeding: refractory anemia
– Obstruction: strictures
– Infection: abscesses
– Perforation: acute or chronic (abscess)
– Neoplasia: dysplasia and cancer

1347
Copyright © Oakstone Publishing, 2021. All Rights Reserved.

Crohn’s: Disease Classification

• Phenotype evolves (80%) over time,


location doesn’t change (15%)
• Inflammatory
• Stricturing
• Penetrating
• Fistulizing

Crohn’s: Classification
• CDAI: Crohn’s disease activity index
– Liquid stool frequency
– Abdominal pain severity
– General well-being
– Extraintestinal symptoms
– Need for anti-diarrheal drugs
– Abdominal mass
– Hematocrit
– Body weight
• HBI: Harvey-Bradshaw index: simply above
– General well-being, abdominal pain, number liquid
stools, abdominal mass, complications

1348
Copyright © Oakstone Publishing, 2021. All Rights Reserved.

Crohn’s: Medications
• Probiotics: ineffective in achieving remission
• Antibiotics: inhibition of pathogenic bacteria
• 5-ASA compounds: rarely used in Crohn's
• Glucocorticoids:
– IV achieve solumedrol
– Budesonide does not maintain remission
• Thiopurines: maintain remission
– side effects: leukopenia, hepatitis, pancreatitis,
increased risk lymphoma
• Methotrexate: maintain remission
– hepatic fibrosis, pneumonitis

Crohn’s: Medications
• Biologic Therapy: anti-TNF’s
– Infliximab
– Adalimumab
– Certolizumab
• Biologic Therapy: Integrin antagonists
– Natalizumab (Tysabri)
– Vedolizumab (Entyvio)
– Ustekinumab (Stelara

1349
Copyright © Oakstone Publishing, 2021. All Rights Reserved.

Crohn’s: Medical Tx after Surgery

• Risk for surgery in Crohn’s:


– 1 year: 16%
– 5year: 33%
– 10 year: 47%
• Endoscopic recurrence: 54% at 5 years
• Need to maintain disease remission

Crohn’s: Surgical Management

• Goals of surgical therapy


– Alleviate symptoms
– Preserve small intestine
– Preserve continence
– Decrease recurrence rates
– Decrease complications

1350
Copyright © Oakstone Publishing, 2021. All Rights Reserved.

Crohn’s: Surgical Management


• 75% at least one bowel resection
• Time to bowel resection after diagnosis
– 1 year: 44%
– 5 year: 61%
– 10 year: 71%
• More likely to require surgery
– Diagnosed < 40 years
– Smokers
– Stricturing/penetrating disease
– Ileal or ileocolic disease > Colonic disease
• RR: 3.2 compared to colonic disease

Crohn’s: Surgical Management


• Anti-TNF therapy reduces risk of surgery
• Most common indication: failed medical mgmt.
– Failure to wean steroids within 6 months
• Obstruction: 25% of Crohn's surgeries
– Inflammatory stricture trial steroids
– Anastomotic stricture can dilate
• 40-80% success with one dilation and good long term results
• Perforation: 1-3% of the time
• Usually toxic colitis: Total abdominal colectomy
w/ileostomy
• Small intestine then SBR.

1351
Copyright © Oakstone Publishing, 2021. All Rights Reserved.

Crohn’s: Surgical Management


• Bleeding: 2-13%
– Tx like regular LGIB
– localize with Tagged Scan, Angio, endoscopy
– Limited resection
• Abscesses: 7-25%
– Microperforation, fistula and abscess forms
– Ileocolic region most common
– Drainage and Abx prior to resection

Crohn’s: Surgical Management


• Fistula’s: 15-25%
– Enteroenteric, enterocutaneous
– Bladder/vagina
• Treat symptomatic fistulas only
– Resect diseased bowel, unaffected organ doesn’t need
resection, repair the wall only

1352
Copyright © Oakstone Publishing, 2021. All Rights Reserved.

Crohn’s: Surgical Management


• Cancer and Dysplasia: 1/1000 pts per year disease
– 2-3 fold increased risk CRC
– 18.75 fold increased risk small. intestine cancer
– Onset Crohn's to cancer is 20 years
– 60% in area of active disease
– 40% in area distant from active disease
– Overall survival worse
– Bypassed bowel cancers 0% survival in some studies
• Use bypass limited fashion
• Remove rectal stumps
• Resect colon strictures (closely monitor)

Crohn’s: Surgical Management


• High Grade Dysplasia in Crohn’s
– Indication for colectomy
– 44% of patients have multifocal dysplasia on colectomy
– On Colonoscopy HGD who have surgery
• 73% had HGD or a cancer
– On Colonoscopy LGD who had surgery
• 36% had HGD on resection

1353
Copyright © Oakstone Publishing, 2021. All Rights Reserved.

Crohn’s: Surgical Management


• Toxic Colitis:
• Disease flare + acute abdomen and toxicity
• Increased BM’s
• Fever
• Tachycardia
• Anemia
• Colonic dilation (toxic megacolon)
• Hypotension
• Confusion

Crohn’s: Surgical Management


• Treatment:
• IVF’s and electrolyte replacement
• Transfusion
• r/o CMV and C. Diff
• Abx if superimposed infection
• DVT prophylaxis
• Corticosteroids
• Infliximab
• Surgery if fails above
• Total abdominal colectomy with ileostomy
• Increased mortality w/perforation (8% non vs 40% in perf)

1354
Copyright © Oakstone Publishing, 2021. All Rights Reserved.

Crohn’s: Surgical Management


• Surgical goals in Crohn’s disease
• Non-curative so preserve small bowel
• Preoperative assessment critical
• Nutrition: malnourished higher mortality and complications
• Immunosuppression:
• Try to get off steroids
• Infliximab no increased complications
• Anemia
• Prior surgeries
• Age

Crohn’s: Surgical Management

• Operative choices
• Resection w/anastomosis
• isolated disease.
• Multiple stricture in close proximity
• Resection with diversion
• To protect a distal anastomosis
• Diversion
• Lots of inflammation and innocent bystander bowel involved
• Strictureplasty
• Multiple strictures separated by long distance of bowel
• Approaching short gut (100 cm) due to ,multiple resections

1355
Copyright © Oakstone Publishing, 2021. All Rights Reserved.

Crohn’s: Surgical Management

• Crohn’s colitis (spared rectum)


• Segmental resection vs total abdominal colectomy with
ileorectal
• Controversial either approach is fine.
• About 50% of patients with segmental colectomy need a repeat
operation.
• Segmental resection better function than IRA
• Both IRA and segmental resection delay stoma formation
• Patients with proctitis not a candidate for this approach

Crohn’s: Surgical Management

• Disease recurrence (one study)


• Endoscopic recurrence: 75% at 1 year (20% symptomatic)
• Endoscopic recurrence: 85% at 3 years (35% symptomatic)
• Risk for second surgery is approximately 30%
• 5 years: 25%
• 10 years: 35%
• Smoking is biggest factor in recurrence

1356
Copyright © Oakstone Publishing, 2021. All Rights Reserved.

Ulcerative Colitis: Surgical Management

• Reasons for surgery


– Failed medical management
– Dysplasia
– Cancer

Ulcerative Colitis: Surgical Management

• Failed medical management


– Medically sick
• Malnourished
• Anemic
• Inpatient
• Multiple immunosuppressives
• Three stage operation
• Total colectomy with ileostomy
• Proctectomy with IPAA and DLI
• Ileostomy reversal

1357
Copyright © Oakstone Publishing, 2021. All Rights Reserved.

Ulcerative Colitis: Surgical Management

• Not medically sick:


– Dysplasia or failed medical management
– Two stage operation
• Total proctocolectomy with IPAA and DLI
• Ileostomy reversal
• CRC risk
• 2% at 10 years
• 8% at 20 years
• 18% at 30 years

Ulcerative Colitis: Surgical Management

• Total Proctocolectomy with end Ileostomy


– Older
• Men >70
• Women >65
– Poor sphincter function
– Dysplasia at rectal cuff

1358
Copyright © Oakstone Publishing, 2021. All Rights Reserved.

Summary IBD
• Heterogeneous diseases often related
• Difficult to differentiate on occasion
• Good results with attention to detail

Question
• A 48 y/o female with a 20 year h/o ulcerative colitis has a
colonoscopy and there is multifocal dysplasia in the right
and left colon the rectum is spared. She has been
maintained on 5-ASA agents for the entire time. The most
appropriate treatment is:

• A. Start biologic therapy with infliximab


• B. Total abdominal colectomy with ileorectal anastomosis
• C. Three stage proctocolectomy with IPAA
• D. Proctocolectomy with IPAA and loop ileostomy.

1359
Copyright © Oakstone Publishing, 2021. All Rights Reserved.

Answer
• A. Start biologic therapy with infliximab
• Cancer risk mandates colectomy

• B. Total abdominal colectomy with ileorectal anastomosis


• Cancer risk can still be in rectum

• C. Three stage proctocolectomy with IPAA


• Reserved for medically ill patients with severe CUC

• D. Proctocolectomy with IPAA and loop ileostomy.


• The answer for a stable patient who is medically fit

1360
Copyright © Oakstone Publishing, 2021. All Rights Reserved.

Emergencies of the Large


Intestine
Jim Yoo
Colorectal Surgery
Brigham and Women’s Hospital
Associate Professor
Harvard Medical School

Disclosures

Advisory Role: Processa


Pharmaceuticals; Auris Health

I have no disclosures relevant to this


presentation.

1361
Copyright © Oakstone Publishing, 2021. All Rights Reserved.

Emergencies of the Large Intestine

Large bowel
obstruction

appendicitis
diverticulitis

Appendicitis

• Inflammation of the appendix


• Most common abdominal surgical emergency

1362
Copyright © Oakstone Publishing, 2021. All Rights Reserved.

Appendicitis - Clinical Presentation

• Periumbilical pain that radiates to the RLQ


• Localized peritoneal signs
• Rovsing’s sign, Psoas sign
• Leukocytosis

Differential Diagnosis
IBD, diverticulitis, TOA, ovarian torsion, ectopic pregnancy

Radiographic studies: u/s, CT scan

Non-perforated Appendicitis: Operative


and Non-Operative Management
• Operative Management
• appendectomy (open or laparoscopic)
• ideally within 12 hrs

• Non-Operative Management
• 90% avoid surgery on the initial hospital
admission
• roughly 30% will develop recurrent symptoms
and may need subsequent surgery within the 1st
year

1363
Copyright © Oakstone Publishing, 2021. All Rights Reserved.

Perforated Appendicitis - Management

• Operative vs. non-operative management:


• Contained vs. free perforation
• Abscess or phlegmon

Non-operative management
• bowel rest
• IV abx
• serial exams
• treatment failure requires surgery

• abscess (>3cm) – IV abx and percutaneous drainage


• phlegmon – depends on the extent of involvement (need for ileocolic resection)

Colonic Obstruction - Clinical


Presentation

• inability to pass flatus or stool


• abdominal distention
• Nausea/emesis

1364
Copyright © Oakstone Publishing, 2021. All Rights Reserved.

Colonic Obstruction

Differential Diagnosis
• Carcinoma
• diverticular disease
• IBD
• Volvulus
• pseudo-obstruction

Radiographic Evaluation:

KUB, CT, gastrograffin enema

Colonic Obstruction – Role of


Endoscopic Stents

• Bridge to Surgery
• Elective resection
• Palliation

1365
Copyright © Oakstone Publishing, 2021. All Rights Reserved.

Colonic Obstruction – Surgical Options


3, 2, and 1 Stage Operations

Resect with Primary Anastomosis

a) Resect/Stoma, Stoma Takedown


b) Resect with Primary Anastomosis
+ diverting ileostomy

Divert, Resect/Anastomosis, Stoma Takedown

Right Sided Obstruction Transverse Obstruction

• Staged operation (1-2) • Staged operation (1-2)


• Stent then surgery

Left Sided Obstruction

• Staged operation (1-3)


• Stent then surgery
• Subtotal colectomy

1366
Copyright © Oakstone Publishing, 2021. All Rights Reserved.

Volvulus

• Sigmoid most common (60%)


• Cecum (20-40%)
• Transverse (3-4%)
• Splenic flexure (2%)
• Ileosigmoid knotting (1%)
Base of volvulus

Radiographic Studies
KUB
CT

Volvulus
Cecal
Cecopexy
resection

Sigmoid
Colonoscopic decompression (rigid vs. flexible)
Resection (elective vs. urgent)

1367
Copyright © Oakstone Publishing, 2021. All Rights Reserved.

Diverticulitis

• Defined as inflammation caused by


perforation of a diverticulum
• Associated with >200,000 hospital
admissions per year in the US
• Leads to ≈50,000 colectomies/yr in the
US

Diverticulosis - Pathophysiology
• Herniation in the bowel
wall
• Pulsion diverticula (not
all layers of bowel wall)
• Most commonly occur in
the sigmoid colon
• Risk factors include age,
obesity, physical
inactivity, aspirin/NSAID
use, genetics

1368
Copyright © Oakstone Publishing, 2021. All Rights Reserved.

Diverticulitis - Pathophysiology

• Inflammation that results from


perforation of a diverticulum (leading to
bacterial infection)
• Not due to an obstruction at the neck of
a diverticula
• Popcorn, seeds and nuts DO NOT
cause diverticulitis
• Develops when colonic pressure
exceeds the wall tension of a
diverticula

Diverticulitis - Pathophysiology

• Inflammation that results from perforation of a


diverticulum
OR
• Inflammation of a diverticulum that may or may not
result in perforation (are antibiotics always
necessary?)
- segmental colitis associated with diverticulosis
- symptomatic uncomplicated diverticular disease (SUDD)

1369
Copyright © Oakstone Publishing, 2021. All Rights Reserved.

Classification

Contained Perforation
Hinchey I Hinchey II
- uncomplicated
- complicated

Free Perforation (10%)


Hinchey III Hinchey IV - Generalized peritonitis
- Systemic symptoms
- Emergency surgery

Diverticulitis - Clinical
Presentation
• Symptoms
• Acute abdominal pain, often in the left lower quadrant
• Fevers, change in bowel habits
• Physical Exam
• Abdominal tenderness
• Abdominal distention
• Laboratory Tests
• Leukocytosis

1370
Copyright © Oakstone Publishing, 2021. All Rights Reserved.

Radiologic Evaluation

• CT scan abdomen/pelvis (oral and IV


contrast)
• Establishes the diagnosis
• Determines the location
• Assesses the severity

Acute Uncomplicated Diverticulitis

Outpatient Management

• Tolerating PO’s • Oral antibiotics x 7-10


• No systemic symptoms days
• Minimal co-morbidities • Low residue diet

Hinchey I

1371
Copyright © Oakstone Publishing, 2021. All Rights Reserved.

Uncomplicated Diverticulitis: Non-


Operative Management
Inpatient Management

• Older pts, systemic • IV antibiotics, NPO


symptoms • No improvement in
• Localized peritonitis 48-72 hours, CT scan
• Not tolerating PO’s • surgery if symptoms
• Pain control worsen/don’t improve

Hinchey I

Hinchey I: Are antibiotics necessary?


AVOD (2003-2010)
• 623 pts with CT-confirmed acute uncomplicated left-sided
diverticulitis (314 treated, 309 observed)
• IV fluids vs. IV fluids + abx (duration: at least 7 days)

• No difference in complications, recurrence, time to recovery,


need for surgery, LOS, symptoms (pain, bowel function)
at 12-month F/U

• Long-term F/U (11 yrs): no difference in recurrences (31.3%),


complications (4.4 vs 5%), need for surgery (6.2 vs 7.1%), QoL

1372
Copyright © Oakstone Publishing, 2021. All Rights Reserved.

Hinchey I: Are antibiotics necessary?


DIABOLO
• 528 pts with CT-confirmed uncomplicated diverticulitis
• amoxicillin-clavulanic acid x 10 days (48hr IV, 8 days PO) vs.
observation

• No difference: complicated/ongoing/recurrent diverticulitis,


need for surgery, readmission, adverse events , mortality

• 24-month F/U: no difference in recurrence,


complicated diverticulitis, mortality

Hinchey I: Are antibiotics necessary?


DINAMO study
• Prospective multi-center RCT (Spain) – 2016-2020
• 480 pts with mild diverticulitis managed in outpatient setting
• Healthy patients
• Ibuprofen/Tylenol ± Amoxicillin-clavulanic acid x 7 days
• Primary endpoint = hospital admission (90 day F/U)

• ED revisits: 7.8% (no abx, n=19) vs. 8.8% (abx, n=21)


• Hospitalization rate: 3.3% (no abx, n=8) vs 5.8% (abx, n=14)
• No patients required emergency surgery

1373
Copyright © Oakstone Publishing, 2021. All Rights Reserved.

Hinchey I: Are antibiotics necessary?


No antibiotics: Increased risk of Needing Elective Surgery?

DIABOLO @ 24 months
• Need for elective surgery (placebo: 7.7% vs. abx: 4.2%)

AVOD @ 12 months
• Need for elective surgery (placebo: 1.9% vs. abx: 0.6%)

Meta analysis (Mege et al. Dis Colon Rectum 2019; 62:371-378.)

• Elective surgery more frequent in observational group than


abx group (2.5% vs. 0.9%, p=0.04)
• Not statistically significant in each individual study

TAKE HOME POINT


In healthy pts with early disease, an antibiotic-free approach may be reasonable

Hinchey II – Non-Operative Management

Hinchey II

Inpatient Management
• IV antibiotics, NPO
• CT-guided percutaneous drainage
• consider surgery if symptoms worsen
or no improvement in 3-4 days

1374
Copyright © Oakstone Publishing, 2021. All Rights Reserved.

Diverticulitis – Operative Management


(Emergent)

1 Stage

2 Stages

3 Stages

What is the risk of recurrence?

• After 1st episode, ≈10% develop a 2nd


episode @ 1 yr
• After 2nd episode, ≈ 20% develop a 3rd
episode @ 1 yr
• Predictors of recurrence
• Prior diverticulitis, + Family Hx, length of
involved colon > 5cm, smoking, obesity

1375
Copyright © Oakstone Publishing, 2021. All Rights Reserved.

Impact of Diverticulitis on QoL

DIABOLO study
• QoL assessment
• 32.2-38.2% of patients had
persistent symptoms after 1-2 years
• Risk factors for persistent symptoms:
prolonged time to recovery (> 28
days) and high pain scores during
the first 10 days after diagnosis

Indications for Elective Surgery

• Multiply recurrent disease


• Worsening symptoms/QoL
• Stricture/obstruction
• ? immunosuppression ?

• Young age
• Prevention of future complications

1376
Copyright © Oakstone Publishing, 2021. All Rights Reserved.

Pre-Operative Evaluation

• CT scan
• confirm the diagnosis/location
• Colonoscopy
• exclude other diagnoses
• ≈6 weeks after an acute episode
• Incidence of cancer <2%

Operative Management (Elective)

1 Stage Approach

1377
Copyright © Oakstone Publishing, 2021. All Rights Reserved.

Impact of Surgery on QoL


DIRECT trial (2010-2014)
• Multicenter, RCT in the Netherlands
• Patients with recurrent diverticulitis (≥3 episodes within 2 yrs)
and/or persistent abdominal complaints (≥ 3 months) attributed
to diverticular disease
• Compared 53 pts (sigmoidectomy) to 56 nonoperative pts
• Primary end point: GIQLI @ 3 + 6 months

• Operative group: 15% anastomotic leak, 21% ostomy


• GIQLI @ 6 months significantly better in resection group

• 5-yr F/U: surgery group continued to have better QOL, improved


physical, mental and pain scores

COSMID Trial

Goal
For patients with QoL-limiting diverticular disease,
is elective colectomy more effective than
best medical management?

1378
Copyright © Oakstone Publishing, 2021. All Rights Reserved.

Conclusions

• Diverticulitis is the most common consequence of


diverticulosis
• Most patients with acute diverticulitis can be effectively
managed medically
• For acute uncomplicated diverticulitis, surgery may be
indicated for recurrent episodes that impact QoL

1379
Copyright © Oakstone Publishing, 2021. All Rights Reserved.

Division of General and GI Surgery

Colon Neoplasms
Jennifer Lynn Irani, MD
Associate Surgeon
Brigham and Women’s Hospital
Assistant Professor of Surgery
Harvard Medical School

Conflict of Interest

No Conflict of Interest

Division of General and GI Surgery


Brigham and Women’s Hospital

1380
Copyright © Oakstone Publishing, 2021. All Rights Reserved.

Outline
• Epidemiology
• Risk Factors
• Clinical Presentation
• Evaluation
• Staging
• Surgical Resection
• Metastatic Disease
• Malignant Polyp
• Polyposis Syndromes

Epidemiology
• Worldwide
• 3rd most commonly diagnosed malignancy– men
• 2nd most commonly diagnosed malignancy – women
• USA
• 3rd most commonly diagnosed malignancy – men and
women
• 2nd most common cause of cancer death overall

1381
Copyright © Oakstone Publishing, 2021. All Rights Reserved.

Epidemiology
• Lifetime incidence of colorectal cancer in average
risk person
• 5% (men>women)
• Great majority (90%) of cases occur after age 50
• Age is the greatest risk factor for sporadic colon ca
• Risk increases with age, but incidence
• Decreasing in older age groups
• Increasing in age <50
• In USA, incidence of CRC in people <50 steadily increasing at a
rate of 2%/year
• Predominantly left-sided cancers, rectal cancer in particular

Increasing incidence of colorectal


cancer with age

Surveillance, Epidemiology, and End Results (SEER) Program, 2002-2006

1382
Copyright © Oakstone Publishing, 2021. All Rights Reserved.

Epidemiology
• United States
• Decrease in distal cancers
• Increase in proximal cancers
• Due to colonoscopy?
• Poor right sided prep
• Incomplete colonoscopy
• Anatomical configurations compromising visibility
• Serrated adenomas more common right colon
• Flatter and more difficult to visualize endoscopically

Risk Factors
• Environmental
• Genetic
• Inherited susceptibility
• Majority of colon cancers are sporadic rather than familial

1383
Copyright © Oakstone Publishing, 2021. All Rights Reserved.

Risk Factors
Environmental
• Fiber, veggies, fruits, physical activity
• Associated with lower CRC risk
• Alcohol, obesity, smoking, processed meats, red
meat, lack of physical activity, diabetes
• Associated with increased CRC risk
• Abdominal Radiation
• Adult survivors of childhood malignancy
• XRT for prostate cancer

Risk Factors
Increased screening
• Hereditary Colorectal Cancer Syndromes
• Personal history of CRC or polyps
• Family history
• First degree relative with CRC -> doubles risk
• Inflammatory Bowel Disease
• Ulcerative colitis – association between risk and extent
and duration of disease
• Crohn’s disease

1384
Copyright © Oakstone Publishing, 2021. All Rights Reserved.

Clinical Presentation
• Symptomatic
• Asymptomatic • Abdominal Pain
• Found on screening • Change in bowel habits
colonoscopy • Hematochezia or Melena
• Better Prognosis* • Occult blood in stool
• Anemia
• Weight loss
• Weakness
• Obstruction
• Nausea/vomiting
• Obstipation
• Perforation

Evaluation
• Full colonoscopy
• Synchronous cancers (3-5%)
• CT colonography vs. Post op if obstructed
• Pathology review
• Including loss of MMR protein expression
• Evaluate for metastatic disease
• CT abdomen/pelvis
• CT chest
• Carcinoembryonic Antigen (CEA)
• Not a screening test
• Useful for post op surveillance to detect persistent disease or
recurrence

1385
Copyright © Oakstone Publishing, 2021. All Rights Reserved.

CEA
• Glycoprotein absent in normal colonic mucosa
• Present in 97% of patients with colon cancer
• Low diagnostic ability due to overlap with benign
disease and low sensitivity for early stage disease
• Gastritis, PUD, diverticulitis, liver disease, COPD, DM, any
acute or chronic inflammatory state
• Not recommended for screening
• Prognostic utility in patients with CRC
• Preoperative serum CEA>5 ng/ml – worse prognosis
• Failure to normalize postop – persistent disease
• Elevation - recurrence

1386
Copyright © Oakstone Publishing, 2021. All Rights Reserved.

Staging
• TNM
• AJCC 8th edition

1387
Copyright © Oakstone Publishing, 2021. All Rights Reserved.

Prognosis
• Stage
• Obstruction and Perforation
• Poor prognosis
• Independent of stage
• Adjacent organ involvement
• Histologic grade
• Signet ring cell histology
• Lymphovascular invasion and perineural invasion
• Microsatellite instability

Lymph Nodes
• Important prognostic factor
• Several observational studies suggest the greater
the number of lymph nodes removed, the better
the outcome
• Goal: 12 lymph nodes
• If fewer obtained, indication for adjuvant
chemotherapy

Chang GJ, J Natl Cancer Inst. 2007 Mar 21;99(6):433-41

1388
Copyright © Oakstone Publishing, 2021. All Rights Reserved.

Spreading Patterns
• Intramural spread – rare
• 5cm margins minimal
• Transmural spread
• Start mucosal, penetrate deeper layers
• If invades other structures, en bloc resection is necessary for R0 resection
• Tumors with serosal involvement can shed viable tumor cells
• Ovaries, omentum, serosal or peritoneal surfaces
• Lymphatic
• Most common mechanism leading to metastatic disease
• Lymphatic invasion correlates with depth of penetration through colon wall
• T1 – 10%
• T2 – 25%
• T3 – 45%
• Hematogenous

Surgical Resection
• Mainstay of treatment for colon cancer
• Oncologic principles
• Wide mesenteric resection
• Ligate vascular pedicle at its origin
• Lymphatic drainage basin
• Minimum 12 lymph nodes
• Complete removal of tumor
• Adequate margins

1389
Copyright © Oakstone Publishing, 2021. All Rights Reserved.

Regional Lymph Nodes


Segment Regional Lymph Nodes
Cecum Pericolic, anterior cecal, posterior cecal, ileocolic, right
colic
Ascending colon Pericolic, ileocolic, right colic, middle colic

Hepatic flexure Pericolic, middle colic, right colic

Transverse colon Pericolic, middle colic


Splenic flexure Pericolic, middle colic, left colic, inferior mesenteric
Descending colon Pericolic, left colic, inferior mesenteric, sigmoid
Sigmoid colon Pericolic, inferior mesenteric, superior rectal
(hemorrhoidal), sigmoidal, sigmoid mesenteric
Rectosigmoid Pericolic, perirectal, left colic, sigmoid mesenteric,
sigmoidal, inferior mesenteric, superior rectal
(hemorrhoidal), middle rectal (hemorrhoidal)

AJCC 7th Edition, 2010

Surgical Technique
• Explore the abdomen
• Assess for resectablility
• Look for metastatic disease

1390
Copyright © Oakstone Publishing, 2021. All Rights Reserved.

Right Hemicolectomy
• Cecal cancer
• Ascending colon ca
• Ligate ileocolic vessels
• Ligate right colic vessels
• Ligate right/hepatic branch
of middle colic vessels
• TI divided
• Mid-transverse colon divided
• Resect the omentum with
the specimen

Extended Right Colectomy


• Hepatic flexure
• Proximal transverse colon
• Ligate
• Ileocolic
• Right colic
• Main middle colic
• Lesser omentum divided
along entire transverse colon
• Mobilize splenic flexure

1391
Copyright © Oakstone Publishing, 2021. All Rights Reserved.

Transverse Colectomy
• Mid transverse colon
• Ligate feeding vessel
• Caveats
• Ensure adequate
lymphadenectomy
• Blood supply to
remaining bowel
• Tension

Left Colectomy
• Splenic flexure
• Ligate
• Left colic
• First sigmoidal branch
• Adequate margins
• Avoid splenic injury

1392
Copyright © Oakstone Publishing, 2021. All Rights Reserved.

Left Colectomy
• Descending colon
• Ligate
• IMA off aorta
• Transect
• Distal transverse colon
• Rectosigmoid
• Resect distal omentum
• Avoid splenic injury

Sigmoid Colectomy
• Sigmoid
• Ligate IMA at origin
• Adequate resection
margins
• Identify and preserve
left ureter

1393
Copyright © Oakstone Publishing, 2021. All Rights Reserved.

Subtotal and Total Colectomy


• Synchronous right and left sided cancers
• Hereditary syndromes
• Ileorectal anastomosis

Laparoscopic vs. Open Colectomy


• No detrimental impact on recurrence or survival –
comparable oncologic outcomes
• Longer operative time
• Shorter length of stay
• Faster recovery
• No significant difference in:
• Intraoperative or postoperative complications
• Perioperative mortality
• Readmission rates
• Reoperation rates
• Surgical wound recurrence

1394
Copyright © Oakstone Publishing, 2021. All Rights Reserved.

Locally Advanced Primary


• Invasion into a contiguous organ
• Multivisceral resection with negative margins of
adjacent structure
• Plane of adherence between tumor and adjacent
organs should not be disrupted
• Improved local control
• Improved overall survival
• Potentially higher perioperative morbidity

Obstruction or Perforation
Proceed to surgery?
• Perforation
• Patient’s overall condition
• Localized or generalized peritonitis
• Primary anastomosis, Diversion
• Obstruction
• Complete vs. Partial
• Primary anastomosis, Diversion
• Stenting to temporize, prep

1395
Copyright © Oakstone Publishing, 2021. All Rights Reserved.

Pathology comes back T3N1


• Stage 3
• Now what?

Colon Cancer Adjuvant Treatment


• Stage 1 – surgical resection alone
• Stage 3 (node positive) - Surgical resection followed by 6
months chemotherapy
• Intent of chemo – eradication of micrometastases, which decreases
disease recurrence and increases cure rates
• Oxaliplatin-containing regimen
• 6 months FOLFOX is standard (IDEA Trial – 3 months Capox)
• Survival benefit – 30% reduction disease recurrence and mortality
• Stage 2 – adjuvant chemo if higher-risk
• Inadequately sampled LNs
• T4 primary lesion
• Tumor perforation or obstruction
• LVI or PNI, poorly differentiated histology (including signet ring and
mucinous, as long as not MSI unstable)

1396
Copyright © Oakstone Publishing, 2021. All Rights Reserved.

Metastatic Disease
• 20% of patients with CRC present with metastatic
disease
• Liver*
• Lungs
• Multidisciplinary Evaluation
• Goal: resect primary and metastases
• Long term survival in 50%
• Clear survival benefit if complete resection achieved
• Fewer than 20% with isolated hepatic metastases are
amenable to potentially curative resection
• Tumor size, location, multifocality, inadequate hepatic reserve

Metastatic Disease
• PET scan for potentially surgically curable
metastatic disease
• Timing of resections – debatable
• Primary
• Chemotherapy
• Metastasis
• If initially unresectable, may become resectable
with chemotherapy

1397
Copyright © Oakstone Publishing, 2021. All Rights Reserved.

Metastatic Disease
• Symptomatic vs. Asymptomatic primary
• Resectable vs. Unresectable mets
• If the mets are resectable, take an aggressive surgical
approach to primary and mets
• Asymptomatic primary with unresectable liver mets
• Primary resection may prolong survival 8 months
• Low risk of bleeding, obstruction, perforation in patients with
asymptomatic primary treated with chemotherapy

Metastatic Disease
• Symptomatic primary, incurable mets
• Surgical palliation
• Resection of cancer and primary anastomosis
• Proximal diverting colostomy
• Bypass procedure
• Nonsurgical palliation
• Endoluminal stents
• May be contraindication to bevacizumab – increased perforation
• Local tumor ablation
• Fulguration, laser ablation

1398
Copyright © Oakstone Publishing, 2021. All Rights Reserved.

Screening Recommendations
• If CRC is suspected based on signs and symptoms
• Colonoscopy – gold standard
• CT colonography – similar sensitivity, less invasive, but
no tissue diagnosis

1399
Copyright © Oakstone Publishing, 2021. All Rights Reserved.

US Preventative Services Task Force


Recommendation Statement

Summary
• Colon cancer incidence increases with age
• Colonoscopy is best prevention and diagnostic tool
• Pathologic stage at diagnosis is best indicator of
long term prognosis
• Surgical resection is the mainstay of treatment for
localized disease
• Aggressive surgical approach for resectable
metastatic disease
• Adjuvant chemo for Stage 3 and high risk Stage 2

1400
Copyright © Oakstone Publishing, 2021. All Rights Reserved.

Malignant Polyp
(Early Cancer (T1) Within Polyp)
• Malignant polyp — The majority of colorectal
cancers arise from polyps (adenomas). The
malignant potential of an adenoma depends on its
size, histology, and degree of dysplasia.
• An invasive cancer, which is defined by penetration
of the muscularis mucosa by malignant cells into
the submucosa (T1), has the potential to
metastasize to lymph nodes and distant sites
• The management of a malignant polyp containing
invasive carcinoma must be individualized

Malignant Polyp
• Endoscopic management is sufficient for pedunculated
or sessile malignant polyps that can be removed in one
piece and have NONE of the following high-risk
features:
• For both pedunculated and nonpedunculated polyps:
• Poorly differentiated histology
• Lymphovascular invasion
• Tumor budding (foci of isolated cancer cells or a cluster of five or fewer
cancer cells at the invasive margin of the polyp)
• For pedunculated polyps, a positive margin variably defined as cancer
present at the resection margin, cancer within 1 mm of the resection
margin, or cancer within 2 mm of the resection margin
• For nonpedunculated polyps, cancer at resection margin or submucosal
invasion depth ≥ 1mm

1401
Copyright © Oakstone Publishing, 2021. All Rights Reserved.

Malignant Polyp
• Polyps with one or more of these high-risk features are
associated with an increased incidence of residual
cancer and/or lymph node metastases. As such, their
presence indicates the need for radical resection. In
addition, any cancer in a nonpedunculated or
pedunculated lesion resected piecemeal or a
pedunculated polyp that could not be properly oriented
in the pathology department to provide optimal
pathologic assessment is an indication for surgery
• Tumors invading through the muscularis propria (T2 or
above lesion) are no longer considered malignant
polyps but bona fide colon cancers and staged and
treated accordingly

Polyposis Syndromes

1402
Copyright © Oakstone Publishing, 2021. All Rights Reserved.

Classification Hereditary
Syndromes
• Awareness and suspicion –key for identifying
• 5-10% of all CRCs arise within a known syndrome
• Personal and family history, physical exam,
documentation of gastrointestinal polyps and
cancers, and identification of extracolonic
manifestations
• Polyposis or Nonpolyposis Syndromes

Hereditary Syndromes
• Polyposis
• Adenomatous
• Familial adenomatous polyposis (FAP)
• MUTYH-associated polyposis (MAP)
• Hamartomatous polyp syndromes
• Peutz-Jeghers syndrome (PJS)
• Juvenile polyposis syndrome (JPS)
• PTEN hamartoma syndrome
• Nonpolyposis
• Lynch Syndrome

1403
Copyright © Oakstone Publishing, 2021. All Rights Reserved.

Familial Adenomatous Polyposis


• Autosomal dominant, caused by germline mutation
in APC gene, chromosome 5q
• 10-25% of germline APC mutations are new in
individuals without a family h/o FAP
• Nearly 100% penetrance of colonic manifestations
• 1:10,000 live births
• Affects both genders equally
• Characterized by thousands of polyps in the colon
• Attenuated FAP – fewer than 100 adenomas

FAP
• Diagnosis – clinically or genetically
• Colorectal cancer risk –
• Nearly 100%, cancers develop at median age 39
• Attenuated FAP – risk of crc 70%, develop avg 58 y

1404
Copyright © Oakstone Publishing, 2021. All Rights Reserved.

FAP
Extracolonic Manifestations
• Gastroduodenal adenomas and carcinoma
• Desmoid disease
• Thyroid cancer
• Osteomas
• Epidermoid cysts
• Congenital hyperplasia of the retinal pigment
epithelium (CHRPE)
• Dental anomalies

FAP Management
Screening
• Colorectal (APC positive and at risk family
members) – start age 10-15y with colonoscopy
every 12 months
• Duodenal and gastric – upper GI endoscopy with
side-viewing scope start age 20-25y
• Desmoids – no recommended screening
• Thyroid – annual ultrasound
• Other neoplasia – individualized based on family dz

1405
Copyright © Oakstone Publishing, 2021. All Rights Reserved.

Surgical Management
• Timing – individualized
• If symptoms, operate
• If asymptomatic teenager, can wait until early twenties
• CRC <age 20y, rare
• Extent of resection
• Total abdominal colectomy with ileorectal anastomosis
• Only if rectal polyps amenable to surveillance and resection
• Requires endoscopic surveillance every 6-12 months

Surgical Management
• Extent of Resection
• Total proctocolectomy (TPC)
• With ileal pouch-anal anastomosis IPAA
• Surveillance of pouch every 6-12 months
• Without restoration – end ileostomy
• Depends on:
• Polyp burden
• Presence of rectal cancer and treatment
• Risk aversion

1406
Copyright © Oakstone Publishing, 2021. All Rights Reserved.

MUTYH-associated polyposis
(MAP)
• Autosomal recessive polyposis, caused by biallelic
mutations in the MUTYH gene
• Associated with increased risk colorectal cancer
and early development of adenomas. Lifetime risk
of developing CRC – 70-75%
• Approximately 0.3% of CRC patients have MAP
• Colonic phenotype variable, but usually 10-100
colorectal polyps
• Diagnosis by genetic testing

MUTYH-associated polyposis
(MAP)
• Management
• If polyp burden unmanageable with colonoscopy or
if cancer -> colectomy
• If rectal polyps management endoscopically, operation is
total abdominal colectomy with ileorectal anastomosis
• Total proctocolectomy (with or without restoration) if
rectal polyp burden unmanageable.
• Endoscopic surveillance thereafter

1407
Copyright © Oakstone Publishing, 2021. All Rights Reserved.

Peutz-Jeghers syndrome (PJS)


• Rare, autosomal dominant, STK11/LKB1 gene mutation
• Characterized by development of multiple
hamartomatous polyps in GI tract, most commonly SB,
colon and rectum
• Develop mucocutaneous pigmentation
• Lips, buccal mucosa, eyes, nostrils, hands, feet
• Occurs in infancy, fades by late adolescence
• Polyps – asymptomatic, blood loss, anemia, obstruction
from intussusception, malignancy
• Symptomatic polyps, and those >1.5cm are removed,
endoscopically or combined laparoscopy/endoscopy
• Risk for extracolonic malignancies – breast, testicular,
pancreatic, uterine, and ovarian – screening needed

Juvenile polyposis syndrome (JPS)


• Rare, autosomal dominant, although family history only
found in 20-50% cases, implying that many patients are
fist to acquire the mutation
• SMAD-4 gene or BMPR1-A gene mutations
• 30-50% cumulative lifetime risk CRC
• Characterized by multiple hamartomatous polyps in GI
tract
• Polyps contain no smooth muscle and can detach from bowel
wall and pass per rectum
• If noted on colonoscopy, remove b/c they can contain
adenomatous dysplasia and progress to adenocarcinoma
• If polyps cannot be removed endoscopically, surgery
indicated with TAC vs. TPC

1408
Copyright © Oakstone Publishing, 2021. All Rights Reserved.

PTEN hamartoma syndrome


• Cowden Syndrome
• Rare, autosomal dominant, PTEN gene mutation
• Most develop polyps in colon and rectum, usually
hamartomatous polyps. Other polyp types possible
• May present with obstruction or bleeding
• 30% pts have macrocephaly; trichilemmomas are
pathognomonic; other benign/malignant lesions of
breast, thyroid, uterus, and skin
Bannayan-Riley-Ruvalcaba Syndrome
• Rare, autosomal dominant, PTEN gene mutation

Lynch Syndrome
• Most common form of inherited colon and rectal
cancer
• Germline mutation in DNA mismatch repair (MMR)
genes (MLH1, MSH2, MSH6, PMS2)
• This genetic alteration is termed microsatellite instability (MSI) and is
characteristic of Lynch-associated cancers.
• Responsible for 3% of all cases of both endometrial and
colon cancer
• Autosomal dominant inheritance, offspring of affected
individuals have a 50% chance of inheriting the
disorder
• Individuals often have more than one index cancer, and
present before age 50

1409
Copyright © Oakstone Publishing, 2021. All Rights Reserved.

Lynch Syndrome
• Cancer risk depends on MMR genetic defect
• Colorectal cancer
• The lifetime risk of CRC in Lynch syndrome varies from 12 percent to
as high as 90 percent
• Uterine/endometrial cancer
• Most common extracolonic cancer
• Ovarian cancer
• Urinary tract cancers
• Upper GI cancers – bile duct, pancreas, duodenum,
gastric

Lynch Syndrome
• CRCs in Lynch syndrome differ from sporadic CRCs in that they are
predominantly right-sided in location.
• Individuals with Lynch syndrome are at increased risk for synchronous and
metachronous CRCs.
• Although most Lynch-associated CRCs are thought to evolve from adenomas,
the adenomas tend to be larger, flatter, are more often proximal, and are more
likely to have high-grade dysplasia and/or villous histology as compared with
sporadic adenomas.
• The adenoma-carcinoma sequence may also progress more rapidly in Lynch
syndrome as compared with sporadic CRC (35 months versus 10 to 15 years).
Adenoma development may also be bypassed altogether, with cancers
developing directly from microscopic colonic mucosal crypts
• However, the overall 10-year survival from CRC in Lynch syndrome is high (88
percent for colon cancer and 70 percent for recto-sigmoid cancers)

1410
Copyright © Oakstone Publishing, 2021. All Rights Reserved.

Identify at risk Individuals


• Amsterdam II criteria (3-2-1 rule)
• Three or more relatives on the same side of the family with histologically verified Lynch-
associated cancers (CRC, cancer of the endometrium or small bowel, transitional cell carcinoma
of the ureter or renal pelvis), one of whom is a first-degree relative of the other two and in
whom familial adenomatous polyposis (FAP) has been excluded.
• Lynch-associated cancers involving at least two generations.
• One or more cancers diagnosed before the age of 50 years.
• Universal screening of CRC and endometrial cancers for microsatellite instability
(MSI) has improved identification of Lynch syndrome among cancer patients
• Tumors in Lynch syndrome demonstrate evidence of high MSI (MSI-H) and loss
of expression of a mismatch repair (MMR) protein on immunohistochemistry.
• CRCs in Lynch syndrome also have distinct histologic features - more often
mucinous, signet ring cell or medullary histologic type, poorly differentiated,
and have a brisk lymphocytic infiltrate or are rimmed by a Crohn-like, germinal
center-producing lymphoid reaction
• A pathogenic germline mutation in the MMR or EPCAM gene is required for a
definitive diagnosis of Lynch syndrome

Lynch Colorectal Cancer


Choice of Operation
• Total abdominal colectomy with ileorectal
anastomosis
• Segmental colectomy - > high risk of metachronous
cancers
• Rectal cancer
• Total proctocolectomy with or without IPAA is the
preferred choice given risk of metachronous cancers
• Individualized
• TAH/BSO recommended for women who have
completed childbearing

1411
Copyright © Oakstone Publishing, 2021. All Rights Reserved.

Screening for Lynch Patients


• Colorectal cancer
• Colonoscopy every one to two years beginning at age 20
to 25 years, or two to five years prior to the earliest age
of CRC diagnosis in the family (whichever comes first)
• Endometrial and ovarian cancer
• There is no proven effective screening strategy for early
detection of either endometrial or ovarian cancer.
• Gastric cancer
• EGD and biopsy every two to four years starting at 30 to
35 years of age

Summary
• Lynch syndrome is most common hereditary form
of colorectal cancer
• Germline mutation in DNA mismatch repair (MMR)
genes (MLH1, MSH2, MSH6, PMS2)
• Colon cancer – TAC with ileorectal anastomosis
• Familial adenomatous polyposis (FAP)
• Caused by mutation in APC gene
• Nearly 100% risk CRC, median age 39
• Gastroduodenal cancer, desmoids, CHRPE

1412
Copyright © Oakstone Publishing, 2021. All Rights Reserved.

Updates in General and GI Surgery:


General Surgery, Colorectal, or Bariatric (Pick
Management of Rectal and Anal Cancer
one)
and Dysplasia Date

Name, Degree
Academic Title, Institution

Nelya Melnitchouk, MD, MSc, FACS


Assistant Professor, Harvard Medical School
Colorectal Surgeon, Brigham and Women’s Hospital *Insert picture related to talk*
Title of Talk

Division of General and GI Surgery


Brigham and Women’s Hospital

Conflict of Interest

Updates in General and GI Surgery:


General Surgery, Colorectal, or Bariatric (Pick
one)
Date
No Conflict of Interest Name, Degree
Academic Title, Institution

Title of Talk

1413
Copyright © Oakstone Publishing, 2021. All Rights Reserved.

Outline

Staging of Rectal Cancer


Date
Rectal Cancer Treatment Options
Name, Degree
Academic Title, Institution
Quality of Life after Surgery in Rectal Cancer

Staging of Anal Cancer

Anal Cancer Treatment

Anal Dysplasia

Case Presentation

Date

Name, Degree
Academic Title, Institution

A middle aged post-menopausal woman presents with a


change in bowel habits over the last three months, rectal
bleeding and pressure.

1414
Copyright © Oakstone Publishing, 2021. All Rights Reserved.

Local Evaluation

Physical exam
Size
Date
Morphology
Location Name, Degree
Academic Title, Institution
Fixation
Sphincter tone
Extrarectal involvement, involvement of other structures

Rigid proctoscopy (biopsy, size, distance from anal verge)

Colonoscopy (rule out synchronous lesions)

Case Presentation

Date
Palpable mass at 6 cm from anal verge, posterior, fixed
Name, Degree
Academic Title, Institution
Colonoscopy: no synchronous lesions, complete to cecum, 5
cm mass in the mid to distal rectum

Biopsy: moderately differentiated adenocarcinoma

1415
Copyright © Oakstone Publishing, 2021. All Rights Reserved.

Staging

Rule out distant disease


Date
Identify any synchronous lesions that may alter resection
Name, Degree
Academic Title, Institution
Identify local extent of disease
Neoadjuvant therapy
Local excision vs Radical Resection
En bloc resection of adjacent organs

Staging

Tumor Depth, not size


T0 Limited to the mucosa
T1 Into the submucosa
T2 Into the muscularis propria Date
T3 Through the wall, perirectal fat Name, Degree
T4 Into an adjacent structure Academic Title, Institution

Nodes
N0 No nodes involved
N1 1-3 nodes
N2 > 4 nodes STAGE
I T1-2
Metastases
M0 None II T3-4
M1 Mets – lung, liver, peritoneum
III +Nodes
IV +Mets

1416
Copyright © Oakstone Publishing, 2021. All Rights Reserved.

Staging

Rectal MRI or Endorectal US: evaluate local/regional extent of


the disease
Date
-Depth of tumor invasions
Name, Degree
-Lymph node status Academic Title, Institution
-Circumferential resection margin
-Sphincter involvement
-Adjacent structures involvement

CT Chest/Abdomen/Pelvis: evaluate for distal metastasis

CEA level

Case Presentation

Date
CT chest/abdomen and pelvis showed no metastatic disease
Name, Degree
Academic Title, Institution
Rectal MRI: T3, N1 disease, no threated CRM, posterior, no
sphincter involvement

1417
Copyright © Oakstone Publishing, 2021. All Rights Reserved.

Management

Depends on Stage
-Systemic chemotherapyDate
-Surgery (local excision orName,
radicalDegree
resection)
-Radiation therapy Academic Title, Institution

Depends on Location of the Tumor


-Sphincter preservation (low anterior resection)
-Abdominoperineal resection with permanent
colostomy
-Local Excision

Local Excision
Date

Name, Degree
Academic Title, Institution
Indications
-T1N0 via EUS/MRI
- <4cm
- <50% of bowel circumference
- Freely mobile on DRE
- Favorable Histology (well or moderately
differentiated, no LVI, no PNI
Considerations
-Low morbidity and mortality
-Organ and sphincter preserving
-No pathologic staging of lymph nodes
-Higher local recurrence rate
Young DO. Surg Clin N Am 2017

1418
Copyright © Oakstone Publishing, 2021. All Rights Reserved.

Radical Resection

Date
Low Anterior Resection (LAR) Name, Degree
-Sphincter preserving Academic Title, Institution
-Remove primary tumor with associated lymph nodes
-Diverting loop ileostomy can be utilized to protect anastomosis while
healing
-Total mesorectal excision (TME)
-LAR syndrome, poor function

Radical Resection

Date
Abdominoperineal resection
Name, Degree
-Results in permanent colostomy Academic Title, Institution
-Sphincter involvement by the tumor or very close
-Poor preoperative sphincter function

1419
Copyright © Oakstone Publishing, 2021. All Rights Reserved.

Radical Resection-TME

Date

Name, Degree
Academic Title, Institution

Radical Resection-TME

Date

Name, Degree
Academic Title, Institution

1420
Copyright © Oakstone Publishing, 2021. All Rights Reserved.

Neoadjuvant Therapy

Utilized for patients with


-T3/T4, N0 Date
-Any T, N1/N2 Name, Degree
Why? Academic Title, Institution
Confined by bony pelvis
-More difficult surgery
-Different course of lymphatic drainage
-Higher local recurrence rate
Able to accurately stage preoperatively
-Easily accessible through anus for EUS
-MRI possible due to limited mobility

Neoadjuvant Therapy

Decreases local recurrence rate


Date
Allows for sphincter preservationName,
in some patients
Degree
Academic Title, Institution
Allows for resectability in selected cases

Opportunity to avoid surgery (watch and wait)

1421
Copyright © Oakstone Publishing, 2021. All Rights Reserved.

Neoadjuvant Therapy

Date
Multiple regimens Name, Degree
Academic Title, Institution

Conventional: Treatment includes neoadjuvant


chemoradiation followed by total mesorectal excision and
adjuvant FOLFOX or CapeOx x 4 months

Chemoradiation can be either capecitabine 825 mg/m2 BID on


treatment days or ci5FU 225 mg/m2/day infused throughout
treatment.

Radiation involves 50.4 Gy to the involved field.

Neoadjuvant Therapy

Date

Name, Degree
Rectal Cancer 2021: Academic Title, Institution
There is growing evidence that total neoadjuvant
therapy may be preferred in many cases due to
improved DFS and increased path complete
response rate

Short course radiotherapy may be as effective as long


course chemoradiotherapy in many cases

1422
Copyright © Oakstone Publishing, 2021. All Rights Reserved.

Neoadjuvant Therapy

Stockholm III

Short course radiotherapy has similar


Date outcomes to long
course chemoradiotherapy. Name, Degree
Academic Title, Institution

Short-course radiotherapy with an eight week delay to


surgery has similar oncologic outcomes compared with
immediate surgery, but fewer complications.

Long-course radiotherapy with delay is similar to both short-


course radiotherapy approaches, but prolongs the treatment
time. It is better tolerated however.

Neoadjuvant Therapy

Stockholm III

Short course radiotherapy has similar


Date outcomes to long
course chemoradiotherapy. Name, Degree
Academic Title, Institution

Short-course radiotherapy with an eight week delay to


surgery has similar oncologic outcomes compared with
immediate surgery, but fewer complications.

Long-course radiotherapy with delay is similar to both short-


course radiotherapy approaches, but prolongs the treatment
time. It is better tolerated however.
J Erlandsson. Lancet Oncol. 2017 Mar;18(3):336-346.

1423
Copyright © Oakstone Publishing, 2021. All Rights Reserved.

Neoadjuvant Therapy

Date
RAPIDO Name, Degree
Academic Title, Institution

T4, N2, EMVI, or CRM threatened

Short course radiation, FOLFOX for nine cycles or CapeOx for


six cycles followed by surgery

Chemoradiotherapy with capecitabine, TME, and FOLFOX for


12 cycles or CapeOx for eight cycles

R Renu J Clin Onc 38, no. 15_suppl (May 20, 2020) 4006-4006.

Neoadjuvant Therapy
RAPIDO

Short course radiation followed by chemotherapy and then


surgery has improved pathologic complete response (27.7%
versus 13.8%, p < 0.001) and disease control rate (23.7%
Date
versus 30.4%, p = 0.02)
Name, Degree
Academic Title, Institution
Short course radiation followed by chemotherapy improved
local recurrences (8.7% vs 6.0%, p = 0.10) and distant
metastasis (19.8% versus 26.6%, p = 0.004) at three years

Short course radiotherapy followed by chemotherapy had


greater up front toxicity but similar long term toxicity

1424
Copyright © Oakstone Publishing, 2021. All Rights Reserved.

Neoadjuvant Therapy
OPRA

Stage II and stage III rectal cancer, similar to Stockholm III


Date
Total neoadjuvant approach Name, Degree
Academic Title, Institution

FOLFOX for eight cycles or CapeOx for six cycles were given
either before (induction) or after (consolidation)
chemoradiotherapy

Restaged eight to 12 weeks after therapy

Watch and wait for complete or near complete responses


J Garcia-Aguilar. J Clin Onc 38, no. 15_suppl (May 20, 2020) 4008-4008.

Neoadjuvant Therapy
OPRA

Organ preservation was improved with chemoradiation followed by


chemotherapy (consolidation)
Date

Numerical improvement in complete responses Name, Degree


Academic Title, Institution

Watch and wait results themselves are still pending


J Garcia-Aguilar. J Clin Onc 38, no. 15_suppl (May 20, 2020) 4008-4008.

1425
Copyright © Oakstone Publishing, 2021. All Rights Reserved.

Watch and Wait

-Clinical approach for patients with rectal cancer after a clinical complete
response (cCR) to neoadjuvant therapy
-cCR means no rectal tumor found by: digital examination, endoscopy, and
magnetic resonance imaging Date
-cCR can occur in up to 30% of patientsName,
treated with total neoadjuvant
Degree
therapy Academic Title, Institution
-Gives patient opportunity to avoid surgery and leads to organ
preservation
-cCR is different than pathologic complete response (pCR), which is an
assessment only made after surgery
Potential risk: patients who do not have a sustained complete tumor
response are at risk for developing local re-growth, requiring delayed total
mesorectal excision and thus compromised oncologic outcomes (i.e.
undetected residual tumor has opportunity to mutate and metastasize
leading to decreased survival)

Watch and Wait

Several small, single institution studies, with varying rectal ca stages;


different protocols for surveillance and definitions of neoadjuvant
therapy/clinical complete response Date
Name, Degree
Data is encouraging: Academic Title, Institution
-Vast majority of regrowth is endoluminal
-Majority of regrowth occurs by 2-3 years
-Patients need to know that regrowth will occur in 15-30% of patients
-Salvage rates with R0 resection exceeding 90% in majority of studies
-Pelvic recurrence after salvage surgery are low (<10%)
-Distant metastasis occurs (~8%)

1426
Copyright © Oakstone Publishing, 2021. All Rights Reserved.

Case Presentation
Date

Name, Degree
Academic Title, Institution

Discussion at tumor board with shared decision making

Underwent short course XRT, systemic chemotherapy and


low anterior resection with temporary ileostomy

Final path: T2N1 with evidence of tumor regression, negative


margins and complete TME

Ileostomy was reversed 8 weeks after LAR

Quality of Life
Bladder dysfunction
Sexual dysfunction
Post LAR syndrome (50% or more)
-Incontinence Date

-Frequency Name, Degree


Academic Title, Institution
-Clustering
-Urgency/pain

Better off with a stoma?


-Importance of preoperative marking
-Support from ostomy nurses
-Parastomal hernia
-Self-image

1427
Copyright © Oakstone Publishing, 2021. All Rights Reserved.

Case Presentation
Date

Name, Degree
Academic Title, Institution

Reports 4-5 BMs per day, small with frequent trips to the
bathroom in the morning
No incontinence but has urgency
Symptoms are somewhat improved after starting fiber
Surveillance imaging: NED

Summary

Updates in General and GI Surgery:


General Surgery, Colorectal, or Bariatric (Pick
one)
Date

Name, Degree
Rectal cancer Academic Title, Institution

Many more choices for management


Very nuanced
Shared decision making
Tumor board discussion of every rectal cancer patient
with review of imaging and pathology

Division of General and GI Surgery


Brigham and Women’s Hospital

1428
Copyright © Oakstone Publishing, 2021. All Rights Reserved.

Anal Cancer

Date

Name, Degree
Anal cancer is rare Academic Title, Institution
9090 new cases diagnosed last year in United States
Rising for many years
Average age of diagnosis is early 60s
Squamous cell carcinoma (SCC) is the most frequent histologic
type
Anal adenocarcinomas are very rare and treated similarly to
rectal cancer

Anal Cancer-Risk Factors


Date

Name, Degree
Academic Title, Institution
HPV infection: the most important risk factor for anal cancer
-Anal warts
-History of cervical, vaginal or vulvar cancer
HIV infection
Smoking
Immunosuppression
Lifetime number of sexual partners
More common in white women and black men

1429
Copyright © Oakstone Publishing, 2021. All Rights Reserved.

Anal Cancer-Staging

Date

Name, Degree
Academic Title, Institution

-Inspection (important to differentiate anal margin cancer


from anal cancer)
-Digital Rectal Exam
-Anoscopy
-Biopsy
-Examinations of the groin
-CT chest, abdomen and pelvis
-PET CT
-For women Gyn exam should be done

Anal Cancer- Staging

Date

Name, Degree
Academic Title, Institution

1430
Copyright © Oakstone Publishing, 2021. All Rights Reserved.

Anal Cancer-Staging

Date

Name, Degree
Academic Title, Institution

Positive lymph nodes in the inguinal, mesorectal, internal and


external iliac distributions are considered N1 disease

Anal Cancer-Management
Date

Name, Degree
Academic Title, Institution

In the absence of metastatic disease majority of patients with squamous


cell cancer of the anal canal are treated with chemoradiotherapy

Fluorouracil with mitomycin and radiation therapy is standard of care

RT fields encompass pelvis, inguinal nodes and anus. If nodes are positive
RT boost is added to affected groin

Can consider local excision for very small tumors, <1 cm in size

1431
Copyright © Oakstone Publishing, 2021. All Rights Reserved.

Anal Cancer-Management
Date

Name, Degree
Academic Title, Institution

SCC of the anus regresses slowly


Clinical assessment at 8-12 weeks following completion of therapy
If complete clinical response follow with DRE, anoscopy and inguinal node
examination every 3-6 months for 5 years and imaging annually for three
years
If patient has persistent disease can be watched for up to six months as
long as there is no progression
If disease progressed or persists abdominoperineal resection is offered

Anal Dysplasia
Date

Name, Degree
Academic Title, Institution

Anal squamous intraepithelial lesions (SIL) (AIN is old nomenclature)


-Low-grade SIL (LSIL)-former AIN1
-High-grade SIL (HSIL)-former AIN 3
-AIN 2 lesions depend on p!6 staining, if p16 negative then LSIL, if
positive then HSIL
Risk factors are similar to anal cancer
HSIL is precursor to anal squamous cell carcinoma

1432
Copyright © Oakstone Publishing, 2021. All Rights Reserved.

Anal Dysplasia-Screening

Date

Name, Degree
Academic Title, Institution

Anal cytology
High-resolution anoscopy if anal cytology is abnormal

Who to screen?
Males who have sex with males living with HIV
Females with HIV, Males with HIV
Females with cervical infection with HPV 16
Immunocompromized patients
ANCHOR study: Treating HSIL reduces risk of progression to anal SCC
among people living with HIV (publication is pending)

Thank you!

Date

Name, Degree
Academic Title, Institution

1433
Copyright © Oakstone Publishing, 2021. All Rights Reserved.

Anal Fissure and Anal Fistula

Ronald Bleday M.D.


Associate Professor of Surgery
Harvard Medical School
Chief, Section of Colon and Rectal Surgery
Brigham and Women’s Hospital

Disclosures
• No disclosures

1434
Copyright © Oakstone Publishing, 2021. All Rights Reserved.

Overview

• Anal Fissure
• Abscess/Fistula

Case Scenario
• A 30 y.o. woman presents with severe anal pain and a perianal “lump”
o Work up of new anal pain: Fissure vs Infection/fistula vs other
o Anal abscess and fistula: Incidence
 How often does an abscess become a fistula?
 Fistula classification in non-IBD patients.
 Work up and treatment
o Anal fissure
 Typical vs Atypical
 Treatment of typical fissures
 Atypical fissures: what should one do?

1435
Copyright © Oakstone Publishing, 2021. All Rights Reserved.

Anal Fissure
• Tear in lining of anal canal

• Posterior midline > anterior midline > lateral


– Posterior 90% of time
– Anterior 10% (90% in women)

• Lateral: IBD, TB, lymphoma, HIV/STD related

Anal Fissure

Sentinel tag Fissure

1436
Copyright © Oakstone Publishing, 2021. All Rights Reserved.

Anal Fissure: etiology


• Traumatic injury to mucosa overlying
sphincter complex
• Cycle: Pain, spasm, no BM, constipation,
reinjury
• Ischemia secondary to spasm

Anal Fissure: diagnosis


• Symptoms
– Hematochezia
– Sharp pain w/defecation (glass/razor blade)
– Post BM spasm

• Physical Exam
– Gentle separation of buttocks, inspection
– DRE and anoscopy only if no pian

1437
Copyright © Oakstone Publishing, 2021. All Rights Reserved.

Anal Fissure

Anal Fissure: treatment


• Treatment goals
– Diminish pain
– Prevent spasm
– Promote healing

• Non-surgical therapy
– Sitz baths, fiber and water, Analpram HC
– Nitoglycerin 0.2% TID to QID to anal canal
– Nifedipine 0.3% with 2% lidocaine compound
– Botox A: 100 IU injection into IAS on either side fissure

1438
Copyright © Oakstone Publishing, 2021. All Rights Reserved.

Anal Fissure: Treatment


• Relaxation of the internal anal sphincter
– Bulk fiber
– Sitz baths
– Topical nitroglycerin
• 0.2%, qid prn
• Side effect: Headache
– Topical Nifedipine
• 0.2%, qid prn
– Botox injection: 20U-100U.

Anal Fissure: nitroglycerin


• Topical nitroglycerin 0.2%
– Relaxes smooth muscle
– Increases blood flow

• Randomized trial 0.2% NTG vs. placebo


– Healing in 68 vs. 8%
– Headaches
• Rectiv is now Rx at 0.4% NTG

1439
Copyright © Oakstone Publishing, 2021. All Rights Reserved.

Anal Fissure: nifedipine


• Topical nifedipine 0.3% compounded with
lidocaine 2% topically QID
– Relaxes smooth muscle
– Increases blood flow
– Less headache
– Better compliance

Anal fissure: surgical therapy


• Lateral internal sphincterotomy

• •95% success rate

• Transient /permanent incontinence rates:


2% - 8%

1440
Copyright © Oakstone Publishing, 2021. All Rights Reserved.

Anal Fissure

Anal Fissure

1441
Copyright © Oakstone Publishing, 2021. All Rights Reserved.

Anal fissure: Surgery


• Lateral internal sphincterotomy
– Stop 2-3 mm distal to the dentate line
• Lord’s dilatation
– Side effects: Decrease in continence
• Balloon Dilatation (Pressure regulated)
• Surgery: Very effective over the long term
– 2-3 % risk of a change in continence.

Atypical Fissure
• Infectious disease workup
– Cultures
– Test for Syphillis and Gonorhea
– Viral swab for HSV
• Colonoscopy to r/o IBD
• If at risk
– Check for TB
– Hematologic work-up

1442
Copyright © Oakstone Publishing, 2021. All Rights Reserved.

Summary: Fissure Treatment


• Break the cycle of pain and spasm
• Medical management:
– fiber, water, Nifedipine vs NTG
• Surgical management
– Botox vs Sphincterotomy
• Recurrence common
• Prevention best treatment
• Often long complicated process

1443
Copyright © Oakstone Publishing, 2021. All Rights Reserved.

Anal Fistula

Abscess/Fistula: etiology
• Cryptoglandular theory abscess
• Occluded gland at dentate line
• Suppuration tracts outward
• Forms fistula 33-50% of the time.

• Fistula
• Epithelial tract b/w anus and external skin
• Chronic abscess

1444
Copyright © Oakstone Publishing, 2021. All Rights Reserved.

Anal glands- Anatomy

Intersphincteric fistula

1445
Copyright © Oakstone Publishing, 2021. All Rights Reserved.

Transphincteric fistula

Extrasphincteric fistula

1446
Copyright © Oakstone Publishing, 2021. All Rights Reserved.

Transphincteric fistula with


extension
• Failure of typical
therapy
• MRI
• EUA, partial
fistulotomy and
placement of a draining
seton
• Porcine plug surgery
– Endoanal flap

Crohn’s disease: Anal Fistula


1447
Copyright © Oakstone Publishing, 2021. All Rights Reserved.

Anorectal Crohn’s: Incidence


• Site of disease
– Small bowel 25%
– Colon 75%
– Rectum 100%
– Anal Canal only 5%

Abscess/Fistula: diagnosis

• Perianal/rectal pain

• Tender/indurated mass/lump

• Fever, malaise, urinary retention

• Spontaneous intermittent discharge


– Bloody
– Mucous
– Purulent

1448
Copyright © Oakstone Publishing, 2021. All Rights Reserved.

Abscess: treatment
• Drainage
• Local anesthesia
• As close to anal verge
• Shortens fistula: 15-50% get

• Sitz baths

• Antibiotics
• Immunocompromised
• Heart valve/vascular graft
• Diabetes

Fistula: treatment

• Eliminate sepsis
• Drain abscess
• Place seton
• Define anatomy
• Locate internal and external openings
• MRI – if needed
• Preserve sphincter

1449
Copyright © Oakstone Publishing, 2021. All Rights Reserved.

Perianal abscess post anal space Fistula tract

Fistula tract Fistula tract

Fistula: treatment
• Fistulotomy
• Simple fistula’s
• Internal sphincter only
• Small amount ext sphincter
• Fibrin glue/Collagen plug
• Cutting Seton
• LIFT Procedure
– Ligation of intersphincteric fistula tract
• Endoanal advancement flap
• Complex/recurrent fistula’s

1450
Copyright © Oakstone Publishing, 2021. All Rights Reserved.

Fistula treatment

Fistula treatment

1451
Copyright © Oakstone Publishing, 2021. All Rights Reserved.

Porcine Plug
• Pig collagen
• Bowel prep
• Curretage of the tract
• Two layer closure
– Suture to the rectal
muscle/wall
– Endoanal flap
• Results

1452
Copyright © Oakstone Publishing, 2021. All Rights Reserved.

Poor prognostic clinical features


• High anal fistulas
• High anal/low anal strictures
• Deep ulcers
• Rectovaginal fistula
• Supralevator abscesses

Summary: Fistula Treatment


• Eliminate sepsis
• Preserve sphincter
• Often long complicated process

1453
Copyright © Oakstone Publishing, 2021. All Rights Reserved.

Initial Assessment in Trauma

Kristin Sonderman MD, MPH


Division of Trauma, Burn, and Surgical Critical Care
Brigham and Women’s Hospital

No disclosures

1454
Copyright © Oakstone Publishing, 2021. All Rights Reserved.

OUTLINE

I. Initial assessment of a trauma patient


i. Primary and secondary survey
i. Management of life-threatening injuries
ii. Role of thoracotomy, REBOA
II. Head trauma
i. Initial evaluation and treatment
ii. ICU management
iii. Facial trauma
III. Neck trauma
i. Blunt
ii. Penetrating

OUTLINE

I. Initial assessment of a trauma patient


i. Primary and secondary survey
i. Management of life-threatening injuries
ii. Role of thoracotomy, REBOA

1455
Copyright © Oakstone Publishing, 2021. All Rights Reserved.

OUTLINE

I. Initial assessment of a trauma patient


i. Primary and secondary survey
i. Management of life-threatening injuries
ii. Role of thoracotomy, REBOA

Pre-hospital preparation
i. Ensure appropriate equipment available and functioning
ii. Establish roles
iii. Communication line/protocol other medical providers

APPROPRIATE EQUIPMENT

Instrument Tray Head/ Neck Airway Thorax Abdomen/pelvis Extremities Shock


• Hemostats/clam • Sutures/staplers • Airway kit (ETT, • Chest tubes/ • Ultrasound • Tourniquets, • IV bags, fluid
ps • ICP kits stylets, bougies, pneumovac • NGTs, foley doppler warming devices
• Retractors • Scopes/light capnography) • Thoracotomy • Pelvic binders • Central line, art
• Pickups source for ENT • Video tray line kits
• Needle drivers exam laryngoscopy • Scalpels
• Cervical collars • Ventilator • REBOA
• ABG kits

1456
Copyright © Oakstone Publishing, 2021. All Rights Reserved.

I. Initial assessment of a trauma patient

Rapid Resuscitate Determine


Assessment and stabilize disposition

I. Initial assessment of a trauma patient


i. Primary survey

1457
Copyright © Oakstone Publishing, 2021. All Rights Reserved.

AIRWAY

I. Airway Assessment
I. Patency “What is your name”
I. Penetrating trauma w/ bloody airway, sit patient upright
II. Airway obstruction from:
I. Excessive bleeding, expanding hematomas, anatomic disruption, secondary
traumatic swelling/edema
II. Evidence of impending airway compromise
I. Severe bleeding in oropharynx, alteration in voice phonation, sensation of
dyspnea, hematoma in neck/face, subcutaneous air in neck/chest
III. Evaluate risk of intubation difficulty

AIRWAY

Reed MJ, Dunn MJ, McKeown DW. Can an airway assessment score predict difficulty at intubation in the emergency department? Emerg Med J.
2005; 22: 99–102.

1458
Copyright © Oakstone Publishing, 2021. All Rights Reserved.

AIRWAY

I. Airway Assessment
I. Patency- “What is your name”
I. Penetrating trauma w/ bloody airway, sit patient upright
II. Airway obstruction from:
I. Excessive bleeding, expanding hematomas, anatomic disruption, secondary
traumatic swelling/edema
II. Evidence of impending airway compromise
I. Severe bleeding in oropharynx, alteration in voice phonation, sensation of
dyspnea, hematoma in neck/face, subcutaneous air in neck/chest
III. Evaluate risk of intubation difficulty
IV. Indications for intubation

AIRWAY
EAST Guidelines (Level 1)
 Trauma Patients:
o Airway obstruction
o Hypoventilation
o Persistent hypoxemia (SaO2 ≤ 90%) despite supplemental oxygen
o Glasgow Coma Scale score ≤ 8)
o Severe hemorrhagic shock, and
o Cardiac arrest

 Smoke inhalation with:


o Airway obstruction
o GCS score ≤ 8
o Major burn (≥40%)
o Major burns and/or smoke inhalation with an anticipated prolonged transport time to definitive care, and
o Impending airway obstruction:
o Moderate-to-severe facial/oropharyngeal burn, moderate-to-severe airway injury seen on EGD
Mayglothling, J. Duane, T., Shah, Kaushal. Endotracheal Intubation Following Trauma. J Trauma. 73(5):S333-S340, November 2012

1459
Copyright © Oakstone Publishing, 2021. All Rights Reserved.

AIRWAY

I. Airway Management
I. Pulse oximetry, End tidal Co2, suction device, supplemental oxygen
II. Maneuvers

AIRWAY

I. Airway Management
I. Pulse oximetry, ECo2, suction device, supplemental oxygen
II. Maneuvers
III. Devices
I. Oropharyngeal/nasopharyngeal airway
II. Extraglottic/ supraglottic devices

1460
Copyright © Oakstone Publishing, 2021. All Rights Reserved.

AIRWAY

I. Airway Management
I. Pulse oximetry, ECo2, suction device, supplemental oxygen
II. Maneuvers
III. Devices
IV. Intubation guided by direct laryngoscopy
i. RSI
ii. Other options
i. Blind insertion supraglottic devices (LMA, combitube, king airway)
ii. Gum-elastic bougie
iii. Video Laryngoscopy

AIRWAY

1461
Copyright © Oakstone Publishing, 2021. All Rights Reserved.

SURGICAL AIRWAY

I. Indications
I. Inability to intubate through vocal
cords, edema of the glottis, fracture
of larynx, severe oropharyngeal
hemorrhage
II. Cricothyroidotomy
I. Open
II. Seldinger
III. Needle

SURGICAL AIRWAY

I. Open cricothyroidotomy

1462
Copyright © Oakstone Publishing, 2021. All Rights Reserved.

AIRWAY

I. Seldinger cricothyroidotomy
I. Needle/catheter onto syringe
with saline
II. Identify landmarks
III. Puncture cricothyroid
membrane while aspirating

SURGICAL AIRWAY

I. Needle cricothyroidotomy
I. Identify landmarks
II. 12-14G large caliber plastic
cannula for adults
III. 16-18G children

1463
Copyright © Oakstone Publishing, 2021. All Rights Reserved.

AIRWAY

I. Cervical spine immobilization

BREATHING

I. Breathing
I. Supplemental oxygen, oximetry
II. Inspect
I. respiratory rate, tracheal position, jugular venous distension, chest wall
excursion (flail chest)
II. Expose the complete chest and neck for trauma (i.e bullet holes, open PTX)
III. Auscultation
I. Ultrasound (eFast)
IV. Identification of:
I. Tension pneumothorax, hemothorax, open pneumothorax, tracheobronchial
tree injuries

1464
Copyright © Oakstone Publishing, 2021. All Rights Reserved.

BREATHING

Tension Pneumothorax
- Air into pleural space, flap mechanism prevents
escape
Physiology - Lung collapse 2/2 increased intrapleural pressure
- Reduce venous return obstructive shock

- Decreased CO hypotension


- Mediastinal shift
Signs and
- Tachypnea/air hunger
symptoms
- Acute respiratory distress
- Subq emphysema
- Absent breath sounds
- Needle or finger decompression followed by chest
tube placement
ED Treatment - Following intubation can convert simple 
tension

BREATHING

I. Finger and Tube Thoracostomy

1465
Copyright © Oakstone Publishing, 2021. All Rights Reserved.

BREATHING

I. Finger and Tube Thoracostomy

BREATHING

Massive Hemothorax
- Rapid accumulation of
>1500ml of blood in the chest
Physiology or >1/3 blood volume
- Penetrating wound to systemic
or hilar vessels
- Acute respiratory distress
Signs and - Absent breath sounds, dull to
symptoms percussion
- Shock

- Large bore IV, IVF, transfuse


- Chest tube placement 28F
ED Treatment
- 1500cc = need for thoracotomy
OR 200 ml/hr x 2-4 hours

1466
Copyright © Oakstone Publishing, 2021. All Rights Reserved.

BREATHING

Open Pneumothorax
- Large injury to chest wall
- Equilibrium between
Physiology intrathoracic and atmospheric
pressure

- Respiratory distress
Signs and symptoms - Tachypnea
- Decreased breath sounds,
- Open wound
- 3-sided occlusive dressing
- Chest tube placement
ED Treatment
- Definitive closure of defect

BREATHING

Tracheobronchial Tree
Injury
- Usually within 1 inch of the carina
- Rapid deceleration
Physiology
- Direct laceration/tearing
- Majority die at scene
- hemoptysis, subcutaneous emphysema
- Air bubbling from neck wound
Signs and symptoms - pneumothorax
- Cyanosis
- Incomplete expansion of lung and large
air leak after CT placement
- Often surgical airway
- Bronchoscopy confirms diagnosis
ED Treatment - Can require 2 chest tubes if significant
leak
- Definitive repair

1467
Copyright © Oakstone Publishing, 2021. All Rights Reserved.

CIRCULATION

I. Blood pressure/Pulses, Access and Hemorrhage Control


I. Large bore IV access in upper extremity stressed by ATLS
II. Cordis lines
III. Interosseous access

CIRCULATION

I. Blood pressure/Pulses, Access and Hemorrhage Control


I. Large bore IV access in upper extremity stressed by ATLS
II. Cordis lines
III. Interosseous access

1468
Copyright © Oakstone Publishing, 2021. All Rights Reserved.

CIRCULATION

CIRCULATION

1469
Copyright © Oakstone Publishing, 2021. All Rights Reserved.

CIRCULATION

CIRCULATION

I. Blood pressure/Pulses, Access and Hemorrhage Control


I. Five Locations

1470
Copyright © Oakstone Publishing, 2021. All Rights Reserved.

CIRCULATION

I. Blood pressure/Pulses, Access and Hemorrhage Control


I. Assess locations
I. Physical exam- LOC, pulse, anatomic exam
II. FAST exam
III. CXR/Pelvis XRAY
IV. Foreign body series

ADJUNCTS
I. FAST and eFAST
I. Low-frequency probe
II. 4 Views
I. Pericardium
II. RUQ
III. LUQ
IV. Pelvis
III. eFAST
I. 2nd or 3rd intercostal space bilaterally
II. diaphragms

1471
Copyright © Oakstone Publishing, 2021. All Rights Reserved.

FAST
I. Right upper quadrant

Normal Fluid in Morison’s Pouch

FAST
I. Left upper quadrant

Normal Free fluid

1472
Copyright © Oakstone Publishing, 2021. All Rights Reserved.

FAST
I. Pelvis

FAST
I. Pericardial View

Normal Pericardial Effusion

1473
Copyright © Oakstone Publishing, 2021. All Rights Reserved.

CIRCULATION

Cardiac Tamponade
- Accumulation of blood in the pericardial
sac
- Decreased inflow  decreased CO
Physiology
circulatory collapse
- Penetrating typically
- Can develop slowly or rapidly
- Hypotension
- Hard to hear heart sounds in ED
Signs and
- May not have distended neck veins 2/2
symptoms
hypovolemia
- FAST- diagnose

- IVFs
- OR sternotomy vs. thoracotomy
ED Treatment
- IF OR not possible, pericardiocentesis
temporary maneuver

eFAST

I. eFAST

1474
Copyright © Oakstone Publishing, 2021. All Rights Reserved.

eFAST

I. eFAST
I. Lung sliding

eFAST

I. eFAST
I. M- mode

Sky: skin/subq tissue

Ocean: muscle

Beach: lung sliding


motion (sandy
appearance)

1475
Copyright © Oakstone Publishing, 2021. All Rights Reserved.

CIRCULATION

I. Blood pressure/Pulses, Access and Hemorrhage Control


I. Assess Locations with adjuvants to primary survey
I. FAST exam
II. CXR/Pelvis XRAY
III. Foreign body series

CIRCULATION

I. Blood pressure/Pulses, Access and Hemorrhage Control


I. External/ internal
I. Direct pressure
II. Tourniquets
III. Pelvic binders

1476
Copyright © Oakstone Publishing, 2021. All Rights Reserved.

CIRCULATION

I. Pelvic Binder placement

CIRCULATION

I. Internal causes of hemorrhage


I. Chest
I. OR or Chest tube or scan
II. Abdominal
I. OR or REBOA or IR or scan
III. Pelvis
I. OR, IR, REBOA, BINDER or scan

1477
Copyright © Oakstone Publishing, 2021. All Rights Reserved.

HEMORRHAGIC SHOCK

CIRCULATION

I. Resuscitation
I. IF obvious hemorrhage shock (Class III or IV)- start with type O BLOOD (O
negative for women of childbearing age)
I. Activate Massive Transfusion Protocol
II. IV crystalloid 20ml/kg isotonic saline
I. Avoid unnecessary IVF resuscitation
II. Permissive hypotension
III. 1:1:1 ratio of plasma, platelets, RBC
IV. Consider Tranexamic acid if within 3 hours of injury

1478
Copyright © Oakstone Publishing, 2021. All Rights Reserved.

TEG

TEG

1479
Copyright © Oakstone Publishing, 2021. All Rights Reserved.

DISABILITY

I. Neurologic Evaluation
I. Level of consciousness, pupillary size and
reactivity, lateralizing signs, and spinal cord
level if present
II. GCS
III. Altered LOC
I. Evaluate oxygenation, ventilation,
perfusion
II. Hypoglycemia, alcohol, narcotics, other
drugs
IV. Prevention of secondary brain injury
I. Maintain adequate oxygenation / perfusion

EXPOSURE AND ENVIRONMENT

I. Exposure
I. Complete with log roll
II. Cover with warm blankets
II. Evaluate/avoid hypothermia
I. External warming devices
II. Warm IV fluids

1480
Copyright © Oakstone Publishing, 2021. All Rights Reserved.

SECONDARY SURVEY

I. AMPLE history
II. Head to toe examination

OUTLINE

I. Initial assessment of a trauma patient


i. Primary and secondary survey
i. Management of life-threatening injuries
ii. Role of thoracotomy, REBOA

1481
Copyright © Oakstone Publishing, 2021. All Rights Reserved.

EMERGENCY DEPARTMENT THORACOTOMY

I. Indications
I. EAST
I. Present pulseless to the emergency department with signs of life after
penetrating thoracic injury
II. Conditional recommendation for pulseless with or without signs of life after
penetrating extra-thoracic injury
III. Conditional recommendation for pulseless without signs of life after
penetrating thoracic injury
IV. Conditional recommendation for pulseless with signs of life after blunt injury
II. WEST
I. Blunt trauma if prehospital CPR <10 minutes
II. Penetrating trauma if prehospital CPR <15 minutes

EMERGENCY DEPARTMENT THORACOTOMY

I. Prognosis
I. Overall survival 9.6%
i. Penetrating injuries 13.3%
ii. Blunt injuries 4.4%
iii. Improved survival over time
i. 7.9% 11.3%
II. Factors associated with survival:
i. Penetrating mechanism
ii. Age <60 years
iii. Signs of life upon arrival
iv. No prehospital CPR
v. ISS
Joseph B, et al. Trauma Surg Acute Care Open 2018;3:e000201. doi:10.1136/tsaco-2018-
000201

1482
Copyright © Oakstone Publishing, 2021. All Rights Reserved.

EMERGENCY DEPARTMENT THORACOTOMY

I. Steps
• 4th or 5th intercostal space above rib
• Scissors/knife subq/muscle
• Rib spreader
• Retract lower lobe, inferior pulm. ligament
• Open pericardium longitudinally, identify
phrenic
• Assess for cardiac injury
• incise mediastinal pleura
• Palpate aorta/esophagus, place clamp
• Hilar injury- clamp
• Chest tube on right, if lots of bleeding-clamshell

EMERGENCY DEPARTMENT THORACOTOMY

I. Cardiac Injury

1483
Copyright © Oakstone Publishing, 2021. All Rights Reserved.

REBOA
Indications:
- Life threatening hemorrhage below the
diaphragm who are unresponsive or transiently
responsive to resuscitation

- Life threatening hemorrhage below the


diaphragm and traumatic arrest

- Severe intra-abdominal or RP hemorrhage, or


those with traumatic arrest (inflation Zone 1)

- Severe pelvic, junctional, or proximal lower


extremity hemorrhage (inflation Zone 3)

REBOA

1484
Copyright © Oakstone Publishing, 2021. All Rights Reserved.

REBOA

REBOA

I. Steps for Placement


I. Arterial access
II. 7F sheath
III. Measure catheter
IV. Advance peel-away sheath over end of catheter
V. Flush catheter
VI. Insert catheter and advance to desired depth
VII. Xray confirmation
VIII.Inflate balloon to appropriate size
IX. Monitor hemodynamic response
X. Secure device
XI. OR/IR

1485
Copyright © Oakstone Publishing, 2021. All Rights Reserved.

REBOA

OUTLINE

I. Initial assessment of a trauma patient


i. Primary and secondary survey
i. Management of life-threatening injuries
ii. Role of thoracotomy, REBOA
II. Head trauma
i. Initial evaluation and treatment
ii. ICU management
iii. Facial trauma

1486
Copyright © Oakstone Publishing, 2021. All Rights Reserved.

HEAD TRAUMA

I. Head trauma
i. Initial evaluation and treatment
i. GCS < or equal to 8 intubate try to obtain best neuro exam
ii. Avoidance of hypoxia, hyperventilation, and hypotension
iii. Recognition of signs of impending cerebral herniation
i. Asymmetrical pupils
ii. Unilateral or dilated pupils
iii. Decorticate or decerebrate posturing
iv. Respiratory depression
v. Cushing Triad- hypertension, bradycardia, irregular respiration

Prompt neuro consult prior to imaging

HEAD TRAUMA

I. Head trauma
i. Initial evaluation and treatment
i. Obtain Head CT
ii. Identify injury

Epidural Hematoma Subdural Hematoma Subarachnoid Hematoma Diffuse axonal injury


Intracranial Hemorrhage

1487
Copyright © Oakstone Publishing, 2021. All Rights Reserved.

HEAD TRAUMA

I. Epidural Hematoma
I. Middle meningeal artery, temporal bone fx
II. Presentation
I. LOC, transient lucid interval, neurologic
deterioration
III. Treatment
I. Surgical evacuation:
I. > 30 mL in volume (regardless of GCS)
II. Acute EDH
III. GCS <9 w/ pupillary abnormalities
II. Non operative management
I. Neuro ICU, repeat exams, repeat CT 6-8
hrs

HEAD TRAUMA

Subdural Hematoma
I. Tearing of bridging veins
II. Presentation: LOC, Headache, confusion, weakness
III. Treatment
I. Surgical evacuation
I. >10 mm thickness
II. Midline shift > 5mm
III. GCS <9, or GCS has decreased by 2 or more
points from time of injury
IV. Asymmetric or fixed or dilated pupils
V. ICP consistently >20 mmHg
II. Non operative management
I. Neuro ICU, repeat exams, repeat CT 6-8 hrs

1488
Copyright © Oakstone Publishing, 2021. All Rights Reserved.

HEAD TRAUMA

Subarachnoid Hematoma
I. Bleed in subarachnoid space, superficial sulci, adjacent
to skull fracture, contusion
II. Presentation: asymptomatic, headache, low GCS
III. Treatment
I. Repeat head CT
II. Reversal of anticoagulation
III. BP control
IV. ICU management if severe

HEAD TRAUMA

Intracerebral Hemorrhage
I. Presentation: headaches, weakness, confusion,
neurologic deficits
II. Treatment
I. Surgical evacuation (not as well defined)
I. Significant mass effect
II. Hemorrhage exceeds 50cm3 in volume
III. GCS 6 or 8 w/ frontal or temporal hemorrhage
>20cm3 w/ midline shift of at least 5mm
II. Nonoperative management
I. ICU management of ICPs
II. Reversal anticoagulation
III. Repeat scans/exam

1489
Copyright © Oakstone Publishing, 2021. All Rights Reserved.

HEAD TRAUMA

Diffuse Axonal Injury


I. Presentation: headaches to coma
II. Treatment
I. Prevention of secondary injury
I. Hypoxia, hypotension, edema, intracranial
hypertension

OUTLINE

I. Head trauma
i. Initial evaluation and treatment
ii. ICU management

1490
Copyright © Oakstone Publishing, 2021. All Rights Reserved.

HEAD TRAUMA

I. Head trauma
i. ICU management
i. Fluid management - prefer isotonic fluids
(normal saline)
ii. Cerebral perfusion pressure
i. CPP= MAP – ICP
ii. ICP monitoring indications
i. GCS <9 and mass effect on CT
ii. Age >40 yrs old, motor posturing, BP
<90
iii.

HEAD TRAUMA

I. Head trauma
i. ICU management
i. Treatment of ICP elevation (>22 mmHg)
i. HOB 30 degrees
ii. CSF drainage
iii. Sedation
iv. Osmotic therapy
i. Hypertonic saline (preferred)
i. 3% continuous infusion, goal sodium 145-155, central line
ii. 23% NaCl boluses
ii. Mannitol
i. Boluses 0.25 to 1 g/kg q 4-6 hours

1491
Copyright © Oakstone Publishing, 2021. All Rights Reserved.

HEAD TRAUMA

I. Head trauma
i. ICU management
i. Ventilation- avoid hyper and hypocarbia, monitor end tidal CO2
ii. Seizure prophylaxis (Keppra), typically a week
iii. Avoidance of fever
i. Maintain normothermia
i. Medications- antipyretics
ii. Cooling blankets
iii. Treat shivering
iv. Treatment of sympathetic hyperactivity
i. Supportive- reduce stimulation, antipyretics
ii. Beta blockers, clonidine, gabapentin, propranolol, bromocriptine

HEAD TRAUMA

I. Head trauma
i. Mild traumatic brain injury (ie, concussion)
i. Most common TBI
ii. GCS 13-15, LOC
iii. Treatment
i. Observation for at 24 hrs, in house or at home by responsible caregiver
ii. Indications for admission
i. GCS <15, abnormalities on head CT, seizures, coagulopathy, neuro
deficits, recurrent vomiting

1492
Copyright © Oakstone Publishing, 2021. All Rights Reserved.

OUTLINE

I. Initial assessment of a trauma patient


i. Primary and secondary survey
i. Management of life-threatening injuries
ii. Role of thoracotomy, REBOA
II. Head trauma
i. Initial evaluation and treatment
ii. ICU management
iii. Facial trauma
i. Orbit
ii. Midface

FACIAL TRAUMA

I. Orbit
I. Classified as open globe, closed globe, or periocular
II. Thorough eye exam, EOM assessment, CT scan
III. Medial and inferior orbital wall more susceptible to
entrapment
IV. If foreign body, leave in place
V. Do not:
I. Apply pressure to eyeball, eyelid retraction, avoid
placing any medications, call Optho

1493
Copyright © Oakstone Publishing, 2021. All Rights Reserved.

FACIAL TRAUMA

I. Orbit
I. Identify orbital compartment syndrome
I. proptosis, eye pain, pupil defect, decreased visual
acuity, swelling
I. Treatment: lateral canthotomy and inferior
cantholysis

FACIAL TRAUMA

I. Midface Injury
I. Complex fractures of face classified using LeFort system

1494
Copyright © Oakstone Publishing, 2021. All Rights Reserved.

OUTLINE

I. Initial assessment of a trauma patient


i. Primary and secondary survey
i. Management of life-threatening injuries
ii. Role of thoracotomy, REBOA
II. Head trauma
i. Initial evaluation and treatment
ii. ICU management
iii. Facial trauma
III. Neck trauma
i. Blunt
ii. Penetrating

NECK TRAUMA
I. Neck trauma
i. Anatomic zones
1

1495
Copyright © Oakstone Publishing, 2021. All Rights Reserved.

NECK TRAUMA
I. Neck trauma
i. Anatomic zones
ii. Hard and soft signs of injury

Hard Signs Soft Signs

Vascular Unstable, pulsatile hematoma, expanding Bruit, pulse deficit


hematoma, active hemorrhage, neurologic
deficit

Respiratory Airway compromise, air bubbling, Hemoptysis, dysphonia, tracheal tenderness,


impressive subq emphysema dyspnea

Digestive -- Hematemesis, odynophagia, dysphagia, subq


emphysema

NECK TRAUMA
I. Neck trauma
i. Anatomic zones
ii. Hard and soft signs of injury
iii. Workup
i. Dependent on stability of patient and zone of injury
ii. CTA, EGD/UGI, Bronchoscopy/Laryngoscopy

1496
Copyright © Oakstone Publishing, 2021. All Rights Reserved.

NECK TRAUMA
Moderate symptoms or
signs or an asymptomatic
patient

Zone 1 Zone 2 Zone 3

Rule out vascular injury Rule out aerodigestive Physical exam Vascular injury Aerodigestive Rule out vascular injury
injury (see Zone 2)

Normal exam, r/o injuries


if track away from CTA vs. arteriography vs. Physical exam, CTA vs.
Chest xray vascular/aerodigestive Larynx, trachea Esophagus
duplex arteriography
structures

CT, laryngoscopy,
CTA UGI, EGD
bronchoscopy

Feliciano DV, Mattox KL, Moore EE. eds. Trauma, 9e. McGraw Hill; 2020

NECK TRAUMA
Overt Signs and
Symptoms

Hemorrhage Suffocation

From compression by
From wound to
Intrapleural (Zone 1) External (Zone 2,3) Into Trachea (Zone 2) vascular hematoma
trachea (II)
(II)

Finger compression, Follow air bubbles or


High anterolateral Small entrance (I, II, Large entrance intubation, enlarge skin hole, ETT vs.
thoracotomy III) (I,II,III) cricothyroidotomy, intubate through Cricothyrotomy
bronchoscopy tracheal defect

Finger or balloon
Finger or gauze pack
through skin or
compression
through ICA

Feliciano DV, Mattox KL, Moore EE. eds. Trauma, 9e. McGraw Hill; 2020

1497
Copyright © Oakstone Publishing, 2021. All Rights Reserved.

NECK TRAUMA
I. Neck trauma
i. Anatomic zones
ii. Hard and soft signs of injury
iii. Workup
iv. Management
i. Vascular injuries
ii. Esophageal injuries
iii. Tracheal injuries

NECK TRAUMA
i. Management
i. Vascular injuries
i. Common and internal carotid injuries
i. Prox and distal control: ICE
ii. Consider heparin before clamping if possible
iii. Usually amenable to primary repair
i. Interrupted 6-0 prolene, or end to end repair if circumferential
ii. Patch angioplasty if adjacent to bifurcation
iii. Large defect- interposition graft (saphenous vein preferred)
ii. Vertebral injuries
i. Control bleeding with bone wax (difficult to access), fogarty catheter,
and IR for embolization
iii. Internal jugular-if time permits can repair, otherwise ligate

1498
Copyright © Oakstone Publishing, 2021. All Rights Reserved.

NECK TRAUMA
i. Management
i. Esophageal injuries
i. When identified early (12-24 hrs) full thickness injuries can be repaired
i. 2 layer fashion
ii. Drains
iii. Muscle flap (especially if tracheal or vascular injury also present)
i. Flap should be in between repairs
iv. Esophageal exclusion if catastrophic injury
i. Place G or J tube

NECK TRAUMA
i. Management
i. Tracheal Injury
i. Primary repair
i. Single layer, interrupted, absorbable suture
ii. Muscle flap over repair (SCM, omohyoid)
iii. Tracheostomy?

1499
Copyright © Oakstone Publishing, 2021. All Rights Reserved.

NECK TRAUMA
i. Blunt Cerebrovascular injury
i. Pathophysiology: stretching or impingement of vessel walls causing
intimal tear, exposure of subintimal layers to blood flow

NECK TRAUMA

EAST RECOMMENDATIONS
DENVER SCREENING CRITERIA

1500
Copyright © Oakstone Publishing, 2021. All Rights Reserved.

NECK TRAUMA

I - Irregularity of vessel wall or dissection/intramural III - Pseudoaneurysm


hematoma with < 25% luminal stenosis
IV - Vessel Occlusion
II - Intraluminal thrombus or raised intimal flap, or
dissection/ intramural hematoma with ≥25% luminal V - Vessel Transection
stenosis

BACKGROUND

• BCVI Management

Scandinavian Protocol

1501
Copyright © Oakstone Publishing, 2021. All Rights Reserved.

Thank you

1502
Copyright © Oakstone Publishing, 2021. All Rights Reserved.

Thoracic Trauma

Geoffrey A Anderson, MD MPH


Brigham & Women’s Hospital Division of Trauma

Disclosures

• NONE

1503
Copyright © Oakstone Publishing, 2021. All Rights Reserved.

Outline
• General Principles • Intercostal vascular bleeding
• Operative approaches • Diaphragm injuries
• Resuscitative thoracotomy • Lung injuries
• Cardiac injuries • Esophageal injuries
• Pericardial Windows • Tracheal injuries
• Sternal & Rib injuries

General Principles
• As always: start with ABCs

• Primary adjuncts: EFAST, CXR

• Stable -> CT

• For GSWs -> missile series

1504
Copyright © Oakstone Publishing, 2021. All Rights Reserved.

General principles
• Most thoracic trauma is non-op

• Permissive hypotension first developed in thoracic trauma

• High ratio blood transfusion works

• Auto-transfusion can be used with special pleur-evac

Indications for the OR


• Hemodynamic instability

• Chest tube output guidelines:


• >1.5 L initially
• >150-200 mL/Hr for 2-4 hours

• Aerodigestive injury

1505
Copyright © Oakstone Publishing, 2021. All Rights Reserved.

Operative Approaches
• Depends on scenario

• Sternotomy is often option #1 for penetrating wounds to anterior


chest

• Left AL thoracotomy for no signs of life

• Clamshell for posterior injuries or if extension into right chest is


needed after left thoracotomy

Resuscitative Thoracotomy
• No signs of life
• resuscitative thoracotomy in the ED
• little to no role for traditional ACLS – vast majority have exsanguinated

• Indications:
• cardiac activity on ultrasound
• Time based: EAST, AAST slightly different criteria
• Of all trauma patients: Penetrating > blunt, chest > abdomen, short downtime
> long down time
• Therefore - Best chance: penetrating chest trauma, low threshold

1506
Copyright © Oakstone Publishing, 2021. All Rights Reserved.

Resuscitative Thoracotomy
• Positioning: supine with left arm overhead

• Splash of betadine, no drapes

• High risk for sharps injury –> Slow is smooth and smooth is fast

• Have 2nd team place right chest tube at the same time

• Get IV access above the diaphragm (bilateral subclavian Cordis lines


works great here)

Resuscitative Thoracotomy Technique


• First cut to the ribs. Sternum, curve into the axilla. Just below the
nipple in a man (inferior mammary crease in woman).

• PITFALL – going straight down to the bed, going too low on chest

• Use large scissors to divide intercostals avoiding NV bundle at inferior


boarder. Holding respiration during pleural entry can reduce chance
of injury

• Insert Finochietto, open ribs, arm of Finochietto away from sternum

1507
Copyright © Oakstone Publishing, 2021. All Rights Reserved.

Resuscitative Thoracotomy Maneuvers


• Sweep lung out of the way, take down inferior pulmonary ligament if
you have to
• PITFALL: avoid pulmonary vein
• Perform pericardotomy cranial to caudal, avoid phrenic nerve on
lateral aspect of heart – cut anteriorly to this nerve
• If tamponade – use scalpel
• If no tamponade – use forceps and mets
• Deliver the heart
• You have now: relieved tamponade, have access to repair the heart,
open cardiac massage, epi into ventricle

Resuscitative Thoracotomy Maneuvers


• Cross clamp the aorta
• Hand hugs posterior ribs and travels to the spine
• Make a hole in the parietal pleura covering the mediastinum
• First structure above spine is the aorta, it will be empty and unimpressive
• People discuss NGT, but this is rarely relevant and is unnecessary
• Place vascular clamp

• Now is a great time to ramp up the MTP and give intra-cardiac epi

• If you get ROSC:


• rush to the OR for definitive repairs
• Don’t forget to tie off IMA’s

1508
Copyright © Oakstone Publishing, 2021. All Rights Reserved.

Clamshell
• Done for:
• More exposure
• Large volume hemothorax on right

• Mirror image incision in Right chest

• Come across lower sternum: bone cutters, Lebsche knife, trauma sheers.
Lower sternum is not has robust as upper

• Finochietto in the right side of incision

• Take down pleura/adhesions between mediastinum and anterior chest wall

Median Sternotomy
• Preferred approach for most penetrating wounds to the anterior
chest

• Good exposure to heart, anterior mediastinum, lungs, distal trachea.


It’s fast, bloodless, less post op respiratory complications

• Poor exposure for posterior mediastinum, does not allow aortic cross
clamping

1509
Copyright © Oakstone Publishing, 2021. All Rights Reserved.

Median Sternotomy Technique


• Skin incision – sternal notch to xiphoid all the way down to sternum

• Clear off tissue above and below the sternum, including


interclavicular ligament

• Score the sternum with bovie

• Divide down the middle with saw or Lebsche, pull up as you go, ask
anesthesia to hold respirations

• Place Finochietto, cheat slightly cephalad

Blunt Cardiac Injuries


• Associated with significant force to chest

• Look for:
• Arrhythmias
• Unexplained hypotension

• Effectively ruled out with normal EKG and negative troponins

• If these are not normal then the next step is a TTE

1510
Copyright © Oakstone Publishing, 2021. All Rights Reserved.

Penetrating Cardiac Injuries


• Often unstable

• Diagnosis:
• Physical (the “Box”)
• FAST
• CXR/CT

• Preferred approach:
• Sternotomy
• Exceptions: Clamshell for resuscitative thoracotomy, SW or GSW from the
back

Pericardial Window Technique


• If low suspicion then consider repeat FAST and formal TTE

• Possible Indications:
• equivocal pericardial FAST with possible trajectory
• injury to the box with left hemothorax
• suspicious and already doing a laparotomy

• Types:
• Subxiphoid – most common
• Transdiaphragmatic – if you are already doing a trauma ex lap

1511
Copyright © Oakstone Publishing, 2021. All Rights Reserved.

Penetrating Cardiac Injuries - Technique


• Open pericardium
• Elevate heart by suturing pericardium to sternum
• Digital control
• Large needle, often 2-0 or 3-0 prolene, +/- pledgets, tie down to the
heart, figure of 8 suture, pericardium can be used as “pledget”
• Look out for coronary vessels, use horizontal mattress under the vessel
• Posterior injuries – elevate heart with lap pads, apex stitch (avoid this
one), dual forceps on the apex, brief period of lift, communicate with
anesthesia, avoid long lift and kink of great vessels

Cardiac Injuries - Pitfalls


• Access to large tapered needle monofilament

• Pulling through myocardium

• Occluding Coronary vessels

• Posterior injuries

1512
Copyright © Oakstone Publishing, 2021. All Rights Reserved.

Sternal and Rib Fractures


• very common, look for underlying hemo/pneumothorax,
pulmonary contusions

• Elderly with multiple rib fx are high risk for respiratory


decompensation and probably warrant ICU admission

• Mainstay is multimodality pain control (Tylenol, NSAIDs,


gabapentin, lidocaine patches, opioids), involve pain team for
regional interventions (epidurals, paraspinal blocks, etc)

Rib Fixation
• Indications
• displaced fractures AND respiratory failure
• significant chest wall deformity
• persistent pain

• Best if done in the first few days after injury

• Optimal fractures to fix are mid ribs (4-9) with lateral fractures

• Pre-Op CT with 3D recons can be helpful

• Often done with VATS and Bronch

1513
Copyright © Oakstone Publishing, 2021. All Rights Reserved.

Flail Chest
• Radiographic flail is more than 2 consecutive ribs broken in 2 or more
places.

• Watch for physiologic flail with paradoxical movement, pulmonary


contusions and respiratory compromise

• Intubate these patients early before they crash

Intercostal Arterial Injuries


• Can bleed a lot

• Can be challenging to control

• IR is an options. Beware there are a lot of collaterals between levels


of intercostals, might require multi level embolization

• Operative control requires good exposure – open the parietal pleura,


use all the tools – suture, clips to temporize, suture around the rib if
you have to

1514
Copyright © Oakstone Publishing, 2021. All Rights Reserved.

Diaphragm Injuries
• CT is ok but misses injuries
• Dx laparoscopy often needed
• These will get bigger over time if untreated
• Left side injuries of any size should be repaired, often right side can
be left alone unless very large
• Common scenario: flank thoracoabdominal stab wound -> Observe
for development of peritonitis. Then do Dx Lap for for diaphragm
• Blunt injuries are less common but can occur, these defects are often
large (7-8cm)

Diaphragm Injuries - Technique


• Can be down Open or Laparoscopically

• For the Dx lap place ports like you would for foregut or bariatric
surgery, use steep reverse Trendelenburg
• During insufflation watch for signs of developing tension capnothorax
• Upsize one of the trocars for laparoscopic suturing

• Grasp the diaphragm with Alice forceps and pull into the operative
field, use the forceps to line up the repair

1515
Copyright © Oakstone Publishing, 2021. All Rights Reserved.

Diaphragm Injuries - Technique


• Repair with interrupted, nonabsorbable figure of 8 sutures, usually
zero monofilament

• In the acute setting you almost never need to do a mesh repair bc


there has not been time for loss of domain

• Suction out the chest before closing the injury

• Place a chest tube

Lung Injuries
• Low pressure system, high levels of tissue thromboplastin -> all this
means most (80-90%) can be managed with just a chest tube

• High rate of associated injuries

• Almost no role for anatomic lung resections

• Pneumonectomy has an extremely high mortality rate

• If you do have to go to the OR: consider double lumen ETT if time


allows, use low tidal volume ventilation to prevent air embolism

1516
Copyright © Oakstone Publishing, 2021. All Rights Reserved.

Tractotomy
• For deep, penetrating wounds without hilar involvement
• Stabilize with duval forceps
• Identify entrance and exit wounds
• Place GIA through the wounds and fire, try to keep the tractotomy
parallel to lung vasculature
• Identify any bleeders/air leak and over sew with figure of 8
absorbable suture on large needle. You can also add tissue sealants
• Can reapproximate tissue with running suture

Wedge Resection
• Ideal approach for peripheral or small injuries

• Grasp the tissue with Duval forceps

• Fire GIA stapler across the lung parenchyma

• Check the staple line


• for hemostasis
• Air leak – over sew if positive

1517
Copyright © Oakstone Publishing, 2021. All Rights Reserved.

Lobectomy
• Anatomic lung resection is rarely needed

• But the basic principle is to first incircle the hilum of the lobe with
your fingers to get digital control and then place a vascular clamp. Use
a TA stapler to take the hilum.

• Tip: place stay suture before firing the TA stapler bc the tissue will
retract away from you after stapling and make it harder to control any
persistent bleeding

Pneumonectomy
• Rarely needed

• Only for severe central hilar injuries in very unstable patient

• Communication with anesthesia is key

1518
Copyright © Oakstone Publishing, 2021. All Rights Reserved.

Pneumonectomy
• Some describe a “hilar twist.” I find this unnecessary and it is just as easy to
encircle the hilum with your hand for initial hemostasis.
• Place a vascular clamp, allow anesthesia to adjust. This maneuver will
result in immediate severe right heart strain
• Avoid phrenic and vagus nerves, take down the inferior pulmonary
ligament
• Come across hilum with TA stapler – can use 1 firing or separate for vessels
and bronchus
• Stay suture placed before transection
• Staple as close to the carina as possible to avoid pooling secretions and
stump break down
• Cover your stump with adjacent tissue: parietal pleura, pericardial fat,
intercostal muscle flap

Lung Injuries - Pitfalls


• Air embolism can be a significant problem with severe pulmonary
injuries
• look out for air bubbles in the coronary vessels, sudden cardiac arrest, or
other arrhythmias
• Place patient in Trendelenburg, elevate apex of heart, aspirate both ventricles

• If large amount of lung is resected make sure to talk to anesthesia


about reducing the tidal volumes to reduce barotrauma and limiting
fluids because of right heart failure

• Don’t forget to do a bronch at the end of the case to aspirate out any
remaining blood

1519
Copyright © Oakstone Publishing, 2021. All Rights Reserved.

Esophageal Injuries
• Rare in blunt trauma
• Diagnosis:
• CT, CT esophagram, esophagram with fluoroscopy (gastrografin then thin barium),
Endoscopy
• Possible need for multiple studies when the index of suspicion is high
• Missed injuries can result in life threatening mediastinal infection
• Preferred approach:
• right thoracotomy for upper and middle injuries
• left thoracotomy for distal injuries
• Laparotomy for the intra-abdominal esophagus and left neck incision for the cervical
esophagus
• Vast majority of injuries can be repaired primarily – it is only the rare injury
with large tissue destruction that would require a conduit (usually gastric
pull up)

Esophageal Injuries - Technique


• Place a double lumen ETT
• Posterolateral thoracotomy from anterior axillary line, extend 1-2
fingers below the tip of scapula and curve up posterior to scapula.
• Divide the lat muscle and preserve the rhomboid posteriorly
• Divide the intercostal muscles, spread the ribs with finechietto, shingle the 6th
rib if needed
• Take an intercostal muscle flap on the way in by taking it bluntly off the rib
• Divide inferior pulmonary ligament and retract lung medially
• Open the mediastinal pleura adjacent to the azygos vein
• Mobilize the esophagus and encircle it with a penrose

1520
Copyright © Oakstone Publishing, 2021. All Rights Reserved.

Esophageal injuries - Technique


• Debride all injured/ischemic tissue
• Incise muscularis longitudinally (superiorly and inferiorly) to expose
the extent of the mucosal injury
• Primary repair of mucosa with interrupted absorbable sutures
• Repair the muscularis with interrupted non-absorbable sutures
• Avoid narrowing the lumen
• Buttress with adjacent tissue and leave drains
• Consider placing some sort of gastric tube for drainage and
jejunostomy tube for feeding

Tracheobronchial Injuries
• High index of suspicion when there is large, persistent air leak in the
chest tube, or based on clinical or CT findings

• Best Dx tool is bronchoscopy

• These injuries are more common with penetrating trauma

• Median sternotomy is often approach of choice for central injuries

• Exposure of the thoracic trachea often involves retraction or even


division of the left innominate vein

1521
Copyright © Oakstone Publishing, 2021. All Rights Reserved.

Tracheobronchial Injuries - Technique


• Primary repair of majority after debridement of devitalized tissue
using interrupted absorbable suture
• Deflate the ETT balloon before passing a needle through the trachea
depending on the level of injury
• Buttress the repair with muscle flap
• Consider a protective tracheostomy
• Perform a bronch at the end of the case to suction out all the blood
• If the injury is large then a resection and anastomosis must be done –
mobilize the trachea but minimize the extent of mobilization to avoid
devascularization, avoid the RLN’s that run laterally in TE groove

Closure of the Chest


• Thoracotomy – close in the usual fashion, use rib approximator, large
needle, figure of 8’s of absorbable suture, approximate but don’t
need to crank down super hard (just more post operative pain),
reapproximate the muscle and close in layers, leave chest tube

• Sternotomy – usual fashion with steel wires. Ensure good hemostasis


after sternal retractor is removed. Leave mediastinal drain and chest
tubes, close the presternal fascia with absorbable suture

1522
Copyright © Oakstone Publishing, 2021. All Rights Reserved.

Take Homes
• Remember to start with the ABCs

• Most injuries can be treated non-operatively

• The CXR and EFAST are important adjuncts to the primary survey

Take Homes
• Know your indications for a resuscitative thoracotomy and make that
decision immediately (yes or no) for a patient in arrest

• Hemodynamically unstable patients, as always, belong in the OR,


triage cavities as best you can – missile survey, EFAST, DPA

• Median Sternotomy is the procedure of choice for most penetrating


injuries to the anterior chest

1523
Copyright © Oakstone Publishing, 2021. All Rights Reserved.

Take Homes
• Cardiac injuries: digital control, then figure of 8 prolenes on a large
taper needle, leave drains

• Rib fractures: multi modality pain control and regional anesthesia

• Don’t miss a diaphragm injury, especially on the left side, this can lead
to significant morbidity later in life

Take Homes
• Lung injuries: tractotomy and non anatomic resection, bronch at the
end, leave a chest tube

• Esophageal injuries: Dx with CT or floro esophagram and/or EGD,


often need 2 modalities if the index of suspicion in high. Primary
repair after fully exposing the mucosal defect, buttress and drain

• Tracheal injuries: Dx with bronchoscopy, most can be primarily


repaired, buttress, bronch at the end

1524
Copyright © Oakstone Publishing, 2021. All Rights Reserved.

Thank you

Geoffrey Anderson
Brigham and Women’s Hospital
Division of Trauma

1525
Copyright © Oakstone Publishing, 2021. All Rights Reserved.

Abdominal Trauma
Nakul Raykar MD MPH
Trauma and Emergency Surgery, Brigham and Women’s Hospital, Boston, MA
Fellowship Director, Program in Global Surgery and Social Change, Harvard
Medical School

Disclosures
None to report

1526
Copyright © Oakstone Publishing, 2021. All Rights Reserved.

The Abdomen is Complicated!


• Initial trauma resuscitation
• Diagnostic modalities
• Basic approach in the operating room
• Management strategies in the
operating room for specific injuries
• Resources for algorithms

The Abdomen
• [picture]

1527
Copyright © Oakstone Publishing, 2021. All Rights Reserved.

Initial Trauma Resuscitation


• The basics of trauma care remain the same as managing any
critically ill patient
• Trauma patients undergo a primary and secondary survey to
address threats to life:
• Airway
• Breathing
• Circulation
• With a focus on treating bleeding, bleeding, bleeding!

Initial Trauma Resuscitation


• Approach to patients with ‘blunt’ and ‘penetrating’ mechanisms
• Use of ultrasound as a diagnostic –
the FAST exam
• Damage control resuscitation

1528
Copyright © Oakstone Publishing, 2021. All Rights Reserved.

Approach To Blunt vs. Penetrating Type of


Injury

(In general…) Blunt


CT

Penetrating OR

Hemodynamically
unstable? OR

The FAST Exam (Ultrasound)


• Focused Assessment with Sonography in Trauma
• Sensitivity for hemoperitoneum ranges from
30-98%. Specificity is consistently high.
• If the FAST is positive, you better act fast!
• Hazard ratio increases by 1.5 for every 10 min increase in time to OR
after positive FAST
Barbosa RR, Schreiber MA et al. J Trauma Acute Care Surg. 2013 Jul;75(1 Suppl 1):S48-52

1529
Copyright © Oakstone Publishing, 2021. All Rights Reserved.

Trauma Triad of Death

Damage Control Resuscitation


• Rapidly evolving science (and guidelines)
• Replace blood with blood (or best attempt)
• 2017 EAST Trauma Guidelines recommend 1:1:1 (PRBC to FFP to
Platelets) ‘balanced’ resuscitation approach
Cannon, JW et al. Damage control resuscitation in patients with severe traumatic hemorrhage, Journal of Trauma and Acute Care
Surgery, March 2017 - Volume 82 - Issue 3 - p 605-617

• In many institutions, 1:1:1 is equal to 4:4:1 or 6:6:1

1530
Copyright © Oakstone Publishing, 2021. All Rights Reserved.

Basic Approach in the OR (Trauma Laparotomy)


• Prep widely – neck to knees, cover genitalia with towel
• Keep it moving
• Swift, definitive entry into the abdomen – “three strokes”
• Pack all four quadrants quickly

Trauma Triad of Death

1531
Copyright © Oakstone Publishing, 2021. All Rights Reserved.

Damage Control Laparotomy


• Two immediate goals of this operation:
(1) control bleeding
(2) control contamination
• Shorter operations,
preferably 90 minutes or less
• Get patients to ICU – warm and resuscitate – and use adjuncts like
angioembolization
• Return to the OR in 24-48 hours for definitive repairs

Vascular Injuries

https://www.facs.org/quality-programs/trauma/education/asset

1532
Copyright © Oakstone Publishing, 2021. All Rights Reserved.

Management of Specific Injuries


• Bowel, small and large
• Liver
• Spleen
• Pancreas
• Duodenum
• Stomach
• Urinary Tract

Management of Specific Injuries


• Bowel, small and large
• Liver
• Spleen
• Pancreas
• Duodenum
• Stomach
• Urinary Tract

1533
Copyright © Oakstone Publishing, 2021. All Rights Reserved.

Bowel
• Small bowel ~ 600 cm; large bowel ~ 150 cm
• ‘Running the bowel’– complete assessment of full length is critical;
beware of the mesenteric borders
• Small denudations/contusions – inversion stitch
• Lacerations – repair with suture

Bowel
• Small denudations/contusions –
inversion stitch / “oversew”
• Lacerations – repair with suture. Single layer absorbable for
small bowel. Large bowel in two layers.
• More extensive injuries or multiple areas of injury to a given
segment – resection with anastomosis; deciding when is an art

1534
Copyright © Oakstone Publishing, 2021. All Rights Reserved.

Bowel – Rectum
• Intraperitoneal rectum – treat like colon
• Extraperitoneal rectum
• If can be approached (and repaired) transanally, repair
• If cannot, then proximal diversion

Management of Specific Injuries


• Bowel, small and large
• Liver
• Spleen
• Pancreas
• Duodenum
• Stomach
• Urinary Tract

1535
Copyright © Oakstone Publishing, 2021. All Rights Reserved.

Liver
• Most commonly injured organ in abdomen
• Highly vascularized
• Portal vein
• Hepatic artery
• Hepatic veins and IVC
• Manipulation of attachments to diaphragm are often key to control
liver bleeding

Liver – Operative Management


• Pack the liver
• reapproximate the shape
• beware of IVC compression
• consider mobilizing attachments
• Pringle maneuver
• Less severe injuries can be managed with electrocautery, energy
devices (Aqua Mantys, Argon beam), large blunt tipped sutures,
hemostatic agents, tractotomy with oversew

1536
Copyright © Oakstone Publishing, 2021. All Rights Reserved.

Liver – Operative Management


• Complex injuries, not slowed by Pringle
• Hepatic vein source
• If not amenable to primary technique, then partial
hepatectomy may be indicated
• If coming from around the liver (or behind it)
• Hepatic vein into IVC disruption
• Very worrisome, high mortality
• “Total hepatic venous isolation”
• Infahepatic control – medial rotation right colon
• Suprahepatic control – (1) Below diaphragm – hard (2) control above
diaphragm, in pericardium either through (a) median sternotomy or (b)
through diaphragm

Management of Specific Injuries


• Bowel, small and large
• Liver
• Spleen
• Pancreas
• Duodenum
• Stomach
• Urinary Tract

1537
Copyright © Oakstone Publishing, 2021. All Rights Reserved.

Spleen
• Located in the left upper quadrant, often partially protected
posteriorly and laterally
by the rib cage
• Attached to stomach, diaphragm,
colon, kidney
• Prone to injury from variety of mechanisms
• Highly vascular and important immunologic functions

Spleen – Non Operative Management


• Splenic preservation is preferred for all hemodynamically normal
patients-use angioembolization as needed
• However, failure of non-operative management has much worse
outcomes, so important to pick your battles
• Many trauma centers employ a routine 48-72 hour post-injury CT
to assess for new vascular pseuodaneurysm development

1538
Copyright © Oakstone Publishing, 2021. All Rights Reserved.

Spleen – Operative
• Hemodynamic instability or severe splenic bleeding while
operating for another indication
• While splenic repair is described, typically splenectomy is
preferred in a trauma scenario
• Rapidly mobilize the spleen from its lateral and posterior
attachments – can be done bluntly; divide the splenocolic
attachments, and take the gastrosplenic ligaments with a vascular
stapler

Spleen – Additional Points


• Splenectomy does leave a patient vulnerable to encapsulated
bacteria! (H. Influenzae, S. pneumoniae, N. Meningitidis)
• Optimal vaccination time:
2 weeks pre-op
• Optimal vaccination in trauma:
2 weeks post-op
• Most common practice recommendation: administer prior to
hospital discharge

1539
Copyright © Oakstone Publishing, 2021. All Rights Reserved.

Management of Specific Injuries


• Bowel, small and large
• Liver
• Spleen
• Pancreas
• Duodenum
• Stomach
• Urinary Tract

Pancreas
• Retroperitoneal location, surrounded by critical structures on all
sides, and overlying the spine
• Must look for pancreatic injury as not always obvious
• Gastrocolic ligament – anterior surface
• Gastrohepatic ligament – superior border
• Kocher maneuver – head of pancreas
• Splenic mobilization – tail and posterior

1540
Copyright © Oakstone Publishing, 2021. All Rights Reserved.

Pancreas Injury – Operative


• Pancreatic injuries and their management can be complex, but
decision-making does not have to be:
1. Stop the bleeding
2. Control the secretions
• Is there major ductal injury and where?
• What does the gland look like?
• Consider a cholangiogram through the gallbladder with
morphine to contract the Sphincter of Oddi

Pancreas Injury – Operative (Western Trauma Association)

1541
Copyright © Oakstone Publishing, 2021. All Rights Reserved.

Pancreas
• Distal pancreatectomy for major bleeding or ductal injury in the
tail (to the left of the SMV)
• Everything else: leave drains
• “Trauma Whipple” – only if the injury does it for you (total
destruction of head of pancreas, duodenum, common duct)

Management of Specific Injuries


• Bowel, small and large
• Liver
• Spleen
• Pancreas
• Duodenum
• Stomach
• Urinary Tract

1542
Copyright © Oakstone Publishing, 2021. All Rights Reserved.

Duodenum
• Housed in retroperitoneum, overlying vertebral column, aorta,
IVC, kidney and associated vasculature
• At risk in both penetrating injury and blunt injury
• Exposure can require medial rotation of colon, takedown hepatic
flexure

Duodenum
• Housed in retroperitoneum, overlying vertebral column, aorta,
IVC, kidney and associated vasculature
• At risk in both penetrating injury and blunt injury
• Exposure can require medial rotation of colon, takedown hepatic
flexure

1543
Copyright © Oakstone Publishing, 2021. All Rights Reserved.

Duodenum

Duodenum - Operative
• Simple laceration – repair in two layers, transverse orientation to
avoid narrowing
• Duodenal hematomas, identified intraoperatively, can be drained
and closed in a single layer
• Complex injuries are…complicated, and care is often individualized

1544
Copyright © Oakstone Publishing, 2021. All Rights Reserved.

Duodenum –
Complex Repair Options
• Primary closure
• Resection of the injured segment of duodenum, mobilization of the
remainder, and primary anastomosis
• If the injury is proximal to the ampulla, closure of the duodenal
wound in two layers (or with a stapler) and gastrojejunostomy
• Roux-en-y duodeno-jejunostomy
• Roux-en-y end of jejunum brought over to the hole
• Jejunal serosal patch? Not recommended

Duodenum –
Additional Points
• Consider pyloric exclusion to protect complex duodenal repairs
• Commonly paired with gastrojejunostomy
• ‘Triple drain approach’ – limited data, consists of gastrostomy
tube (drainage), retrograde jejunostomy (drainage), anterograde
jejunostomy (feeding)
• Complete, complex destruction to duodenum may be indication for
a trauma Whipple

1545
Copyright © Oakstone Publishing, 2021. All Rights Reserved.

Management of Specific Injuries


• Bowel, small and large
• Liver
• Spleen
• Pancreas
• Duodenum
• Stomach
• Urinary Tract

Stomach
• Relatively mobile pouch connected between the more fixed
esophagus and duodenum
• Easy to miss injuries near GE junction and posterior
• Lacerations and small injuries can be closed in one or two layers
• Wedge resection or closure with GIA stapler also options

1546
Copyright © Oakstone Publishing, 2021. All Rights Reserved.

Management of Specific Injuries


• Bowel, small and large
• Liver
• Spleen
• Pancreas
• Duodenum
• Stomach
• Urinary Tract

Kidneys, Ureter, Bladder


• Kidney located in retroperitoneum
• CT scan is the key diagnostic modality in patients who are
hemodynamically stable
• Renal preservation is goal; prioritize angioembolization efforts
over surgery, when possible
• Nephrectomy for a devascularized kidney or a shattered kidney, if
in the OR for other reason
• Like spleen, non-operative management for high-grade injuries
prompts repeat CT in 48-72 hours in most centers

1547
Copyright © Oakstone Publishing, 2021. All Rights Reserved.

Kidneys –
Operative Management
• Zone II retroperitoneal hematoma – explore in penetrating injury,
with expansion of hematoma (in blunt or penetrating) injury, or
with hemodynamic instability in any type of injury
• Obtain vascular control early – retract up small bowel and open
peritoneum over aorta superior to the IMA, trace this up until you
encounter the left renal vein; arteries are posterior/superior
• Alternatively, medialize the colon and access it as it enters
Gerota’s fascia

Kidneys –
Operative Management
• Parenchymal injury-- cautery, topical agents, thin absorbable
suture
• Urinary collecting system injury – running 4-0 Vicryl or PDS
• Vascular injury – shunt if amenable and/or vascular repair if
patient is stable and can tolerate it
• Shattered kidney – nephrectomy only if >2/3rds not viable.
Otherwise, apply damage control procedures (pack, hemostatic
agents)

1548
Copyright © Oakstone Publishing, 2021. All Rights Reserved.

Kidneys –
Operative Management
• Parenchymal injury-- cautery, topical agents, thin absorbable
suture
• Urinary collecting system injury – running 4-0 Vicryl or PDS
• Vascular injury – shunt if amenable and/or vascular repair if
patient is stable and can tolerate it
• Shattered kidney – nephrectomy only if >2/3rds not viable.
Otherwise, apply damage control procedures (pack, hemostatic
agents)

Ureters –
Operative Management
• Base management on hemodynamic status of the patient – if
unstable, can always ligate or exteriorize ureter with a stent
• Ureteral repair strategies
• Very proximal: Re-anastomose back to renal pelvis
• Very distal: Re-anastomose to the bladder
• Middle: Primary anastomosis with 5-0 absorbable suture over
a stent; ureteroureteral anastomosis; Boari flap; Psoas flap

1549
Copyright © Oakstone Publishing, 2021. All Rights Reserved.

Bladder Injuries
• The bladder is located in the pelvis, and has a posterosuperior
free surface that is in the peritoneum, and an anteroinferior
surface that is extraperitoneal.
• Intraperitoneal injuries require operative management when
identified on preoperative CT
• Extraperitoneal bladder injuries may be managed non-
operatively with catheter decompression

Bladder Injuries – Operative Management


• Repair the intraperitoneal defect in two layers with absorbable
suture
• In complex trauma, where there is an extraperitoneal injury as
well as an intraperitoneal injury, you can repair the
extraperitoneal defect through the bladder defect, typically in a
running, single-layer closure

1550
Copyright © Oakstone Publishing, 2021. All Rights Reserved.

Important Resources – Western Trauma Algorithms

https://www.westerntrauma.org/western-trauma-association-algorithms/

Important Resources – EAST Trauma Guidelines

https://www.east.org/education-career-development/practice-management-guidelines

1551
Copyright © Oakstone Publishing, 2021. All Rights Reserved.

Management of > 20% TBSA


Burns: The First 24 Hours
Stephanie Nitzschke, MD
Brigham and Women’s Hospital

• No disclosures

1552
Copyright © Oakstone Publishing, 2021. All Rights Reserved.

Objectives
• Initial evaluation and management of the
burn patient

• Burn pathophysiology

• Fluid resuscitation

• Quick review of burn depth assessment

Priorities in Burn Care

1553
Copyright © Oakstone Publishing, 2021. All Rights Reserved.

Initial Management

Initial Management
• Primary Survey • Secondary Survey
Airway Complete History
Breathing Head to Toe Exam
Circulation TBSA
Disability Fluid Resuscitation
Exposure

1554
Copyright © Oakstone Publishing, 2021. All Rights Reserved.

History
Flame/scald Chemical/Electric
• How did the burn occur • What was the agent?
• Inside/outside • Duration of contact
• Did the clothes catch fire? • What has been done for
• Temperature of the liquid decontamination?
• Was the cloth removed • What kind of electricity was
• Abuse? involved, voltage
• LOC/CPR?

Management Principles
• Start fluid resuscitation
– Typically LR

• Monitor extremity perfusion

• Airway assessment

• Pain management

• Normothermia

1555
Copyright © Oakstone Publishing, 2021. All Rights Reserved.

Considerations
• Delay in presentation

• Alcohol or substance abuse

• Elderly

• Inhalation injury

• Need to transfer

Severity of Burn

• Extent of burn
– Rule of 9s/Lund Browder
– Scattered burns

• Depth of the burn


– Temperature
– Duration of contact
– Thickness of the dermis

1556
Copyright © Oakstone Publishing, 2021. All Rights Reserved.

Pathophysiology of burn shock

Pathophysiology: Severe Burn


> 20% TBSA
• Gradual hypovolemia

• Burn Shock =Time limited

• Extensive capillary leak


• In burned and unburned tissue

1557
Copyright © Oakstone Publishing, 2021. All Rights Reserved.

Pathophysiology of Burn Shock


Physiologic changes during burn shock

High

Normal

Low

0 hours 8 hours 16 hours 24 hours

Cardiac output Capillary leak PVR

Pathophysiology: Severe Burn


> 20% TBSA
• In the first 24 hours post burn
– Cardiac suppression
– PVR increases
– Massive extensive capillary leakage resulting in a
decrease in circulating plasma volume (20%)

• > 24 hours post burn


– Capillary leakage begins to resolve
– Cardiac output is supra-normal
– PVR is decreased

1558
Copyright © Oakstone Publishing, 2021. All Rights Reserved.

Fluid Resuscitation

Fluid Resuscitation
• For patients > 20% TBSA

• Several formulas exist


– Parkland
– Modified Brooke
– US Army Institute of Surgical Research

• Much debate about over- and under-resuscitation

• Crystalloid vs Albumin

1559
Copyright © Oakstone Publishing, 2021. All Rights Reserved.

Resuscitation Rates & Volumes


• Modified Brooke Formula:
kg x % TBSA x 2 cc = Volume/24hr

• Parkland Formula:
kg x % TBSA x 4 cc = volume/24hr

• ½ volume in 1st 8hrs; ½ volume over 16 hrs.

80kg patient, burned over 50% TBSA

• 80kg x 50% x 2ml/kg/TBSA% = 8L/24hr


– 8L divided by 2
– 4L in first 8 hours or 500ml/hr
– 4L in next 16 hours or 250ml/hr

• 80kg x 50% x 4ml/kg/TBSA% = 16L/24hr


– 16L divided by 2
– 8L in first 8 hrs or 1L/HR
– 8L in next 16 hrs or 500ml/hr

1560
Copyright © Oakstone Publishing, 2021. All Rights Reserved.

8.0

7.0

6.0

5.0

Modified Brooke
4.0
Parkland

3.0

2.0

1.0

0.0
mls/kg/%TBSA

The Fluid Creep


• Elderly, large cutaneous burns, inhalation
injury

• Hard to determine perfusion


– Urine output, lactate, ABG, SVO2, etc.

1561
Copyright © Oakstone Publishing, 2021. All Rights Reserved.

ISR ‘Rule of Ten’


1. Estimate Burn Size to the nearest 10%
• Use the Rule of Nine’s

2. Burn Size X 10 = INITIAL FLUID RATE


• Applies to ADULT patients 40-80 kg

3. For every 10 kg above 80 kg, add 100cc/hr to


the rate.

ABA 2008

ISR Rule of Ten

1562
Copyright © Oakstone Publishing, 2021. All Rights Reserved.

What’s your initial rate?

• 30% TBSA • 300 cc/hr


• 40% TBSA • 400 cc/hr
• 70% TBSA • 700 cc/hr
• 40% TBSA, • 400+200 =
100 kg 600 cc/hr

Resuscitate using basic critical


care principles…titrate the rate
up and down

Evaluation of Fluid Resuscitation


• Hemodynamics

• UOP (0.3cc/kg - 0.5cc/kg)

• Lab values (pH, bicarb, base deficit, lactate,


SVO2, Creatinine)

1563
Copyright © Oakstone Publishing, 2021. All Rights Reserved.

Resuscitation Strategy
• Classically managed with hourly titration of
fluids

• Fluids are titrated up and down by 20% based


on UOP (AND critical care principles)

• Generally NO fluid boluses

Fluid Resuscitation

1564
Copyright © Oakstone Publishing, 2021. All Rights Reserved.

How much is too much?

• >250 cc/kg in 24 hours


– Approximately 20L in an 80kg patient
– Associated with Abdominal Compartment
Syndrome

• Ivy Index: >6 cc/kg/%TBSA in 24 hours

Ivy et al. J Trauma 2000;49:387-91.

For the ‘Runaway Resuscitation’

• Albumin, FFP, Hypertonic Saline


– Poor data
– Findings of decreased overall volume infused, less
Intra-abdominal hypertension

• Other adjuncts
– Vit C
– CVVH

Jeng J. Problems in General Surgery 2003;20:37-46


O’Mara et al. J Trauma 2005;58:1011-1018
Tanaka et al. Arch Surg 1997;132:158-161

1565
Copyright © Oakstone Publishing, 2021. All Rights Reserved.

Complications of Resuscitation
• Total – 32 decompressive
laparotomies

• Overall 78% mortality


– >60% TBSA with almost
100% mortality

*Markell et al. JACS 2009

Optimal Resuscitation

Prefer slightly
under

1566
Copyright © Oakstone Publishing, 2021. All Rights Reserved.

Type of Fluid

1567
Copyright © Oakstone Publishing, 2021. All Rights Reserved.

Summary
• Initial management of the burn patient is the
same as the trauma patient

• Begin fluid resuscitation using ISR rule of ten

• Titration of fluids based on critical care


principles

Burn Injury Depth Assessment


• Superficial (1st degree)

• Partial Thickness (2nd degree)


– Superficial
– Deep

• Full Thickness (3rd degree)

1568
Copyright © Oakstone Publishing, 2021. All Rights Reserved.

Superficial
• Top layers of epidermis

• Painful

• NO blister

• Tx: topical moisturizer

• Will heal in 3-5 days

Courtesy Google Images

Superficial Partial Thickness


• Heals within 5-10 days

• Pink, blanches, wet,


painful

• Tx: with topical


antimicrobial

1569
Copyright © Oakstone Publishing, 2021. All Rights Reserved.

Deep Partial Thickness


• Heals within 10-20 days

• May need grafting

• Tx: with topical


antimicrobials

Full Thickness

• White, leathery,
dry

• Non-tender

• Likely need
grafting

1570
Copyright © Oakstone Publishing, 2021. All Rights Reserved.

Referral to a Burn Center

Transport
• Stabilize patient for transport
– Respiratory and Circulatory Support
– Appropriate fluid resuscitation Wound Care: Cover
with dry, sterile dressings
– Normothermia – Keep them warm!

• If delayed or long transport time


– Consult with our Burn MD on call
– Monitor compartments, escharotomy only if indicated

1571
Copyright © Oakstone Publishing, 2021. All Rights Reserved.

Burn Center Transfer Criteria


1. Partial thickness burns greater than 10%
body surface area (TBSA)
2. Burns involving the face, hands, feet,
genitalia, perineum, or major joints
3. ANY full thickness burns (3◦)
4. Electrical burns (lightning strike)
5. Chemical burns

American Burn Association, 2015

Burn Center Transfer Criteria


6. Inhalation injury
7. Burns along with pre-existing medical history
that could complicate management
8. Burns along with concomitant trauma where the
burn is greatest risk for M&M
9. Pediatric burns
10.Burn injury in patients who require special
social, emotional or rehabilitative intervention
American Burn Association, 2015

1572
Copyright © Oakstone Publishing, 2021. All Rights Reserved.

• Thank you

1573
Copyright © Oakstone Publishing, 2021. All Rights Reserved.

Top 6 Surgical
GI Emergencies

Reza Askari, MD
Trauma/Burns and Critical Care
Department of Surgery
Brigham and Women’s Hospital
Harvard Medical School

No financial disclosures

1574
Copyright © Oakstone Publishing, 2021. All Rights Reserved.

Top 6 Surgical GI Emergencies


Acute appendicitis

Acalculous / perforated cholecystitis

Perforated hollow viscus

Obstructed hernia

Acute mesenteric ischemia

Abdominal compartment syndrome

Case #1
53 yo F with PMH of HTN, OSA, who presents with 1 day history of abdominal pain.
Patient initially awoke from sleep with epigastric abdominal pain. Pain is described as
sharp and stabbing, progressively worsening, and now migrated to the RLQ. Pain is
also exacerbated by movement and by the bumps in the road on the drive to the
hospital. He also endorses minimal PO intake due to mild nausea, though denies
emesis. Denies fevers, chills, diarrhea. Last colonoscopy was 3 years ago and was
reportedly normal. In the ED, remains afebrile, tachycardic to HR 110s, normotensive.
Labs are notable for WBC 11.5, Cr 1.4, lactate 1.2. CT demonstrates dilated appendix
to 1cm with mild periappendiceal fat stranding.

Presentation consistent with acute uncomplicated appendicitis.


The patient would prefer to avoid surgery at this time due to an
upcoming business trip. How would you counsel him?

1575
Copyright © Oakstone Publishing, 2021. All Rights Reserved.

• 5 year observational follow-up of patients in the Appendicitis Acuta (APPAC) multicenter randomized clinical trial
comparing open appendectomy versus antibiotic therapy (IV ertapenem x3d followed by PO
levofloxacin/metronidazole x7d)
• 530 patients = 273 appendectomy, 257 antibiotics
• From antibiotics group, those requiring appendectomy (cumulative incidence):
• 16% within 90d
• 27.3% within 1yr
• 34% within 2yrs
• 35.2% within 3yrs
• 37.1% within 4yrs
• 39.1% within 5yrs
• 5 year overall complication rate (SSI, incisional hernia, pain, obstruction) – 24.4% appendectomy, 6.5% antibiotics
•  Antibiotic treatment alone is a feasible alternative to appendectomy for uncomplicated acute appendicitis

Salminen P, Tuominen R, Paajanen H, et al. Five-year follow-up of antibiotic therapy for uncomplicated
acute appendicitis in the APPAC randomized clinical trial. JAMA. 2018;320(12):1259-65.

• Randomized trial comparing appendectomy versus 10d antibiotics


• 1552 adults = 776 appendectomy, 776 antibiotics
• From antibiotics group, 29% underwent appendectomy by 90d
• 41% with appendicolith, 25% without appendicolith
• Higher rate of complications in antibiotics group -- missed less time from work, but more common to have visits
to the ED and hospitalizations
• Possibly attributed to those with appendicolith given higher risk of adverse events
•  Antibiotics are noninferior to appendectomy on the basis of a standard health-status measure (European
Quality of Life-5 dimensions Questionnaire)
• ~7 of 10 patients in the antibiotics group avoided surgery and able to be treated in the outpatient setting

CODA collaborative; Flum DR, Davidson GH, Monsell SE, et al. A randomized trial comparing antibiotics
with appendectomy for appendicitis. N Engl J Med. 2020 Nov 12;383(20):1907-19.

1576
Copyright © Oakstone Publishing, 2021. All Rights Reserved.

Case #2
79 yo M with PMH of CAD and NSTEMI s/p PCI, CHF (EF 30%), T2DM, ESRD s/p renal
transplant is admitted to the MICU for fevers and hypotension of unknown etiology.
Pressor support is immediately initiated, and he soon also requires intubation for
hypoxemic respiratory failure. Labwork demonstrates WBC 14.2, Cr 2.1, AST/ALT
243/198, Tbili 1.4, lactate 1.5. He is found to have Enterococcus faecalis bacteremia, as
well as acute PEs. CT abdomen/pelvis and abdominal US demonstrate a distended
gallbladder, gallbladder wall thickening to 5mm, and pericholecystic fluid.

Presentation likely reflects acalculous cholecystitis.


The MICU team is asking if cholecystectomy is warranted.

Management of acalculous cholecystitis

• Due to bile stasis, chronic distention, gallbladder ischemia,


hypoperfusion
• Often complicated by gangrene and perforation
• Mortality rate 30-60%, related to initial clinical severity
• Primary concerns with cholecystectomy:
• Patients are generally critically ill and poor surgical candidates
at the time
• High rate of conversion to open procedure
• Management:
• Pain control
• IVF resuscitation
• IV antibiotics
• Definitive therapy for gallbladder drainage – percutaneous
cholecystostomy tube versus cholecystectomy

1577
Copyright © Oakstone Publishing, 2021. All Rights Reserved.

Systematic review of literature 2000-2016


• PCT is typically 1st line of treatment for
acalculous cholecystitis
• Unless gallbladder is gangrenous or
perforated
• Non-surgical PCT versus surgical CCY  followed
• If low surgical risk, can consider CCY, but both
non-surgical PCT group for >3yrs
options are effective and PCT generally • No statistical difference in mean hospital LOS
associated with lower post-treatment • Surgical group had significantly more post-
morbidity treatment complications (18.8% vs 2.4%)
• PCT for acalculous cholecystitis may be •  Non-surgical PCT outcome is not inferior to
definitive treatment without need for surgical CCY
subsequent elective CCY given a low risk for
recurrence (studies range 2.3-9.8%)

Soria-Aledo V, Galindo-Iniguez L, Flores-Funes D, Carrasco-Prats M, Aguayo-Albasini JL. Is cholecystectomy the treatment of choice for acute acalculous
cholecystitis? A systematic review of the literature. Rev Esp Enferm Dig 2017;109(10):708-18.

Simorov A, Ranade A, Parcells J, et al. Emergent cholecystostomy is superior to open cholecystectomy in extremely ill patients with acalculous cholecystitis: A
large multicenter outcome study. Am J Surg. December 2013;206(6):935-41.

Kim SB, Gu MG, Kim KH, Kim TN. Long-term outcomes of acute acalculous cholecystitis treated by non-surgical management. Medicine 2020;99:7(e19057).

Case #3
82 yo F with PMH of HTN, T2DM, tobacco use presents to the ED in the middle of the
night with abdominal pain. She reports onset of epigastric pain 6d ago. Pain is sharp,
radiating to her RUQ and around to her back, and has been exacerbated with PO
intake. Over the past 2d she has developed persistent nausea and PO intolerance. Also
notes fever to 101.2, chills, and inability to sleep. She admits she has been scared to
bring herself to the hospital because of the pandemic. In the ED, her vitals include
temp 101.8, HR 118, BP 86/62. She is extremely tender on exam. Imaging reveals a
distended gallbladder with wall thickening and an area of irregular discontinuity along
the posterior wall with mild wall hypoenhancement and surrounding free fluid.

You suspect perforated cholecystitis.


How urgent should intervention be considered?

1578
Copyright © Oakstone Publishing, 2021. All Rights Reserved.

Management of gallbladder perforation

• Perforated gallbladder is often a complication from advanced


gallbladder inflammation, including gangrenous cholecystitis
• Treatment generally consists of emergency cholecystectomy
• Significant differences with cholecystectomy for perforated versus
non-perforated cholecystitis:
• Longer duration of surgery
• Higher rate of conversion from laparoscopic to open procedure
• Higher rate of bile duct injury
• Higher rate of re-intervention to manage perioperative
complications
• More likely to need ICU level of care
• Overall LOS

Jansen S, Stodolski M, Zirngibl H, Godde D, Ambe PC. Advanced gallbladder inflammation is a risk factor for gallbladder
perforation in patients with acute cholecystitis. World J Emerg Surg. 2018;13(9)

Jansen S, Doerner J, Macher-Heidrich S, Zirngibl H, Ambe PC. Outcome of acute perforated cholecystitis: A register study of
over 5000 cases from a quality control database in Germany. Surg Endosc. 2017 Apr;31(4):1896-1900.

Case #4
62 yo F with PMH of GERD, DVT (on apixaban), metastatic rectal cancer to the liver,
currently admitted for cycle 2 of chemotherapy, who developed acute onset of
abdominal pain overnight. Patient initially attributed it to gas pain, but the pain
persisted and progressively worsened throughout the night, preventing him from
sleeping. Also endorses worsening abdominal distention and 1x episode of bilious
emesis this morning. Labs are notable for WBC 15.5, Cr 1.39, and worsening lactic
acidosis from 1.9 to 2.9. CT abdomen/pelvis demonstrates free intraperitoneal fluid,
moderate pneumoperitoneum, and bowel wall thickening in the proximal duodenum.

Presentation likely consistent with gastroduodenal perforation.


What are some approaches to surgical management?

1579
Copyright © Oakstone Publishing, 2021. All Rights Reserved.

Surgical approach to gastroduodenal perforation

• Key considerations:
• Oversew ulcer versus Graham patch
• Biopsy ulcer margin to rule out gastric carcinoma
• Highly important to limit delay to OR!
‒ Every hour of surgical delay from hospital admission associated
with increased postoperative morbidity and mortality
• Laparoscopic versus open approach
• Laparoscopic – significantly less postoperative pain and wound
infections
• No significant differences with overall postoperative suture repair
leak, intra-abdominal abscess, reoperation rate, mortality
•  Reasonable to approach laparoscopically for stable patients and
with appropriate technically-skilled surgeon

Tarasconi A, Coccolini F, Biffl W, et al. Perforated and bleeding peptic ulcer: WSES guidelines. World J Emerg Surg. 2020;15(3)

Cirocchi R, Soreide K, Di Saverio S, et al. Meta-analysis of perioperative outcomes of acute laparoscopic versus open repair of perforated
gastroduodenal ulcers. J Trauma Acute Care Surg. 2018 Aug;85(2):417-25.

If small perforation < 2cm If large perforation > 2cm


• Primary repair Gastric
• Multiple studies now show the “standard” • Resection + reconstruction
addition of the omental ‘Graham’ patch does • Frozen pathology (to rule out malignancy)
not add benefit to simple suture primary
repair, though it significantly increases Duodenal
operative time • Resection, versus repair +/- pyloric exclusion
• Similar results between simple primary or external biliary drainage
repair and omentopexy • Only D1 can be easily resected without risking
• Consider adding omental patch for larger injury to bile duct or pancreatic head
ulcers with friable edges • Antrectomy +/- D1-D2 resection with
diversion
• Damage control procedure = pyloric exclusion,
gastric decompression via NGT or G-tube,
external biliary diversion via T-tube

Tarasconi A, Coccolini F, Biffl W, et al. Perforated and bleeding peptic ulcer: WSES guidelines. World J Emerg Surg. 2020;15(3)

1580
Copyright © Oakstone Publishing, 2021. All Rights Reserved.

Case #5
68 yo F with PMH of HTN, HLD, GERD, recurrent diverticulitis, who presents with 2wk
history of abdominal pain and intermittent diarrhea. Yesterday, her symptoms acutely
worsened, and she noticed new onset chills and mild nausea. Pain is mostly in the LLQ
and radiates across to the RLQ. Denies blood in her diarrhea. In the ED she remains
AVSS. Labwork demonstrates WBC 14.8, Cr 1.3, lactate 0.6. CT abdomen/pelvis reveals
scattered foci of gas in the peritoneum and a small fluid collection adjacent to multiple
small and large bowel loops with extensive colonic diverticulosis. She is admitted to
the floor for bowel rest, IV antibiotics, and close monitoring. Two days later, she
becomes progressively tachycardic with worsening pain and new peritoneal signs.

Presentation likely represents perforated diverticulitis with failed


medical management.
What are some approaches to surgical management?

Surgical approach to colonic perforation

• Key considerations:
• Oversew small perforation – if tissue appears healthy
and well-vascularized
• Resection – if larger perforation and devascularized
colonic wall
• May need to consider damage control surgery
• Risk factors: old age, delayed diagnosis, HD instability,
immunosuppressed, sepsis, high surgical risk
• Laparotomy, control contamination, temporary
abdominal closure  resuscitation, return to OR in 24-
48hrs for definitive repair and closure

Halim H, Askari A, Nunn R, Hollingshead J. Primary resection anastomosis versus Hartmann’s procedure in Hinchey III and IV diverticulitis. World J Emerg Surg. 2019 Jul 11;14:32.

Sartelli M, Weber DG, Kluger Y, et al. 2020 update of the WSES guidelines for the management of acute colonic diverticulitis in the emergency setting. World J Emerg Surg. 2020;15(32).

De’Angelis N, Di Saverio S, Chiara O, et al. 2017 WSES guidelines for the management of iatrogenic colonoscopy perforation. World J Emerg Surg. 2018;13(5).

1581
Copyright © Oakstone Publishing, 2021. All Rights Reserved.

Surgical approach to colonic perforation


Primary anastomosis +/- diverting ostomy, versus sigmoidectomy + end colostomy (Hartmann’s)
• Depends on patient comorbidities and clinical condition – generally primary anastomosis for Hinchey I/II, and
Hartmann’s for Hinchey III/IV
• Though studies often do not show significant differences in outcomes
• 2017 DIVERTI multicenter RCT (Primary vs Secondary Anastomosis for Hinchey Stage III-IV Diverticulitis) –
comparable complications; and at 18 months, stoma reversal able to be performed in 96% of primary
anastomoses and 65% of Hartmann’s
•  Primary anastomosis appears to be possible safe alternative for select patients needing emergent surgery

Bridoux V, Regimbeau JM, Ouaissi M, et al. Hartmann’s procedure or primary anastomosis for generalized peritonitis due to perforated
diverticulitis: A prospective multicenter randomized trial (DIVERTI). J Am Coll Surg. 2017 Dec;225(6):798-805.

Case #6
58 yo M with PMH of afib, GERD, obesity s/p open RYGB, recurrent ventral hernias,
presents with 2d history of abdominal pain with associated nausea/vomiting. He states
that in the past 20yrs of having recurrent hernias, it has always been reducible, though
this time given the significant pain he did not try reducing the hernia. He can’t
remember when he last passed flatus. Work-up is notable for temp 101.6, HR 100s,
and WBC 14.8. Exam is remarkable for 4x4cm infraumbilical hernia with overlying
erythema, exquisite tenderness with palpation, and +rebound and guarding. CT
abdomen/pelvis reveals a small segment of distended, fluid-filled loops of small bowel
through the hernia defect with mild bowel wall thickening.

You suspect an incarcerated ventral hernia with an associated


small bowel obstruction.
How urgent should intervention be considered?

1582
Copyright © Oakstone Publishing, 2021. All Rights Reserved.

• 2341 patients with SBO across 131 UK hospitals


• 17.7% had SBO due to a hernia  75.2% of these required surgical intervention, with 63.5% needing
bowel resection and 17.3% needing stoma creation
• Mortality rates: operative patients 8.3%, nonoperative patients 12.6%
• Improved outcomes for those who underwent prompt surgery for clinically apparent bowel ischemia
• Patients unsuitable or too sick for surgery managed nonoperatively
• Other studies identify the importance of quick diagnosis and decision for surgical intervention
• Delay in surgery >24hrs of admission:
‒ 3x increase in mortality
‒ Higher rates of complications, need for bowel resection, and longer LOS

National Audit of Small Bowel Obstruction Steering Group and National Audit of Small Bowel Obstruction Collaborators. Outcomes of obstructed abdominal wall hernia: Results from the
UK national small bowel obstruction audit. BJS Open. 2020 Oct;4(5):924-34.

Gonzalez-Urquijo M, Tellez-Giron VC, Martinez-Ledesma E, et al. Bowel obstruction as a serious complication of patients with femoral hernia. Surg Today. 2021 May;51(5):738-44.

Case #7
48 yo M with PMH of tobacco use, CAD, afib, T2DM, CVA, metastatic bladder cancer
presents with acute onset of diffuse severe abdominal pain. Pain has been unrelieved
with positioning or medications. Denies nausea/vomiting though has not had any PO
intake due to the severity of the pain. Has never had pain like this before. On
evaluation, he is tachycardic to HR 130s though normotensive, and exam is notable for
slightly distended abdomen with diffuse moderate tenderness and mild rebound and
voluntary guarding. Lab work includes WBC 11.2, Cr 1.37, INR 0.9, lactate 7.9, anion
gap 18. CT abdomen/pelvis demonstrates occlusion of the proximal SMA with bowel
dilatation and bowel wall thickening of the small bowel and cecum.

Presentation is suggestive of acute mesenteric ischemia, likely of


embolic etiology.
How do you approach management?

1583
Copyright © Oakstone Publishing, 2021. All Rights Reserved.

Management of acute mesenteric ischemia

• Surgical intervention is not always indicated and is dependent


on most likely etiology
• Management:
• Fluid resuscitation
• Broad-spectrum antibiotics
• NGT decompression, bowel rest
• +/- Systemic anticoagulation
• +/- Emergent surgery
‒ Indications: peritoneal signs, metabolic acidosis, lactic
acidosis, pneumatosis intestinalis, portal venous gas,
bowel ischemia

Occlusive mesenteric ischemia (mesenteric


arterial embolism, mesenteric arterial
thrombosis, mesenteric venous thrombosis) Non-occlusive mesenteric ischemia (NOMI)
• Goals of surgery: re-establish blood supply to • Intestinal hypoperfusion most commonly at
ischemic bowel, resect non-viable bowel, preserve watershed areas of colon with limited
viable bowel collateralization (splenic flexure, rectosigmoid
• Procedure(s): laparotomy, SMA embolectomy, junction)
bowel resection and left in discontinuity, temporary • Typically as a consequence of a “low flow state”
abdominal closure  2nd look laparotomy, bowel (heart failure, sepsis)
anastomosis, delayed abdominal closure • More gradual onset
• If uncertain about bowel viability, can use • Worsening hypotension
intraoperative Doppler • Progressive organ failure
• If unsuccessful embolectomy, other • Treat underlying etiology of hypoperfusion
revascularization options:
‒ Thromboendarterectomy
‒ Mesenteric bypass
‒ Mesenteric stent

Bala M, Kashuk J, Moore EE, et al. Acute mesenteric ischemia: Guidelines of the World Society of
Emergency Surgery. World J Emerg Surg. 2017 Aug 7;12:38.

1584
Copyright © Oakstone Publishing, 2021. All Rights Reserved.

Case #8
71 yo M with PMH of HTN, HLD, afib, who initially presented with ruptured AAA
requiring massive transfusion and s/p emergent repair. He remains critically ill in the
ICU. On postoperative day 1, the ICU nurse notices new firm abdominal distention.
Due to a concern for intra-abdominal hypertension, a bladder pressure is measured to
be 16 mmHg. Throughout the morning, the patient develops increasing ventilatory
requirements with elevated peak airway pressures, as well as decreased urine output.
A trend of hourly bladder pressures is recorded: 18 mmHg, 16 mmHg, 20 mmHg, 23
mmHg, 21 mmHg.

You are concerned about abdominal compartment syndrome.


How do you decide if, and when, surgery is indicated?

• Intra-abdominal • Abdominal compartment


Intra- Hypertension syndrome
abdominal • Bladder Pressure
>12mmHg (15cmH20)
• Bladder pressure>20
mmHg (29cmH2O)
Hypertension • with evidence of
pressure related
and organ dysfunction
Abdominal
Compartment
Syndrome

2004 International ACS Consensus Definitions Conference

1585
Copyright © Oakstone Publishing, 2021. All Rights Reserved.

Measurement
• Direct • Indirect
of Intra- • Measuring pressure via • Transduced pressure
abdominal a catheter introduced
directly into the
from inside accessible
organs
Pressure peritoneal cavity • Intragastric
• Intracolic
• Intravesicle
• IVC catheter

Measurement of
Intra-abdominal
Pressure
• Prospective blinded trial of
staff physician ability to
detect intra-abdominal
hypertension
• < 50% of the time was the
clinician able to determine
when IAP was elevated.

Kirkpatrick, Can J Surg 2000

1586
Copyright © Oakstone Publishing, 2021. All Rights Reserved.

Bladder Pressure

• Instill 50cc saline into bladder via


Foley
• Tubing at collecting bag clamped
• Transducer introduced at sampling
port
• Transducer zeroed at level of
symphysis pubis

Bladder Pressure

• May be inaccurate in the setting of


• Intraperitoneal adhesions
• Pelvic hematoma/fracture
• Abdominal packing
• Neurogenic bladder
• Pelvic tumor

1587
Copyright © Oakstone Publishing, 2021. All Rights Reserved.

Management of abdominal compartment syndrome (ACS)

Supportive care and temporizing measures


• Evacuate intraluminal contents via decompression
• NGT, foley, rectal tube
• Evacuate intra-abdominal space-occupying lesions
(ascites, hematoma)
• Improve abdominal wall compliance
• Pain control, paralytic, sedation
• Optimize fluid administration
• Avoid excess fluid resuscitation
• Judicious diuresis if stable
• Optimize systemic perfusion
• Goal-directed fluid therapy
• Monitor hemodynamics to guide resuscitation

Coccolini F, Roberts D, Ansaloni L. The open abdomen in trauma and non-trauma patients: WSES
guidelines. World J Emerg Surg. 2018(13):7.

Surgical consideration for abdominal compartment syndrome

• Need to consider 3 factors:


• Intra-abdominal pressure
• Characteristics of organ dysfunction
• Nature and course of underlying etiology
• Intervention should be expected to have a beneficial
impact on intra-abdominal pressure and organ function
• Urgency and timing of intervention depend on:
• Value of intra-abdominal pressure
• Rate at which intra-abdominal pressure is increasing
• Degree of organ dysfunction

De Laet IE, Malbrain MLNG, De Waele JJ. A clinician’s guide to management of intra-abdominal hypertension and
abdominal compartment syndrome in critically ill patients. Crit Care. 2020 Mar 24;24(1):97.

1588
Copyright © Oakstone Publishing, 2021. All Rights Reserved.

Management of abdominal compartment syndrome (ACS)

Surgical intervention
• Decompressive laparotomy, temporary abdominal
closure, return to OR in 24-48hrs
• Definitive abdominal closure
• Once resuscitated, resolved acidosis, restored
perfusion
• May require multiple trips to OR for attempts at
closure – do not want to risk premature closure
and recurrence of ACS
• Decompression can cause severe ischemia-reperfusion
requiring high supportive measures

Coccolini F, Roberts D, Ansaloni L. The open abdomen in trauma and non-trauma patients: WSES guidelines. World J Emerg Surg. 2018(13):7.

De Laet IE, Malbrain MLNG, De Waele JJ. A clinician’s guide to management of intra-abdominal hypertension and abdominal compartment syndrome in
critically ill patients. Crit Care. 2020 Mar 24;24(1):97.

Top 6 Surgical GI Emergencies


Acute appendicitis

Acalculous / perforated cholecystitis

Perforated hollow viscus

Obstructed hernia

Acute mesenteric ischemia

Abdominal compartment syndrome

1589
Copyright © Oakstone Publishing, 2021. All Rights Reserved.

Questions?

Question #1
72 yo M presents to the ED with 3d history of abdominal pain and diarrhea. He reports pain
across his entire lower abdomen, though denies nausea/vomiting. Vitals include temp 101.4,
HR 120s, BP 96/58. Exam is notable for a distended abdomen, severe tenderness to palpation
diffusely, and rebound tenderness. CT abdomen/pelvis demonstrates small amount of
pneumoperitoneum, moderate free fluid, and thickening of the sigmoid colon. You suspect
perforated diverticulitis, Hinchey classification III-IV, and bring him to the OR. Intraoperatively
you identify frank stool in the abdomen from a large colonic perforation. Throughout this time,
the patient continues to deteriorate despite fluid resuscitation and requires increasing pressor
support. What is the next step in management?

A) Oversew perforation site, omental patch


B) Resection with primary anastomosis and diverting ostomy
C) Sigmoidectomy with end colostomy
D) Resection, temporary abdominal closure, return to the OR for 2nd look in 24-48hrs

1590
Copyright © Oakstone Publishing, 2021. All Rights Reserved.

Question #1
72 yo M presents to the ED with 3d history of abdominal pain and diarrhea. He reports pain
across his entire lower abdomen, though denies nausea/vomiting. Vitals include temp 101.4,
HR 120s, BP 96/58. Exam is notable for a distended abdomen, severe tenderness to palpation
diffusely, and rebound tenderness. CT abdomen/pelvis demonstrates small amount of
pneumoperitoneum, moderate free fluid, and thickening of the sigmoid colon. You suspect
perforated diverticulitis, Hinchey classification III-IV, and bring him to the OR. Intraoperatively
you identify frank stool in the abdomen from a large colonic perforation. Throughout this time,
the patient continues to deteriorate despite fluid resuscitation and requires increasing pressor
support. What is the next step in management?

A) Oversew perforation site, omental patch


B) Resection with primary anastomosis and diverting ostomy
C) Sigmoidectomy with end colostomy
D) Resection, temporary abdominal closure, return to the OR for 2nd look in 24-48hrs

Answer #1
The patient is starting to show hemodynamic instability and peritoneal signs in the ED. This,
coupled with Hinchey classification III-IV, prompts the decision to proceed with surgical
intervention. In the OR, his clinical condition continues to worsen in the setting of pressor
support despite fluid resuscitation. While studies show that either primary anastomosis +/-
diverting stoma or sigmoidectomy with end colostomy can be suitable for perforated
diverticulitis, in this situation the patient’s deterioration warrants damage control surgery.
Once the source of contamination is controlled by resecting the perforated segment of colon,
place a temporary abdominal closure dressing and allow the patient to be resuscitated in the
ICU prior to returning to the OR in 24-48hrs for delayed repair and closure.

1591
Copyright © Oakstone Publishing, 2021. All Rights Reserved.

Question #2
42 yo F is admitted to the SICU after a fall from 30ft, sustaining SDH, multiple spinal fractures,
and a grade III splenic laceration managed nonoperatively. Since admission, she has received
10L crystalloids and 2u pRBC. She is sedated and mechanically ventilated on AC mode. You are
called by the SICU team for slowly rising peak airway pressures without any other changes in
ventilatory parameters. The patient is suctioned with scant amount of bloody secretions. Her
urine output has also decreased from 70cc/hr to 15cc/hr. What is the next most appropriate
step in management?

A) Neuromuscular blockade
B) Bladder pressure measurement
C) Fluid bolus with 1L crystalloid
D) Change to PC ventilation

Question #2
42 yo F is admitted to the SICU after a fall from 30ft, sustaining SDH, multiple spinal fractures,
and a grade III splenic laceration managed nonoperatively. Since admission, she has received
10L crystalloids and 2u pRBC. She is sedated and mechanically ventilated on AC mode. You are
called by the SICU team for slowly rising peak airway pressures without any other changes in
ventilatory parameters. The patient is suctioned with scant amount of bloody secretions. Her
urine output has also decreased from 70cc/hr to 15cc/hr. What is the next most appropriate
step in management?

A) Neuromuscular blockade
B) Bladder pressure measurement
C) Fluid bolus with 1L crystalloid
D) Change to PC ventilation

1592
Copyright © Oakstone Publishing, 2021. All Rights Reserved.

Answer #2
This patient’s slowly rising peak airway pressures indicate a different process than pressures
that may rise suddenly. The patient has received large-volume fluid resuscitation from the
trauma resuscitation and throughout her hospital course so far for her multiple injuries. This
can lead to an increase in intra-abdominal pressure from visceral edema, hemorrhage, and
acute ascites formation. Measuring the intra-abdominal pressure using bladder pressure will
assess for intra-abdominal hypertension and abdominal compartment syndrome.

1593

You might also like